You are on page 1of 375
BELEIE TY BHI RED Abel ely a ee INCLUDING 720 SOLVED PROBLEMS bas ‘t ol SCHAUM’S OUTLINE OF ys THEORY AND PROBLEMS oF THEORETICAL MECHANICS with an introduction to Lagrange’s Equations and Hamiltonian Theory BY MURRAY R. SPIEGEL, Ph.D. Professor of Mathematics Rensselaer Polytechnic Institute SCHAUM’S OUTLINE SERIES McGRAW-HILL BOOK COMPANY ‘New York, St. Louis, San Francisco, Toronto, Sydney Copyright © 1967 by MoGraw-Hill, Ine, All rights reserved, Printed in the United States of America. No part of this publication may be reproduced, stored in a retrieval system, or transmitted, in any form or by any moans. ‘lectronte, mechanical, photocopying, recording, or otherwise, without the prlor ‘written permission of the publisher. one 810 11 12 18 1415 SH SH 75 Preface In the 17th century, Sir Isaac Newton formulated his now famous laws of mechanics. ‘These remarkably simple laws served to describe and predict the motions of observable objects in the universe, including those of the planets of our solar system. Early in the 20th century it was discovered that various theoretical conclusions de- rrived from Newton's laws were not in accord with certain conclusions deduced from theories of electromagnetism and atomic phenomena which were equally well founded experimentally. ‘These discrepancies led to Einstein's relativistic mechanics which revolutionized the con- cepts of space and time, and to quantum mechanics, For objects which move with speeds much less than that of light and which have dimensions large compared with those of atoms and molecules Newtonian mechanics, also called classical mechanics, is nevertheless quite satisfactory. For this reason it has maintained its fundamental importance in science and engineering. It is the purpose of this book to present an account of Newtonian mechanics and its applications. The book is designed for use either as a supplement to all current standard textbooks or as a textbook for a formal course in mechanics. It should also prove useful to students taking courses in physics, engineering, mathematics, astronomy, celestial me- chanics, aerodynamics and in general any field which needs in its formulation the basic principles of mechanics. Each chapter begins with a clear statement of pertinent definitions, principles and theorems together with illustrative and other descriptive material. This is followed by graded sets of solved and supplementary problems. The solved problems serve to illustrate and amplify the theory, bring into sharp focus those fine points without which the student continually feels himself on unsafe ground, and provide the repetition of basic principles 80 vital to effective learning. Numerous proofs of theorems and derivations of basic re- sults are included in the solved problems. The large number of supplementary problems with answers serve as a complete review of the material of each chapter. Topics covered include the dynamics and statics of a particle, systems of particles and rigid bodies. Vector methods, which lend themselves so readily to concise notation and to geometric and physical interpretations, are introduced early and used throughout the book. An account of vectors is provided in the first chapter and may either be studied at the be- ginning or referred to as the need arises. Added features are the chapters on Lagrange’s equations and Hamiltonian theory which provide other equivalent formulations of ‘Newtonian mechanics and which are of great practical and theoretical value. Considerably more material has been included here than can be covered in most courses. ‘This has been done to make the book more flexible, to provide a more useful book of reference and to stimulate further interest in the topics. I wish to take this opportunity to thank the staff of the Schaum Publishing Company for their splendid cooperation. M. R. SPIEGEL Rensselaer Polytechnic Institute February, 1967 CONTENTS VECTORS, VELOCITY AND ACCELERATION . : Mechanics, kinematics, dynamics and statics. Axiomatic foundations of me chanics. Mathematical models. Space, time and matter. Scalars and vectors, Vector algebra. Laws of vector algebra, Unit vectors, Rectangular unit vee- tors. Components of a vector. Dot or scalar product, Cross or vector product. ‘Triple products, Derivatives of vectors, Integrals of vectors. Velocity. Ac celeration. Relative velocity and acceleration. Tangential and normal acceler- ation. Circular motion. Notation for time derivatives, Gradient, divergence and curl. Line integrals. Independence of the path. Free, sliding and bound vectors. NEWTON'S LAWS OF MOTION. WORK, ENERGY AND MOMENTUM : s cee ‘Newton's laws. Definitions of force and mass. Unite of force and mass. Inertial frames of reference. Absolute motion. Work. Power. Kinetic energy. Conservative force fields. Potential energy or potential. Conservation of energy. Impulse. Torque and angular momentum. Conservation of momentum, * Conservation of angular momentum. Non-conservative forces, Statics or equi- librium of a particle. Stability of equilibrium. Chapter 3 MOTION IN A UNIFORM FIELD. FALLING BODIES AND PROJECTILES ....... 7 ry Uniform force fields. Uniformly secelerated motion, Weight and acceleration due to gravity. Gravitational system of units. Assumption of a flat earth. Freely falling bodies. Projectiles. Potential and potential energy in a uniform foree field. ‘Motion in a resisting medium. Isolating the system. Constrained motion, Friction. Statics in a uniform gravitational fil THE SIMPLE HARMONIC OSCILLATOR AND THE SIMPLE PENDULUM .. ee : ‘The simple harmonic oscillator. Amplitude, period and frequency of simple ‘harmonic motion. Energy of a simple harmonic oscillator. ‘The damped har- ‘monic oscillator. Over-damped, critically damped and under-damped motion. Forced vibrations, Resonance. The simple pendulum. ‘The two and three dimensional harmonic oecillator. 62, 86 CENTRAL FORCES AND PLANETARY MOTION Central forces, Some important properties of central force filds. Equations of motion for a particle in a central fleld. Important equations deduced trom ‘the equations of motion. Potential energy of a particle in a central field. Con- servation of energy. Determination of the orbit from the central force. Deter ‘mination of the central force from the orbit, Conic sections, ellipse, parabola and hyperbola, Some definitions in astronomy. Kepler's laws of ‘planetary motion. Newton's universal law of gravitation. Attraction of spheres and other objects, Motion in an inverse square field, 6 CONTENTS Page MOVING COORDINATE SYSTEMS ‘Non-inertial coordinate systems. Rotating coordinate operators, Velocity in a moving system. Acceleration in a moving Coriolis and centripetal acceleration. Motion of a particle relative to the earth, Coriolis and centripetal force, Moving coordinate systems in general. The Foucault pendulum, SYSTEMS OF PARTICLES . ee ie, Diserete and continuous systems. Density. Rigid and clastic bodies. Degrees of freedom. Center of mass, Center of gravity. Momentum of a system of articles, Motion of the center of mass. Conservation of momentum. Angular momentum of « system of particles. Total external torque acting on a system. Relation between angular momentum and total external torque. Conservation of angular momentam, Kinetic energy of a system of particles. Work. Po- tential energy. Conservation of energy. Motion relative to the center of mass. Impulse. Constraints, Holonomie and non-holonomic constraints. Virtual dis- placements. Statice of a system of particles. Principle of virtual work. Equi- Tibrium in conservative fields. Stability of equilibrium. D'Alembert’s principle APPLICATIONS TO VIBRATING SYSTEMS, ROCKETS AND COLLISIONS ........0-.0:000c0ceccecceceess 198 Vibrating systems of particles, Probloms involving changing mass. Rockets. Collisions of particles, Continuous systems of particles. ‘The vibrating string. Boundary-value problems, Fourier series. Odd and even functions. Con- vergence of Fourier series. PLANE MOTION OF RIGID BODIES Rigid bodies, Translations and rotations. Euler's theor ‘axis of rotation. Degrees of freedom. General motion of a rigid body. Chasle's theorem, Plane motion of a rigid body. Moment of inertia. Radius of gyra- tion. Theorems on moments of inertia, Parallel axis theorem. Perpendicular fares theorem, Special moments of inertis. Couples. Kinetie energy and Angular momentum ebout a fixed axis, Motion of a rigid body about a fixed axis, Principle of angular momentum. Principle of conservation of energy. Work and power. Impulse. Conservation of angular momentum. ‘The com- pound pendulum. General plane motion of a rigid body. Instantaneous center. Bpace and body centrodes, Statics of a rigid body. Principle of virtual work fand D’Alember(’s principle, Principle of minimom potential energy. Stability. Se Chapter 10 SPACE MOTION OF RIGID BODIES ceveeess 258 General motion of rigid bodies in space. Degrees of freedom. Pure rotation of rigid bodies. Velocity and angular velocity of a rigid body with one point fixed. Angular momentum, Moments of inertia. Products of inertia, Moment of inertia matrix or tensor. Kinetic energy of rotation. Principal axes of {nertia, Angular momentum and kinetic energy about the principal axes. The llipeoid of inertia. Euler's equations of motion. Force free motion. The in- Variable line and plane. Poineot’s construction. Polhode. Herpolhode. Space lind body cones. Symmetric rigid bodies. Rotation of the earth. The Euler tangles, Angular velocity and kinetic energy in terms of Euler angles, Motion of a spinning top. Gyroscopes, Chapter 11 CONTENTS LAGRANGE'S EQUATIONS ee ee General methods of mechanics. Generalized coordinates. Notation. Tra formation equations. Classification of mechanical systems. Scleronomic Theonomic systems. Holonomie and non-holonomie systems. Conservative and non-conservative systems. Kinetic energy. Generalized velocities. Gener= ‘lized forces, Lagrange's equations. Generalized momenta. Lagrange’s equa- tions for non-holonomic aystems. Lagrange’s equations with impulsive forces. Page Chapter 12 HAMILTONIAN THEORY = a foto Hamiltonian methods. The Hamiltonian. Hamilton's equations. The Hamil. tonian for conservative systems. Ignorable or cyclic coordinates. Phase space, Liouville’s theorem. ‘The calculus of variations, Hamilton's principle. Cat onical or contact transformations. Condition that a transformation be canoni- cal. Generating functions. The Hamilton-Jacobi equation. Solution of the Hamilton-Jacobi equation. Case where Hamiltonian is independent of time, Phase integrals. Action and angle variables 31 APPENDIX A UNITS AND DIMENSIONS .... APPENDIX B ASTRONOMICAL DATA .. APPENDIX C SOLUTIONS OF SPECIAL DIFFERENTIAL EQUATIONS . APPENDIX D INDEX OF SPECIAL SYMBOLS AND NOTATIONS INDEX .. Chapter 1 MECHANICS, KINEMATICS, DYNAMICS AND STATICS ‘Mechanics is a branch of physics concerned with motion or change in position of physical objects. It is sometimes further subdivided into: 1. Kinematics, which is concerned with the geometry of the motion, 2. Dynamics, which is concerned with the physical causes of the motion, 8. Statics, which is concerned with conditions under which no motion is apparent. AXIOMATIC FOUNDATIONS OF MECHANICS An axiomatic development of mechanics, as for any science, should contain the following basic ingredients: 1. Undefined terms or concepts. This is clearly necessary since ultimately any definition must be based on something which remains undefined. 2. Unproved assertions. These are fundamental statements, usually in mathematical form, which it is hoped will lead to valid descriptions of phenomena under study. In general these statements, called azioms or postulates, are based on experimental observations or abstracted from them. In such case they are often called laws. 8. Defined terms or concepts. These definitions are given by using the undefined terms or concepts. . Proved assertions. These are often called theorems and are proved from the definitions and axioms. An example of the “axiomatic way of thinking” is provided by Euclidean geometry in which point and line are undefined concepts. MATHEMATICAL MODELS A mathematical description of physical phenomena is often simplified by replacing actual physical objects by suitable mathematical models. For example in describing the rotation of the earth about the sun we can for many practical purposes treat the earth and sun as points. SPACE, TIME AND MATTER From everyday experience, we all have some idea as to the meaning of each of the following terms or concepts. However, we would certainly find it difficult to formulate completely satisfactory definitions, We take them as undefined concepts. 1 2 VECTORS, VELOCITY AND ACCELERATION (CHAP.1 1. Space. This is closely related to the concepts of point, position, direction and displacement. Measurement in space involves the concepts of length or distance, with which we assume familiarity. Units of length are fect, meters, miles, ete. In this book we assume that space is Euclidean, ie. the space of Euclid’s geometry. 2. Time. This concept is derived from our experience of having one event taking place after, before or simultaneous with another event. Measurement of time is achieved, for example, by use of clocks. Units of time are seconds, hours, years, ete. 8. Matter. Physical objects are composed of “small bits of matter” such as atoms and molecules. From this we arrive at the concept of a material object called a particle which can be considered as occupying a point in space and perhaps moving as time goes by. A measure of the “quantity of matter” associated with a particle is called its mass, Units of mass are grams, kilograms, ete. Unless otherwise stated we shall assume that the mass of a particle does not change with time. Length, mass and time are often called dimensions from which other physical quantities are constructed. For a discussion of units and dimensions see Appendix A, Page 339. SCALARS AND VECTORS ‘Various quantities of physics, such as length, mass and time, require for their specifica- tion a single real number (apart from units of measurement which are decided upon in advance). Such quantities are called scalars and the real number is called the magnitude of the quantity. A scalar is represented analytically by a letter such as t, m, ete. Other quantities of physics, such as displacement, require for their specification a direction as well as magnitude. Such quantities are called vectors. A vector is repre- sented analytically by a bold faced letter such as A in Fig. 1-1. Geometrically it is represented by an arrow PQ where P is called the initial point and @ is called the terminal point. The magnitude or length of the vector is then denoted by [Al or A. Fig. 1-8 Fig. 1 VECTOR ALGEBRA ‘The operations of addition, subtraction and multiplication familiar in the algebra of real numbers are with suitable definition capable of extension to an algebra of vectors. ‘The following definitions are fundamental. 1. Two vectors A and B are equal if they have the same magnitude and direction regardless of their initial points. Thus A=B in Fig. 1-2 above. 2. A vector having direction opposite to that of vector A but with the same length is denoted by —A as in Fig. 1-3 above. 8, ‘The sum or resultant of vectors A and B of Fig. 1-4(a) below is a vector C formed by placing the initial point of B on the terminal point of A and joining the initial point of A to the terminal point of B [see Fig. 1-4(0) below]. We write C = A+B. ‘This definition is equivalent to the parallelogram law for vector addition as indicated in Fig. 1-4(¢) below. CHAP. 1] VECTORS, VELOCITY AND ACCELERATION 3 : a Fig 4 Extensions to sums of more than two vectors are immediate. For example, Fig. 1-5 below shows how to obtain the sum or resultant E of the vectors A,B,C and D. 8\ | 5 Y arate Fig. 1-5 4, The difference of vectors A and B, represented by A—B, is that vector C which when added to B gives A. Equivalently, A—B may be defined as A+(-B). If A=B, then A—B is defined as the null or zero vector represented by 0. This has ‘& magnitude of zero but its direction is not defined. 5. The product of a vector A by a scalar p is a vector pA or Ap with magnitude [p| times the magnitude of A and direction the same as or opposite to that of A according as p is positive or negative. If p=0, pA=0, the null vector. LAWS OF VECTOR ALGEBRA If A,B and C are vectors, and p and q are scalars, then 1 A+B=B+A Commutative Law for Addition 2 A+(B+C) = (A+B)+C Associative Law for Addition 8. p(GA) = (pa)A = a(pA) Associative Law for Multiplication 4. (p+ QA = pPA+qA Distributive Law 5. p(A+B) = pA+2B Distributive Law Note that in these laws only multiplication of a vector by one or more scalars is defined. On pages 4 and 5 we define products of vectors. UNIT VECTORS Vectors having unit length are called unit vectors. If A is a vector with length A > 0, then A/A=a is a unit vector having the same direction as A and A= Aa. RECTANGULAR UNIT VECTORS ‘The rectangular unit vectors i, j and k are mutually perpendicular unit vectors having directions of the positive 2, y and z axes respectively of a rectangular coordinate system 4 VECTORS, VELOCITY AND ACCELERATION [cHAP.1 (see Fig. 1-6]. We use right-handed rectangular coordinate systems unless otherwise specified. Such systems derive their name from the fact that a right threaded serew ro- tated through 90° from Oz to Oy will advance in the posi- tive z direction. In general three vectors A, B and C which have coincident initial points and are not coplanar are said to form a right-handed system or deztral system if a right threaded screw rotated through an angle less than 180° from A to B will advance in the direction C [see Fig. 1-7 below] Fig.7 Fig. 18 COMPONENTS OF A VECTOR Any vector A in 8 dimensions can be represented with initial point at the origin O of ‘a rectangular coordinate system (see Fig. 1-8 above]. Let (Ai, As, As) be the rectangular coordinates of the terminal point of vector A with initial point at 0. ‘The vectors Aui, ‘Aaj and Ask are called the rectangular component vectors, or simply component vectors, of A in the x, y and 2 directions respectively. As, Az and A: are called the rectangular components, or simply components, of A in the x, y and z directions respectively. ‘The sum or resultant of Axi, Aaj and Ask is the vector A, so that we can write A= Aut Adj + Ack « ‘The magnitude of A is A= al = VAT+4T+ AE @ In particular, the position vector or radius vector x from O to the point (2,y,2) is written ra cityitek ® and has magnitude r Vetere. DOT OR SCALAR PRODUCT ‘The dot or scalar product of two vectors A and B, denoted by A-B (read A dot B) fs defined as the product of the magnitudes of A and B and the cosine of the angle between them. In symbols, A+B = ABcosé, 05025 ) ‘Note that A+B is a scalar and not a vector. CHAP. 1] VECTORS, VELOCITY AND ACCELERATION 5 The following laws are valid: 1. A-B= BA Commutative Law for Dot Products 2. A+(B+C) = A-B+A-C Distributive Law 3. p(A+B) (pA)-B = A+(pB) = (A-B)p, where p isa scalar. 4 ied ke 1 i tk = ki = 0 5. If A= Ad+Asj+Ask and B= Bit B:j+Bsk, then ACB = A,B, + AsBa+ AaB AA = A? = At+AE+ AE BB = BY = Bi+Bi+ BY 6. If A+B =0 and A and B are not null vectors, then A and B are perpendicular. CROSS OR VECTOR PRODUCT The cross or vector product of A and B is a vector C = AXB (read A cross B). The magnitude of AX B is defined as the product of the magnitudes of A and B and the sine of the angle between them. The direction of the vector C = AXB is perpendicular to the plane of A and B and such that A, B and C form a right-handed system. In symbols, AXB = ABsingu, 0505 ) where u is a unit vector indicating the direction of AXB. If A=B or if A is parallel to B, then sing=0 and we define AXB = 0. ‘The following laws are valid: 1. AXB = -BxA (Commutative Law for Cross Products Fails) 2 AX(B+C) = AXB+AxC _ Distributive Law 8. p(AXB) = (pA)XB = AX(pB) = (AXB)p, where p is a scalar. 4. ixi Kxk=0, ixjsk, jxk=i, kxi=j 5. If A= Ad+Asj+Adk and B= Bi+Bij+Bek, then iik AXB A, Az As By Bs Bs 6. JAX) = the area of a parallelogram with sides A and B. 7. If AxB=0 and A and B are not null vectors, then A and B are parallel. TRIPLE PRODUCTS ‘The scalar triple product is defined as Ai Ar As A+(BxC) By By Bs @ Ci Gr Cs where A = Ai+Asj+Ask, B= Bii+Baj+Buk, C = Ci+Caj+Cok. It represents the volume of a parallelepiped having A,B, C as edges, or the negative of this volume according as A,B,C do or do not form a right handed system. We have A+(BXC) = B+(CX A) = C-(AxB). The vector triple product is defined as AX (BXC) = (A+B - (A-B)C ” Since (AxB)XC = (A-C)B—(B-C)A, it is clear that Ax (BXC) * (AXB)xC. 6 VECTORS, VELOCITY AND ACCELERATION [CHAP. 1 DERIVATIVES OF VECTORS _If to each value assumed by a sealar variable u there corresponds a vector A(u), or briefly A, then A(u) is called a (vector) function of u. ‘The derivative of A(u) is defined as tA ig Ale+aw) — Aw) ue jim, au ® du Fras) provided this limit exists. If A(u) = A,(w)itAz(u)j+Aa(u)k, then (9) Similarly we can define higher derivatives, For example the second derivative of A(u) if it exists is given by du @A _ PA, PAs @A at = aa + ae) + ae (10) Example, If A = (2?—3ui +5 cosuj—sinuk, then = au ‘The usual rules of differentiation familiar in the calculus can be extended to vectors, although order of factors in products may be important. For example if 4(u) is a scalar funetion while A(u) and B(u) are vector functions, then = unt = Sanus Seman, BE = ah ~ Goons + Sanuk da — dA fea = 9+ Ba ra ad au B) (12) faxes) = axB, Bye (as) INTEGRALS OF VECTORS Let A(w) = Au(u)i + Ax(w)i + Aske be a vector function of u. We define the indefinite integral of A(u) as Sacoau = if Adan + if Aswan + ef Asnaw aay If there exists a vector function B(u) such that A(u) = 2-(B(u)), then Sacau = f Fem du = Bw +e (a) where ¢ is an arbitrary constant vector independent of u. The definite integral between limits u=a and w= is in such case, as in elementary calculus, given by Sf moves ‘The definite integral can also be defined as a limit of a sum analogous to that of elementary caleulus. S Fea = wey te = BH - Be — ao) VELOCITY ‘Suppose that a particle moves along a path or curve C (Fig. 1-9 below]. Let the position vector of point P at time ¢ be r=r(t) while the position vector of point Q at time t+At is CHAP. 1] VECTORS, VELOCITY AND ACCELERATION 7 r+ar=s(t+at). Then the velocity (also called the instantaneous velocity) of the particle at P is given by de vag = img = tim Heta)— no et eae an and is a vector tangent to C at P. If r= x(t) = a(tit+u(ti tel) = ai tui + ek, we can write de do dy as v= HF = Gis Wis Se 8) Fig. 1-9 ‘The magnitude of the velocity is called the speed and is given by = day, (eY , (ty oe = VG) + (+ @) = & (9) where s is the arc length along C measured from some seme ACCELERATION If v = dr/dt is the velocity of the particle, we define the acceleration (also called the ‘instantaneous acceleration) of the particle at P as av (B+ Bus + Bylo) ABs + ABaleJ + ABgls ke + AgBieh + AgByj +4 + AgBal eke FASB + Ayah} + AgBake ke A.B, + ABs + ABs k = 1 and all other dot products are zero. u since 1+ =H LiL. If A= Ai+Asj+Ask, show that A = VAvA = VAT+AT+ AR. +A = (AYA) co80° = AX Then A= VATA. ‘Also, A*A = (A+ Agh+ Ash) “(Asi + Asi + Ask) (ANA) + (AAs) + (ADNAD = AT+ AE + AB by Problem 1.10, taking B= A. ‘Then A = VAvA = VATTAE4 AB is the magnitude of A. Sometimes A+A is written A® y B (46,0) 1.12, Find the acute angle between the diagonals of a quadrilateral having vertices at (0,0,0), (8,2,0), (46,0), (1,3, 0) (Fig. 1-21). We have OA=8i+2), OB=4i+6), OC=i+95 from which CA = OA-0C = 21-5 ‘Then (0B-CA = [OB||CAl cos ¢ he. (i+ 6+ 1-H) = VOFF OF VEFF HD cos & ‘from which cose = 2/(VB2 VB) = 1240 and ¢ = 82°63’. THE CROSS OR VECTOR PRODUCT 1.13. Prove AXB = —BXxA. re @ Fig. 122 AXB=C has magnitude AB sin ¢ and direction such that A, B and C form a right-handed system (Fig. 1-22(a) above]. u ‘VECTORS, VELOCITY AND ACCELERATION [cHar.1 D has magnitude BA sin and direction such that B, A and D form a right-handed 1-2(8) above) ‘Then D has the same magnitude as C but is opposite in direction, i ‘The commutative law for cross products is not valid. C=-D or AXB xa, 1.14. Prove that AX(B+C) = AXB+AxC for the case where A is perpendicular to B and also to C. Since A is perpendicular to B, AXB vector perpendicular to the plane of A and B and having magnitude AB sin90° = AB or ‘magnitude of AB. This is equivalent to mul- tiplying vector B by A and rotating the resultant vector through 90° to the position shown in Fig. 1-28, Similarly, AX C is the vector obtained by ‘multiplying C by A and rotating the resultant ‘vector through 80° to the position shown, In like manner, AX (B+ C) is the vector ‘obtained by multiplying B+ C by A and rotat- {ng the resultant vector through 90° to the position shown. Since AX (B+C) is the diagonal of the parallelogram with AX'B and AX C as sides, we have AX(B+C) = AXB+AXC, 115. Prove that Ax (B+C) = AXB+AxC in the general case where A, B and C are non-coplanar. See Fig. 1-24. Resolve B into two component vectors, one perpendicular to A and the other parallel to A, and denote them by B, and By, respectively, ‘Then B= B, +B, If 6 is the angle between A and B, then B, = Being. Thus the magnitude of AXB is AB sing, the same as the magnitude of AX. ‘Also, the direction of AX B is the same as the direction of AX. Hence AXB, =AXB. ‘Similarly if C is resolved into two component vectors C,, and C,, parallel and perpendicular respectively to A, then AXC, =AXC. Fig. 12 Also, since B+ = By +B, +C, +, = B,+C,)+(B,+C,) it follows that AX@B,+C,) = Ax(B+O) Now B, and C, are vectors perpendicular to A and so by Problem 1.14, AX(BL+C,) = AXBL+AxC, ‘Then AX (B+O) = AXB+AxC nd the distributive law holds. Multiplying by —1, using Problem 1.18, this becomes (B+) x A = BXA+CXA. Note that the order of factors in cross products is important. The usual laws of algebra apply only if proper order is maintained, tik 116. If A= Ad+Agj+Ack and B= Bi+B,j+Bsk, prove that AXB =| A: As As |. By B: By CHAP. 1) VECTORS, VELOCITY AND ACCELERATION 15 AXB = (Ad +Agd + Aso x (Blt Bi + Bd) AMX (Byl+ Bab + Bye) + AaS x (Byl + Bal + Buk) + Aske x (B+ By} + Bye) A:BAXA+ AyBgh XG + Ayah Xe + AaB XL + AaB XJ + AQBS Ke + AgBkX 1+ Ayah X 4+ ABs Xe ae Ar As ds By By By (AgBy— AgBDH + (AyBy~ ABO) + (AyBa— AB = LIZ. If A=8i-j+2k and B=2i+3j—k, find AxB. ijk eee ene 1.18 Prove that the area of a parallelogram with sides A and B is [AX Area of parallelogram = /|B| A {al ain 1B axB| Note that the area of the triangle with sides A and 5 Bis HAXB). Fig. 1-25 1.19, Find the area of the triangle with vertices at P(2,8,5), Q(4,2,—1), R(8, 6,4). PQ = 4-914 2-9) + (1-H = 2-1-6 PR = (8-21 + (6-9) + (4—Ok = 1+ 8)—k Area of triangle = $/PQXPR) = $|(2—§—6k) x (1+3)—W0] tik = alle -1 -6|) = g)ir—a+m 1 8-1 AVF CRF = 4 VRE ‘TRIPLE PRODUCTS 1.20. Show that A- (BX C) is in absolute value equal to the volume of a parallelepiped with sides A,Band C. Let m bo a unit normal to parallelogram I, having the direction of BX C, and loth be the height of the terminal point of A above the parallelogram Z. Fig. 1.28 ‘Volume of parallelepiped = (height A)area of parallelogram 2) (Army e) = As(BXC|a} = A-BxXO) If A, B and C do not form a right-handed system, A+n <0 and the volume = [A+ (BXO)|. 121, (a) If A = Ad+Asj+Adk, B= Bi+Bij +B, C = Cit+Caj+Csk show that Ay As As A(BxC) = | Bi Bs Bs C1 Ch Cs 16 VECTORS, VELOCITY AND ACCELERATION (CHAP. 1 (©) Give a geometric significance of the case where A- (Bx C) Li. (@ Axo = A-lB, BB Cy Cy Cs = (Ask + Aad + Ask) + [(B,04— BCH + (BsC,— BCD + (ByC— BAC] Ay Ay As = A,(BaCy— ByCs) + Ay(ByC,— BiG) + ABC,—ByCy) = | By By By % OO (®) By Problem 1.20 if A+(BXC)=0 then A,B and C are coplanar, ie, are in the same plane, and conversely if A, B,C are coplanar then A+(BX C) =0. 1.22, Find the volume of a parallelepiped with sides A = 8i-—j, B= j+2k, C=i+5j+4k. B= 0 By Problems 1.20 and 1.21, volume of parallelepiped = |A+(BxC)| = |] 1 211 15 4 = |e) = %-Bj+k, C = 4j—Bk, find (a) (AXB)XxC, (b) Ax (BxC). k ij ok ° =5k. Then (AXB)XC =| 1-1 5 | = 28149)+4K. 1 o 4s ik ia @ Bxe=|2 -3 1) = o1+6j+8k Then Axxo 1 ai—aj+k. 4-8 6 It follows that, in general, (AXB)XC # AX (BX C). DERIVATIVES AND INTEGRALS OF VECTORS ere de lat) Pt lee 1A, It r= (P+21-3e-%}+2sin5tk, find (0) $F, (0) [FF], (Se @ eal at t=0. (0) $= See sao + Se-te-ms + Le wins = e401 + 66-4) + 10 conden a pees (®) From (a), |dr/dt| = V@PF 6+ do) = Vid = 2785 at t=0. 00 SE = £(B) = eansans cory touts) = an 1-H —sinste oe Ope ee. 4B, dA.y where A and B are differentiable func- au * du 1.25, Prove that Xia) =A: hu tions of 1. CHAP. 1) VECTORS, VELOCITY AND ACCELERATION 17 Method 1. 2 (a+b) = jim At8a)-(B+ a8) — vB AaB + SAB + aAsaB a a = tim (043 4 M4. 4 Asap) = = am (Ad + Sie + Shae) = Method 2 Let A= AM+ Ag+ Age, B= BA+ B+ Byk. Then @ FAB) = GAB Aw + Awd a (at aS atte) + (Mt a rg Bees 1.26, It g(z,y.2) =e and A= S2tyi+yetj— zak, find Pad (GA) at the point (1, -2,—1). 4A = (otra) Qa + ys —aeky) = Baty + ays) — ae Zeomy = Leen t ayes ay%) = selyti + sty) — 2a aigtom = Zeoerms setyss—aatyae) = Coty + Gaby) ~ Boh ‘this becomes —12i — 12) + 2k. eesty= 121, Bvaluate [Addu if Aw) = Gut—2)k + Qu—B) + (w— dud. ‘The given integral equala So Gen + n= + = 4019 Shaadi + (Wt Bud + COU 20 | = (B-21+ 4-H) + A6—HK) — {1 —Di + L-3) + OBR) = 6+ 8k VELOCITY AND ACCELERATION 128, A particle moves along a curve whose parametric equations are = = 8e-*, y = 4 sin3t, 2 = 6 cosSt where ¢ is the time. (a) Find its velocity and acceleration at any time. (®) Find the magnitudes of the velocity and acceleration at ¢=0. (@) The position vector x of the particle is r= abt yj tick = SoM + Asingt) + 5 cosdtk ‘Then the velocity is v= de/dt = ~6e-M + 12 cos3tj — 16 sin3tk fand the acceleration is a = dv/dt = @e/d® = 120-Mi — 86 sinBtj — 45 const () At t=0, v = de/dt = 61412) and a = dr/dt? = 121— 45k, Then ‘magnitude of velocity at t= 0 is VCOOFT (RF = 6V5 ‘magnitude of acceleration at t= 0 is VUZFF (157 = sv Bi 18 VECTORS, VELOCITY AND ACCELERATION (omar. 1 1.29. A particle travels so that its acceleration is given by a = 24 + Beostj ~ 3sintk If the particle is located at (1,—8,2) at time t=0 and is moving with a velocity given by 4i—3j+2k, find (a) the velocity and (6) the displacement of the particle at any time t>0, a= Bn = wets bets sae Intertne, w= fens seut)—ssincioae se v= aise v= “tet Sites + temte ta (6—2e-i + (6 sint — 3)j + (3 cost — 1)k ® () Relcngv by dfn 1) and Integrating, we hve b= Sf U6-2 + ame O) + Bonet aml at (is teh ~ ent An) + Game oH Sic he pct std ot (82) at 20, we hae t= Lape ae £=0, wo Ba Logit Siete or ee tas ae ‘Thus F = (6t+2e-t—1)i + (2— 5 cost—3t)j + (8 sint—t+ 2k @ RELATIVE VELOCITY AND ACCELERATION 1.30, An airplane moves in a northwesterly direction at 125 mi/hr relative to the N ground, due to the fact that there is a = westerly wind [ie. from the west] of Ss 50 mi/hr relative to the ground. Deter- mine (a) graphically and (b) analyti- el cally how fast and in what direction ® Vo the plane would have traveled if there > were no wind. > (0) Graphical Lat W = wind velocity w Va = velocity of plane with wind ‘Y= velocity of plane : without wind. ‘Then [see Fig. 1-27] Va = Vo+W or Vy Vp has magnitude 6.5 units (©) Anatyticalty. Letting 1 and j be unit vectors in directions E and N respectively, we see from Fig, 1-27 that Ve W = Vet (-w. 168 mifhr and direction 88° north of west. Ve = 125 cos45° 1 + 125.ind5°j and W = 501 ‘Then Vp = Va—W = (~125 cos45° — 60)i + 125 sin 45° j = ~198.501 + 88.99). Thus the magnitude of V, is V(—19830"+ (GB50F = 164.2 mi/hr and the direction is tan—1 88.80/188.89 = tan-1.6387 = 32°84! north of west. CHAP. 1] VECTORS, VELOCITY AND ACCELERATION 19 131. Two particles have position vectors given by m1 = 2ti—tj+(8f*—4t)k and m2 = (5—12t+4)i+¢j—8¢k. Find (a) the relative velocity and (6) the relative acceleration of the second particle with respect to the first at the instant where t=2. (a) The velocities of the particles at =2 are respectively vy =H = 244+ (6a) = at ok ve = fy = origi + aej— ak] = at + 12) — ok velocity of particle 2 with respect to particle 1 = vem = (G+12)— Bk) — (i 4j+8k) = 6 + 16) — 11k Relati (®) The-accelerations of the particles at t=2 are respectively sh =H = toe 21+ veto i = tte], = asa Relative acceleration of particle 2 with respect to particle 1 (101+ 12)) — (~2)+ 6k) = 108 + 14) - 6 TANGENTIAL AND NORMAL ACCELERATION 1.82, Given a space curve C with position vector r = Beos2ti + Ssin2tj + (8t—4)k (a) Find a unit tangent vector T to the curve. (®) If r is the position vector of a particle moving on C at time ¢, verify in this case that v=0T. (@) A tangent vector to C is de/dt = —Gsin2ti + 6 cost) + Sk ‘The magnitude of this vector is lde/ae| = delat = VHT BP + C cost + GF = 10 ‘Then a unit tangent veetor to C is = Seldt, _ delat _ de _ —Osin2ti + 6cos2tj + Oe ‘de/dt ~ delat ~ da ~ i0 = —Bsin2ti + Posts + gk (©) This follows at once trom (a) since v = de/dt = ~6sin2t1 + Gcon2tj + Bk (Oy sin 21 + Beoozts + fy = oF Note that in this case the speed of the particle along the curve is constant, 188. If Tis a unit tangent vector to a space curve C, show that dT/ds is normal to T. Since T is @ unit vector, we have T+T = 1. Then differentiating with respect to ¢, wo obtain Tie so rE Ben = oe E = 0 or which states that dT/de is normal, ie. perpendicular, to 20 VECTORS, VELOCITY AND ACCELERATION (cHaP.1 IE N is a unit vector in the direction of d¥/de, we have a@t/de = N and we call N the unit principal normal to C. ‘The scalar « = |@T/da| is ealled the curvature, while R= Ve is called the radiue of curvature. 1.34, Find the (a) curvature, (b) radius of curvature and (¢) unit principal normal N to any point of the space curve of Problem 1.82. (@) From Problem 1.82, T = —2sin2¢i+ fcos2tj+ fk. Then at _ awe 15) com Bt i — (6/5) sin 2¢ | de aalae = 1 —Zeos2ti ~ Reinzes = Ve Feo + (Hsin aye = & (6) Radius of curvature = B = i/e = 25/8 (©) From (a), (6) and Problem 1.38, = 1a nde Thos he creator in « = [| ar RE = —cos2ti — sin2ti 135. Show that the acceleration a of a particle which travels along a space curve with velocity v is given by Par a= Sr+n a’ +R where T is the unit tangent vector to the space curve, N is its unit principal normal and R is the radius of curvature. Velocity v = magnitude of v multiplied by unit tangent vector 7, or vow Dierntating, a= f= dum = Bree ut BeBe. ene 8 Then a= rs o(M) = Ss gy ‘This shows that the component of the acceleration is du/dt in a direction tangent to the path and ‘vi/R in the direction of the principal normal to the path. The latter acceleration is often called the centripetal acceleration or briefly normal acceleration. CIRCULAR MOTION 136. A particle moves so that its position vector is given by r = coset i+ sinet j where » is a constant. Show that (a) the velocity v of the particle is perpendicular to x, (®) the acceleration a is directed toward the origin and has magnitude proportional to the distance from the origin, (c) rx v = a constant vector. de @ v = $= -wsinetitecosut}. Then Fly = [eosut + sin at j]+[-w sin at i+ w coset j) (cos.at)(-w sin wt) + (sinut\(o coset) = 0 ‘and x and v are perpendicular. CHAP. 1] VECTORS, VELOCITY AND ACCELERATION 21 Pr _ dv _ 7 ‘ 0) BE =H = ~reosut tot sinat | = et [eonot 1+ sin ot j] = —ote ‘Then the acceleration is opposite to the direction of r, ie. it is directed toward the origin. Its magnitude is proportional to |r| which is the distance from the origin. (© eXv = [coset + ainot j] X [-w sinat 1+ 0 cos ot j] i ie comet sinut 0 —wsinat weosut 0 Physically, the motion is that of a particle moving on the circumference of a circle with constant angular speed «. The acceleration, directed toward the center of the circle, is the centripetal acceleration. = loos? at + sin? wt constant vector. GRADIENT, DIVERGENCE AND CURL 137, If g = ye! and A= aci—y'j+2z*yk, find (@) V4, (0) V-A, (0) VXA, (@) div (¢A), (¢) curl (A). w v6 = ( eg ae ahitde)e = rs isthe Reeve + Zeer + Zeer = tev + tah + sete w via = (Sit Sis du)- tet vi + 28m a 2 a = Ren + RM + ey = ey + birdy) xet-vis 28 i k se i te ee yh Raty re = 2 (aa) — = (Saen— dew) + (Le0-£ = Qe + (e—4ey)j eon) + (ew - Fle) (@ div(gA) = V+(~A) = V+ (atystt — atysedj + 2rtytatk) = Reeves + Leann + Lasts) oye — ae + be (@) curl (Ga) = VX (A) = x (obysth — atyAetj + BetyAahR) to k az aly alae sty —atytet aly aye! + Batya + (rye — Satya) — Bays + 2 188. (a) If A = Qzy+ sit (22+ 2y)i + (Bee*—2)k, show that VXA = (b) Find a scalar function ¢ such that A= V9. ' i x tee aley ae Bey tat att Dy Bast @ 9xa =0 22, ‘VECTORS, VELOCITY AND ACCELERATION [omap.1 (b) Method 1. = eet ee eee (©) Method 1. It A= vp = 38143854 38K th 2 hay oS 2 @) f= tts) Ba attty HE = aoa Integrating, we find Wo = yt ets ins) (9 = sty tytt Fylea) (©) 6 = eh -2t Pyle) Comparing these we must have Fy(y,2) = 2-2, Fyle,s) = 228-22, Fle) = atytyt and a0 g = ty bast + ye Oe, Method 2. We have if A=, ode = (2814 %54%y). cde = ($8i4 S54 38k) -eets ast arn) = Beet Hays Ha = wy tn enact dierent. For tha cae, dy = Avde = Chey te de + (4 2p dy + (tet) de (tay ade + a8 dy + Bethe] + By dy ~ Be d(xty + xz!) + diy) + d(—22) dlxty + 228 + y®— 22) Then 9 = yeast atte, Note that an artery constant can alo be aed to & LINE INTEGRALS AND INDEPENDENCE OF THE PATH 189. If A = (B2—Gyz)i + (2y+8z2)j + (1—dzye%)k, evaluate [ Asds from (0,0,0) to (1,1,1) along the following paths C: @e (b) the straight lines from (0,0,0) to (0,0,1), then to (0,1,1), and then to (1,1,1). (0) the straight line joining (0,0,0) and (1, 1,1). Sarde =f (ae synn + cyt Beni + aeveri (aed + dy) + del) = fet ounde + yt sendy + O~ aeveyde (0) Ite=ty=0, = #8, points (0,0,0) and (1,1,1) correspond to t= 0 and t=1 respectively, Then Sra =f" er-ceyena + ae ssa + a—aoenen ae Sf (ooo ae + Wertorae + (e—tennd = 2 Another method. ‘Along C, A= (Q0— 6091+ (QU-+900) + (1— 40% and x = akt yi tak = 48+ 0%, ar 2H Bey de, Then Sra = [oma + uatona + oouma = 8 () Along the straight line from (0,0, 0) to (0,0,1), 2=0, y=0, de =0, dy=0 while « varies from Otol, Then the integral over this part of the path is L., 8OF-sOVA0 + O)+80K—)0 + A-dOVOMGE = J" ee = 1 CHAP. 1) VECTORS, VELOCITY AND ACCELERATION 23 Along the straight line from (0,0,1) to (0,1,1), #=0, from 0 to 1. ‘Then the integral over this part of the path is F., Gor-cwnenyo + v+sOO) ay + A-souarI = fade = 2 , dz =0, d2=0 while y varies Along the straight line from (0,1,1) to (1,1,1), v=1, from 0 to 1. Then the integral over this part of the path’ f* aeroonnnae + eons satyo + a—aavanne =f" ae-aee = 5 , dy=0, de=0 while « varies (©) Along the straight line joining (0,0,0) and (1,1,1) we have 2=¢, de = dy = ds = dt, Sine , #=t, Then since So smae + arssenay + tte JS ae-ayae + aevamae + dua = fiery = on Note that in this ease the value of the integral depends on the particular path. 140. If A = ey +2%)i + (2*+2u)i + (Sx2?—2)k show that oS, A-dr is independent of the path C joining the points (1,~1,1) and (2,1,2) and (6) find its value, By Problem 188, VXA=0 or Arde = dp = d(sty-+as'+y?—22), Then the integral is independent of the path and its value is, SP nde =f dentate to : = tytadty—as(7P? = MISCELLANEOUS PROBLEMS 141. Prove that if a and b are non-collinear, then za+yb = 0 implies « ‘Suppose #0. ‘Then za+yb =0 implies za=—yb ora to the same line (collinear), contrary to hypothesis, Thus 142, Prove that the diagonals of a parallelogram bi- sect each other. Let ABCD be the given parallelogram with diagonals intersecting at P as shown in Fig. 1-28. Since BD+a a, Then BP = x(b—a). (a+b). But AB=AP+PB=AP-BP, a= vatd) — 2b—a) = et ya+ Wad. ‘Since « and b are non-collinear we have by Problem 14, 2ty=1 and y—2=0, Le e=y=4 and P is the midpoint of both diagonals. 24 143, 146, VECTORS, VELOCITY AND ACCELERATION (cHaP.1 Prove that for any vector A, (@) A= (A-Di+ (A+Dj + (A-kKk () A = A(cosai + cos j + cosy k) where a, B,y are the angles which A makes with i,j,k respectively and cosa, cos, cos y are called the direction cosines of A. (@) Wehave A= Ait Agi+Agk. Then Atl = (Ag+ Ags+Agri = Ay Ash = Abt AS + Add Ask = (Ad +A, +Ayk"k = Ay ‘Thus A (ADE (ACD + (ACW © = Acosa cose = Acosy ‘Then from part ( a (ASDF (ASHI + (ASW = Alcos.ai + cos Bj + cosy) Prove that V¢ is a vector perpendicular to the surface 4(2,y,2) constant. Let t= ait vit ak be the position vector to any point P(e,y,2) on the surface, ‘Then de = dei + dyj+dek lies in the plane tangent to the surface at P. But , where c is a to = Maas Maye ae = 0 or (Sn 488s 4 Mu) otaet eas alm) te Verde 0 0 that V4 is perpendicular to dr and therefore to the surface. Find a unit normal to the surface 2x*+4yz—52? = —10 at the point P(8, By Problem 1.44, a veetor normal to the surface is eee + dye 68) = dal + deh + ly—0k = 12 +} — 2k at (1,2) 14 eee a+ Bo Vrrerea T baa ak hae ‘Then a unit normal to the surface at P is Another unit normal to the surface at P is A ladder AB of length a rests against a vertical wall OA (Fig. 1-29]. The foot B of the ladder is pulled away with constant speed v. (a) Show that the midpoint of the ladder describes the are of a circle of radius a/2 with center at O. (b) Find the velocity and speed of the midpoint of the ladder at the instant where B is distant o> that (nde a sing i+ cosej © (6) ‘These results follow by solving the simultaneous equations (8) and (6) for 4 and 5. 26 VECTORS, VELOCITY AND ACCELERATION (CHAP. 148. Prove that (a) ix= 60: (0) 6:= (@) From (8) of Problem 1.47 we have nya 2 do or dt * 90 dt (OG) + (ain + cone 5nd) = ae doy _ dr, Odo > at rata = (OF) + (coset —sine 8) = Ory 149. Prove that in polar coordinates (a) the velocity is given by v= intr hye, + (7B + 27804 (@) We have r=rry 0 that or v= at by Problem 1.48(a). ee Ht rG = ith = it rig () From part (a) and Problem 1.48 we have ae aie a= B= Font Fat Ay + Fry + HG (Fa rine, + B+ BF, Supplementary Problems VECTOR ALGEBRA 150, Given any two vectors A and B, illustrate geometrically the equality 4A+S(B—A) = A+3B. 151. Given vectors A, B and C, construct the vectors (@) 2A—9B+4C, (8) C—4A+4B. 152 If A and B are any two non-zero vectors which do not have the same direction, prove that pA + ¢B is a vector lying in the plane determined by A and B. 1.58, (a) Determine the vector having initial point (2,—1,8) and terminal point (8,2,—4). (6) Find the distance between the two points in (a). Ans. (a) 1+3)—Tk, (6) V59 154. A triangle has vertices at the points A(2,1,—1), B(—1,3,2), C(1,—2,1). Find the length of the median to the side AB. Ane. 4V6 155, A man travels 25 miles northeast, 15 miles due east and 10 miles due south. By using an ‘appropriate scale determine (a) graphically and (0) analytically how far and in what direction he is from his starting position. Ans. 88.6 miles, 132° north of east. CHAP. 1) VECTORS, VELOCITY AND ACCELERATION 27 1.56, Find a unit vector in the direction of the resultant of vectors A = 2i—j+k, B C= B-2+ 4k. Ane, + (61-254 TH/VED ie i+ 2k, ‘THE DOT OR SCALAR PRODUCT 157. Evaluate (A+B)-(A~B)| if A= 2-3)+5k and B= Sit+j—% Ane. 24 158. Find a so that 21—3)+5k and Sitaj—2k are perpendicular. Ans. a= ~4/3 158, If A= 2i+j+k, B= i-2)+2k and C = Si-4j+2k, find the projection of A+C in the direction of B. | Ama, 17/8 1.60, A triangle has vertices at A(2,8,1), B(-1,1,2), C(1,-2,9). Find the acute angle which the median to side AC makes with side BC. | Ans. cos! VOT/14 161, Prove the law of covines for triangle ABC, ic. 6? = at+02—Zab cos C. (Hint, ‘Take the sides as A,B,C where C = A-B. Then use C+C (a-B)] 1.62. Prove that the diagonals of a rhombus are perpendicular to each other. ‘THE CROSS OR VECTOR PRODUCT 162, If A= 21—j+k and B= i+2)—9k, find [(2A4B) x (A-2B)), Ana, 253 164, Find a unit vector perpendicular to the plane of the vectors A= 31—2)+4k and B=i+j—2k Ane, = (25+ KE 1.65. Find the area of the triangle with vertices (2,~8,1), (11,2), (-1,2,8). Ane. $V 1.66, Find the shortest distance from the point (8,2,1) to the plane determined by (1,1,0), (8,—1,1), ae ee Let, Prove the lw of snes for triangle ABC, te, 2A = HRB. she, [dHint. Consider the sides to be A,B,C where A+B+C = sides with A and B respectively.) and take the eross product of both ‘TRIPLE PRODUCTS 168, If A= M+j—Sk, B=1—3+k and C= —i+J)—dk, find (o) A+ (BXO), (6) C+(AXB) (©) Ax BX), () (AXB)XC. Ame. (a) 20, (6) 20, (6) 8L—19)—k, (d) 251—15)— 10k 169. Prove that A+(BXC) = (AXB)+C, Le. the dot and the cross can be interchanged. 170, Find the volume of a parallelepiped whose edges are given by A = 2i+8)—k, B C= Bij. Ane. 81 242k, LI, Find the volume of the tetrahedron with vertices st (21,1), (ly Ane. 4/8 4,2), (0,1,—1), (,—22). 12, Prove that (a) A+(BX€) = B+(CXA) = C+(AXB) () AX(BXO) = BIA+©) - CA. 118, (a) Lat ry ryny be porition vectors to three points P,, Py, respectively. Prove that the equation (em) = en) x (e—n)] = 0, where r= at +y}+ a, represents an oquation for the plane determined by Py, Py and Py. (8) Find an equation for the plane passing through (2,—1,—2), 12,-8), 1,0). | Ane) 2ety Be = 9 DERIVATIVES AND INTEGRALS OF VECTORS LT, Let A = Bti— (+0) + (20k, Find (0) dA/dt and (0) PA/dE at Ans. (a) 81—8)—k, (b) ~2)+2k 11S. If r= acosut+bsinut, where a and b are any constant non-collinear vectors and w is a constant sealer, prove that (a) xX dr/dt = u(ax), (8) dr/det+ utr 28 VECTORS, VELOCITY AND ACCELERATION (CHAP. 1 176, If A= fi—sintk and B ee rove that Ziaxp) = ax 44a Lit, Prove that Z(axp) = ax 92+9%xB where A and B are differentiable functions of u. Lm Tf A) = deu— DI ut 94 + 6, evaluate (@) J" ACen, @) f” uk—24)-Ale de Ans. (a) 61-8) + 38k, (b) —28 a : 179, Find the vector B(u) auch that @?B/du? = 6ui—48ut}+12k where B= 2%i—Sk and dB/du He Rear Rabie se error oh auth tan rove tat fA ERG = Ax dR whee 6 a conan vs ae m= etapa ave ted PEt he pk 8-0, Am 1a It A= e—yitek and B= vital—ayek, find 5S Ane. 4148) (AX) at the point (1,—1,2). VELOCITY AND ACCELERATION 183, A particle moves along the space curve r = (+0i+(8—2)+(@—40%k. Find the (@) velocity, (b) acceleration, (¢) speed or magnitude of velocity and (d) magnitude of accelera- tion at time ¢ ‘An. (a) 5i+3}+8k, (6) 21+ 16k, (0) TV2, (d) 2V65 LB, A particle moves slong the space curve defined by x = e~t cost, y = ¢ ‘the magnitude of the (a) velocity and (4) acceleration at any time t. Ana, (a) VEe~', (b) VB e-t sint, #=e-% Find 1.85, The position vector of a particle is given at any time ¢ by r= acosuti+ bsinutj + ot. (@) Show that although the speed of the particle increases with time the magnitude of the ‘acceleration is always constant, (6) Describe the motion of the particle geomettically. RELATIVE VELOCITY AND ACCELERATION 196, ‘The position vectors of two particles are given respectively by ry = HO} +(2t+ 9k and my = (2t—B¢)1+4t]— ok, Find (a) the relative velocity and (b) the relative acceleration of the Second particle with respect to the first at t=1. Ams. (a) —81+6)—Bk, (b) —1+2)—6k 187, An automobile driver traveling northeast at 26 mi/hr notices that the wind appears to be coming ‘from the northwest. When he drives southeast at 90 mi/hr the wind appears to be coming from 60° south of west, Find the velocity of the wind relative to the ground. ‘Ans. 52 mi/hr in a direction from 80° south of west 188, A wn in a boat on one side of a river wishes to reach a point directly opposite him on the other ‘of the river. Aswuming that the width of the river is D and that the speeds of the boat and current are V and vb. (a) Show that the particle moves in an ellipse. (b) Show that the force acting on the particle is always directed toward the origin. vector atu and s0 © =acosut, y=) sinut which are the parametric equations of an ellipse having semi-major and semi-minor axes of lengths a and b respectively (see Fig. 2-8). Since Gla} + (y/b* = costut + sintat = 1 the ellipse is also given by 23/a?+ yt/b2 = (a) The posi acosuti + bsinat i Fig.23 CHAP.2] NEWTON'S LAWS OF MOTION. WORK, ENERGY AND MOMENTUM 39 (b) Assuming the particle has constant mass m, the force acting on it is v= mt = mith = mile canatl + (otro = mfnote cog tt 2 sin = —mut[acosut i+ 8 sin ut j}) = —moe which shows that the force is always directed toward the origin. 23. ‘Two observers 0 and 0’, fixed relative to two coordinate systems Oxyz and O'x'y’2’ respectively, observe the motion of a par- ticle P in space [see Fig. 2-4]. Show that to both observers the particle appears to have the same force acting on it if and only if the coordinate systems are moving at constant velocity relative to each other. Lt the position vectors ofthe particle inthe Ozyz and O'2'y'2’ coordinate aystems be r and r” respectively and let the position vector of O° ‘with respect to O be R= rr Fig. 24 Relative to observers © and 0’ the forces acting on P according to Newton's laws are given respectively by ae ™ ae ‘The difference in observed forces is and this will be zero if and only if = constant i.e. the coordinate systems are moving at constant velocity relative to each other, Such coordinate ystems are called inertial coordinate systems. ‘The result is sometimes called the classical principle of relativity. 24, A particle of mass 2 moves in a force field depending on time ¢ given by F = 24ti + (6t-16)j ~ 12tk Assuming that at t=0 the particle is located at re = Si—j+4k and has velocity Vo = 61+15j—8k, find (a) the velocity and (b) the position at any time t. (@) By Newton's second law, Bdvidt = 240% + (96¢—16)) — 120k dvidt = 121 + G88 — 6 Integrating with respect to ¢ and calling ¢; the constant of integration, we have v= ai + GH —8N) — Se + 6 Since v = vq = 61-+15)—Bk at ¢=0, we have ej = 61+16j—8k and 20 v = GP+6 + @A—Bt+15)) — G+ 8k 40 NEWTON'S LAWS OF MOTION. WORK, ENERGY AND MOMENTUM —[CHAP. 2 (@) Since v iefdt, we have by part (a) ae a Integrating with respect to ¢ and calling ey the constant of integration, r= (460i + GH 42 +150) — (480K + = UP+OL + (OP —8+ 1B) — E+E Since BI J+ dk at £=0, we have e = Si—j-+4k and 20 r= (HGF OL + BH 4e4 5-H) + 4-880 25, A constant force F acting on a particle of mass m changes the velocity from v; to ve in time r. (@) Prove that F = m(vo—vi)/r. (®) Does the result in (a) hold if the force is variable? Explain. (@) By Newton's second law, w_F o HF o ‘Then if F and m are constants we have on integrating, v = @/me +e At t=0, v=; #0 that ¢ v= (met y « At t=, vev, so that v2 = Om ty ie. F = mvy—vi/r “ Another method. Write (1) as mdv=Fadt. Then since Somer = fora oe minnw =F which yields the required result, (8) No, the result does not hold in general if F is not a constant, since in such case we would not ‘obtain the result of integration achieved in (a). 26. Find the constant force in the (a) cgs system and (b) mks system needed to accelerate a mass of 10,000 gm moving along a straight line from a speed of 54 kan/hr to 108 km/hr in 5 minutes. Assume the motion to be in the direction of the positive 2 axis, Then if v; and vg are the velocities, we have from the given data v, = Géikm/hr, vz = 108i km/hr, m = 10,000 gm, t= 6 min, (2) In the egs system m = 10'gm, vy, = bAi km/hr = 5X 10° em/sec, vg = 8.0% 10% em/see, t = 800 see 5) = anew (SE) (05 x 1051 gm em/sec? = 5X 104 dynes ‘Thus the magnitude of the force is 60,000 dynes in the direction of the positive axis. ‘Then F = me (®) In the mks system m = 10kg, vy = Séb km/hr = Si m/sec, vz = SOi m/sec, ¢ = 800 sec CHAP.2] NEWTON'S LAWS OF MOTION. WORK, ENERGY AND MOMENTUM aL mee we (2G) = conn (EH) 0.51 kg m/sec? = 0.51 newtons ‘Thus the magnitude is 0.5 newtons in the positive = direction. ‘This result could also have been Obtained from part (a) on noting that I newton = 10% dynes or 1 dyne = 10~* newtons. In this simple problem the unit vector 1 is sometimes omitted, it being understood that th force F will have the direction of the positive 2 axis. However, it is good oractice to work thi tnd similar problems with the unit vector present so as to emphasize the vector character of force, Nelocty, ete. This is especially important in cases where velocities may change their directions, See, for example, Problem 2.48, page 56. 2.7. What constant force is needed to bring a 2000 Ib mass moving at a speed of 60 mi/hr to rest in 4 seconds? We shall assume that the motion takes place in a straight line which direction of the axis. Then using the English absolute system of ur mm = 20001, vy, = 604 Then v= m= n(27%) = ir = 881 ft/sec, = 44x 1044 ft Ib/ace? = 4.4 1044 poundals ‘Thus the force has magnitude 4.410! poundals in the negative x direction, i.e. in a direction ‘opposite to the motion. This is of course to be expected. WORK, POWER, AND KINETIC ENERGY 28. A particle of constant mass m moves in space under the influence of a force field F. ‘Assuming that at times t; and f2 the velocity is vi and vs respectively, prove that the work done is the change in kinetic energy, i.e. "pede = dmuf — dmv} ‘Work done , = fined = nfl ea i a ee Sorte = forse }, ™ae in fSaen = dwell = deed = dot 2.9. Find the work done in moving an object along a veetor r= 8i+2j—5k if the applied force is F=2i—j—k. Refer to Fig. 25. ‘Work done = {magnitude of force in direction ‘of motion) (distance moved) F cos (7) = @l-j—W)+ i+ 2}— 5K) = 6-245 =9 42, NEWTON'S LAWS OF MOTION. WORK, ENERGY AND MOMENTUM —[CHAP.2 210. Referring to Problem 2.2, (a) find the kinetic energy of the particle at points A and B, (b) find the work done by the force field in moving the particle from A to B, (o) illustrate the result of Problem 2.8 in this case and (d) show that the total work done by the field in moving the particle once around the ellipse is zero. (0) Velaiy = y= det = ~ anasto eat Rieti energy = fmt = fet unset + cat, inate eeey ot A [vier cout = 2, anat =0) = fut fietaee as reese y = jo () Motto 1. From pet (of Problem 22, Workame =f rede =f Cmanede = mt free = nine fo aren = ime? = Gmetad — mst? = fmar{a2— 04) Method 2 We can assume that at A and B, and t 120 respectively. Then: Work done = 7 Fede (ee S TO publ B) aint cost dt yrnot(o2 — 09) int at | rato? — ba) (©) From parts (a) and (6), ‘Work done Hmst{at— 0) = meta? — Jmat? kinetic energy at A — kinetic energy at B (@) Using Method 2 of part (b) we have, since t goes from 0 to t=2x/w for a complete cireuit ‘around the ellipse, ow Work dene =f meat 00 sine cnet dt 1" t t i $ a Method 1 can also be used to show the same result. ‘211. Prove that if F is the force acting on a particle and v is the (instantaneous) velocity of the particle, then the (instantaneous) power applied to the particle is given by P= Fy By definition the work done by a force F in giving a particle a displacement dr is W = Pedr ‘Then the (instantaneous) power is given by aw Pee as required. CHAP. 2} NEWTON'S LAWS OF MOTION. WORK, ENERGY AND MOMENTUM 43 212, Find the (instantaneous) power applied to the particle in Problem 2.1 by the force field. By Problem 2.1, the velocity and force are given respectively by v= (6+ Ii + (128209) — 4th F = 604i + (180!2—10)} — 120k ‘Then the power (by Problem 2.11) is given by = Fey = (Go0(6e-+1) + (180e*— 10)(120— 20) + (120)(240) = 216008 — 12008 + 29608 218. Find the work done by the force in (a) Problem 2.6, (b) Problem 2.7. (@) Im the egs system: vj = |v] = 1.5% 108 cm/sec, v = |v] = 8.0% 108 cm/sec, m = 101 gm. ‘Then by Problem 28, Work done = change in kinetic energy = dm} = 4 emy9.0 x 108 — 2.26 x 109 SB = 3.38 x 101 SE emt ‘nom = 838 x 100mm 338x108 (E22 (om) = 888% 10 dyneem = 8.58% 100 erge Tn the mks system we have similarly: Work done = 4(10 ke(900 — 225) 2 = ase x10e(4EE) (my = 598% 10° newton meters (©) As in part te Work done = 42000 1(¢8" on £8, = mimeo BB) = roca CONSERVATIVE FORCE FIELDS, POTENTIAL ENERGY, AND CONSERVATION OF ENERGY 2.14, Show that the force field F defined by F = (ye —Gze4i + Qrys'j + (Bay's —62%2)k is a conservative force field. Method 1. ‘The foree field F is conservative if and only if curl F = VXF = 0. Now i i oy alae alty alee vet —Gzst Days? Saye ~ Bete vxXF = 1 [Zane e - Zam] + s[Zure—onn ~ Lanne oro] | + «[ Ze - Loreen] ‘Then the force fleld is conservative. 44 NEWTON'S LAWS OF MOTION. WORK, ENERGY AND MOMENTUM — [CHAP.2 Method 2, ‘The force field F is conservative if and only if there exists a scalar function or potential Vie,y,2) such that F = —gradV = —VV Then = e604 tl + tao M17 emt wm so V ch at Vida = Gxa?~ ytet, OV/ay = —2eys8, aV/az = Griz — Sayts® @ F = -wW Integrate the first equation with respect to 2 keeping y and constant. Then V = Setst — yet + gulnn2) © ‘where 9:(¥,2) is « function of y and =. Similarly integrating the second equation with respect to y (keeping » and z constant) and the third equation with respect to z (keeping = and y constant), we have V = ~2¥ts + osle,2) O V = tated — xyted + ose.) w Equations (2), (8) and (4) yield a common V if we choose ane) = sary constant, and it follows that V = satet—nyted +e + grles2) = Batet te, ox(esy) = @ © where ¢ is any art fs the required potential nents - Mane) Vim fleece = =f ede + anten + Gayest = WS" aay - sete) = atta +o where ¢ = syvded— Se3ef, 215. Prove Theorem 2.2, page 85: If the force acting on a particle is given by F = —VV, then the total work done in moving the particle along a curve C from P; to P; is w= fred = vi) - Ve ‘We have fees = Sa = -v| = vey - veg 2.16. Find the work done by the force field F of Problem 2.14 in moving a particle from the point A(-2, 1,8) to B(1,—2,—1). Work done =f" Fede ova Se eee pn CHAP. 2} NEWTON'S LAWS OF MOTION. WORK, ENERGY AND MOMENTUM 45 2.17. (a) Show that the force field of Problem 2.2 is conservative. (0) Find the potential energy at points A and B of Fig. 2-8. (c) Find the work done by the force in moving the particle from A to B and compare with Problem 2.10(6). (@) Find the total energy of the particle and show that it is constant, the principle of conservation of energy. . demonstrate (@) Prom Problem 2(0), F = mate = —mo%(2i+ yi). ‘Then i ik VxXF alas alty aloe Hence the field is conservative. (Since the fel i conservative there exits a potential Yauch that P= most mi) = wy = ~~ yy ‘Then aV/ax = mute, aV/ay = muty, aV/oz = 0 trom which, omitng the centant, we have V = dmatat + gmetyt = fmaXet ty) = jmutrt hich isthe regivd potent (e) Potential at point A of Fig. 28 [where r=a] = gmu%at. Potential at point B of Fig. 28 [where r=8] = mut, Then Work done from A to 8 = Potential at A — Potential at B Ymatat — Ymetbt = 4met(at— 9) agreeing with Problem 2.10(b). (@) By Problems 2.10(a) and part (6), Kinetic energy at any point = T = mvt = jmit Jn(ota? aint ot + 428? cost wt) fate Jrnat(a? cost ut + BF sin? wt) ‘Thus at any point we have on adding and using sin? at + cost et = 1, THV = fmter +o Potential energy at any point = V which is a constant, IMPULSE, TORQUE, ANGULAR MOMENTUM, AND CONSERVATION OF MOMENTUM 218 Prove Theorem 2.6, page 86: The impulse of a force is equal to the change in momentum. By definition of impulse (see (19), page 86] and Newtor second law, we have Sova = S$ komae = fame) = me 46 NEWTON'S LAWS OF MOTION. WORK, ENERGY AND MOMENTUM — (CHAP. 2 2.19. A mass of 5000 kg moves on a straight line from a speed of 540 km/hr to 720 km/hr in 2 minutes. What is the impulse developed in this time? ‘Method 1. ‘Assume that the mass travels the direction of the positive + aux, In the mke ayatem, ae 2 cin 100m my = soe = Se 15x 10% ‘7201 x 1000 m ve = 72018 = 20x 104 "2800 eee ‘Then from Problem 2.18, Impulse = m(va—v;) = (6000 ke)(0.5 x 108i m/sec) = 265X108 kg m/sec = 2.5 X 105% newton see since Inewton = 1kgm/see? or 1 newton sec = 1 kg m/sec. ‘Thus the impulse has magnitude 2.5 X 105 newton sec in the positive 2 direction. ‘Method 2. Using the ces system, vy = 540i km/hr = 1.6% 10¢iem/see and vy = 7201 km/hr = 2.0% 1044 em/see. Then Impulse " ‘amiva—v) = (6000 x 108 grn(0.5 % 1044 em/see) 2.50 X 10101 gm em/see = 2.50% 101i dyne sec since 1dyne = 1 gmem/sect or 1 dyne see = 1 gm cm/sec Note that in finding the impulse we did not have to use the time 2 minutes as given in the statement of the problem. 2.20. Prove Theorem 2.7, page 86: The moment of force or torque about the origin O of a coordinate system is equal to the time rate of change of angular momentum. ‘The moment of force or torque about the origin O is a A = exe = ext ‘The angular momentum or moment of momentum about O is @ = mrxv) = x (mv) ae ae wl - Lex = Exon + 2x Zeon Now we have $2. A ee wx (ne) +x Lom) = 0+ rxP = A which gives the required result, 2.21. Determine (a) the torque and (0) the angular momentum about the origin for the particle of Problem 2.4 at any time ¢. (@) Torque = rxXF [(t0-+ Gt + a)k + (e402 + 15¢— 19) + (4 — Bey] X [BEE + (Bor — 16 — 120K) i i k = |e#+orta se-aetuse—1 4-8 sr ae 36t 16 —a2e (3248 + 10862 — 260¢ + 6a) — (1245 + 19208 — 1684" 368)j — (86e8 — Boe! + S606 — 24008 — 12¢ + 48) CHAP. 2] NEWTON'S LAWS OF MOTION. WORK, ENERGY AND MOMENTUM aT ne moe ga 81H + 8 a0 xilaet ont eee i644 Om i ; . wre oe se+ 1s 8-8 = (Bt4+ 3648 — 18018 + 64 — 104)i — (2e + 4844 — B66 — 180% — 96)) — (Ge — 1605 + 9044 — Boe! — Bet + 48¢— 102) Note that the torque is the derivative with respect to ¢ of the angular momentum, illustrating the theorem of Problem 2.20. 222. A particle moves in a force field given by F= rr where r is the position vector of the particle. Prove that the angular momentum of the particle is conserved. ‘The torque acting on the particle is A= EX = rx (tr) = rirxe) = 0 ‘Then by Theorem 2.9, page 87, the angular momentum is constant, Le. the angular momentum is ‘conserved. NON-CONSERVATIVE FORCES 228, Show that the force field given by F = 2*yzi—zyz"k is non-conservative. We have toi ok ofas aay ales aye 0 at ‘Then since V x F s 0, the field is non-conservative. vxP = = ath + (ety + ah) - otek STATICS OF A PARTICLE 224. A particle P is acted upon by the forces F.,F,F1,F.,Fs and Fe shown in Fig. 2-6. Represent geometrically the force needed to prevent P from moving. Fe Fig.26 Fig.27 ‘The resultant R of the forces F.,F;,Fs,FiyFs and Fy can be found by vector addition as indicated in Fig. 27. We have R= F\+#,+F,+Fj+F,+¥e The force needed to prevent P from moving is -R which is a vector equal in magnitude to R but opposite in direction and sometimes called the equilibrant. 48 NEWTON'S LAWS OF MOTION. WORK, ENERGY AND MOMENTUM —[CHAP. 2 2.25. A particle is acted upon by the forces F, = 5i—10j+15k, F: = 10i+25j-20k and Fs = 15i—20j+10k. Find the force needed to keep the particle in equilibrium. ‘The resultant of the forces is B= Fy +R, + Fy = (Gi~10j + 10k) + (101+ 25j~ 20k) + (151-20) + 10k) 801 — 5) + 5k ‘Then the force needed to keep the particle in equilibrium ig — 801 + 5] — 5k. 2.28. The coplanar forces as indicated in Fig. 2-8 act on a particle P. Find the resultant of these forces (a) analytically and (b) graphically. What force is needed to keep the particle in equilibrium? Fig. 28 Fig.29 (©) Analytically, From Fig. 2-8 we have, Fy = 160(cos 45° 1+ sin 45° j), Fy = 100(—cos 30° i + sin 80° j), Fy = 120(~ cos 60° 1 ~ sin 60° j) ‘Then the resultant R is R= th +Fy = (160 cos 45° — 100 cos 80° ~ 120 60s 60°)t + (160 sin 45° + 100 sin 30° — 120 sin 60°) 88.461 + 59.21) Writing R= Reosai+ Raine} where a is the angle with the positive # axis measured counterclockwise, we see that Recess = 38.48, Rina = 59.21 ‘Thus the magnitude of Ris R= V(~3848FF GORI = 68.0 Ib, and the direction « with the positive x axis is given by tana = 59.21/(~83.46) = ~1.770 or « = 119° 28". (©) Graphically. Choosing » unit of 20 1b as shown in Fig. 20, we find that the retultant has ‘magnitude of about 68 1b and direction making an angle of about 61° with the negative axis (using « protractor] #0 that the angle with the positive 2 axis is about 119°, A force ~R, ic, opposite in direction to R but with equal magnitude, is needed to keep P {in equilibrium. STABILITY OF EQUILIBRIUM 227, A particle moves along the x axis in a force field having potential V=4cr*, «>0. (a) Determine the points of equilibrium and (b) investigate the stability. (@) Equilibrium points occur where VV Vidz = 2 = 0 or 2 =0 or in this case ‘Thus there is only one equilibrium point, at 2 =0. CHAP.2} NEWTON'S LAWS OF MOTION. WORK, ENERGY AND MOMENTUM 49 (8) Method 1. Since @V/ds? = « > 0, it follows that at page 38, 2=0 is a point of stability. ‘This is also seen from Problem 2.36 where that the particle oscillates about #=0. Visa minimum. ‘Thus by Theorem 2.10, ‘ie shown ve Method 2. Wehave F=—vv =—20i=—eel, Then when 2>0 the particle undergoes a force to the left, and when <0 the particle under- ‘goes a force to the right. Thus 2=0 Is a point of stability. Method 3. ‘The fact that 2 =0 is a minimum point ean be seen from a graph of Vie) viz (Pig. 2-10]. Fig.2-10 MISCELLANEOUS PROBLEMS 2.28. Show how Newton's laws can be used to develop definitions of force and mass Let us first consider some given particle P, assuming for the present that its mass mp is not defined but is simply some constant scalar quantity associated with P. Axiom 1 states that if P moves with constant velocity (which may be zero) then the force acting on it is zero. Axiom 2 states that if the velocity is not constant then there is a force acting on P given by mpap where fap is the acceleration of P. Thus force is defined by axioms 1 and 2 (although axiom 1 is unnecessary since it ean in fact be deduced from axiom 2 by letting F=0). Tt should be noted that foree is vector and thus has all the properties of vectors, in particular the parallelogram law for vector addition. To define the mass mp of particle P, let us now allow it to interact with some particular particle which we shall consider to be a standard particle and which we take to have unit mass. If ap and ag are the accelerations of particle P and the standard particle respectively, it follows from axioms 2 and 3 that mpap ‘Thus the mass mp can be defined as ~as/ay 2.29, Find the work done in moving a particle once around a circle C in the zy plane, if the circle has center at the origin and radius 3 and if the force field is given by Fo = Q@x-yt2)i + (e+y—2)j + (Bx—-2y +4) In the plane F = 22— yh t (e+ yi) + @e—2y)ke and de = deit dy} so that the work done is Sree = Seems +My + eB et + dy] Seema + etna Choose the parametric equations of the circle as = cost, t "et varies from 0 to 2» (se Fig. 211), Then the Tine invogeal equate (le $120 ont) — 5 ain ei-B ine de + [cont +8 sing con at NN : In traversing C we have chosen the counterclockwise direction indi cated in Fig. 2-11. We call this the positive direction, or say that Ct = a+ yi hhas been traversed in the positive sense. If C were traversed in the Beostit Bsint i clockwise (negative) direction the value of the integral would be —18r. Fig. 241 50 2.80. (a) (b) @ @ NEWTON'S LAWS OF MOTION. WORK, ENERGY AND MOMENTUM — [CHAP. 2 If F=—vyV, where V is single-valued and has continuous partial derivatives, show that the work done in moving a particle from one point P; = (21,4:,2:) in this field to another point P: = (2, y2,z2) is independent of the path joining the two points, Conversely, it f° F-dr is independent of the path C joining any two points, show that there exists a function V such that F=—yV. = -f" ova v5 + y+ Be) (acts ays + deny Way 4 ¥ : ety + ee fs ~ SP av = ve) vie) = Vienne) — Venrned ‘Then the integral depends only on points P, and P; and not on the path joining them, ‘This is true of course only if V(r,y,2) is single-valued at all points P and P. Let F = Fut Fy + Fok. By hypothesis, ff F+de is independent of the path C joining any ‘two points, which we take as (24, vs,2) and (#,y,2) respectively. ‘Then - fo rede =~ rae + Pay + Faas) sma evn is independent of the path joining (ey,4s,2)) and (#,y,2)- Thus Veena) Vanna) = —Sihitenée+ Reinet Poems where C is a path joining (x,¥1.#) and (2,042). Let us choose as a particular path the straight line segments from (2, v4.2) to (2,414) to (e,¥.21) to (2,42) and call V(e,y,2) the ‘work done along this particular path. ‘Then Si riemande ~ S"rewed ~ Sf" roennde Veeva) Ie follows that ©. penn wv _ (7%, a A eclee B= rene — Sf Peonae = rene - Sf Ztenne = -Flensy — tena’ Faved = Fala) + Fale. yad Paley) ” oF, Fie uy a) ~ * oF ese) ay -S Geewade oe = Fileyon) — Filewa) | — Few) Frew) + Fema) Fle vy (ua) + Flewe) = —Fyle.ms) CHAP.2] NEWTON'S LAWS OF MOTION. WORK, ENERGY AND MOMENTUM BL Then FS Pithit hk = wy My = -ww ‘Thus necessary and sufficient condition that a fleld F be conservative is that curl F = Vx F 231. (a) Show that F = (ey+2)i+24j + 8zz*k is a conservative force field. (6) Find the potential. (c) Find the work done in moving an object in this field from (1, ~2, 1) to (31,4). (a) A necessary and sufficient condition that a force will be conservative is that curl F = VX = 0. i ik afez fay aide Boyt at Bet Now VxXF = 0. Thus F is « conservative force field (®) As in Problem 2.14, Methods 2 or 8, we find V = ~(2%y +224). (9 Warkaane = ~1ty + 2382, Prove that iff" Fedr is independent of the path joining any two points P, and Ps ina given region, then F-dr = 0 for all closed paths in the region and conversely. Let P)AP,BP, [se0 Fig. 2-12] be a closed curve, ‘Then fo e 5 . = fra- fra =o since the integral fom Pt Py slong « path through A is) the same as that along @ path through B, by hypothesis. = Conversely it § Rede = 0, then Petit So fee (ee fee ie a 2.88. (a) Show that a necessary and sufficient condition that Fidz + Fady + Fsdz be an exact differential is that VxF = 0 where F = Fui+F:j+Fak. (©) Show that (y*2* cos 2 ~ daz) dz + 22'y sin z dy + (8y'#* sinz ~ 2')dz is an exact differential of a function ¢ and find ¢. (0) Soppone Ryde + Ryd + Ryde = dy = Bde + May +See, an exact diferent, Then since 2,9 and + are independent vaslbles, 52 NEWTON'S LAWS OF MOTION. WORK, ENERGY AND MOMENTUM (CHAP. 2 snd wo P= FAAP tM = Mie BBE = ve Thos VER = UXO Conversely if VXF = 0, then P= Vo and so Pedr = Verde = dg, te. Fide + Fray + Fydz = dg, an exact differential (0) F = (y's? cos ~ 42%) i+ Baty sine j + (By2st sine ~ 24) k and VF is computed to be zero, 40 that by part (a) the required result follows. 2.84, Referring to Problem 2.4 find (a) the kinetic energy of the particle at t=1 and t=2, (b) the work done by the field in moving the particle from the point where t=1 to the point where ¢=2, (c) the momentum of the particle at t=1 and t=2 and (4) the impulse in moving the particle from t=1 to t (@) From part (a) of Problem 24, v= (484 6) + (OH —8¢-4 15))~ (9+ Bk Land ¢=2 are My = 101+ 16) 11h, vy = 98+ 95) — 20K and the Kinetic energies at (=1 and ¢=: 7, = Jot = HOQ08+GoR+ 1] = 477, Ty = dowd = 060 (®) Work done = Sf, 2001+ ane 10}— ina -[49 +014 068419) ~ Bose = FF eure +0 + abr —aeR—ee + aren aie = sn Note that by part (a) this is tho same as the difference or change in kinetic energies 8069477 = 2692, ‘illustrating Theorem 2.1, page 95, that Work done = change in kinetic energy. (0) By part (a) the momentum at any time ¢ is p= my = Bv = (BH+ ID + (18H 164 + 90)j — (64 + 16) Land ¢ 201 + 82) — 22k, py = Toi + 70} 40K. ‘Then the momenta at Sire = ff 240+ ses —sangat = 1+ 28) (@ Impulse Note that by part (2) this is the same as the difference or change in momentum, i, a — By = (T6L+70j—40k) — (201+ 82)—22k) = 561 + 88j— 18k, illustrating Theorem 2.6, page 86, that Impulse = change in momentum. 285. A particle of mass m moves along the x axis under the influence of a conservative force field having potential V(z). If the particle is located at positions 2; and 2% at respective times t1 and ts, prove that if Z is the total energy, aon VES ee CHAP. 2] NEWTON'S LAWS OF MOTION. WORK, ENERGY AND MOMENTUM 53 By the conservation of energy, Kinetic energy + Potential energy = amidzidyt + Ve) 5 ‘Then (dala = (aime — Vee), ® from which we obtain on considering the positive square root, at = Vmi8 (dx/VE—Vea)) Hence by integration, ao ae ann= 3 Rr] @ (a) If the particle of Problem 2.85 has potential V = 4x2? and starts from rest at =a, prove that = acosyx/mt and (b) describe the motion. (a) From (1) of Problem 2.35, (dz/dt)* = (2/m)(E— ea). Since dz/dt=0 where 2=a, we find E = fea? ao that (delat? = (xtmyat—2t or dala aF = Velme dt Integration yields sin-1(e/a) = = ViJmt+ ey. Since 2: #2. Then at 8, a sin-1(2/a) = tVelmt + 2/2 or 2 = asin(e/2*Velmt) = aconValmt (0) ‘The particle oscillates back and forth slong the 2 axis from x=a to #=—a. The time for fone complete vibration or oscillation from 2a back to 2=a again is called the period of the oscillation and is given by P = 2x Vals. A particle of mass 3 units moves in the zy plane under the influence of a force field having potential V = 122(8y—4z). The particle starts at time t=0 from rest at the point with position vector 10i—10j. (a) Set up the differential equations and conditions describing the motion. (b) Solve the equations in (a). (c) Find the position at any time. (d) Find the velocity at any time. (a) Sinco V = 122(8y—42) = S6ey— 4822, the force field is Fe -w = -%_¥, = (-86y + 962)4 ‘Then by Newton's second law, 3GE = (-86y +9601 — 3625 Ga = (My +962)i — 8625 or in component form, using x = i+ vi, Pela = —12y + 822, Pylded = 122 0 where 2=10, 2=0, 10, 7=0 at t=0 @ the fact that the particle starts at r= 101~10) with velocity v= (6) Prom the second equation of (1), yields = jy @v/dt?, Substitution into the first equation of (1) diyldts ~ 32.dy/de ~ 4dy = 0 ® If « is constant then y= ett is a solution of (8) provided that A B2et 14 = 0, de, (e+ Ale —96) = 0 or 54 NEWTON'S LAWS OF MOTION. WORK, ENERGY AND MOMENTUM — [CHAP. 2 Thus solutions are. et, and the general solution Is Y= ecosdt + epsingt + eget + cere w Thus from © =~ @y/de we find, wing (4), = focondt + fepsin at — dott ~ Soyo Using the conditions (f) in (4) and (6), we obtain Hei Bey Bey = 10, Gep— 8p + 1804 ete te = 10, Bey + 665 ~ 66 = 0 eM, eM or cost, sin 2, et, [in terme of real functions) © Solving simultaneously, ¢,=—6, ¢,=0, e5=—2, eg=—2 so that © = 6 cos2t—2e%—2e-t, y = 2 con Bt + Gott + Gee (©) The position at any time is ¥ = ait yh = (6 conte — 208 — 20-891 + (-2 cos Bt + Belt + BoA (@) The velocity at any time is ati In terms of the hyperbolic functions sinh at = jee vei 12 ain 2¢ — 120M + 120A) + (A sin BE + 2664" — 380-M) 0, combat = (ort emety we can also write (6 cos 2t ~ 4 cosh 62) + (-2 con 2¢ + 12 cosh 60) (42 ain 2¢ — 24 inh 60) + (4 sin 2¢ + 72 sinh 604 2.38. Prove that in polar coordinates (r, 6), av. lav, Ve oe rae Let WV = Gr + He o where G and H are to be determined. Since dr = del+dyj we have on using z= cose, y=rsing and Problem 1.47(0), page 25, de = (cossdr —r sino)(con er, ~ sin oe,) + (sing dr + cone de)(sin or, + cos 04) or de = deny troy ® ee Now vvede = av = Bars May Using (1) and (2) this becomes (On, + Heyetire, trdee) = Ode + Hae = Bars ay so tat ¢ own (0 come wy = By the, 2.39. According to the theory of relativity, the mass m of a particle is given by mo me vi-we = Vvi-# where » is the speed, mo the rest mass, c the speed of light and f = v/c. CHAP.2] NEWTON'S LAWS OF MOTION. WORK, ENERGY AND MOMENTUM 65 (a) Show that the time rate of doing work is given by a au met $01 (b) Deduce from (a) that the kinetic energy is T= (m— malo? = moct{(1— 6-7 = 1) (©) If v is much less than ¢, show that T= 4mv* approximately. (a) By Newton's second law, rte ES) Fores GS) «ke as proved by direct differentiation, () Since Work done = change in kinetic energy, we have ‘Time rate of doing work = time rate of change in kinetic energy ox by part, = = wh (Zhe) yet Integrating, T= To determine o note that, by definition, T=0 when v=0 or 8=0, s0 that ¢; Hence we required, (©) For <1 we have by the bit 1 ie qe = oor re ee he be Then ]a met = foe aprocinatty a Supplementary Problems ‘NEWTON'S LAWS 240. A particle of mass 2 units moves along the space curve defined by x = (4t2— di ~ Bt] + (t#— 2k. Find (a) the momentum and (8) the force acting on it at t=1. Ans. (a) 10i—10}+8k, (B) 41+ Dake 241, A particle moving in a foree field F has ita momentum given at any time t by p= Ben — 2eosti ~ Ssintk Find F, Ans, —Se- + 2sintj— 3 cos tk 242, Under the influence of a foree field a particle of mass m moves along the ellipse F = acosutl + beinwt | If p is the momentum, prove that (a) rx p= mabsk, (8) r+p = Jm(6%— a) sin ut 243, 2M, 246, aan, 2a, 249, NEWTON'S LAWS OF MOTION. WORK, ENERGY AND MOMENTUM —[CHAP. 2 If F is the force acting on the particle of Problem 2.42, prove that rx FF means physically. Explain what this ‘A force of 100 dynes in the direction of the positive 2 axis acts on a particle of mass 2 gm for 10 minutes. What velocity does the particle acquire assuming that it starts from rest? Ane. 3x 104 cm/sec Work Problem 2.44 if the force is 20 newtons and the mass is 10 kg. Ama. 1200 m/sec (a) Find the constant force needed to accelerate a mass of 40 kg from the velocity 41 —5) +k ‘m/sec to 81+3j—Bk m/sec in 20 seconds. (6) What is the magnitude of the force in (a)? Ans. (a) 81+16)—16k newtons or (81+16j— 16k) x 105 dynes (®) 24 newtons or 24X10" dynes ‘An elevator moves from the top floor of tall building to the ground floor without stopping. (@) Explain why a blindfolded person in the clevator may believe that the elevator is not moving at all. (8) Can the person tall when the motion begins or stops? Explain. A particle of unit mass moves in a force field given in terms of time ¢ by F = (6t—8)i — 6003} + (200+ 260% Tits inital position and velocity are given respectively by ry = 2i—dk and vy = 61+4), Find the (a) position and (b) velocity of the particle at ¢=2. Ans. (a) 4188) +7Mk, (6) 1286) +176 Th fre acing on pte of mane i given In terme of ne F = ecoratt + inet} 1 the partic is inl tre a he oii, nd () poston and) velo a any Inter tne + ha eonuo Ans. (a) 2501 — coswt) i + Pa(ot — sint) j, (0) 58 WORK, POWER AND KINETIC ENERGY 250, 251, 252, 253, 254, 256, 257. ‘A particle is moved by a force F = 201~80j+15k along a straight line from point A to point 5 with position vectors 21+ 7)~8k and 5i—3)—6k respectively. Find the work done. Ane. 315 Find the kinetic energy of a particle of mass 20 moving with velocity 8i—6j+4k. Ana, 500 (9201+ 8)— Hk, ‘to the point Due to a foree field F, a particle of mass 4 moves along the space curve r Find the work done by the fleld in moving the particle from the point where t= where ¢=2. Ane. 2454 [At one particular instant of time a particle of mass 10 is traveling along a space curve with velocity kiven by 4i+ 16k. At a later instant of time its velocity is $i—20}, Find the work done on the particle between the two instants of time. Ans. 192 Verify Theorem 2.1, page 85 for the particle of Problem 2.52. ‘A particle of mase m moves under the influence of the force field given by F = a(sin ut i+ cost ). I the particle i initially at rest at the origin, prove that the work done on the particle up to time & is given by (a?/mat)(1 — cos ut). Prove that the instantaneous power applied to the particle in Problem 2.65 is (o3/mu) sin wt. ‘A particle moves with velocity 5i—8]-+6k under the influence of a constant force F = 201+ 0j'+ 16k. What is the instantaneous power applied to the particle?’ Ams. 160 CHAP. 2} NEWTON'S LAWS OF MOTION, WORK, ENERGY AND MOMENTUM 7 CONSERVATIVE FORCE FIELDS, POTENTIAL ENERGY AND CONSERVATION OF ENERGY 258, 259, 260. 261 262, 263, 264, 265, 266, (a) Prove that the force field F = (y?—2eysi)i + (8-4 22y ~%)j + (624 — Sxty24%k is conservative, (6) Find the potential V associated with the force field in (a). Ans. (0) zy? — atys) + By +94 A particle moves in the force field of Problem 2.68 from the point (2,~1,2) to (—1,8,—2). Find the work done. Ane, 55 (a) Find constants ,6,¢ 0 that the force field defined by F = (e+ 2y+aa)i-+ (bx —3y—2) + (de + cy +22) ss conservative (8) What is the potential associated with the force field in (a)? Ans. (a) a=4,6=2, c=-1 (b) V = Jetty? at oy deste Find the work done in moving a particle from the point (1,—1,2) to (2,8,—1) in a force field with potential V = 23—y8+2y—y?+4s. Ans. 15 Determine whether the force field F = (2ty—25)i + (Szya-+220)j + (2etys+ysk is conservative, Ans. Not conservative Find the work done in moving a particle in the force field F = Sati + (222—y)} + sk along (othe straight line from (0,0) to (21,8), (8) the space curve x = 22, y = bs 2 from ¢ isthe work indopendent of the path? Explains Ana, (a) 16, (6) 142 (obra § Fede whee P= (eS (9~ 20 and te ced cove ne ae 2 =2cost, y=Ssint from t=0 to t=2e. (b) Give a physical interpretation to the result in (a). ‘Ans. (a) 6s if C is traversed in the positive (counterclockwise) direction. (a) Show that the force field F = ~rr'r is conservative. (8) Write the potential energy of a particle moving in the foree field of (a). () Ifa particle at mass m moves with velocity v= dr/dt in this field, show that if K is the constant total energy then 4m(dr/dt)? + fer® = E. What important physical principle does this illustrate? 200r/r3. (a) Show that the fleld is ‘A particle of mass 4 moves in the force field defined by F ‘with speed 20, what will conservative and find the potential energy. (6) If a particle starts at r bbe its speed at r=2? Ans. (a) V = 200/r, (6) 15V2 IMPULSE, TORQUE AND ANGULAR MOMENTUM. CONSERVATION OF MOMENTUM 261, 270 an, A particle of unit mass moves in a force field given by F = (3t?—40)l + (12t—6)) + (6-120 where ¢ is the time. (a) Find the change in momentum of the particle from time {=1 to t=2. () If the velocity at t=1 is 4i—5}+10k, what is the velocity at ¢=2? Ans. (a) i+ 12) ~ 19k, (0) 81+ 1 ~ 9k A particle of mass m moves along a space curve defined by r= aos oti+ bsinut}. Find (@) the torque and (6) the angular momentum about the origin. Ans. (a) 0, (6) 2mabuk. A particle moves in a foree fleld given by F = (r)r. Prove that the angular momentum of the particle about the origin is constant, Find (a) the torque and (8) the angular momentum about the origin at the time t=2 for the particle of Problem 2.67, assuming that at ¢=0 it is located at the origin. Ana. (a) ~ (861+ 1285+ 60k), (B) — 44+ 62) + 16k Find the impulse developed by a force given by F = 4ti + (6t—2)j+ 12k from t=0 to t=2, Ans. 81+ 12) + 24th 58. NEWTON'S LAWS OF MOTION. WORK, ENERGY AND MOMENTUM — (CHAP. 2 272, What is the magnitude of the impulse developed by 51-34 7k m/sec to 2+3)+k m/sec? — An. mass of 200 gm which changes its velocity from 1.8 * 108 dyne see or 1.8 newton see STATICS OF A PARTICLE, 278. A particle is acted upon by the forces F,=21+4)~3k, Fy =Sitej+bk, Fy =bi—Bj-+T%, F,=ci—6j+ak. Find the values of the constants a,b,c in order that the particle will be in equilibrium. Ans. a=7,6=11,0=4 274, Find (@) graphically and (b) analytically the result- fant force acting on the mass m of Fig. 218 where all forces are in a plane. Ans. (0) 19.5 dynes in a direction making an angle 85°22" with the negative 2 axis 215, The potential of a particle moving in the zy plane is given by V = 22? Bayt Sy2+62—Ty. (a) Prove that there will be one and only one point at which a particle will remain in equilibrium and (b) find the coordinates of this point. Ana. (®) (1, 2) 216, Prove that a particle which moves in a force field of potential Vs at + yt + at — dy — dye + Bae ~ de + By — 4 can remain in equilibrium at infinitely many points land loeate these points Ana, All points on the plane #~2y-+2=2 Fig. 248 STABILITY OF EQUILIBRIUM 277, A particle moves on the # axis in a force field having potential V (a) Find the pointe at equilibrium and (b) investigate their stability. Ane, 2=0 i8-8 point of stable equilibrium; «=4 is a point of unstable equilibrium 262). 248, Work Problem 2.77 if (a) V = 2t—829—6zt-+ 242, (b) V = at. Ana, (a) 2 = 1,2 are points of stable equilibrium; * = —1 is a point of unstable equilibrium. () #=0 isa point of stable equilibrium 219, Work Problem 2.77 if V=sin 2ez. Ang, Hn = 0,21,22,28,... then = ts of unstable equilibrium. tn are points of stable equilibrium, while 2 = +n 280, A particle moves in a force field with potential V=a+y?+2t— 82+ 16y~ de, Find the points of stable equilibrium. Ane. (4,—8,2) MISCELLANEOUS PROBLEMS 281. (a) Prove that F = (y2 cons + 21 + (2y sine —4)j + (x2" + 2k ‘a conservative force field. (6) Find the potential corresponding to F. (0) Find the work done in moving a particle in this eld from = 0,1,-1) to (2,-1.2). J ‘Ane. (a) V= yPaing + 228 — dy +220 (6) 16+ de - 282, A particle P is acted upon by 8 coplanar forces as indicated in avon] Fig. 214, Find the force needed to prevent P from moving. ‘Ans, 823 lb in a direction opposite to 150 Ib force 288, (a) Prove that F = r#r in conservative and (8) find the correspond Fig. 214 ing potential. Ana. () V=—4rt te CHAP. 2] NEWTON'S LAWS OF MOTION, WORK, ENERGY AND MOMENTUM 59 281. Explain the following paradox: According to Newton's third law a trailer pulls back on an auto- ‘mobile to which itis attached with as much force as the auto pulls forward on the trailer. Therefore the auto cannot move, 285. Find the potential of a particle placed in a force field given by F=—xr-"r where « and n are constants. Treat all cases. 286. A waterfall 500 ft high has 440,000 ft? of water flowing over it per second. Assuming that the density of water is 62.5 Ib/ft and that 1 horsepower is 850 ftIb/see, find the horsepower of the waterfall. Ans. 25% 108 hp 287. The power applied to a particle by a force field is given as a function of thme t by (0) = St— 4t+2. Find the work done in moving the particle from the point where ¢=2 to the point where t= 4. Ane. 36 288. Can the torque on a particle be zero without the force being zero? Explain. 289. Can the force on a particle be zero without the angular momentum being zero? Explain. 290. Under the influence of a force fold F a particle of mass 2 moves slong the space curve Gti = St2} + (4¢9—5)k. Find (a) the work done in moving the particle from the point where 10 to the point where ¢=1, (b) the power applied to the particle at any time. Ans. (a) 756 (6) T2e(ABt + 8L+1) 291. A foree field moves a particle of ma power is required? (6) Discuss physically the ca ‘m along the space curve r = acos uti+ bain ut). (a) What Gab, Ans, (a) ma? 82) sin ut cos ut 292, The angular momentum of a particle is given as a function of time ¢ by = Gel — (at +1) + G20 — se Ans, 121 —2)-+ 20 Find the torque at the time 293, Find the constant force needed to give an object of mass 36,000 Ib a speed of 10 mi/hr in 6 minutes starting from rest. Ans, 1760 poundals 294, A constant foree of 100 newtons is applied for 2 minutes to a 20 kg mass which (a) What is the speed achieved? (b) What is the distance traveled? Ans, (a) 600 m/see, (6) 36,000 m 295. A particle of mass m moves on the 2 axis under the influence of a foree of attraction toward origin given by F = —(e/2%)i. If the particle starts from rest at z=, prove that it will arrive at O in a time given by Jeavina/. 296. Work Problem 2.95 if F = —(«/ai)i, 297. A particle of mass 2 units moves in the force field F = Ci — St} +(¢+2)k where ¢ is the time, (a) How far does the particle move from t=0 to t=8 if it is initially at rest at the origin? () Find the kinetic energy at times t=1 and ¢=3. (c) What is the work done on the particle by the field from ¢= 1 to €=87 (€) What ia the power applied to the particle at t=17 (e) What is the Impulse supplied to the particle at ¢=1? fs at rest at the origin. If it is acted upon by a force F = 100te~84, 298. At ¢=0a particle of unit ms = 1 to t=2, (6) the velocity find (a) the change in momentum of the particle in going from time after a long time has elapsed. Ane. (a) 26¢-%(8—5e-Hi, (b) 25 2.100, 20, 2102, 2.108, 2104, 2.106. 2.106. 2407, 2108, 2.109, 2110, NEWTON'S LAWS OF MOTION. WORK, ENERGY AND MOMENTUM —[CHAP. 2 ‘A particle of mass 2 unita moves in the zy plane under the influence of a force field having potential V = Get+12y!+ 86zy — 4822, Investigate the motion of the particle if it is displaced slightly from ita equilibrium position. [Hint, Near #=0,y=0 the potential is very nesrly 362y—482* since 62% and 124? are negligible.) 4 partie of nit mae mover on the = axis under te infence of «fore ld having potential {2 2), (a) Show that «=1 is a position of stable equilibrium. (6) Prove that if the masa ‘iaplaced slightly from its postion of equilrium it will owcllate about it with period equal to 40V8. (Hint, Let e 1 +m and neglect terms in w of degree higher than one,] ‘A particle of mass m moves in a force field F = —eai. (a) How much work is done in moving the particle from =, to #24? (6) If a unit particle starts at x=, with speed v, what is its speed fn reaching 2=2,2 Ans. (a) $s(st— al), (0) Vl @imle@e— =D ‘A particle of mass 2 moves in the zy plane under the influence of a foree field having potential V222+ 42. ‘The particle starts at time £=0 from rest at the point (2,1). (a) Set up the differential ‘equations and conditions describing the motion. (8) Find the position at any time ¢. (c) Find the velocity at any time t, ‘Work Problem 2.102 if V=8ey. Does Theorem 2.7, page 36, hold relative to @ non-inertial frame of reference or coordinate system? Prove your answer. () Prove that if a particle moves in the zy plane under the influence of a force field having potential Y= 12e(8y— 42), then #=0, y=0 is a point of stable equilibrium. () Discuss the relationship of the result in (a) to Problem 2.37, page 58. (@) Prove that a suficent condition for the point (c,8) to be a minimum point of the funetion V(z,y) is that at (a,) sv ye) _ (079 ov aa, wy a= (S)- (SEY > 0 mt Boe (bie (o to ovat the point of sat of «particle moving na force ld having potential Pee ete at) The poi Ua) pant ot stably Suppose that a particle of unit mass moves in the force field of Problem 2.106, Find its speed at any time, [A particle moves once around the circle r = a(cos@ i+ sing j) in a force field F = Gi- wert) (@) Find the work done. (6) Is the force fleld conservative? (c) Do your answers to (a) and (b) contradict Theorem 2.4, page 85? Explain. It is sometimes stated that classical or Newtonian mechanics makes the assumption that space and time are both absolute, Discuss what is meant by this statement. called the average force acting on a particle from time t, to ty. ah Does the result (8) of Problem 2.5, page 40, hold if F is replaced by Fay? Explain, CHAP.2} NEWTON'S LAWS OF MOTION. WORK, ENERGY AND MOMENTUM 61 2IIL, A particle of mass 2 gm moves in the force field F = Sxyl + (422—82)j — Syl dymes. If it has 1 speed of 4 em/see at the point (—1,2,—1), what is its speed at (1,~1,1)? Ans, 6 cm/sec 2112, (a) Find positions of stable equilibrium of a particle moving in a force field of potential V = ierte: (0) If the particle is released at r (e) Find the peri find the apeed when it reaches the equilibrium position. for small oscillations about the equilibrium position. 2118, According to Einatein’s epecial theory of relativity the mass m of a particle moving with speed » relative to an observer is given by m = my/VI=o8/e where e is the epeed of light [186,000 rifsee] and mo is the rest mass. What is the percent increase in rest mass of (a) an airplane moving ‘at 700 mi/hr, (0) a planet moving at 25,000 mi/hr, (c) an electron moving at half the speed of light? ‘What conclusions do you draw from these results? 2114, Prove that in cylindrical coordinates, w= where ¢,,€¢,€, are unit vectors in the direction of increasing p, # and + respectively. 2415. Prove that in spherical coordinates, vw “where @,€9/¢4 are unit vectors in the direction of increasing r, 6, ¢ respectively. Chapter 8 MOTION in a UNIFORM FIELD FALLING BODIES and PROJECTILES. UNIFORM FORCE FIELDS A force field which has constant magni- tude and direction is called a uniform or con- stant force field. If the direction of this field is taken as the negative z direction as indi- cated in Fig. 3-1 and the magnitude is the constant Fs > 0, then the force field is given by F = -Fok (en) Fig. 81 UNIFORMLY ACCELERATED MOTION If a particle of constant mass m moves in a uniform force field, then its acceleration is uniform or constant. The motion is then described as uniformly accelerated motion. Using F = ma in (1), the acceleration of a particle of mass m moving in the uniform force field (1) is given by @ WEIGHT AND ACCELERATION DUE TO GRAVITY It is found experimentally that near the earth’s surface objects fall with a vertical acceleration which is constant provided that air resistance is negligible. This acceleration is denoted by g and is called the acceleration due to gravity or the gravitational acceleration. ‘The approximate mag- nitude of g is 980 cm/sec, 9.80 m/sec* or 82 ft/sec according as the cgs, mks or fps system of units is used. This value varies at different parts of the earth's surface, increasing slightly as one goes from the equator to the poles. Assuming the surface of the earth is repre- sented by the zy plane of Fig. 8-2, the force acting ona particle of mass m is given by W = -mgk (8) ‘This force, which is called the weight of the par- ticle, has magnitude W = mg. Fig.3-2 62 CHAP, 8] MOTION IN A UNIFORM FIELD. FALLING BODIES AND PROJECTILES 63 GRAVITATIONAL SYSTEM OF UNITS Because W = mg, it follows that m = W/g. This fact has led many scientists and engi- neers, who deal to a large extent with mechanics on the earth’s surface, to rewrite the equations of motion with the fundamental mass quantity m replaced by the weight quantity W. Thus, for example, Newton's second law is rewritten as -% Ee ® In this equation W and g can both vary while m= W/g is constant. One system of units used in (4) is the gravitational or English engineering system where the unit of F or W is the pound weight (Ib wt) while length is in feet and time is in seconds. In this case the unit of m is the slug and the system is often called the foot-slug-second (fss) system. Other systems are also possible. For example, we can take F or W in kilograms weight (kg wt) with length in meters and time in seconds. ASSUMPTION OF A FLAT EARTH ‘Equation (3) indicates that the foree acting on mass m has constant magnitude mg and is at each point directed perpendicular to the earth’s surface represented by the zy plane. In reality this assumption, called the assumption of the flat earth, is not correct first because the earth is not flat and second because the force acting on mass m actually varies with the distance from the center of the earth, as shown in Chapter 5. In practice the assumption of a flat earth is quite accurate for describing motions of objects at or near the earth's surface and will be used throughout this chapter. However, for describing the motion of objects far from the earth’s surface the methods of Chapter 5 must be employed. FREELY FALLING BODIES If an object moves so that the only force acting upon it is its weight, or force due to gravity, then the object is often called a freely falling body. If r is the position vector and m is the mass of the body, then using Newton's second law the differential equation of motion is seen from equation (3) to be =-1 = Ge =—mok or OF ok 6) ‘Since this equation does not involve the mass m, the motion of a freely falling body is independent of its mass. PROJECTILES An object fired from a gun or dropped from a moving airplane is often called a projectile. If air resistance is negligible, a projectile can be considered as a freely falling body so that its motion can be found from equation (5) together with appropriate initial conditions. If air resistance is negligible the path of a projectile is an arc of a parabola (or a straight line ‘which can be considered a degenerate parabola). See Problem 3.6. 64 MOTION IN A UNIFORM FIELD. FALLING BODIES AND PROJECTILES [CHAP.3 POTENTIAL AND POTENTIAL ENERGY IN A UNIFORM FORCE FIELD ‘The potential of the uniform force field, or potential energy of a particle in this force field, is given by V = Fz) @ where zo is an arbitrary constant such that when 2=z, V=0. We call z level 1 the reference In particular for a constant gravitational field, F»=mg and the potential energy of the article is, V = mglz-2) ” ‘This leads to Theorem 3.1. The potential energy of a particle in a constant gravitational field is found by multiplying the magnitude of its weight by the height above some prescribed reference level. Note that the potential energy is the work done by the weight in moving through the distance z— 20. MOTION IN A RESISTING MEDIUM In practice an object is acted upon not only by its weight but by other forces as well. An important class of forces are those which tend to oppose the motion of an object. Such forces, which generally arise because of motion in some medium such as air or water, are often called resisting, damping or dissipative forces and the corresponding medium is said to be a resisting, damping or dissipative medium. It is found experimentally that for low speeds the resisting force is in magnitude propor- tional to the speed. In other cases it may be proportional to the square [or some other power] of the speed. If the resisting force is R, then the motion of a particle of mass m in an otherwise uniform (gravitational) force field is given by Gt = mgk - B ® If R=0 this reduces to (5). ISOLATING THE SYSTEM In dealing with the dynamics or statics of a particle [or a system of particles, as we shall see later] it is extremely important to take into account all those forces which act on the particle [or on the system of particles]. This process is often called isolating the system. CONSTRAINED MOTION In some cases a particle P must move along some specified curve or surface as, for example, the inclined plane of Fig. 8-3 or the inner surface of a hemispherical bow! of Fig. 3-4 below. Such a curve or surface on which the particle must move is called constraint and the resulting motion is called constrained motion, Just as the particle exerts a force on the constraint, there will by Newton's third law be a reaction force of the constraint on the particle. This reaction force is often described by giving its components N and f, normal to and parallel to the direction of motion respectively. In most cases which arise in practice, f is the force due to friction and is taken in a direction opposing the motion. CHAP. 3] MOTION IN A UNIFORM FIELD. FALLING BODIES AND PROJECTILES 65 Fig. 34 Problems involving constrained motion can be solved by using Newton's second law to arrive at differential equations for the motion and then solving these equations subject to initial conditions. FRICTION In the constrained motion of particles, one of the most important forces resisting motion is that due to friction. Referring to Fig. 3-5, let N be the magnitude of the normal component of the reaction of the con- straint on the particle m. Then it is found experi- mentally that the magnitude of the force f due to friction is given by f= uN ® Fig. 35 where 1 is called the coefficient of friction. ‘The direction of f is always opposite to the direction of motion. The coefficient of friction, which depends on the material of both the particle and constraint, is taken as a constant in practice. STATICS IN A UNIFORM GRAVITATIONAL FIELD ‘As indicated in Chapter 2, a particle is in equilibrium under the influence of a system of forces if and only if the net force acting on it is F = 0. Solved Problems UNIFORM FORCE FIELDS AND UNIFORMLY ACCELERATED MOTION 3.1. A particle of mass m moves along a straight line under the influence of a con- stant force of magnitude F. If its in speed is vs, find (a) the speed, (b) the velocity and (c) the distance traveled after time ¢. Fi ne 66 32 @ ® Cy MOTION IN A UNIFORM FIELD. FALLING BODIES AND PROJECTILES (CHAP. 3 Assume that the straight line along which the particle P moves is the « axis, as shown in Fig. 8-6 above. Suppose that at time ¢ the particle is at a distance z from origin O. If { is a unit veetor in the direction OP and v is the speed at time t, then the velocity is vi. By Newton's second law we have 2 oe Lino = rt ae mi F ins a= Ea ee fam fEn le w= Etta 0 where o; is a constant of integration. To find e we note the initial condition that v= at E=O so that from (2), = and v= Etem or “ From (8) the velocity at time ¢ is wa nit Za or where v= vi, vp =v and F= Fi, Since v= dz/dt we have from ($), &e = Zz Bathe or ae = (m+Ee)ar ‘Then on integrating, assuming ¢, to be the constant of integration, we have F z= ut+(Z)ere Since #=0 at ¢=0, wefind cp =0. Thus . F z= ut+(Ze » Referring to Problem 8.1, show that the speed of the particle at any position « is given by v = Vos + @F/m)z. we find 2 = (m/2F\(ot— 0). Solving for v we obta ‘or since v= de/dt, Integrating, Since v= vy when = ‘Method 1. From (8) of Problem 3.1, we have mm(v—vpV/F. Substituting into (6) and si ‘the required result. Method 2. ‘From (1) of Problem 8.1, we have ie vdv = Eas gE Estes /2 and hence v= Voq+ @F/me. 0, we find, Method 3. Change in kinetic energy from t=0 to any time ¢ ‘Work done in moving particle from 2=0 to any position # or gmv!— fmol = Fle—0). Then v= Vogt @F/mz. CHAP. 8] MOTION IN A UNIFORM FIELD. FALLING BODIES AND PROJECTILES er LINEAR MOTION OF FREELY FALLING BODIES 33. An object of mass m is thrown vertically up- ‘ward from the earth’s surface with speed v. Find (a) the position at any time, (b) the time taken to reach the highest point and (c) the ‘maximum height reached. (a) Let the position vector of m at any time t be r=altyj+2k. Assume that the object starts at r=0 when t=0. Since the foree acting on where v is the velocity at time t. Integrating (1) once yields v = ~otk be ® ‘Since the velocity at ¢=0 [ie. the initial velocity] is wok, we have from (2), ej = wjk 0 that v= —otk +k = (v0—otk o or & = (v—o0k © Integrating (4) yields F = (yt fot +e o ‘Then since when £=0, e=0. Thus the position vector is F = (vot 400k © or, equivalently, 220, y=0, 2 = Ot ott o ie, at time t= v9/g. (b) ‘The highest point is reached when v = (v)—9f)k = (@) At time t=vg/o the maximum height reached is, from (7), spe. Another method. If we assume, as is physically evident, that the object must always be on the + axis, we may avoid vectors by writing Newton’s law equivalently as (see equation (1) above and place +r = 2k) @e/d8 = ~9 we find 2 = wt doe fas above. ‘The answers to (b) and (c) are then obtained as before. from which, using , deldt =p at Find the speed of the particle of Problem 3.3 in terms of its distance from origin 0. ‘Method 1. From Problem 8.8, equations (8) and (7), we have v= mot 2 = et dor Solving for ¢ in the frst equation and substituting into the second equation, we find Method 2, From equation (1) of Problem 8.8 we have, ®o, eee au See @ Then on integrating, 0/2 = ~gs-+ey. Since v=u at 2=0, cy=0f/2 and thus of = of—2ys. ‘Method 8. See Problem 8.9 for a method using the principle of conservation of energy. 68 MOTION IN A UNIFORM FIELD. FALLING BODIES AND PROJECTILES [CHAP.3 MOTION OF PROJECTILES 35. A projectile is launched with initial speed vy at an angle « with the horizontal. Find (a) the position vector at any time, (b) the time to reach the highest point, (c) the maximum height reached, (d) the time of flight back to earth and (e) the range. (2) Let ¥ be the position vector of the projec- fand v the velocity at any time t. Then by Newton's law ee i noe Fig. 88 oo fu or =~ ® Integration yields eer e ‘Assume the initial velocity of the projectile is in the yz plane so that the initial velocity vy = vce) + tysinak w Since v= vy at €=0, we find from (6), v= vpconas + (eosina— 9k o Replacing v by de/dt in (6) and integrating, we obtain £ = (wpconalts + (lve sinalt~ $othe rc) = (psin alt = jot® o or, equivalently, = (v9 08 It follows that the projectile remains in the yz plane. ‘the ke direction is zero. Thus (b) At the highest point of the path the component of velocity sn se (Ve he va fated i (fl rom at cenen bcttnaind = ian) ("2 : : (Toe wo igh bok arth th tn we ‘atin jt = tops) fol = 8 t bast (10) 7 Note that this is twice the time in (6). (e) The range is the value of y at the time given by (10), ic. mene) ie _ Range = (oy coe0)(( ‘Show that the path of the projectile in Problem 3.5 is a parabola. From the second equation of (7) in Problem 3.5, we have ¢ = v/(vocos into the third equation of (7) in Problem 3.5, we find = (eosinelo/ry cons) — Jolvloy coma)? or # = vtana— (O/B? sect which is @ parabola in the ys plane, Substituting this CHAP, 3) MOTION IN A UNIFORM FIELD. FALLING BODIES AND PROJECTILES 69 8.7. Prove that the range of the projectile of Problem 3.5 is a maximum when the launch- ing angle a= 45°. By Problem 3.6(c) the range is (vf sin 2a)/g. Thi or «= 45°, js a maximum when sin 2a=1, Le, 22 = 90° POTENTIAL AND POTENTIAL ENERGY IN A UNIFORM FORCE FIELD 8.8 (a) Prove that a uniform force field is conservative, (b) find the potential correspond- ing to this field and (c) deduce the potential energy of a particle of mass m in a uniform gravitational force field. (a) If the force field is as indicated in Fig. 8-1, then rok: We have i 3s vx = | ae ay az) = 0 0 0 =F ‘Thus the foree fleld is conservative. = Pyke = —vv = —% 5 x () F = Fok = -0v = ~ Ba Hj— then "y from which VERete I V=0 at then ©=—Fozp and so V = Fy(z—x)- (0) For a uniform gravitational force field, F to Fy=mg. ‘Then by part (6) the potential or potential energy is V —mgk (see Fig. 3-2, page 62] and corresponds ‘male 20). 3.9. Work Problem 3.4 using the principle of conservation of energy. ‘According to the principle of conservation of energy, we have PE ate=0 + KE ate=0 = PRate + KBate 0 dmd = mor + dt Then v= ef 20e MOTION IN A RESISTING MEDIUM 3.10. At time t=0 a parachutist [Fig. 3-9] having § ——[—————— weight of magnitude mg is located at z= 0 and is traveling vertically downward with speed If the force or air resistance acting on the parachute is proportional to the instantaneous speed, find the (a) speed, (b) distance traveled and (¢) acceleration at any time > 0. (a) Assume the parachutist (considered as a particle fof mass m) is located at distance z from origin O. If k is « unit vector in the vertically downward a ‘direction, then the weight is mgk while the force of air resistance is Buk so that the net force ane ™ is (mg— Bek. ‘Thus by Newton's law, o) Fig. 39 70 aL. 312, MOTION IN A UNIFORM FIELD. FALLING BODIES AND PROJECTILES [CHAP. 3 may u mie men ae or BH ae Integrating, — Fm (mg Av) = the @ Since v= at £=0, e = —Fin(mg—fry). Then from (2), = Fim (mg— pv) — Finimg— po) = ‘Thus oro = Me (Ete @ (0) From (8), de/dt = mo/p + (vy—mg/p)e-#™. ‘Then by integration, 2 = MEW (oy Mem + ay nan Since 2=0 at t= 0, ¢y = (m/B)vy—mo/f) and thus 2 = Moby My, MOY — g-aum ih (ay BE tm (6) From (8), the acceleration is given by C} ‘Show that the parachutist of Problem 3.10 approaches a limiting speed given by mg/p. Method 1. From equation (3) of Problem 8.10, v = mg/f + (vy—mg/ple~/m, ‘Then as ¢ increases, » approaches mg/B so that after a short time the parachutist is traveling with speed which is practically constant. Method 2. If the parachutist ia to approach limiting speed, the limiting acceleration must be zero. ‘Thus from equation (1) of Problem 310 we have mg~Pvim=0 oF Yim = mo/B- A particle of mass m is traveling along the x axis such that at t= 0 it is located at and has speed ve. The particle is acted upon by a foree which opposes the motion and has magnitude proportional to the square of the instantaneous speed. Find the (a) speed, (b) position and (c) acceleration of the particle at any time t>0. (@) Suppose particle P is at a distance 2 from 0 at and has speed v [see Fig, $10], ‘Then the Rone force F=~pv'i where A> 0. is a constant of proportionality. By Newtons law, | « __} Fig. 80 atm o which is the speed. Bin (t+) ee CHAP. 8] MOTION IN A UNIFORM FIELD, FALLING BODIES AND PROJECTILES n B13. 814, Since 2=0 at = -in(@). mm =o Bam) 7 ) gu (+) O (©) From (a), mst o= = S(nets) * aaa o Note that although the speed of the particle continually decreases, it never comes to rest. Determine the (a) speed and (b) acceleration of the particle of Problem 3.12 as a function of the distance z from O. Method 1, From parts (a) and (6) of Problem 3.12, md = oe en Min(2) or v= vge-Aarm " gu(2) ow and the acceleration ia given in magnitude by = Pde SP te ae ae 7 ‘which can also be obtained from equation (4) of Problem 8.12, Method 2. From equation (1) of Problem 8.12 we have fo dete Syl eae ee 0 040, mit =p and = —Be, Interting, nv = ~pelm tay Snes v= ms when #=0, Hine The In(cle) = "anim or v= oe-tm ‘Suppose that in Problem 8.5 we assume that the projectile has acting upon it a force due to air resistance equal to Av where A is a positive constant and v is the instan- taneous velocity. Find (a) the velocity and (b) the position vector at any time. (a) ‘The equation of motion in this ease is PE = —mok- pv or mGE+ py = —mgk ” Dividing by m and multiplying by the integrating factor eS" = eht/m, the equation can be written as : Gloomy = ~oetom Integration yields omy = ME om te cy The initial velocity or velocity at #=0 i ve = %coned + wsinek ® ‘Using this in (2) we find = memes + vysinak + a 2 MOTION IN A UNIFORM FIELD. FALLING BODIES AND PROJECTILES [CHAP.3 ‘Thus (2) becomes on dividing by e®/m, = (Wpconad + ty sina kent — MBCA ~ etme ® (&) Replacing: v by dr/dt in (4) and integrating, we find F = ~Beogcone d+ og nina ier — MES Mo-timie + e = Bvcwait susined + Mu 0 Using (6) in (6), we Sind r= cosa 5 + sina kd —e-ttimy — PO (4 4 Menpum — pone 5+ wna WL —e-tmy — ME (44 me o 3.15. Prove that the projectile of Problem 3.14 attains a limiting velocity and find its value, Method 1. Refer to equation (4) of Problem 3.14, As ¢ increases, velocity approaches a limiting value equal to Vig = —(g/lk -6t/m approaches zero, ‘Thus the Method 2. If the projectile is to approach a limiting velocity ite limiting acceleration must be zero, Thus from equation (1) of Problem 2.14, —mgk ~ Avi = 0 OF Viyq = —(mg/A)k. CONSTRAINED MOTION 8.16. A particle P of mass m slides without rolling down a frictionless inclined plane AB of angle « [Fig. 8-11]. If it starts from rest at the top A of the incline, find (a) the acceleration, (b) the velocity and (c) the distance traveled after time t. (a) Since there is no friction the only forees acting on P are the weight W=—mpk and the re ‘action force of the incline which is given by the normal force N. Let ¢ and e, be unit vectors parallel and perpendicular to the incline respectively. If we denote by # the magnitude of the displacement from the top A of the inclined plane, we have bby Newton's second law Bg) = = m Zl) = WHEN = mosinae, o since the resultant equal to W++N is mg sinae,, as indicated in Fig, 8-11, From (1) we have @eldt® = g sina ® ‘Thus the acceleration down the incline at any time t is a constant equal to g sin =. (®) Since v= ds/dt is the speed, (2) can be written dv/dt = gsine or v = (gsinat te on integrating. Using the initial condition v speed at any time t is oa a ‘The velocity is ve, =(g sina)te, which has magnitude (g sina)t in the direction e, down ‘the incline. so that the at t=0, we have o= CHAP. 3] MOTION IN A UNIFORM FIELD. FALLING BODIES AND PROJECTILES 3 (©) Since v=dv/dt, (8) can be written delat = (gsinalt or 8 = Hasina tex ‘on integrating. Using the initial condition «=0 at £=0, we find ¢p=0 so that the required distance traveled is # = dosing w 3.17. If the length AB of the incline in Problem 3.16 is 1, find (a) the time + taken for the article to reach the bottom B of the incline and (b) the speed at B. (a) Since #=1 at B, the time + to reach the bottom is from equation (4) of Problem 3.16 given by T= 4@ sina? or += VEgaing). (8) The speed at B is given from (s) of Problem 8.16 by v= (¢sinalr = V2gl sina. MOTION INVOLVING FRICTION 8.18. Work Problem 8.16 if the inclined plane has ‘a constant coefficient of friction j. (a) Im this case there is, in addition to the forces W and N acting on P, « frictional foree f [see Fig. 8-12} directed up the ineline [in a direction oppe- site to the motion] and with magnitude ae [ Le. f = —pmg cosa e @) eae ae ae m : — ® ‘Thus the acceleration down the incline has sing > pcose or tana > [otherwise the frictional force is so great tht rot move at all]. the constant magnitude g(sine— cose) provided st the particle will (8) Replacing a/dt? by do/dt in (4) and integrating a3 in part (0) of Problem 8.16, we find the speed at any time ¢ to be os ii = (@) Replacing v by de/dt in (5) and integrating as in part (c) of Problem 3.16, we find = folsine ~ n cose) o 8.19. An object slides on a surface of ice along the horizontal straight line OA (Fig. 8-13]. ‘At a certain point in its path the speed is ve and the object then comes to rest after traveling a distance zo, Prove that the coefficient of friction is v*/2gz0. Let # be the instantaneous distance of the object of mass m from O and suppose that at time t=0, 2=0 and de/dt =. & ‘Three forces act on the object, namely (1) the “| A weight W= mg, (2) the normal force N of the = VW = me {ee surface on the object, and (@) the frietions force f. Fig. 313 By Newton’s second law we have, if v is the instantaneous speed, WHNet o 4 MOTION IN A UNIFORM FIELD. FALLING BODIES AND PROJECTILES [CHAP.3 Bu since N= —W ad the magni ot fs f= a8" png ao that C= yma, (1) beamen fi . wR emt Bene o Method 1. Write (2) as ge-- o Then Integrating, sing the fast that = nq at #0, we find we =~ +4 ) Then since ¥=0 when = 5, (4) becomes wor + Wy2 = 0 or 4 = W/2oz © Method From @ we have on integrating and ving the fact that = vp at # = nat or deli = Hy pat © Integrating sgn, ong the fact that = 0 at €=0, we find 2 = tot — fagt® om From (7) we see that the object comes to rest (i, »=0) when Ment = 0 or t= why Substituting this into (7) and noting that © =a, we obtain the required result, STATICS IN A UNIFORM GRAVITATIONAL FIELD 820. A particle of mass m is suspended in equilibrium by two inelastic strings of lengths and b from pegs A and B which are distant ¢ apart. Find the tension in each string. Fig. 814 ig. 815 Let W denote the weight of the particle and ‘T, and 7 the respective tensions in the strings of lengths a and b as indicated in Fig. 3-14, ‘These forces are aleo indicated in Fig, 9-15 and are assumed to Ile in the plane of unit vectors j and k. By resolving T, and T into horizontal and vertical components it is clear that T = Tysinak~Tyeonaj, TN; = Tesin pk + Tyeosps where 7, and 7, are the magnitudes of T, and T, respectively and where a and f are the respective angles at A and B. Also we have Ws mek ‘Since the particle is in equilibrium if and only if the net force acting on it is zero, we have Fo+h+W T,sinak — Ty cosas + Tyain fk + Tyco | — mak = (Py 0088 = 7; cosadi + (Py sina + Ty sin p ~ mak ° CHAP. 8} MOTION IN A UNIFORM FIELD. FALLING BODIES AND PROJECTILES © From this we must have Ty c08 8 ~ Ty co. Ty sine + Ty sinp—~mg = 0 Solving simultaneously, we find r = m9 £08; = mg cose = sinet ay’ T= Gina rA) ‘The angles « and pcan be determined from the law of cosines as (Sa), = a (E=*) From these the tensions can be expressed in terms of a, b, ¢- MISCELLANEOUS PROBLEMS 3.21. An inclined plane [Fig. 3-16) makes an angle « with the horizontal. A projectile is, launched from the bottom A of the incline with speed v in a direction making an angle f with the horizontal. (a) Prove that the range R up the incline is given by R= 20§ sin (P — a) cos 8 9 COS Fig. 3-16 (b) Prove that the maximum range up the incline is given by % ~ 91 ¥ sina) and is achieved when f = 1/4 + a/2. Ras (@) As in Problem 3.5, equation (6), the position vector of the projectile at any time ¢ is F = (vo cos ty + {{v sin Aye ~ hott) ny or ¥ = (epcossyt, 2 = (op sina)t— jot? ® ‘The equation of the incline [which is a line in the ys plane] is 2 = ytane cy) ‘Using equations (2) in (8) we see that the projectile’s path and the incline intersect for those values of t where (vg sin p)t ~ Jot? = [ve cos )t] tana 2us(sin f cos a ~ cos # sin) cosa le. t=0 and ¢ ‘The value #=0 gives the intersection point A. ‘The second value of t yields point B which is the required point. Using this second value of ¢ in the frst equation of (2), we find that the required range R up the incline is 20g sin 8 a) 8 (®) Method 1. ‘The range R can be written by using the trigonometric identity sind cosB = j{sin(A +B) + sin(A—B)) xo fs as R= saeep (sin @p—«) ~ sina) 6 3.22, 3.23, MOTION IN A UNIFORM FIELD. FALLING BODIES AND PROJECTILES [CHAP. 3 This is a maximum when sin (p=) =1, be a cite eprom i - = _tagansing = v costa? ) ofl — sin®a) a ) ofl + sina) Method 2. ‘The required result can also be obtained by the methods of differential calculus for finding maxima and minima, ‘Two particles of masses m: and ms respectively are connected by an inextensible string of negligible mass which passes over a fixed frictionless pulley of negli- gible mass as shown in Fig. 3-17. Describe the motion by finding (a) the acceleration of the particles and (0) the tension in the string. Let us first isolate mass my. There are two forces acting on its (1) its weight omg = myok, and (2) the foree due to the string which isthe tension ‘T= Tk. If we call’ a~ ok the acceleration, then by Newton's law ‘mak = moe — Tk wo Next we isolate mass mp ‘There are two forces acting omit (0) its weight, mug'= magk, and (2) the tension T==Tk [the tension ie the same throughout the string since the mass of the string is assumed neglisible and i extensible), Since the string is inextensible, the acceleration of ma ig a= ~ak, ‘Then by Newton’ Fig. 3-17 mak = mgk — Tk ® Prom (1) and (2) we have ma = mg- 7, -ma = my —T Solving simultaneously, we find my my 2mymy a, mt my Fm mm ‘Thus the particles move with constant acceleration, one particle rising and the other falling. In this pulley system, sometimes called Atwood’s machine, the pulley can rotate. However, since it is frictionless and’has no mass [or negligible mass] the effect is the same as if the string passed over a smooth or frictionless peg instead of a pulley. In ease the mass of the pulley is not negligible, rotational effects must be taken into account and are considered in Chapter 9. A particle P of mass m rests at the top A of a frictionless fixed sphere of radius b. The par- ticle is displaced slightly so that it slides (with- out rolling) down the sphere. (a) At what posi tion will it leave the sphere and (b) what will its speed be at this position? ‘The particle will slide down a circle of radius a which we choose to be in the zy plane as indieated in Fig. 848, The forces acting on the particle are: ) its weight W =—mgj, and (2) the reaction force N of the sphere on the particle normal to the sphere. Method 1. (a) Let the position of the particle on the circle be ‘measured by angle @ and let ry and @ be unit. vectors. Resolving W into components in direc tions ry and @, we have as in Problem 1.43, page 24, CHAP. 3} @ MOTION IN A UNIFORM FIELD. FALLING BODIES AND PROJECTILES 7 W = (Were, + (Weeyey = (emoisepes + (mol opie, = ~ma sino x, — mo cose 4 Also, N= Nn Using Newton's second law and the result of Problem 1.49, page 26, we have F = ma = mili rity + (8 + Bhd) W+N = (N— mg sine)r, — mg cose ey a ‘Thos ri) = N— mgs mod + 2) = —mg cone ® ‘While the particle is'on the circle (or sphere), we have r=B. Substituting this into (2), 0 the particle stays on the sphere; but when N= 0. the particle will be Just about to lenve the sphere. ‘Thus the required angle fs given by Ssin¢—2=0, Le, sing = 2/8 or @ = sin-12/3 (6) Putting sine = into (4), we find i = 2/86 o Then if in the speed, we have v= 3 eo that (7) yields v= Gbo or 9 = VGPV. Method 2. By the conservation of energy, using the + axis as reference level, we have PEatA + KEatd = P.EatP + KEatP mgb + 0 = mob sing + pret or vt = gb — sins) ® Using the result of Problem 1.85, page 20, together with Newton's second law, we have, since the radius of curvature is b, F = oma = WEN = (N= mg sin sje, — mg cose oy Using only the r, component, we have vb = N—mgsine @ From (@) and (@) we find_N = mg(8sin@ ~ 2) which yields the required angle sin (f) Method 1. ‘The speed is then found from (6). 8 ‘MOTION IN A UNIFORM FIELD. FALLING BODIES AND PROJECTILES (CHAP. 3 Supplementary Problems ‘UNIFORM FORCE FIELDS AND LINEAR MOTION OF FREELY FALLING BODIES 32, 3.35, 3.26, 3a, 3.28, 323, 230, sat, 332, 3.38. ‘An object of mass m is dropped from a height H above the ground. Prove that if air resistance fs negligible, then it will reach the ground (a) in a time VZH/g and (b) with speed V2gH. ‘Work Problem 2.24 if the object is thrown vertially downward with an initial velocity of magi- tude vp. Ana. (@) (Vag + 20H — vila, (0) VoE# 26H Prove that the object of Problem 8.3, page 67, returns to the earth's surface (a) with the same speed as the initial speed and (b) in a time which is twice that taken to reach the maximum height, ‘A ball which is thrown upward reaches its maximum height of 100 f¢ and then returns to the Starting point. (a) With what speed was it thrown? (b) How long does it take to return? ‘Ans. (a) 80 ft/sec, () 5 s0e ‘A ball which is thrown vertically upward reaches particular height J after a time 7, on the ‘way up and a time z, on the way down. Prove that (a) the inital velocity with which the ball was thrown has magnitude do(ry +) and (8) the height Hf = dorsra In Problem 8.28, what is the maximum height reached? Ana. do(es + 13¥ ‘Two objects are dropped from the top of a cliff of height H. ‘The second is dropped when the first has traveled a distance D. Prove that at the instant when the first object has reached the bottom, the second object is at a distance above it given by 2/DH—D. ‘An elevator starts from rest and attains speed of 16 ft/sec in 2sec. Find the weight of a 160 1b man in the elevator if the elevator is (a) moving up (6) moving down. ‘Ans. (a) 200 1b, (B) 120 1b A particle of masa 8 kg moving ina straight line decelerates uniformly from a speed of 40 m/sec to 20 m/sec in a distance of 900m. (a) Find the magnitude of the deceleration. (b) How much further does it travel before it comes to rest and how much longer will this take? Ama. (a) 2 m/sec, (6) 100m; 10 see In Problem 3.82, what is the total work done in bringing the particle to rest from the speed of 40 m/sec?" Ane. 2400 newton meters (or joules) MOTION OF PROJECTILES 336 3s, 3.8, ‘A projectile is launched with a muzzle velocity of 1800 mi/hr at an angle of 60° with « horizontal and lands on the same plane. Find (e) the maximum height reached, (8) the time to reach the maximum height, (c) the total time of flight, (d) the range, (e) the speed after 1 minute of flight, () the speed at a height of 82,000 ft. Ans. (a) 65 mi, (6) THA see, (€) 1428 see, (4) 86.7 mi, (e) 984 mi/hr, (9) 1668 mile (o} What is the maximum range possible for a projectile fired trom a eannon having muzzle velocity 1 mi/see and (2) what is the height reached in this case? Ans. (@) 185 mi, (8) 41.28 mi ‘A cannon has its maximum range given by Rngx. Prove that (a) the height reached in such case fs Lac and (0) the time of Might is VRmay/20. It is desired to launch a projectile from the ground so a8 to hit a given point on the ground which is at a distance less than the maximum range. Prove that there are two possible angles for the launching, one which is less than 45° by a certain amount and the other greater than ‘A projectile having horizontal range R reaches a maximum height H. Prove that it must have ck taanehed with e) am inital speed equal to Va TOHPVBH and (0) at an angle with (QH/V RET 16H). the horizontal given by sin CHAP. 8] MOTION IN A UNIFORM FIELD. FALLING BODIES AND PROJECTILES 9 339, ‘A projectile is launched at an angle « from a cliff of height H above soa level. If it falls into the ea at a distance D from the base of the clif, prove that its maximum height above sea level is D2 tanta ‘HY THF D tend) MOTION IN A RESISTING MEDIUM 3.40, sat, 3.42, 344, 3.45. 3a. 3.48, ‘An object of weight W is thrown vertically upward with speed vp. Assuming that air resistance is proportional to the instantaneous velocity and that the constant of proportionality is x, prove ‘that (a) the object will reach a maximum height of Wen wa ( 2) -Bin(1+ ot ao 80" w fand that (b) the time taken to reach this maximum height is oo(+) ‘A man on a parachute falls from rest and acquires a limiting speed of 15 mi/hr. Assuming ‘that air resistance is proportional to the instantaneous speed, determine how long it takes to reach the speed of 14 mi/hr. Ans. 1.86 sec ee APD le ae ey Pee Sette ad min (Pa ee 2G) ‘A particle of mass m moves in a straight line acted upon by a constant resisting force of magni- ‘tude F. If it starts with a speed of vy, (a) how long will it take before coming to rest and (@) what distance wil it travel in this time? Ame. (a) mvg/F, (0) mvd/2F ‘Can Problem 8.48 be worked by energy considerations? Explain. ‘A locomotive of mass m travels with constant speed vp along a horizontal track. (a) How long ‘will it take for the locomotive to come to rest after the ignition is tumed off, if the resistance to the motion is given by «+ Aut where v ia the instantaneous speed and a and p are constants? (H) What is the distance traveled? Ane. (0) Vm/p tan-¥(vgVpla), (8) (2A) In (1 + Avb/a) ‘A particle moves along the # axis acted upon only by a resisting foree which is proportional to the cube of the instantaneous speed. If the initial speed is ve and after a time r the speed is ‘4%o, prove that the speed will be 4 ia time Br. Find the total distance traveled by the particle of Problem 2.46 in reaching the speeds (@) 4, (©) Feu Ane. (0) Fron, () vor Prove that for the projectile of Problem 14, page 71, Presa) (ethene to sec the hich pnt ie Sn (1+ 7 meg sin ve sin (8) the maximum height i OBES — Ein (1+ CONSTRAINED MOTION AND FRICTION 3.49, ‘A weight of 100 Ib slides from rest down 2 60° incline of length 200 ft starting from the top. Neglecting friction, (a) how long will it take to reach the bottom of the incline and (#) what is the speed with which it reaches the bottom? Ans. (a) 8.80 sec, (6) 105.3 ft/sec 80 MOTION IN A UNIFORM FIELD. FALLING BODIES AND PROJECTILES (CHAP. 3 350. Work Problem 2.49 if the coefficient of friction is 0.3. Ans. (a) 4.18 sec, (0) 96.7 ft/sec 851, (a) With what speed should an object be thrown up a smooth incline of starting from the bottom, so as to just reach the top and (8) what is Ans. (a) VigT sina, (6) VIG sine) 352. If it takes a time + for an object starting from speed vp on an iey surface to come to rest, prove that the cocficient of friction is vy/gr. ele @ and length 1, time taken? 353, What force is needed to move @ 10 ton truck with uniform speed up an incline of 80° if the coefficient of friction is 0.1 ‘Ans. 587 tons 54, A mass m rests on a horizontal piece of wood. The wood is tilted upward until the mass m just begins to slide, If the angle which the wood makes with the horizontal at that instant is o, rove that the coeficient of friction is» = tana. 355. A 400 kg mass on a 80° inclined plane is acted upon by © force of 4800 newtons at angle 30° ‘with the incline, as shown in Fig. 9-20, Find the acceleration of the mass if the incline (a) is frictionless, (0) has coefficient of friction 0.2. Ans. (a) 5.5 m/sect, (6) 5.0 m/sec? Fig. 3-20 Fig. 321 856, Work Problem 8.55 if the force of 4800 newtons acts as shown in Fig. 3-21. Ans. (a) 5.5 m/sec?, (8) 2.6 m/sec? STATICS IN A UNIFORM GRAVITATIONAL FIELD 357, A 100kg weight is suspended vertically from the center of a rope as shown in Fig. 8-22, Determine the tension T in the rope, Ans. T = 100 ke wt = 980 nt Fig. 8.22 Fig. 2-23, 358, In Fig. 8-28, AB and AC are ropes attached to the celling CD and wall BD at C and B respectively. ‘A weight W is suspended from A. If the ropes AB and AC make angles 4, and ¢, with the ‘wall and ceiling respectively, find the tensions 7, and Tin the rope W cose Wesingy Ans. 7; wae 359, Find the magnitude of the force F neoded to keep mass m in equilibrium on the inclined plane ‘of Fig. 5-24 if (a) the plane is smooth, (6) the plane has coefficient of frietion mgsine yy p= malsina— a core) Ane. (0) F = MESES, Se CHAP, 3) MOTION IN A UNIFORM FIELD. FALLING BODIES AND PROJECTILES aL 360. How much foree is needed to pull a train weighing $20 tons from rest to a speed of 15 mi/hr 20 seconds if the coefficient of friction is 0.02 and (a) the track is horizontal, (b) the track is inclined at an angle of 10° with the horizontal and the train is going upward? [Use sin 10° = 1787, cos 10° = 9848] Ane. (a) 17.4 tons, (0) 129.6 tons 361. Work Problem 3.60(b) if the train is going down the incline, Ans, 8.6 tons 362, A train of mass m is coasting down an inclined plane of angle « and coefficient of friction ‘with constant speed vy. Prove that the force needed to stop the train in a time + is given by ‘ma(sin a ~ 1 C080) + mvolr. MISCELLANEOUS PROBLEMS 262, A stone is dropped down a well and the sound of the splash is heard after time +r. Assuming the ‘speed of sound is ¢, prove that the depth of the water evel in the well is (Ve?+ ger ~ oF /20. Oe ee 2v5 sin y cos (7 (an cane tga ty B= PEERY ONCT A 4 th ttm rn em Incline is, Bras = = sina)” S65 A canon ecto hil which ah shape ot nln lane of ane with thera Ceea ae eee era ea ocpuchitad maine irenge fit Ses te tht nore forth projece eM th ll rotay we mat have # = en (E282), 3.66. Suppose that two projectiles are launched at angles a and f with the horizontal from the ‘same place at the same time in the same vertical plane and with the same initial speed. Prove that during the course of the motion, the line Joining the projectiles makes constant angle ‘with the vertical given by H(e+ A). 367. In {€ ponsible to solve equation (1), page 83, by the method of separation of variables? Explain, 368, When launched at angle @, with the horizontal a projectile falls a distance D, short of its target, while at angle ¢, it falls a distance D, beyond the target. Find the angle at which the projectile should be launched 20 as to hit the target. 38.69. An object was thrown vertically downward. During the tenth second of travel it fell twice as far fas during the fifth second. With what speed was it thrown? Ans, 16 ft/sec 810. A gun of muzzle speed uy is situated at height A above a horizontal plane. Prove that the angle ‘ti 'which it must bo fred s0 as to achieve the greatest range ‘on the plane is given by Oa" f cor ahi 0M). ‘B71, In Fig. 3-25, AB is a smooth table and masses m, = and’, are connected by a string over the smooth peg at B. Find (a) the acceleration of mass m: 4 & nd) the tension im the string mM n> Ane (0) Mg, mm, m™ mma @ meg 312, Work Problem 8.11 if the table AB has coefficient of friction ». 473, The maximum range of a projectile when fired down an inclined plane is twice the maximum ‘range when fired up the inclined plane. Find the angle which the incline makes with the horizontal. Ane, sin“11/2 82 MOTION IN A UNIFORM FIELD. FALLING BODIES AND PROJECTILES [CHAP.3 374, Masses m, and m, are located on smooth inclined planes of angles a, and ay respectively and are connected by fan inextensible string of negligible mass which passes lover a smooth peg at A (Fig. 3-26). tions of the masses. Ane. The accelerations are in magnitude equal to ‘my sin ay — my sin og mm 815. Work Problem 8.74 if the cooficient of friction between the mastes ms 608.09 816, Prove that the least horizontal fore F needed to pull ler of radius a and weight W over an obstacle of height (sce Fig. 8-27] is given in magnitude by WVEQa—Bya~b). 417. Explain mathematically why a projectile fired from ‘cannon A at the top of a lif at height H above the ground can reach a cannon B located on the ground, while a projectile fired from cannon B with the same ‘muzzle velocity will not be able to reach eannon A. 318, In Fig. 3-28 the mass m hangs from an inextensible string OA. It ie pulled aside by a horizontal string AB so that OA makes fan angle @ with the vertical. Find the tension in each string. Ans. ‘Tension in AB = mg tana; inOA = mg seca 378, A particle moving slong the + axis is acted upon by a resisting force which is such that the time ¢ for it to travel a is given by t= Azt+Ba+C where A, B and C are constants. Prove that the magnitude of the resisting force is proportional to the cube of the instantaneous speed, 380, A projectile is to be launched s0 as to go from A to B [which are respectively at the bases of a double inclined plane having angles « and fas shown in Fig. 8.28] and Just barely mise a pole of height H. If the distance between A and B is D, find the angle with the horizontal fat which the projectile should be launched. BBL. A. particle of mass m moves on a frictionless inclined plane of angle a and length 1. If the particle starts from rest at the top of the incline, what will be its ‘speed at the bottom assuming that air resistance is equal to av where is the instantaneous speed and « is con- stant? Fig. 8-29 382, Suppose that i Problem 9:29 the particle P is given an initial speed v at the top of the circle (or sphere). (a) Prove that if vp = Vb, the angle ¢ at which the particle leaves the circle is given by sin-1(j + of/3g8), (@) Discuss what happens if v9 > V96. 382. A cannon is situated at the top of a vertical cliff overlooking the sea at height H above sea level. ‘What should be the least muzzle velocity of the cannon in order that a projectile fired from it will reach ship at distance D from the foot of the cliff? 384, In Problem 8.88, (a) how long would it take the projectile to reach the ship and (b) what is the velocity on reaching the ship? CHAP. 3] MOTION IN A UNIFORM FIELD. FALLING BODIES AND PROJECTILES 88. 385, 3.36, 3.88, 3.0, 395, 3.96. A. uniform chain of total length @ has a portion 0 yp, then a necessary and sufficient condition that W, move relative to the plane while W, not move relative to Ws is that aA + We) ~—% tose sine © cosa sina Fig. 3.88 Discuss the results in Problem 8.100 if any of the conditions are not satisfied. Give a generalization of Problem 3.100, Describe the motion of the particle of Problem 8.97 if E and B are constants, and have the same direction. ‘A bead of mass m j located on a parabolic wire with its axi vertical and vertex directed downward as in Fig. 8-34 and ‘whose equation is cz =a, If the coefficient of friction a my find the highest distance above the 2 axis at which the Particle will be in equilibrium Ams. jute Work Problem 2.104 if the parabola is replaced by a vertical cirele of radius 6 which is tangent to the 2 axis. Fig. 3.34 ‘A weight W is suepended from 3 equal strings of length I which are attached to the 8 vertices of @ horizontal equilateral triangle of side s. Find the tensions in the strings. Ans. WUVOR= Bet ‘Work Problem 8.106 if there are n equal strings attached to the m vertices of a regular polygon having 7» sides. CHAP. 3] MOTION IN A UNIFORM FIELD. FALLING BODIES AND PROJECTILES 85 8.108, 30, sant, 3.112, A rope passes over a fixed pulley A of Fig. 3-85. At one end of this rope a mass M, is attached. At the other end of the rope there is a pulley of mass Mz over which passes another rope with masses m, and m, attached. Prove that the accel- eration of the mass mm, is given by BmgMy — miMs ~ mMa ~ mM; Gia mV, + MG) + Arg dong ‘An automobile of weight W with an engine having constant instantaneous power P, travels up an incline of angle «. Assuming that resistance forces are r per unit weight, prove ‘that the maximum speed which can be maintained up the nes 2 incline 88 FEE Fig. 8.5 ‘An automobile of weight W moves up an incline of angle a, powered by an engine having ‘constant instantaneous power ®. Assuming that the resistance to motion is equal to xv per unit ‘weight where v is the instantaneous speed and « is a constant, prove that the maximum speed which is possible on the incline is (VW7aintat GW — W sin a)/24W. {A chain hangs over a smooth peg with length a on one side and length 5, where 0<#-<4, on the weevil Va- ve)” ether sd, Prove that the ine an fo the hn tale of even by [2B n(: Prove that a bead P whichis placed anywhere on a vertical frictionless wire (see Fig. 3-86] in the form of cycloid 2 = We tsing, = BL cose) ‘will reach the bottom in the same time regardless of the starting point and find this time. Ans. 2V8I9 Fig. 336 Chapter 4 The SIMPLE HARMONIC OSCILLATOR and the SIMPLE PENDULUM THE SIMPLE HARMONIC OSCILLATOR » In Fig. 4-1(a) the mass m lies on a friction- less horizontal table indicated by the 2 axis. : Itis attached to one end of a spring of negligible mass and unstretched length ! whose other end is fixed at E. If m is given a displacement along the x axis @ [see Fig. 4-1(b)} and released, it will vibrate or oscillate back and forth about the equilibrium position O. To determine the equation of motion, note 1 — that at any instant when the spring has length fey lhe L+a [Fig. 4-1(b)] there is a force tending tore- SOOO store m to its equilibrium position. According 7 to Hooke's law this force, called the restoring G force, is proportional to the stretch x and is given by Fig. 1 Fy = ai w where the subscript R stands for “restoring force” and where « is the constant of propor- tionality often called the spring constant, elastic constant, stiffness factor or modulus of elasticity and i is the unit veetor in the positive ¢ direction. By Newton’s second law we have nO a asi ae nEtee = 0 o ‘This vibrating system is called a simple harmonic oscillator or linear harmonic oscillator. This type of motion is often called simple harmonic motion. AMPLITUDE, PERIOD AND FREQUENCY OF SIMPLE HARMONIC MOTION If we solve the differential equation (2) subject to the initial conditions #=A and de/dt=0 at t=0, we find that x = Acoset where « = Vm @) For the case where A=20, m=2 and x=8, see Problem 4. Since cos ut varies between —1 and +1, the mass oscillates between x =—A and x= A. A graph of « vs. t appears in Fig. 4-2. 86 CHAP. 4) THE SIMPLE HARMONIC OSCILLATOR AND THE SIMPLE PENDULUM 87 a] ‘The amplitude of the motion is the distance A and is the greatest distance from the equilibrium position. The period of the motion is the time for one complete oscillation or vibration [some- times called a cycle) such as, for example, from x=A to z=—A and then back to 2=A again. If P denotes the period, then P = 2xlo = 2uVmie w The frequency of the motion, denoted by f, is the number of complete oscillations or eycles per unit time. We have zsilseved Spm a © In the general ease, the solution of (2) is 2 = Acoet + Bsinet where » = yal Cy where A and B are determined from initial conditions. As seen in Problem 4.2, we can write (6) in the form 2 = Ceos(wt—9) where » = yam ” and where Cc = VA*+B* and ¢=tan™'(B/A) (8) ‘The amplitude in this case is C while the period and frequency remain the same as in (4) and (6), ie. they are unaffected by change of initial conditions. The angle ¢ is called the phase angle or epoch chosen so that 0S =r. If ¢=0, (7) reduces to (8). ENERGY OF A SIMPLE HARMONIC OSCILLATOR If 7 is the kinetic energy, V the potential energy and E=7+V the total energy of simple harmonic oscillator, then we have T= gmvt, V = eat ) and E = dmo! + dea? (10) See Problem 4.17. THE DAMPED HARMONIC OSCILLATOR In practice various forces may act on a harmonic oscillator, tending to reduce the magnitude of successive oscillations about the equilibrium position. Such forces are some. times called damping forces. A useful approximate damping force is one which is propor- tional to the velocity and is given by F, = pv = -poi = -9 $4 (a) 88 THE SIMPLE HARMONIC OSCILLATOR AND THE SIMPLE PENDULUM (CHAP. 4 where the subscript D stands for “damping force” and where f is a positive constant called the damping coeficient. Note that F, and v are in opposite directions. If in addition to the restoring force we assume the damping force (11), the equation of motion of the harmonic oscillator, now called a damped harmonic oscillator, is given by @: de @z , od mig = ~~ BG or mop t+ eget = 0 (2) on applying Newton's second law. Dividing by m and calling Bim = 2, dm = (as) this equation can be written aero ay where the dots denote, as usual, differentiation with respect to t. OVER-DAMPED, CRITICALLY DAMPED AND UNDER-DAMPED MOTION ‘Three cases arise in obtaining solutions to the differential equation (14). Case 1, Over-damped motion, y*>, ie. > 4xm In this case (14) has the general solution 2 = em(Aet+ Bes) where a = Waa (15) and where the arbitrary constants A and B can be found from the ini Case 2, Critically damped motion, y'= ot, ie. p= 4am In this case (14) has the general solution 2 = "(A + Bt) (18) where A and B are found from initial conditions. Case 3, Under-damped or damped oscillatory motion, y" CHAP. 4] THE SIMPLE HARMONIC OSCILLATOR AND THE SIMPLE PENDULUM 89 between two successive maxima or minima] in the under-damped [or damped oscillatory] motion of Fig. 4-3 is called the period of the motion and is given by P ae Qe Arm (18) and the frequency, which is the reciprocal of the period, is given by cei gy (FF _ Vim # Pe ee (29) Note that if @=0, (18) and (19) reduce to (4) and (5) respectively. The period and frequency corresponding to 6=0 are sometimes called the natural period and natural Frequency respectively. The period P given by (18) is also equal to two successive values of t for which cos (At—4) = 1 [or cos(At—¢) = —I] as given in equation (17). Suppose that the values of « corresponding to the two successive values fy and ty+1= te +P are ty and znss respec tively. ‘Then Balin ‘The quantity 8 = In (xs/tnss) = yP (21) which is a constant, is called the logarithmic decrement. emerath = ee (20) FORCED VIBRATIONS ‘Suppose that in addition to the restoring force ~xri and damping force —évi we impress on the mass m a force F(t)i where F() = Focosat (22) Then the differential equation of motion is Gz = er - pl + Feconat (29) or E+ Ryd + ote = f,cosat (4) where y= Alam, t= alm, fy= Fylm (25) ‘The general solution of (24) is found by adding the general solution of ° (ee) B 4 Dyk + wt [which has already been found and is given by (15), (16) or (17)] to any particular solution of (24). A particular solution of (24) is given by [see Problem 4.18] 7) where (28) Now, as we have seen, the general solution of (26) approaches zero within a short time and we thus call this solution the transient solution. After this time has elapsed, the motion of the mass m is essentially given by (27) which is often called the steady-state solution. ‘The vibrations or oscillations which take place, often called forced vibrations or forced oscillations, have a frequency which is equal to the frequency of the impressed force but lag behind by the phase angle ¢. 90 THE SIMPLE HARMONIC OSCILLATOR AND THE SIMPLE PENDULUM (CHAP. 4 RESONANCE ‘The amplitude of the steady-state oscillation (27) is given by (29) ie (29) assuming y 0, i.e. 6 ~ 0, so that damping is assumed to be present. The maximum value of of in this case occurs where the frequency a/2x of the impressed force is such that oe (20) assuming that y7< Jot [see Problem 4.19]. Near this frequency very large oscillations may occur, sometimes causing damage to the system, The phenomenon is called resonance and the frequency a,/2r is called the frequency of resonance or resonant frequency. ‘The value of the maximum amplitude at the resonant frequency is f Aux = —— (81) Pera ‘The amplitude (29) can be written in terms of ay as a (32) VeF= aa) + A= 7) A graph of of vs. a? is shown in Fig. 4-4. Note that the graph is symmetric around the resonant frequency and that the resonant frequency, frequency with damping and natural frequency (without damping) are all different. In case there is no damping, ie. y=0 or 8 =0, all of these frequencies are identical. In such case resonance occurs where the frequency of the impressed force equals the natural frequency of oscillation. The general solution for this case is t 2 = Acoset + Bsinot + Be sinet (83) From the last term in ($3) it is seen that the oscillations build up with time until finally the spring breaks. See Problem 4.20. enonantfeequeney Froquncy with damping Fig. tt ‘THE SIMPLE PENDULUM A simple pendulum consists of a mass m [Fig. 4-5] at the end of a massless string or rod of Iength 1 [which always remains straight, i.e. rigid). If the mass m, sometimes called the pendulum bob, is pulled aside and released, the resulting motion will be oscillatory. Calling @ the instantaneous angle which the string makes with the vertical, the differential equation of motion is [see Problem 4.23] CHAP. 4) THE SIMPLE HARMONIC OSCILLATOR AND THE SIMPLE PENDULUM 91 ge = -Gsine (84) assuming no damping forces or other external forces are present. For small angles (e.g. less than 5° with the vertical], sin 4 is very nearly equal to 6, where 6 is in radians, and equation ($4) becomes, to a high degree of approximation, a _ 9 -— 1? (85) This equation has the general solution 9 = Acos Vgilt + Bsin Voit (86) where A and B are determined from initial conditions. For example, if @= 4, 9=0 at t=0, then 8 = Bcos gilt (87) In such case, the motion of the pendulum bob is that of simple harmonic motion. The period is given by P = 2nJflig (88) and the frequency is given by 1 fp = gval (69) If the angles are not necessarily small, we can show [see Problems 4.29 and 4.80] that the period is equal to T pra_ao po vi ee g S vi-Psinté . Ty 4 (2) p04 (1:8)! = ayia +() +(ES) i +( where = sin (0/2). For small angles this reduces to (88). For cases where damping and other external forces are considered, see Problems 4.25 and 4.114. (40) THE TWO AND THREE DIMENSIONAL HARMONIC OSCILLATOR ‘Suppose a particle of mass m moves in the zy plane 7 under the influence of a force field F given by F =~ ti ati un) where «, and x, are positive constants. In this case the equations of motion of m are given by @r_ ey mg = a2, mob = uy 2) and have solutions Fig. 4-6 = AjcosVa/mt + B,sinVaJmt, y = A,cosyVnjmt + B,sinyeymt (48) where Ai, Bs, As, Bs are constants to be determined from the initial conditions, The mass m subjected to the force field (41) is often called a two-dimensional harmonic oscillator. The various curves which m describes in its motion are often called Lissajous curves or figures. ‘These ideas are easily extended to a three dimensional harmonic oscillator of mass m which is subject to a force field given by F = ~eti — 60) — tk (4a) where «)*,% are positive constants. 92. ‘THE SIMPLE HARMONIC OSCILLATOR AND THE SIMPLE PENDULUM (CHAP. 4 Solved Problems SIMPLE HARMONIC MOTION AND THE SIMPLE HARMONIC OSCILLATOR 4d. 42. A particle P of mass 2 moves along the axis attracted toward origin O by a force whose magnitude is numerically equal to 8x (see Fig. 4-7). If it is initially at rest at z= 20, find (a) the differential equation and initial conditions describing the motion, (b) the position of the particle at any time, (0) the speed and velocity of the particle at any time, ' asi and (@) the amplitude, period and frequency of the ae vibration, . (@) Let r= a1 be the position vector of P. ‘The acceleration 0; ae De ' ot Pic Set) = S21, The net force acting on P is : “Bei, Then by Newton's second law, @s, ® 22 = act or SE tte 0 Which is the required differential equation of motion. The initial conditions are 2=20, dx/dt=0 at t=0 ® (©) The general solution of (1) is 2 = Acon2t + Bainzt ® When ¢=0, 2=20 sothat A=20. Thos = meos2t + Bain2e ® Then de/dt = —40 sin2e + 20 cone ® 40 that on putting ¢= 0, ds/dé=0 we find B= 0, Thus (8) becomes = 20.cos2t © Which ives the position at any time (0) From (@) da/dt =~40 sin2t which gives the speed at any time, ‘The velocity is given by a 4 40 sine (@) Amplitude = 20. Period = 2r/2 =x. Frequency = I/period = Ws. (a) Show that the function A coset +Bsinet can be written as Ccos(ot—9) where C=A?+B and ¢=tan'(B/A). (b) Find the amplitude, period and frequency of the function in (a). ) Acoust + Bainet = VAPFR(—A— coset + —B— sinet ee (aR Jaa) = VATE ong conat + sn sine!) = VET Bem(et—9) = Combt—9 where cosy = A/VAPFB® and sing = B/VATTB, ie, tang=B/A or ¢=tan-'B/A, and C= VAT+H, We generally choose that value of ¢ which lies between 0° and 180°, ke OS 9am. * () Amplitude = maximum value = C = VATFBR Period = 2x/o. Frequency = w/2z. CHAP. 4) THE SIMPLE HARMONIC OSCILLATOR AND THE SIMPLE PENDULUM 93 43. 44, Work Problem 4.1 if P is initially at x = 20 but is moving (a) to the right with speed 30, (b) to the left with speed 80. Find the amplitude, period and frequency in each case. (@) The only difference here by dz/dt=30 at t Problem 4.1. becomes that the condition dz/dt=0 at t=0 of Problem 4.1 is replaced ‘Then from (5) of Problem 41 we find B=15, and (3) of = 20cos2t + 15 sin 2 @ Which gives the position of P at any time. ‘This may be written [see Problem 42) as 2 = VOOrE eR {Boat + Fett ne} Veo? + 05) Vi20y + (15) 25(4 con2e+ fsin31) = 25cos(2t—9) t @ ‘The angle ¢ which can be found from (2) Is often called the phate angle or epoch. Since the cosine varies between —I and +1, the amplitude = 25. ‘The period and fre- queney are the same as before, Le. period = 2s/2= and frequency = 2/2r = 1s. where cosé = fang (8) Im this case the condition de/dt=0 at 0 of Problem 41 is replaced by da/dt at t=0. ‘Then B= 15 and the position is given by = 20cos2t ~ 15 sin 2¢ which as in part (a) ean be written & = 25($ cos2t— B sin 20) 2B{cosy cos2t + siny sin2t) = 26 eos (2t—y) where cosy = 4, siny =—B. ‘The amplitude, period and frequency are the same as in part (a). The only difference fs in the phase angle, ‘The relationship between y and is y= +s. We often describe this by saying that the two motions are 180° out of phase with each other. ‘A spring of negligible mass, suspended vertically from one end, is stretched a distance of 20 cm when a 5 gm mass is attached to the other end. The spring and mass are placed on a horizontal frictionless table as in Fig. 4-1(a), page 86, with the suspension point fixed at Z. The mass is pulled away a distance 20 cm beyond the equilibrium position O and released. Find (a) the differential equation and initial conditions describing the motion, (b) the position at any time f, and (c) the amplitude, period and frequency of the vibrations. 15(080) dynes = 4900dynes, ‘Then since 4900 dynes stretches the spring 20cm, the spring constant is 45 dynes/em. Thus when the spring is stretched a’ distance 2 em beyond the equilibrium position, the restoring force is —245z1. Then by Newton's second law we have, if r=ai is the position vector of the ms ta) ee sHED = -o4sai or G54 492 = 0 o ‘The initial conditions are x = 20, dzx/dt=0 at t= 0 ® (8) The general solution of (1) is 2 = AcosTt + BsinTt ® Using the conditions (2) we find A= 20, B=0 sothat x= 20 cost (©) From © =20.cos7t we see that: amplitude = 20 em; ps Be! aoe; frequency = T/2z vvibjece oF T/2x eycles/see. 4 46. ‘THE SIMPLE HARMONIC OSCILLATOR AND THE SIMPLE PENDULUM (CHAP. 4 A particle of mass m moves along the « axis, attracted toward a fixed point O on it by a force proportional to the distance from O. Initially the particle is at distance 7, from O and is given a velocity v, away from O. Determine (a) the position at any time, (b) the velocity at any time, and (¢) the amplitude, period, frequency, and maximum speed. (@) The force of attraction toward O is ~xei where « ' posv ontant of propertly. ‘Then by Newton's mGE = —oi or an) Solving (2), we find 2 = Acosvelmt + B sin Vadmet (e) We elso have the initial conditions Figs Hay deldt=ry att ® From =) at ¢=0 we find using (0), that A= ap Thus 2 = mgconValmt + B sin vaTme © to that delat = xy Velmsin Valmet + Batra cos Valmet © From da/dt = vy at ¢=0 we find, using (6), that B= vpVmi/e ‘Thus (4) becomes 2 = 2ycon Velmt + v9 Vmile sin Velmt @) Using Problem 42, this ean be written = = VAT Way cos (Valmet @) o where = tenn! (volte) Vint @ (0) “The velocity is, using (6) oF (7), # Tm sin VaTmit + vp cos Vadim tt = Valen VaEF wae sin (VTE #4 = VET oalym sin (VeTmt— 9) Oy amplitude is given from (7) by Va + mole. From (7), the period is P= 2rVem- The frequency is # = 1/P = 2eVim/m. this speed ia Vag Feagim v= & ( The From (9), the speed is a maximum when sin (Veit ~ ¢) An object of mass 20 kg moves with simple harmonic motion on the « axis. Initially (t=0) it is located at the distance 4 meters away from the origin + =0, and has velocity 15 m/sec and acceleration 100 m/sec* directed toward #=0. Find (a) the position at any time, (b) the amplitude, period and frequency of the oscillations, and (e) the force on the object when f= =/10 sec. (a) If # denotes the position of the object at time f, then the initial conditions are dz/ét = 16, dz/dt = 100 at t=0 o Now for simple harmonic motion, ‘2 = Acosut + Bainot o Differentiating, we find de/dt_ = —Awsinet + Bu cosot ” Boldt = —Ast coset ~ Bot sinat w CHAP. 4] ‘THE SIMPLE HARMONIC OSCILLATOR AND THE SIMPLE PENDULUM 95 4. 49. Using conditions (1) in (2), (8) and (4), we find 4=A, 15 = Bo, —100=—Aw2 Solving simultaneously, we find A= 4, w=65, B=~8 so that 2 = Acos5t ~ 3sin dt © which ean be written, = Beos(Gt—9) where cos =f, sing = —f Cc) () From (6) we sce that: amplitude = 6m, period = 2/6 sec, frequency = 5/2 vib/see. (©) Magnitude of acceleration = d2/dt? = —100 cos 5t +75 sin St = 15 m/sec? at Force on object = (mass)(acceleration) = (20 kg)(7S m/sec?) = 1500 newtons, fo. A 20 Ib wt object suspended from the end of a ver- tical spring of negligible mass stretches it 6 inches. LLMs (a) Determine the position of the object at any time if initially it is pulled down 2 inches and then re- leased. (b) Find the amplitude, period and fre- quency of the motion. (a) Let D and E (Fig. 49] represent the position of the fend of the spring before and after the object is put on on the spring. Position H is the equilibrium position of the object. Choose a coordinate system as shown in Fig. 49 s0 that the positive 2 axis is downward with origin at the equilibrium position, By Hooke's law, since 20 Ib wt stretches the spring Jt, 40 Ibwt stretches it 1 ft; then 40(5 +2) Ibwt stretches it (5+2)ft. Thus when the object is at position F there is an upward force acting on it of ‘magnitude 40(5++ 2) and a downward force due to its weight of magnitude 20. By Newton's second law we thus have BE = omarion or Miso = 0 savin, f= Acces + Binet Now at €=0, #= 5 and defdt=0; thus A= j, B= 0 and sm beat ® () From (¢): amplitude = § ft, period = 24/8 = x/4 sec, frequency = 4/= vib/see. Work Problem 4.7 if initially the object is pulled down 3 inches (instead of 2 inches) and then given an initial velocity of 2 ft/sec downward. In this ease the solution (1) of Problem 4.7 still holds but the initial conditions are: at == and di/dt=2, From these we find A=} and B={, s0 that = = feos8t + pains = EA com (8 #14) ‘Thus amplitude = VI/4 ft, period = 25/8 = 2/4 and frequency are unaffected by changing the init , frequency = 4/r vib/aee. Note that the period 1 condi A particle travels with uniform angular speed » around a circle of radius b. Prove that its projection on a diameter oscillates with simple harmonic motion of period 2n/a about the center. ‘Choose the cirele in the zy plane with center at the origin O aa in Fig. 4-10 below. Let @ be ‘the projection of particle P on diameter AB chosen along the # axis. 96 THE 4.10. ‘THE SIMPLE HARMONIC OSCILLATOR AND THE SIMPLE PENDULUM [CHAP. 4 If the particle is initially at B, then in time ¢ we will have 2BOP=#= ot. Then the position of P at time ¢ is F = deosutit bsineti a ‘The projection @ of P on the » axis is at distance ris 2 = beoset from O at any time t. From (2) we see that the projection Q oscillates with simple harmonic motion of period 2/2 about the center O. Fig. 4-10 DAMPED HARMONIC OSCILLATOR ‘Suppose that in Problem 4.1 the particle P has also a damping force whose magnitude is numerically equal to 8 times the instantaneous speed. Find (a) the position and (b) the velocity of the particle at any time. (c) Ilustrate graphically the position of the particle as a function of time ¢. y (a) In this case the net force acting on P is Fig. 4-11] ~8:i 898i. Then by Newton's sec- ond law, ey taal = at "P @2, yee on Peat ae = 0 Fig t-it ‘This has the solution [see Appendix, page 852, Problem C.14] 2 = eA + BH When t=0, 2=20 and do/dt=0; thus A=20, B=40, and 2= 200 the position at any time & (+20 gives (b) The velocity is given by ae Sei = sore 24 (©) The graph of 2 vs. tis shown in Fig. 412, Tt is seen that the motion is non-oscillatory. ‘The par- tiele approaches O slowly but never reaches it. ‘This is an example where the motion is eritically damped. Fig. 412 A particle of mass 5 gm moves along the « axis under the influence of two forees: (i) a force of attraction to origin O which in dynes is numerically equal to 40 times the instantaneous distance from O, and (ii) a damping force proportional to the instantaneous speed such that when the speed is 10 cm/sec the damping foree is 200 dynes. Assuming that the particle starts from rest at a distance 20cm from O, (a) set up the differential equation and conditions describing the motion, (b) find the position of the particle at any time, (c) determine the amplitude, period and frequeney of the damped oscillations, and (d) graph the motion. (@) Let the position vector of the particle P be denoted by = al as indicated in Fig. 4-18, Then the force of attr ton (directed toward 0) is Taotierani 02 ny ‘The magnitude of the damping force f is proportional to at the speed, 40 that. f= idz/dé where @ is constant. “Then | 2} ——e since / = 200 when d2/dt=10, we have @=20 and = —[ ng J =20deldt. To get f, note that when dz/dé>0 and P 2 >0 the particle is on the positive x axis and moving to Fig. 4418 CHAP. 4) THE SIMPLE HARMONIC OSCILLATOR AND THE SIMPLE PENDULUM 97 the right, ‘Thus the resistance force must be directed toward the left, This ean only be secom- plished if de t= -20%f ” ‘This same form for f is easily shown to he correct if z>0, di/dt<0, x <0, de/dt>0, £<0, dafdt <0 {sce Problem 4.45). Hence by Newton's second law we have te ae, sTei = -20%8 — 4001 ® Pe, ye or Ser ste = 0 w Since the particle starts from rest at 20 em from 0, we have 20, dx/dt=0 at t=0 © ‘where we have assumed that the particle starts on the positive le of the 2 axis [we could just ag well assume that the particle starts on the negative side, in which ease z= —20). (8) 2 = eat is a solution of (6) if @tdet8 = 0 or «= de Vien 8) = 222i ‘Then the general solution is 2 = eA con2t + B sin 2) © Since 2=20 at t=0, we find from (6) that A = 20, i, 2 = 67420 cos2t + B sin 2) o ‘Thus by differentiation, alae = (e240 sin 2¢ + 2B e08 24) + (~2¢~%)(20 cos 2t + B sin 2) © Since dz/dt=0 at ¢=0, we have from (8), B= 20. Thus from (7) we obtain x = 206-(cos 2¢ + sin 2t) = 20VBe-* cos (2t ~ =/4) © using Problem 42. (c) From (9): amplitude = 20yZe-em, period = 2e/2 =r see, frequency = 1/s vib/see. (@) ‘The graph is shown in Fig. 4-14, Note that the amplitudes of the oscillation decrease toward zero as ¢ increases, T freer} Fig. 414 4.12. Find the logarithmic decrement in Problem 4-11. Method 1. ‘The maxima (or minima) of # occur where dz/dt = 0, Prom (9) of Problem 4.11, dafdt = —80e-* sin at = 0 when t= 0,=/2,2,89/2,22,52/2,.... ‘The maxima occur when ¢=0,7,2x,...; the minima ‘occur when t= +/2,8x/2, 9/2, -.- ‘The ratio of two successive maxima is ¢-M®/e-%0 or e-%/e-229, ete, Le, o®, Then the logarithmic decrement is 8 = In (e*) = 2r. 98 4.13, 44, 415. ‘THE SIMPLE HARMONIC OSCILLATOR AND THE SIMPLE PENDULUM [CHAP. 4 Method 2. From (9) of Problem 4.11, the difference between two successive values of t, denoted by ty and y41, for which cos (2¢~#/4) = 1 (or 1) is , which is the period, ‘Then 20v2 aa aias Or and 8 = Ineyfegys) = Be Method 8. From (19), (18) and (20), pages 88 and 88, we have dem anf a= we = (2) (or) Fae = 3) Vien = Then 20, «=40 [Problem 4.11, equation (9), 5 Determine the natural period and frequency of the particle of Problem 4.11. ‘The natural period is the period when there is no damping. In such ease the motion is sriven by removing the term involving di/dt in equation (3) or (4) of Problem 4.11. ‘Thus Prd + Be = 0 or = = AcosavEt + Bsin2VFe ‘Then: natural period = 2-/2VF sec = x/VZ sec; natural frequency = V3/x vib/see. For what range of values of the damping constant in Problem 4.11 will the motion be (a) overdamped, (b) underdamped or damped oscillatory, (c) critically damped? Denoting the damping constant by f, equation (8) of Problem 4.11 is replaced by ae dey or Me 4 Bae $24 — dori Sts bE tee = 0 ‘Then the motion is: (a) Overdamped if (/5}* > 82, ie. p> 20y2. (©) Underdamped if (8/5)? < 82, Le. p< 2073. [Note that this is the case for Problem 4.11 where = 20.) 2oy2. (©) Critically damped if (A/5)2 = 82, Le. Solve Problem 4.7 taking into account an external damping force given numerically in Ib wt by fv where v is the instantaneous speed in ft/sec and (a) £=8, (b) = 10, (0) p= 12. ‘The equation of motion is 20 ate ~ ade 52H. oe = wey — aon — anoren pik or Gates oe = 0 (a) If p=8, then ds/dt? + 128de/dt+ 642 =0. ‘The solution 2 = e-M(A cos 4.8t + B sin 4.81) 1/6, defdt=0 at 1=0, we find A= 1/6, B= 2/9 so that, Using the conditions = fgets conse + 4 sin g.8t) ‘The motion is damped oscillatory with period 25/4.8 = e/12 sec. 5 5 64t cos (4.8¢ —58°8)) Be (48t—58°8) (®) If #=10, then da/de? + 16deldt + 642 ©. ‘The solution is emia + BO) ye then Solving subject to the i gore tan. CHAP. 4) ‘THE SIMPLE HARMONIC OSCILLATOR AND THE SIMPLE PENDULUM 99 ‘The motion is critically damped since any decrease in p would produce oscillatory motion. (© If p=126 then as/dt® + 20de/de + 642 = 0. The solution is a= Aes Bet Solving subject to initial condi ives A= 1/6, B= —1/24; then 2 = ‘The motion is overdamped. ENERGY OF A SIMPLE HARMONIC OSCILLATOR 4.16. (a) Prove that the foree F=—szi acting on a simple harmonic oscillator is con- servative. (b) Find the potential energy of a simple harmonic oscillator. i yk (@) Wehave ¥xE = | 2/82 aay afaz| = 0 so that F is conservative, mr 0 0 (he per pein hen WV whe B= PY ot ey ae a met = — (Eee te) ‘Then aV/ex =x, V/dy=0, aV/02 responding to 2 =0, we find from which V ao that V = feat, yer? be, Assuming V=0 com 4.17. Express in symbols the principle of conservation of energy for a simple harmonic oscillator. By Problem 4.16(6), we have Kinetic energy + Potential energy = Total energy or fmt get = 5 which ean also be written, since v= dz/dt, as 4m(dz/dt?+ feat = BL ‘Another method. The differential equation for the motion of jimple harmonic oscillator is m@eld = ex ‘Since dz/dt = v, this can also be written as deg de de, mot =, s or m= Ws, ie, mogd = —ae Integration yields jo + fea? = EL FORCED VIBRATIONS AND RESONANCE 4.18. Derive the steady-state solution (27) corresponding to the differential equation (24) on page 89. ‘The differential equation ty, 9s E+ ays ote = fycosat w Consider particular solution having the form e = ecosat + sina ® 100 ‘THB SIMPLE HARMONIC OSCILLATOR AND THE SIMPLE PENDULUM (CHAP. 4 where o, and cy are to be determined. Substituting (2) into (1), we find (Cate, + Brees + wtey) conat + (—atey — 2yae; + u%ey) sinat = fy cosat {rom which matey + Byace + whey = fo, whey 2ymey + eg ® or (at — toy — 2yaey = —fo, Brae, + (a? ate, = 0 6) Solving these simultaneously, we find fale? = at) Bove 0° Saar ae > Bare o ‘Thus (2) becomes Jollet ~e!)cosat + 2ya sin at] (a= at}? + aya 2 Now by Problem 4.2, page 92, (?— a) cosat + 2yasinat = V(u?— al}? + dyFa? cos (at — 9) o where tang = 2ya/(a*—w?), 0S ¢ 57. Using (7) in (6), we find as required tnt Vea att ae # 4.19. Prove (a) that the amplitude in Problem 4.18 is a maximum where the resonant fre- quency is determined from «= /u?—2y' and (b) that the value of this maximum amplitude is f/(2yVa?— 7). Method 1. The amplitude in Problem 4.18 is LIVE AP RE a It is a maximum when the denominator [or the square of the denominator] is a minimum. ‘To find this minimum, write (ot oA + pte = at — But —2yA)at + ot at — Bo BME + (8 2yPP + at — (ot — By [at — (ot 298) + Aye? — 7) ‘This fs a minimum where the first term on the last line is zero, ie. when a? value is then 4y%(4!—7!), ‘Thus the value of the maximum amplitude is HaWe=~. Method 2. The function U = (ew)! + 4y4 has a minimum or maximum when Wa awit oe = 0 oe oem = 0 0 VHB where 0, Thus a= Ve=2y? gives the minimum value. 420. (a) Obtain the solution (33), page 90, for the case where there is no damping and the impressed frequency is equal to the natural frequency of the oscillation. (b) Give a physical interpretation. (a) The case to be considered is obtained by putting 7=0 or # = (28) or (24), page 89. We thus must solve the equation Beate = fycosut o and «=. in equations CHAP. 4) ‘THE SIMPLE HARMONIC OSCILLATOR AND THE SIMPLE PENDULUM 101 ‘To find the general solution of this equation we add the general solution of Btot = 0 @ to a particular solution of (1). Now the general solution of (2) is 2 = Acosut + Bsinot cH ‘To find a particular solution of (1) it would do no good to assume a particular solution of the form fe = cyeosut + ey sin at ® since when we substitute (4) [which is identical in form to ()] into the left side of (1), we would got zero. "We must therefore modify the form of the assumed particular solution (4). As seen in Appendix C, the assumed particular solution has the form 2 = tle, covet + ey sinet) “6 ‘To see that this yields the required particular solution, let us differentiate (6) to obtain B= ten, sinot + ve, coset) + (cy cost + ey sin vt) o He we, coat — ay sin wt) + 2(— 26) sin ot + aes c08 w!) ” Substituting (6), (6) and (7) into (2), we find after simplifying —2we, sin ut + Beep coset = fa coset from which ¢ ‘quired particular solution (6) is ‘The general solution of (1) is therefore x = Acosut + Bainat + fol2s)tsinut (6) © and y= fol2s. ‘Thus the re- (u/2a)t sin st. (6) The constants A and B in (8) are determined from the initial conditions. Unlike the case with damping, the terms involving A and B do not become small with time, However, the last term involving ¢ increases with time to such an extent that the spring will finally break. A graph of the last term shown in Pig, 4-16 indicates how the oscillations build up jn magnitude. 0 a force hangs in jon of the 421. A vertical spring has a stiffness factor equal to 8 Ibwtper ft. At given in Ibwt by F(t) = 12sin4t, t= 0 is applied to a 6b weight whi equilibrium at the end of the spring. Neglecting damping, find the posi weight at any later time t. Using the method of Problem 4.7, we have by Newton's second Iw, oe SSE = we + sina ee or Foe 16s = Ot sinae o Solving, 2 = Acosdt + Bsint — 8 cosdr then A=0, B=2 and 4t — Bt con de ® [As ¢ gets larger the term —8¢ cos4t increases numerically without bound, and physically the spring will ultimately break, The example illustrates the phenomenon of resonance. Note that the natural frequency of the spring (4/2 ~2/r) equals the frequency of the impressed force, 102 422. Work Problem 4.21 if F(t) = 80 cos6t, ¢ THE SIMPLE HARMONIC OSCILLATOR AND THE SIMPLE PENDULUM In this case the equation (1) of Problem 4.21 becomes Pld + 162 = 160 cost fand the initial conditions are =0, defdt=0 at t=0 ‘The general solution of (2) is A cost + B sin4t ~ 8 cost Using conditions (2) in (8), we find A=8, B=0 and = = Sleosdt—cosst) = 8(cos(St—1) — cos (6t+#)) = 16sint sinSt (CHAP. 4 © ® ® ‘The graph of 2 vs. t is shown by the heavy curve of Fig. 4-16, The dashed curves are the curves £16 sin t obtained by placing sin ‘#1. If we consider that 16 sin¢ is the amplitude of sin t, we see that the amplitude varies sinusoidally. ‘The phenomenon is known as amplitude ‘modulation and is of practical importance in communications and electronics. THE SIMPLE PENDULUM 428, Determine the motion of a simple pendulum of length J and mass m assuming small vibrations and no resisting forces. Let the position of m at any time be determined by 4, the arclength measured from the equilibrium position O [seo Fig. 4-17]. Let ¢ be the angle made by the pendulum string with the vertical. If T is a unit tangent vector to the circular path of the pendulum bob m, then by Newton's second law Gir = —mosino® o + since my. Fh = —Ssine ® For small vibrations we can replace sin # by # 0 that to a high degree of accuracy equation (2) can be replaced by ae Teste = 0 ® which has solution Fig. 47 @ = Acos Veit + Bsinvalt king as 4 = syconvalt From this we see that the period of the pendulum is 2eVi/g. Initial conditions @=%, de/dt=0 st t=0, we find A=6, B=0 and so CHAP. 4) THE SIMPLE HARMONIC OSCILLATOR AND THE SIMPLE PENDULUM 103 424. Show how to obtain the equation (2) for the pendulum of Problem 4.23 by using the principle of conservation of energy. We see from Fig, 4-17 that OA = OC ~AC = 1—1-coss = I(L— cos). ‘Then by the conserva. tion of energy [taking the reference level for the potential energy as a horizontal plane through the lowest point 0] we have Potential energy at B + Kinetic energy at 2 = Total energy = E = constant mgl(d — cose) + 4m(daldt? = FE a Since # 1, this becomes mgl(l ~ cos) + fmbdo/dey® = B ® Differentiating both sides of (2) with respect to f, we find mmglsine 6 + mPbS = 0 or T+ (sine = 0 {in agreement with equation (2) of Problem 4.23. 425, Work Problem 4.23 if a damping force proportional to the instantaneous velocity is taken into account. In this case the equation of motion (1) of Problem 4.23 is replaced by ae, Gt = ging - mmg sino — pr or 2 = -gaine- 2H ing sine by # for small vibrations, this becomes fo dey Get ktete m dt ° ‘Three eases arise: Cave 1. pe/Am? < g/t 0 = ePVIM(A copat + Bainut) where » = Voll F/Em® ‘This i the case of damped oscillations or underdamped motion Case 2. pt/Am® = g/t = eo Pm(A + BH ‘This is the case of critically damped motion. Case 8. pt/Am® > oft 9 = emvIm(Ae + Be“) where = VEFZAm® = gil ‘This is the case of overdamped motion In each ease the constants A and B can be determined from the initial conditions. In Case 1 there are continually decreasing oscillations. In Cases 2 and 3 the pendulum bob gradually returns to the equilibrium position without oscillation. ‘THE TWO AND THREE DIMENSIONAL HARMONIC OSCILLATOR 4.26. Find the potential energy for (a) the two dimensional and (b) the three dimensional harmonic oscillator. (a) In this case the force is given by FS mest ey Since VX¥ = 0, the force field is conservative. Thus a potential does exist, ic. there exists fa function V such that F = —VV. We thus have Fos aia) = VV = ova ae! oy 104 427. THE SIMPLE HARMONIC OSCILLATOR AND THE SIMPLE PENDULUM (CHAP. 4 from which aV/0x = ey2, 0V/0y = xy, 8V/32=0 oF V = geet + few? ‘choosing the arbitrary additive constant to be zero. This is the required potential energy. (6) Im this case we have F = —rjxi—sarj—sp2k which is also conservative since VXI =0, We then find as in part (a), aV/oz=n2, aV/0y = nxy, aV/az nye from which the required potential energy is V = deat + deat + daw? A particle moves in the zy plane in a force field given by F = —ezi—xyj. Prove that in general it will move in an elliptical path. If the particle has mass m, its equation of motion is eat = ay o or, since r= 2ityi, ® Pe Then GE = —ex, ‘These equations have solutions given respectively by Ay cos VaTnt + Ay sinVeTmt, y= BycosVelmt + BysinValmt (8) © the particle is located at the point whose position vector is Let us suppose that at Fea Pnd more with wlahy ne eh oh Using ee eedene ae Sed B,=b, Ay=viVmkk, By = vyVmin and so ee ee o where ‘uVinkk, d= vyViijx. Solving for sinut and cosut in (4) we find, if ad + be, wet = SE, nat = TE Savaring and adding, wing the fact that covet + alt=, we ad (dx — ey)? + (ay — bx)? = (ad — be)® . (24 ae — aed abaya ey? = (od be? © Now the equation At + Bry + Cy =D — where A>0,C>0,D>0 is an ellipse if B2—44C <0, «parabola To determine what (5) is, we see that A Bea 4A we, B ©, and a hyperbola if B*—4AC > 0. Bed +ab), C= a? +e? so that BE 4AC = Aled + aby? — 4(08+ aa +02) = ~Mad—bo)2 < 0 provided ad # de. Thus in general the path is an ellipse, and if A=C it is a circle, If ad=be the ellipse reduces to the straight line ay = br MISCELLANEOUS PROBLEMS 428, A cylinder having axis vertical floats in a liquid of density ¢. It is pushed down slightly and released. Find the period of the oscillation if the cylinder has weight W and cross sectional area A. CHAP, 4) THE SIMPLE HARMONIC OSCILLATOR AND THE SIMPLE PENDULUM 105 Let RS, the equilibrium position of the eslinder, be distant, + from the liquid surface PQ at any time t. By Archimedes! principle, the buoyant force on the eylinder is (Az)o. Then by Newton's second Taw, . Fog we Solving, = creonV@AalWt + ein Vonalt and the period of the oscillation is 2eVW7aAs. Fig. 8 429. Show that if the assumption of small vibrations is not made, then the period of a simple pendulum is ail” a ‘The equation of motion for a si [equation (84), page 91] = —fsine Let de/dt =u. Then fe _ du a ~ i and (1) Becomes, ) Integrating (2) we obtain . B= Leste Now when @= 4, «=0 0 that ¢= (9/9 conty, Thus (8) can be written 2 = @olt\eone — cons) or delat = =VBllfeond = wanes) ® If we restrict ourselves to that part of the motion where the bob goes from #= 4 to @=0, which represents a time equal to one fourth of the period, then we must use the minus sign in (4) zo that it becomes delat = — V{(2gli\{eos# = cos 6) Separating the variables and integrating, we have 7 as ce —— VeS Taam Since t=0 at @=4 and ¢ , where P is the period, © Making use of the trigonometric identity cos# = 2 sin®(¢/2)—1, with a similar one replacing @ by oy (6) can be written Toe ee NS aan am : Now let sin (0/2) = sin (0/2) sin g o 106 430. 431. THE SIMPLE HARMONIC OSCILLATOR AND THE SIMPLE PENDULUM (CHAP. 4 ‘Then taking the differential of both sides, eon 9/2) de = sin (0/2) cos 9 ds fo calling & = sin (¢4/2), (40/2) cos 9 dg Vi- Paints 0; and when Also from (7) we see that when # fas required, 2/2 Hence (8) becomes, OS ta ° Note that if we have emall vibrations, ic. if k is equal to zero very nearly, then the period (8) o ao ee . oo ‘The integral in (8) is called an elliptic integral and cannot be evaluated exactly in terms of elementary function, The equation of motion of the pendulum ean be solved for ¢ in terms of elliptic funetions which are generalizations of the trigonometric functions. Show that pel 2 avita{. +(3ye+ Gaye +( The binomial theorem states that if |e| <1, then y= PN ye 4 MP=WP=D yo 4... (tae = 1+ pe + MBEDee 4 APRN Ds 1d given in Problem 4.29 can be written as If p=~4, this can be written gta = tobe dda a8 at ta — gae™ * Letting »=—K aint and integrating from 0 to #/2, we find Pn OS wif fi + datamte + ESe = evn {i + (3)'e + (Gye + (E where we have made use of the integration formula Je sin 9 dp = ‘The term by term integration possible since kj <1. ‘A bead of mass m is constrained to move on a frictionless wire in the shape of a eycloid [Fig. 4-19 below] whose parametric equations are a(g — sing), y = a(1 — cos¢) @) which lies in a vertical plane. If the bead starts from rest at point 0, (a) find the speed at the bottom of the path and (b) show that the bead performs oscillations with period equivalent to that of a simple pendulum of length 4a. CHAP, 4] THE SIMPLE HARMONIC OSCILLATOR AND THE SIMPLE PENDULUM 107 4.82, A particle of mass m is placed on the inside (@) Let P be the position of the bead at any time ¢ and let s be the arclength along the eycloid meas- lured from point 0. By the conservation of energy, measuring potential energy relative to line AB through the minimum point of the eyelofd, we have PBatP + KEatP = PE,atO + KE. atO ‘mg(2a~y)+ drm(de/at)? = mg(2a)+0 (2) Thus vt = (da/dt® = 2gy or v = deldt = Vigy Oy V20(a) = 2V G0. At the lowest point y= 2a the speed is () From part (a), (de/de)? = 2oy. But (ds/at}t = (delat? + (dylaty® = aX — cos 9)%4* + a? sint g g* = 2a%(1 — cos g)$# ‘Then 2a%(1 — cos)? = 2ga(1—coss) or $?=g/a. Thus dyldt = Vola and @ = Volat+ey w When ¢=0, ¢=0; when $= 2, t=/2 where P is the period. Hence from the second ‘equation of (4), ae ae ‘and the period is the same as that of a simple pendulum of length 1 4a, For some interesting applications see Problems 4.86-4.88. of a smooth paraboloid of revolution having equation cz =2*+y* at a point P which is at height H above the horizontal assumed as the zy plane]. Assuming that the particle starts from rest, (a) find the speed with which it reaches the vertex O, (b) find the time + taken, and (¢) find the period for small vibrations. It is convenient to choose the point P in the ys plane so that 2=0 and ez =y2. By the principle of conservation of energy we have if Q is any point on the path PQO, PEatP + KEatP = PEat@ + KEat@ moh + 4m(oyt mgt + fmnda/atyt where ¢ is the arelength along OPQ measured from 0. Thus (aajan = 2g(tt- 2) a or astét = - y2qH=3) ® using the negative sign since # is decreasing with t. (0) Putting ), we see that the speed is V3gH at the vertex. () We have, since 2=0 and es =¥%, (ay = (G+ @)'+ GY = @Y+FQy - 0+ V8) Tho) em bm writen + MGV = Rt 19. Then vee = oe de eR on ww = ay 108 © THE SIMPLE HARMONIC OSCILLATOR AND THE SIMPLE PENDULUM [CHAP. 4 Integrating, using the fact that += and thus y= Vel at ¢ we have 0 while att = elrer—~—sS=— 0 is constant, If at ¢=0, 2=0 and de/dt=0, find (a) = as a function of ¢, (6) the period of the external force for which resonance occurs Ans. (a) #= (Bsinat—de sin 2/6—e) if oA & () ©=2 oF period sin2t—2t cost if o=2 456. A. vertical spring having constant 17 Ibwt per ft has a 321b weight suspended from it. An external foree given as a function of time t by F(t) =65 sin4t, ¢=0 is applied. A damping force given numerically in Ib wt by 2v, where v is the instantaneous speed of the weight in ft is assumed to act. Initially the weight is at rest at the equilibrium position. (a) Determine position of the weight at any time. (b) Indicate the transient and steady-state solutions, giving physical interpretations of each. (c) Find the amplitude, period and frequency of the steady-state solution, (Use g = 82 ft/sec!) Ana, (a) # = 4e~t con t+ ain 4t — 4 con at (6) Transient, 4e~t cos dt; steady-state, sin 4t—4 con dt (o) Amplitude = IT ft, period = +/2 sec, frequency fm vibysce 451. A spring is stretched 5 em by a force of 50 dynes, A mass of 10 gm is placed on the lower end fof the spring. After equilibrium has been reached, the upper end of the spring is moved up and down so that the external force acting on the mass is given by F(t) = 20 cost, t= 0. (@) Find the position of the mass at any time, measured from its equilibrium position. (6) Find the value of w for which resonance occurs. Ama. (a) # = (20 cos uf)/(1~ 2) —20 cost, (0) w= 1 458, A periodic external force acts on a 6 kg mass suspended from the lower end of a vertical spring hhaving constant 150 newtons/meter. ‘The damping force is proportional to the instantaneous speed of the mass and is 80 newtons when the speod is 2meters/sec. Find the frequency at which resonance occurs. Ans. 6/62 vib/sec ‘THE SIMPLE PENDULUM 459, Find the length of a simple pendulum whose period is 1 second. Such a pendulum which registers seconds is called a eeconde pendulum. Ana, 99.8 em or 3.96 ft 460, Will a pendulum which registers seconds at one location lose or gain time when it is moved to another location where the acceleration due to gravity is greater? Explai An. Gain time 461, A simple pendulum whose length is 2 meters has its bob drawn to one side until the string makes fan angle of 30° with the vertical. The bob is then released. (a) What is the speed of the bob ft passes through ite lowest point? (b) What is the angular speed at the lowest point? (c) What is the maximum acceleration and where does it occur’ Ans. (a) 2.98 m/sec, (b) 146 rad/sec, (c) 2 m/sec CHAP. 4] THE SIMPLE HARMONIC OSCILLATOR AND THE SIMPLE PENDULUM SEEN 482. Prove that the tension in the string of a vertical simple pendulum of length 1 and mass m is given by mg cos# where @ is the instantanoous angle made by the string with the vertical, 463. A seconds pendulum which gives correct time at a certain location is taken to another location ‘where itis found to lose seconds per day. Determine the gravitational acceleration at the second location. Ans. g(1~ 7/86,400)? where is the gravitational acceleration at the first location 464, What is the length of a seconds pendulum on the surface of the moon where the acecleration due to gravity is approximately 1/6 that on the earth? Ans, 16.5 cm 465. A simple pendolum of length 1 and mass m hangs vertically from a fixed point O. ‘The bob is given ‘an inital horizontal velocity of magnitude 2. Prove that the are through which the bob swings in one period has a Tength given by 4l cas~*(1— 9f/2¢0) Find the minimum value of vy in Problem 4.65 in order that the bob will make a complete vertical circle with center at 0. Ans, 2Vgi. THE TWO AND THREE DIMENSIONAL HARMONIC OSCILLATOR 467. A particle of mass 2 moves in the zy plane attracted to the origin with a force given by F=-18ri—50yj. At ¢=0 the particle is placed at the point (3,4) and given a velocity of magnitude 10 in a direction perpendicular to the axis. (a) Find the position and velocity of the particle at any time. (b) What curve does the particle describe? Ans. (a) r= 8 cos3t i+ [4 cos 5t-+2 sin t]j, v= ~9 ain S¢ i+ [10 cos 5¢~ 20 sin tj 468. Find the total energy of the particle of Problem 4.67. Ane. 581 469, A two dimensional harmonic oscillator of mass 2 has potential energy given by V = 8(224+4y?). If the position vector and velocity of the oscillator at time t=O are given respectively by r= 2i—j and vo= 41+8), (a) find its position and velocity at any time #>0 and (b) deter- ‘mine the period of the motion. ‘Ans, (a) 1 = (2 cos dt +sin 401+ (sin 8t — cos 85, @) e/8 (cos 448 sin 48+ (8 cos 8t-+8 sin 82) 410, Work Problem 4.69 if V=8(22+2y2). Is there a period defined for the motion in this ease? Explain. 471, A particle of mass m moves in a 3 dimensional force field whose potential is given by V = |a(z* + 4y? +163"), (@) Prove that if the particle is placed at an arbitrary point in space other than the origin, then it will return to the point after some period of time. Determine this ‘time. (2) Is the velocity on returning to the starting point the same as the initial velocity? Explain, 472 Suppose that in Problem 4.71 the potential is V = Je(s?+2y?4 52%), Will the particle return to the starting point? Explain, ‘MISCELLANEOUS PROBLEMS. 413, A vertical spring of constant « having natural length Jis supported at a fixed point A. A mass m is placed at the lower end of the spring, lifted to a height h below A and dropped. Prove that the lowest point reached will be at a distance below A given by I-+mgle+Vm'g%@+ Bnghle. 474, Work Problem 4.78 if damping proportional to the instantaneous velocity is taken into account. 4.75 Given the equation m+ Az +.x = 0 for damped oscillations of a harmonic oscillator. Prove that it B= mit gest, then B= —p3t, Thus show that if there is damping the total energy 2 decreases with time. What happens to the energy lost?” Explain. 2 416, an. 478, an, 480, ast, 492, 433, 435, 436, ‘THE SIMPLE HARMONIC OSCILLATOR AND THE SIMPLE PENDULUM (CHAP. 4 ee ina where A ‘Ax sings + Azsings’ i Ay cos tae). (b) Use (@) to demonstrate that the sum of two simple harmonic motions of the same frequency ‘and in the same straight line is simple harmonic of the same frequency. Give a vector interpretation to the results of Problem 4.76. Discuss Problem 4.76 in case the frequencies of the two simple harmonic motions are not equal. Is the resultant motion simple harmonic? Justify your answer. ‘A particle oscillates in a plane o that ite distances x and y from two mutually perpendicular faxes are given as functions of time £ by 2 = Acos(atts), y = Beoslet +e) (a) Prove that the particle moves in an ellipse inserihed in the rectangle defined by = y = =B. (b) Prove that the period of the particle in the elliptical path is 2x/s. ‘Suppose that the particle of Problem 4:79 moves so that 2 = Acos(att oy), y= Beos(ut+ett on) here « is assumed to be a positive constant which Is assumed to be much smaller than « Prove That the particle oscillates in slowly rotating ellipses inscribed in the rectangle #= =A, y = =B. Mlustrate Problem 4.80 by graphing the motion of a particle which moves in the path = Boos (2tta/t), y= 40824) In Fig, 4-21 2 mass m which is on a frictionless table is connected to fixed points A and B by thro springs of eaual navaral length, of neg PN ns ible Das and opring constants 6, and iq Te rato [= vit] , mls dlplaced horizontally : Prove that the period of ‘oscillation is given by P= 2xVmiln +). |A spring having constant « and negligible mass has ‘one end fixed at point A on an inclined plane of fangle « and a mass m at the other end, as indicated m in Pig. ¢-22, "If the mass m is pulled down a distance i below the equilibrium position and released, find the displacement from the equilibrium position at any time if (a) the incline is frictionless, (2) the incline hhas coefficient of friction 1. Fig. 422 [At times ty, 2f and ty it is ‘i _ Arto located at = a,b and ¢ respectively. Prove that the period of oscillation is ——T(q + o/b" |A particle moves with simple harmonic motion slong the # axis 'A seconds pendulum giving the correct time at one location is taken to another location where fh ivocs B rainutes per day. By how much must the pendulum rod be lengthened or shortened in order to give the correct time? |A. vertical pendulum having a bob of mass m is sus- ° pended from the fixed point 0. As it oscillates, the String winds up on the constraint curves ODA. [or OC] y a indicated in Fig. 4-28. Prove that if curve ABC is a {ycloid, then the Period of oscillation will be the same A regardless of the amplitade of the oscillations. The pen- Gulum in this case is called a oyeloidal pendulum. The B curves ODA and OC are constructed to be evoluter of tthe eyeloid, (Hint, Use Problem 4.31,] Fig. 428 CHAP. 4] THE SIMPLE HARMONIC OSCILLATOR AND THE SIMPLE PENDULUM 113 47, 38, 498 495, 4.96, 491, 498. 49, 4.100. 4.01, ‘A bead slides down a frictionless wire located in a vertical plane. It is desired to find the shape of the wire so that regardless of where the bead is placed on the wire it will slide under the influence of gravity to the bottom of the wire in the same time. This is often called the tautochrone problem. Prove that the wire must have the shape of a cycloid, [Hine, Use Problem 4.31,] Prove that the curves ODA and OC of Problem 486 are cycloids having the same shape as the eyeloid ABC. A simple pendulum of length { has its point of support moving back and forth on a horizontal line 40 that its distance from a fixed point on the line ig A sinwt, t= 0, Find the position of the pendulum bob at any time ¢ assuming that it is at rest at the equilibrium position at t= Work Problem 4.89 if the point of support moves vertically instead of horizontally and if at 4=0 the rod of the pendulum makes an angle ¢ with the vertical. A particle of mass m moves ina plane under the influence of forces of attraction toward fixed points which are directly proportional to ite instantaneous distance from these points. Prove that in general the particle will describe an ellipse. A vertical elastic spring of negligible weight and having its upper end fixed, carries a weight W at its lower end. ‘The weight is lifted so that the tension in the apring is zero, and then released, Prove that the tension in the spring will not exceed 217. A vertical spring having: constant « has @ pan on top of it with a weight W on it [see Fig. 4-24]. Determine the largest fre- quency with which the spring can vibrate so that the weight ‘will remain in the pan, A spring has @ natural length of 50 em and a force of 100 dynes is required to stretch it 25 em. Find the work done in stretching the spring from 75 em to 100 em, assuming that the elastic limit not exceeded s0 that the spring characteristics do not change. Ans, 3750 ergs ‘A particle moves in the sy plane so that its position is given by 2 =A cosut, y= B coset. Prove that it describes an are of a parabola, A. particle moves in the 2y plane so that ite position is given by 2 =A cos (t+), ¥ = B cos (ot + ¢3). Prove that the particle describes a closed curve or not, according as oi/o; rational or not. In which eases is the motion periodic? ‘The position of a particle moving in the zy plane is described by the equations z/d!2 aylde =~de, At time ¢=0 the particle is at rest at the point (6,3). Find (@) and (0) its velocity at any later time'#. Find the period of simple pendulum of length 1 meter if the maximum angle which the rod makes with the vertical is (a) 80°, (6) 60°, (e) 90°. A simple pendulum of length 8 ft is suspended vertically from a fixed point. At ¢=0 the bob is given a horizontal velocity of 8 ft/sce. Find (a) the maximum angle which the pendulum rod makes with the vertical, (6) the period of the oscillations Ans. (a) cos-12/8 = 41° 48", (6) 1.92 see Prove that the time averages over a period of the potential energy and kinetic energy of a simple harmonic oscillator are equal to 2s42/P? where Ais the amplitude and P is the period of the motion. A cylinder of radius 10 ft with its axis vertical oscillates vertically in water of density 62.5 Ib/ft ‘with a period of 5 seconds, How much does it weigh? Ans. 8.08% 10° Ibwt ‘A particle moves in the xy plane in a force field whose potential is given by V=2?+2y-+y%. If the particle is initially at the point (8,4) and is given a velocity of magnitude 10 in a direction parallel to the positive « axis, (a) find the position at any time and (6) determine the period of the motion if one exists. 14 4.103 408, 4.05, 4.106, 4.108, 400. san, 4a, ans. sau ‘THE SIMPLE HARMONIC OSCILLATOR AND THE SIMPLE PENDULUM [CHAP. 4 In Problem 4.96 suppose that oj/s; is irrational and that at ¢=0 the particle is at the particular point (29,4) inside the rectangle defined by 2 =A, y= +B, Prove that the point (qo) will never be Teached again but that in the course of its motion the particle will come arbitrarily close to the point. [A particle oscillates on a vertical frictionless cycloid with its vertex downward. Prove that the projection of the particle on a vertical axis oscillates with simple harmonic motion. ‘A mass of 5 kg at the lower end of a vertical spring which has an elastic constant equal to 20 newtons/meter oscillates with a period of 10 seconds. Find (a) the damping constant, (2) the natural period and (c) the logarithmic decrement, Ans. (a) 19 ntsec/m, (6) 3.14 see ‘A mass of 100 gm is supported in equilibrium by two identical springs of negligible mass having elastic constant equal to 50 dynes/em, In the equilibrium position shown in Fig. 4-25 ‘the springs make an angle of 30° with the horizontal and are 100 et in length. If the mass is pulled down a distance of 2em and released, find the period of the resulting oscillation. ‘A thin hollow circular cylinder of inner radius 10 em is fixed ‘0 that its axis is horizontal, A particle is placed on the inner frictionless surface of the cylinder s0 that its vertical distance above the lowest point of the inner surface is 2em. Find 100 gm (a) the time for the particle to reach the lowest point and (8) the period of the oscillations which take place. Fig. 4-25, A cubical box of side a and weight W vibrates vertically in water of density . Prove that the period of vibration is (@r/a)Vag/W- |A spring vibrates #0 that its equation of motion is mesa +e = FO ala , find # as a function of time t. 2G afin (tu) de eS. Au) sin Vai (tu) ome ‘Work Problem 4.109 if damping proportional to dz/dt is taken into account. |A spring vibrates so that its equation of motion miPaldt? + xx = Beosut + 2 cosSut It 2=0, #=vy at £=0, (@) find + at any time ¢ and (6) determine for what values of resonance will oceur. ‘A vertical spring having clastic constant x carries a mass m at its lower end, At t= 0. the ‘pring is in equilibrium and its upper end is suddenly made to move vertically so that its distance {rom the original point of support is given by A sinet, t= 0. Find (a) the position of the mass mm lat any time and (0) the values of w for which resonance occurs. dzldt=0 at t=0, and (2) give a physical (a) Solve dBx/de + 2 = tsint + cost where = interpretation. Discuss the motion of @ simple pendulum for the ease where damping and external forces are present. CHAP. 4) THE SIMPLE HARMONIC OSCILLATOR AND THE SIMPLE PENDULUM 115 4118, Find the period of small vertical oscillations of a cylinder of radius @ and height A floating with its axis horizontal in water of density o. 4.116. A vertical spring having elastic constant 2 newtons per meter has a 60 ¢m weight suspended from it. A force in newtons which is given as a function of time ¢ by F(t) =Gcostt, t20 is applied, Assuming that the weight, initially at the equilibrium position, is given an upward velocity of 4 m/sec and that damping is negligible, determine the (a) position and (8) velocity of the weight st any time. 4117. In Problem 4.55, ean the answer for the limit as 2? Justify your answer. 2 be doduced from the answer for w#2 by taking 4118, An oscillator has a restoring force acting on it whose magnitude is —cx — 2? where « is small compared with x. Prove that the displacement of the oscillator [in this case often called an anharmonic oscillator] from the equilibrium position is given approximately by f= Acontet=9) + AZ ont 9) 8) where A and ae dterined fom the nial eons 4118, Prove that if the oscillations in Problem 4.92 are not necessarily amall, then the period is given by p= afi (ye (aye (ee)'e- +} Chapter 5 CENTRAL FORCES and PLANETARY MOTION CENTRAL FORCES Suppose that a force acting on a particle of mass m is such that [see Fig. 5-1} (a) it is always directed from m toward or away from a fixed point 0, (b) its magnitude depends only on the distance r from 0. ‘Then we call the force a central force or central force field with O as the center of force. In sym- ols F is central force if and only if F = f(r)n = fryer 7) where r= 1/r is a unit vector in the direction of r. ‘The central force is one of attraction toward O or repulsion from O according as f(r) <0 or f(r) > 0 respectively. Fig. 5-1 SOME IMPORTANT PROPERTIES OF CENTRAL FORCE FIELDS If a particle moves in a central force field, then the following properties are valid. 1. The path or orbit of the particle must be a plane curve, i.e. the particle moves in a plane. This plane is often taken to be the xy plane. See Problem 5. 2. The angular momentum of the particle is conserved, i.e. is constant, See Problem 5.2. 8, The particle moves in such a way that the position vector or radius vector drawn from O to the particle sweeps out equal areas in equal times. In other words, the time rate of change in area is constant. This is sometimes called the law of areas. See Problem 5.6. EQUATIONS OF MOTION FOR A PARTICLE IN A CENTRAL FIELD By Property 1, the motion of a particle in a cen- tral force field takes place in a plane. Choosing this plane as the zy plane and the coordinates of the par- ticle as polar coordinates (r, 4), the equations of mo- tion are found to be [see Problem 5.3] m(# — rit) = f(r) @) m(ré + 248) = 0 () where dots denote differentiations with respect to time t. 116 Fig. 5-2 ir9) CHAP. 5) CENTRAL FORCES AND PLANETARY MOTION 7 From equation (3) we find . r= constant = h ® This is related to Properties 2 and 3 above, IMPORTANT EQUATIONS DEDUCED FROM THE EQUATIONS OF MOTION ‘The following equations deduced from the fundamental equations (2) and (3) often Prove to be useful. eR _ fe) L 7-4 = © (6) 2 ae + =~ aap ltl) © where u=1/r. @r — 2/dr\? _ tin . aa 7(a5) —" = SAP ” POTENTIAL ENERGY OF A PARTICLE IN A CENTRAL FIELD A central force field is a conservative field, hence it can be derived from a potential. ‘This potential which depends only on r is, apart from an arbitrary additive constant, given by vo) = ~f fear @ ‘This is also the potential energy of a particle in the central force field. ‘The arbitrary additive constant can be obtained by assuming, for example, V=0 at r=0 or V>0 as roe, CONSERVATION OF ENERGY By using (8) and the fact that in polar coordinates the kinetic energy of a particle is 4m(i*+r%i), the equation for conservation of energy can be written dm +19) + V(r) = EB (9) or amit) — fymar = EB (10) where E is the total energy and is constant. Using (4), equation (10) can also be written as 2 T/de\2 Re [(H) +] - Sra = & (11) and also as Z(#+8)- Sima = & (12) Tn terms of u=1/r, we can also write equation (9) as Gy +e = gn do a oan (as) DETERMINATION OF THE ORBIT FROM THE CENTRAL FORCE If the central force field is prescribed, i.e. if f(r) is given, it is possible to determine the orbit or path of the particle. This orbit can be obtained in the form r= 16) (4) us CENTRAL FORCES AND PLANETARY MOTION (OHAP. 5 ie. r as a function of 4, or in the form r= nt), @ = ot) (0) which are parametric equations in terms of the time parameter t. To determine the orbit in the form (14) it is convenient to employ equations (6), (7) or (11). To obtain equations in the form (15), it is sometimes convenient to use (12) together with (6) or to use equations (4) and (5). DETERMINATION OF THE CENTRAL FORCE FROM THE ORBIT Conversely if we know the orbit or path of the particle, then we can find the correspond- ing central force. If the orbit is given by r= (0) or u= (0) where u=1/r, the central force can be found from no = MEGS - 2G) - >} (10) or rary = ~mine {su} an which are obtained from equations (6) and (7) on page 117. The law of force can also be obtained from other equations, as for example equations (9)-(13). It is important to note that given an orbit there may be infinitely many force fields for which the orbit is possible. However, if a central force field exists it is unique, ie. it is the only one. CONIC SECTIONS, ELLIPSE, PARABOLA AND HYPERBOLA Consider a fixed point O and a fixed line AB distant D from O, as shown in Fig. 5-8. Suppose that a point P in the plane of O and AB moves so that the ratio of its distance from point 0 to its distance from line AB is always equal to the positive constant « ‘Then the curve described by P is given in polar coordinates (r,4) by = Pp T= [Fee (28) See Problem 5.16. ‘The point O is called a focus, the line AB is called a directriz and the ratio « is called the eccentricity. ‘The curve is often called a conic section since it can be obtained by intersecting ‘a plane and a cone at different angles. Three possible types of curves exist, depending on the value of the eccentricity. 1. Ellipse: «<1 [See Fig. 5-4 below.] If C is the center of the ellipse and CV = CU =a is the length of the semi-major aris, then the equation of the ellipse can be written as a(l-e) T+ ccosd (9) Note that the major azis is the line joining the vertices V and U of the ellipse and has length 2a. CHAP. 5] CENTRAL FORCES AND PLANETARY MOTION 119 If b is the length of the semi-minor azis [CW or CS in Fig. 5-4) and c is the distance CO from center to focus, then we have the important result, Ve=E = a (20) A circle can be considered as a special case of an ellipse with eccentricity equal to zero. 2 Parabola: «= 1 [See Fig. 5-5.] The equation of the parabola is =~? _ "= T¥eosd en We can consider a parabola to be a limiting case of the ellipse (19) where «= 1, which means that ac [ie. the major axis becomes infinite] in such a way that a(l-2) 8. Hyperbola: «> 1 [See Fig. 5-6.) ‘The hyperbola consists of two branches as indicated in Fig. 5-6. ‘The branch on the left is the important one for our purposes. The hyperbola is asymptotic to the dashed lines of Fig. 5-6 which are called its asymp- totes. The intersection C of the asymptotes is called the center. The distance CV =a from the center C to vertex V is called the semi-major ais [the major axis being the distance between vertices V and U by anal- oxy with the ellipse]. ‘The equation of the hyperbola can be written as a(é=1) T+ cco Fig. 55 r (22) Fig. 56 ‘Various other alternative definitions for conic sections may be given. For example, an ellipse can be defined as the locus or path of all points the sum of whose distances from two fixed points is a constant. Similarly, a hyperbola can be defined as the locus of all points the difference of whose distances from two fixed points is a constant. In both these cases the two fixed points are the foci and the constant is equal in magnitude to the length of the major axis. SOME DEFINITIONS IN ASTRONOMY A solar system is composed of a star (such as our sun] and objects called planets which revolve around it. The star is an object which emits its own light, while the planets do not emit light but can reflect it. In addition there may be objects revolving about the planets. These are called satellites. In our solar system, for example, the moon is a satellite of the earth which in turn is a planet revolving about our sun. In addition there are artificial or man-made satellites which can revolve about the planets or their moons. 120 CENTRAL FORCES AND PLANETARY MOTION (CHAP. 5 ‘The path of a planet or satellite is called its orbit. The largest and smallest distances of a planet from the sun about which it revolves are called the aphelion and perihelion respectively. The largest and smallest distances of a satellite around a planet about which it revolves are called the apogee and perigee respectively. ‘The time for one complete revolution of a body in an orbit is called its period. This is sometimes called a sidereal period to distinguish it from other periods such as the period of earth’s motion about its axis, ete. KEPLER’S LAWS OF PLANETARY MOTION Before Newton had enunciated his famous laws of motion, Kepler, using voluminous data accumu- lated by Tycho Brahe formulated his three laws concerning the motion of planets around the sun [see Fig. 5-7). 1. Every planet moves in an orbit which is an ellipse with the sun at one focus. Fig. 5-7 2. The radius vector drawn from the sun to any planet sweeps out equal areas in equal times (the law of areas, as on page 116) 8. The squares of the periods of revolution of the planets are proportional to the cubes of the semi-major axes of their orbits. NEWTON'S UNIVERSAL LAW OF GRAVITATION By using Kepler's first law and equations (16) or (17), Newton was able to deduce his famous law of gravitation between the sun and planets, which he postulated as valid for any objects in the universe [see Problem 5.21] Newton's Law of Gravitation. Any two particles of mass m: and mz respectively and distance r apart are attracted toward each other with a force — Gm ay (28) where G is a universal constant called the gravitational constant. By using Newton's law of gravitation we can, conversely, deduce Kepler's laws (see Problems 5.18 and 5.23]. ‘The value of G is shown in the table on page 342. F= ATTRACTION OF SPHERES AND OTHER OBJECTS By using Newton's law of gravitation, the forces of attraction between large objects such as spheres can be determined. To do this, we use the fact that each large object is composed of particles. We then apply the law of gravitation to find the forces between particles and sum over these forces, usually by methods of integration, to find the resultant force of attraction. An important application of this is given in the following Theorem 5.1. Two solid or hollow uniform spheres of masses m: and mz respectively which do not intersect are attracted to each other as if they were particles of the same mass situated at their respective geometric centers. Since the potential corresponding to = _Gmm r= -SOre (24) CHAP. 5) CENTRAL FORCES AND PLANETARY MOTION 121 is v= - Sum (25) it is also possible to find the attraction between objects by first finding the potential and then using F=—VV. See Problems 5.26-5.38 MOTION IN AN INVERSE SQUARE FORCE FIELD As we have seen, the planets revolve in elliptical orbits about the sun which is at one focus of the ellipse. In a similar manner, satellites (natural or man-made) may revolve around planets in elliptical orbits. However, the motion of an object in an inverse square field of attraction need not always be elliptical but may be parabolic or hyperbolic. In such cases the object, such as a comet or meteorite, would enter the solar system and then leave but never return again. ‘The following simple condition in terms of the total energy E determines the path of an object. (if <0 the path is an ellipse (ii) if B=0 the path is a parabola (ii) if 2 >0 the path is a hyperbola Other conditions in terms of the speed of the object are also available. See Problem 5.37. In this chapter we assume the sun to be fixed and the planets do not affect each other. Similarly in the motion of satellites around a planet such as the earth, for example, we assume the planet fixed and that the sun and all other planets have no effect. Although such assumption is correct as a first approximation, the influence of other planets may have to be taken into account for more accurate purposes. The problems of dealing with the motions of two, three, ete., objects under their mutual attractions are often called the two body problem, three body problem, ete. Solved Problems CENTRAL FORCES AND IMPORTANT PROPERTIES 5.1. Prove that if a particle moves in a central force field, then its path must be a plane ane Tet F=f) be the central force eld. ‘Ten ea. rs since 6 6 «unt vetor inthe det of the poon ver, Since F = dvd, tha canbe written rx dv/dt = 0 (2) : dexw =e Inepatng, we . where h is a constant vector. Multiplying both sides of (4) by r+, rh =o 6 ‘Thus ¥ is perpendicular to the constant vector I, fe shall assume that this plane is taken to be the using the fact that r+(Xw) = (eX2)+¥ and s0 the motion takes place in a plane. ‘zy plane whose origin is at the eenter of force. 122 CENTRAL FORCES AND PLANETARY MOTION [CHAP. 5 52. Prove that for a particle moving in a central force field the angular momentum is conserved. From equation (4) of Problem 5.1, we have rxv ah ‘where Wis a constant vector. Then multiplying by mass m, (exw) = mh ® Since the left side of (2) is the angular momentum, it follows that the angular momentum is ‘conserved, ie. is always constant in magnitude and direction, EQUATIONS OF MOTION FOR A PARTICLE IN A CENTRAL FIELD 53. Write the equations of motion for a particle in a central field. By Problem 5.1 the motion of the particle takes place in a plane, Choose this plane to be the xy plane and the coordinates deseribing the position of the particle at any time t to be polar coordinates (r,). Using Problem 149, page 27, we have (mass)(acceleration) = _net force mF rityr, + °F 4 BPH) = Heyy o ‘Thus the required equations of motion are given by mG rit) = fr) ® mir + 2%) = 0 @) 54, Show that r’) =, a constant. Method 1, Equation (a) of Problem 5.3 can be written does Thus Sot) = 0 and s0 A where A is # constant, Method 2. By Problem 1.49, page 27, the velocity in polar coordinates is v= int rig ‘Then from equation (4) of Problem 5.1 hes ny = Hexen) trdexe) = rik ® since Xr, =0 and x0, =rk_where k is the unit veetor in a direction perpendicular to the plane of motion [the sy plane], ie. in the direction xxv. Using h= hk in (2), we see that roan 5.5. Prove that r= 2A where A is the time rate at which area is swept out by the position vector r. ‘Suppose that in time At the particle moves from M to N [see Fig. 5-8), The area ad swept out by the position vector in this time is approximately half the area of a parallelogram with sides + and Ar oF [see Problem 1.18, page 15] BA = Hlex ar] Dividing by S¢ and letting at ais, ie CHAP. 5] CENTRAL FORCES AND PLANETARY MOTION 123 56. 5. ie, A= glexol = ge using the regult in Problem 5.4. Thus r2é= 2A, as required. The vector quantity A= dk = yiexe) = yori is often called the areal velocity. Prove that for a particle moving in a central force field the areal velocity is constant. By Problem 5.4, 723 ‘a constant, Then the areal velocity is hk 4h, a constant vector ‘The result is often stated as follows: If @ particle moves in a central force field with O as center, then the radius vector drawn from © to the particle sweeps out equal areas in equal times, This result is sometimes called the law of areas. ‘Show by means of the substitution r= 1/u that the differential equation for the path of the particle in a central field is ce era) de mie From Problem 6.4 oF equation (9) of Problem 53, we have Pes hor Alt = hut @ Substituting into equation (2) of Problem 5.3, we find ) = Hr) ® Now if = Iu, we have é cy oe se 8 Co) From this we see that (2) ean be written n(hPat ule — Kas) = f(t) © or, as required, - Ly c) ‘nha POTENTIAL ENERGY AND CONSERVATION OF ENERGY FOR CENTRAL FORCE FIELDS 58. (a) Prove that a central force field is conservative and (b) find the corresponding potential energy of a particle in this field. Method 1. If we can find the potential or potential energy, then we will have also incidentally proved that the field is conservative, Now if the potential V exists, it must be such that Fede = av wo where F = /lr)r, is the central force. We have Pedr = finysde = f0)E+de = fridr since r+ dr=rdr. ‘Since we can determine V such that =av = finde 124 CENTRAL FORCES AND PLANETARY MOTION (CHAP. 5 for example v= Snow it follows that the field is conservative and that (2) represents the potential or potential energy. Method 2. We can show that VxF=0 directly, but this method is tedious although straightforward. 59. Write the conservation of energy for a particle of mass m in a central force field. [Problem 1.49, page 27) i trie, noth ot = vey = + eet Method 1. The velocity of @ particle expressed in polar coordinates ‘Thon the principe of conservation of enerEy can be expressed as fut tv = Boor amity finer = F ‘whore E is constant. Method 2 The equations of motion for a particle ina central fld are, by Problem 5.8, mF — ré2) = fir) a m(ré + 278) = 0 @) Multiply equation (2) by #, equation (2) by ré and add to obtain mre + 88 + vray = fore s) ‘his can be writen indaesnin = 2 fae _ ‘Then integrating both sides, we obtain jnie +r — fi finde = © 5.10. Show that the differential equation describing the motion of a particle in a central field can be written as BG) +e]- Snow = = From Problem 59 we have by the conservation of eneray, nGe+ oi — Psmar = B w We also have eee @ Substitoting (2) into (1), we find an [(B) te] ~ f sora since #= h/t, BAL (a) If w=1r, prove that v* = i + 1%? = WY (du/de)* + wy. (B) Use (a) to prove that the conservation of energy equation becomes (du/doy? + ut = 2(8 - Vyfmhe CHAP. 5] CENTRAL FORCES AND PLANETARY MOTION 125 (@) Prom equations (2) and (9) of Problem 5.7 we have 3 = hut, #=—hdu/de, ‘Thus FA aE = WAduldeyt + (Ma (hutye = W((du/dey? +) (®) From the conservation of energy [Problem 5.9] and part (a), foe? = fit tri) = BV or (dufdoyt + ut = 208 — Vm? DETERMINATION OF ORBIT FROM CENTRAL FORCE, OR CENTRAL FORCE FROM ORBIT 5.12. Show that the position of the particle as a function of time ¢ can be determined from the equations t= Ste eas t iSve where oo) = 2+ 2 fina - gee Vr in the equation for conservation of energy of Problem 5.9, smo + wire) ~ fi yodr = B or # 2S rae —2 = am ‘Then assuming the positive square root, we have aria = VE land so separating the variables and integrating, we find 1 =f tora ‘The second equation follows by writing $= h/r? as dt=rtde/h and integrating. 5.13. Show that if the law of central force is defined by fr) = -Ki*, K>0 i.e. an inverse square law of attraction, then the path of the particle is a coni Method 1. Tn th wwe find cease /(l/u) = Ku, Substituting into the differential equation of motion in Problem 5.7, Pula + Kimae ‘This equation has the general solution ue = Acoso + Baing + K/mk? ® or using Problem 42, page 92, Kimk + C cos(o—9) ® 1 se, = Ria Casena) 2 It is always possible to choose the axes so that ¢=0, in which case we have 1 ° = Rimi + Ceose 0 126 Bad. CENTRAL FORCES AND PLANETARY MOTION (CHAP. 5 ‘This has the general form of the conic [see Problem 5.16) ae ae sconanne pee Me T¥ «cos ifp + (ep) cose @ hen snoai sd (0) at p= Kini, ep = 0 . pe mig, «= wok ® Metod 2 since /0)= “Ki, weve v= finde = -Kir ta ee ve Ke wn maz fae)? . K mG) +8] - 2+ wn oe trom we ee aE us By separating variables and integrating [see Problem 5.66] we find the solution (6) where C is expressed in terms of the energy E. (a) Obtain the constant C of Problem 5.18 in terms of the total energy B and (b) thus show that the conic is an ellipse, parabola or hyperbola according as E <0, E=0, E> 0 respectively. Melted (0) The potential exrey Y= Sina = ff xnnae = Kir = ~kx ee) ee TEN lee Prete Bis a eee, o ‘Thun from Problem 6.110) together with (0), hae comer + (ys ccae)' = 2% my oe : o- Bt ow © tenming €> 0. (0) Uaing th van of tn part, the equation ofthe cone Decree oe Eat wee Behe fie} compesing ths wth (6 ot Problem 66, we ame that th exantrolty : o that the conie is an ellipse if # <0 (but greater than —K2/2mh}, a parabola From this we 1 and «>1 respectively. if B=0 and a hyperbola if E>, since in such cases «<1, Method 2. ‘The value of C ean also be obtained as in the second method of Problem 5.13. CHAP. 5} CENTRAL FORCES AND PLANETARY MOTION 127 5.15. Under the influence of a central force at point O, a particle moves in a circular orbit which passes through 0. Find the law of force. of a circle of radius @ passing through O is [see Fig. 5-9] r= 2acose ‘Then since a we have Fig.59 ‘Thus by Problem 5:7, cnnana (S021e + sec 0 tant 6 + see 2a fats) = —miet (G+ 0) mit (aes : Mo sees AME (ecto + sec (tants +1) = —~ I s 2 sect ~mitoe smhtat or fo = ~ ae ‘Thus the foree is one of attraction varying inversely as the fifth power of the distance from 0. Method 2. Using r= 2a-cos@ in equation (16), page 118, we have = ee os Feose e CONIC SECTIONS. ELLIPSE, PARABOLA AND HYPERBOLA 5.16. Derive equation (18), page 118, for a conic section. Referring to Fig. 53, page 118, by definition of a conic section we have for any point P on it, d= er d= 9h a Corresponding to the particular point Q, we have pD=+ op o ® But D = dt rcs = Lt rene = Tteomn O ‘Then from (2) and (8), we have on eliminating D, pa ritecrn oo r= pha w ‘The equation is a circle if fe>d 0, am ellipse if 0 < «<1, a parabola if ¢=1 and a hyperbola 5.17. Derive equation (19), page 118, for an ellipse. Referring to Fig. 5-4, page 119, we see that when 6 OU. Thus using equation (4) of Problem 5.16, r= OV and when @=s, 128 CENTRAL FORCES AND PLANETARY MOTION (CHAP. 5 ov = plate, ov = plato © Bat since 20 the length of the major axis, OV+OU = 20 or pili +) + 9/19 = 20 ® from which p= at Oo ‘Thus the equation of the elipae fs . ® 518 Prove that in Fig. 5-4, page 119, (a) OV = a(1—.), @) OU =a(l +9. (e) From Problem 5.17, equation (8) and the fist equation of (2, ov = Py = HEA = a9 oy (0) From Problem 617, equation (8) and the second equation of (1, ou = = ato © 5.19. Prove that =a where c is the distance from the center to the focus of the ellipse. ais the length of the semi-major axis and « is the eccentricity. From Fig, 5-4, page 119, we have = CO = CV-OV. ‘An analogous result holds for the hyperbola [see Problem 5.73(c), page 189]. alt 2) = ae 5.20. If a and ¢ are as in Problem 5.19 and b is the length of the semi-minor axis, prove that (a) ¢= Ve—B, (b) b=aVI—é (@) From Fig. 5-4, page 119, and the definition of an ov _ cv-co . Ve ao aeve Vee ee ‘Also since the eccentricity is the distance from O to W divided by the distance from W to the directrix AB [which is equal to CE], we have nse, we have a OWICE = « or, using (1) and the result of Problem 6.19, OW = «CE = dCV+VE) = da+(e-ol] = ote Then (OW) = (OCF + (CW) or at = BEC, ie, = VORP () From Problem 6.19 and part (a), @? = 040% or b= aVi—a. KEPLER'S LAWS OF PLANETARY MOTION AND NEWTON'S UNIVERSAL LAW OF GRAVITATION 5.21. Prove that if a planet is to revolve around the sun in an elliptical path with the sun ‘at a focus {Kepler's first law], then the central force necessary varies inversely as the square of the distance of the planet from the sun. If the path is an ellipse with the sun at a focus, then calling r the distance from the sun, we have by Problem 5.16, CHAP. 5] CENTRAL FORCES AND PLANETARY MOTION 129 5.23. 524. 5.25. teem oF wa ha fescue w where «<1. ‘Then the central force is given as in Problem 5.7 by f(Alu) = -mkQutideulde® +x) = —mi2elp ® on substituting the value of u in (1). From (2) we have on replacing u by 1, Hin) = —mi8/pr® = Kit ® Discuss the connection of Newton’s universal law of gravitation with Problem 5.21. istorically, Newton arrived at the inverse square law of force for planets by using Kepler’s first law and the method of Problem 5.21. He was then led to the idea that perhaps all objects of the universe were attracted to each other with a force which was inversely proportional to the square of the distance r between them and directly proportional to the product of their masses. This led to the fundamental postulate cM: — ait, « F ‘where G is the universal gravitational constant. Equivalently, the law of force (8) of Problem 5.21 is the same as (1) where K = GMm @ Prove Kepler's third law: The squares of the periods of the various planets are proportional to the cubes of their corresponding semi-major axes. If a and b are the lengths of the semi-major and semi-minor axes, then the area of the ‘llipse is sa. Since the areal velocity has magnitude A/2 [Problem 5.6), the time taken to sweep over area rab, ic. the period, is p = mb _ Bead 4 5 a Now by Problem 6.17 equation (8), Problem 6.20(6), and Problem 5.18 equation (8), we have b= aVi=4, op = ae) = miiK ® ‘Then from (1) and (2) we find P= demiltqii2/KV2 or PY = detma¥/K ‘Thus the squares of the periods are proportional to the eubes of the semi Prove that GM = gR?, On the earth's surface, ie. r= where R is the radius, the force of attraction of the earth ‘on an object of mass m is equal to the weight mg of the object. Thus if M is the mass of the earth, GMmiRt = mg or GM = gR* Calculate the mass of the earth, From Problem 524, GM=gR? or M= RG. Taking the radius of the earth as 6.98 X10" em, g = 980 cm/sec? and G@=6.67%10-*egsunits, we find M = 5.98% 10%" gm= 12 x 108 Tb. ATTRACTION OF OBJECTS 5.26. Find the force of attraction of a thin uniform rod of length 2a on a particle of mass m placed at a distance b from its midpoint. 130 52. CENTRAL FORCES AND PLANETARY MOTION [omar. 5 Choose the » axis along the rod and the y axis perpendicular to the rod and passing through ite center 0, as shown in Fig. 5-10. Let o be the mass per unit length of the rod, The foree of attraction AF between an element of mass ede of the rod and ‘m is, by Newton's universal law of gravitation, Gmade ; de = GMeGe (ging i — cose) Gmordz ; __Gmabde , err oeea! — reaped Fig.5-10 since from Fig. 5-10, sing = z/VH4T, cose = b/Va"FE. Then the total force of attraction is Let #=b tan in this Integral, Then when == ‘Thus the intewral becomes = 0; and when 2 = tan (a/t). oo. ; Fm any fae, A Since the mass of the rod is M ae, this can alo be written as ‘Thus we see that the force of attraction is directed from m to the center of the rod and of magnitude 2Gmoa/bVa?+ BF or GMm/bvat +08, A mass m lies on the perpendicular through the center of a uniform thin circular plate of radius a and at distance b from the center. Find the force of attraction between the plate and the mass m. ‘Method 1. Let n be a unit vector drawn from point P where m is located to the center O of the plate. Subdivide the cireular plate into circular rings [such as ABC in Fig. 5-11] of radius » and thickness dr. If ¢ is the mass per unit area, then the mass of the ring is o(2zrdr) Since all points of the ring are at the same distance V8 +62 from P, the force of attraction of the ring on sm will be Geter dry. e+e Go2erdrmd (ee DD where we have used the fact that due to aymmetry the resultant force of attraction is in the direction m. By integrating over all rings from r=0 to r=a, we find that the total attraction is Fig. 51 «nae f= scoot fata ® cS cong ® when ‘To evaluate the integral, let 2+ b2= wi ao that rdr=udu. Then since u and w= VaGBE when =a, the result is CHAP. 5) CENTRAL FORCES AND PLANETARY MOTION 131 5.28, VOTH du 3 F = 2eGombn f" $e = 2eGemn (1 ) , * Verte, I we let « be the value of ¢ when v=o, this can be written F = 2eGomn(t ~ cosa) “ ‘Thus the force is directed from m to the center O of the plate and has magnitude 2eGem(1 — eos). Method 2. "The method of double integration can also be used. In such case the clement of area at A is rarde where @ is the angle measured from a line {taken as the < axis) in the plane of the circular plate and passing through the center 0, Then we have as in equation (1), ae = Gelrdrdéymd 5 land by integrating over the circular plate Genta So, Soe 2er dr r Tee oe = 2Gem nt ~ cosa) A uniform plate has its boundary con- le sisting of two concentric half circles of inner and outer radii a and 6 respec- tively, as shown in Fig. 5-12. Find the F oh force of attraction of the plate on a ‘mass m located at the center 0. 4% Tt is convenient to use polar coordinates (7.0) The element of area of the plate [shaded DE Ot A in Fig. 6-12) is dA=rdrde, and the mane is ardrds. ‘Then the force of attraction between GA and 0 is Fig. 5.12 Glor dr doy . ae = Gerd dPYm (eos ¢ i+ sin) ‘Thus the total foree of attraction is Be fl Sf artes Gomn(#) f" cones sina ar = 26omin(8)s Since M = o(frb?—jrai), we have © =2M/e(6—ai) and the force can be written cum (8) = aete (2) + sine) ‘The method of single integration can also be used by dividing the region between =a and =b into cirealar rings as in Problem 5.27. Find the force of attraction of a thin spherical shell of radius a on a particle P of mass m at a distance r >a from its center. Let © be the center of the sphere, Subdivide the surface of the sphere into circular elements such as ABCDA of Fig, 5-13 below by using parallel planes perpendicular to OP. "The area of the surface element ABCDA as seen from Fig. 6-13 is 2ela sin g)(ade) = Bea? sing de since the radius is @ ing [so that the perimeter is 2z(a sin é)] and the thickne if-e is the mass per unit area, the mass of ABCDA is 2rate sin dé. 182, CENTRAL FORCES AND PLANETARY MOTION (CHAP. 5 Since all points of ABCDA are at the same dit tance w= AP from P, the force of attraction of the element ABCDA on m is Gi2rate sino de)m ar coopn where we have used the fact that from symmetry the net force will be in the direction of the unit vector n from P toward 0. Now from Fig. 5-13, coup = HE = PO=EO , rasemt wy Using (2) in (2) together with the fact that by the eee es ee @) wwe find ay = Sirstesne deine — ace ‘Then the total force is (mae F = tose f o T= Bar con om We can ersluste the intopral hy using the vrible w tren by (8) in place of. When 420, w= a@—2art+= (ra)? so that w=r—a if r>a. Also when @=7, Sidertet syrah we that we rte, In addon, we hove Dw dw = 2ar sine do : - @ttout) _ wtowaet | ieee z ‘Then (6) becomes ye magne (at te = sedate 5.0. Work Problem 5.29 if r a this yields v If ra the force is r= -w = -v( and if aetin te » = fq- 2am Les 3 Rae Ay Using Problem 6.24, this can be written, oR : Rte bt (0) As H~~, the limiting speed (6) becomes: VE=ROMTR or | VOOR o H : since Jim. yea ‘The minimum initial speed occurs where (7) is zero oF where vy = VIGMIR = VIgR © This minimum speed is ealled the escape called the escape velocity from the earth's surface. speed and the corresponding velocity 5.85. Show that the magnitude of the escape velocity of an object from the earth's surface we find % Now since r= 771, is about 7 mi/sec. From equation (8) of Problem 584, v= V29R. Taking g = 82 ft/sec? and R= 4000 mi, 96 mi/see Prove, by using vector methods primarily, that the path of a planet around the sun is an ellipse with the sun at one focus. Since the force F between the planet and sun is av _ _GMm mae 2 dv _ OM a F n o we have s Te, Also, by Problem 5.1, equation (6), we have rv oh 0 dr rr,, Thus from (8), ary. dr . mix (Gitta) = As w CHAP. 5] CENTRAL FORCES AND PLANETARY MOTION 135 From (2), i = = ms, x (2%) = now {(n$)n eee} = ow ‘using equation (6) above and equation (7), page 5. But sine his «constant ver, fick = ivi) wo tat at a ary Sox = ow St Integrating, vxh = GMn +e ‘from which re(vxXh) = GMren + rve = GMr + mre = Gr + recone ‘where ¢ is an arbitrary constant vector having magnitude c, and ¢ is the angle between ¢ and ry. Since r+(vXh) = (eXv)+h= h+h= At [see Problem 1.72(a), page 27), At = Gir + recone : oe alae 1 = Git ews ~ TF CGM cose which is the equation of a coni ‘required result is proved. Since the only conie which is a closed curve is an ellipse, the 5.87, Prove that the speed v of a particle moving in an elliptical path in an inverse square field is given by 2. @ = 2) mS m\oa where a is the semi-major axis. By (8) of Problem 5.18, (s) of Problem 5.14 and (s) of Problem 5.17, we have p= B= cane = «(-2a) © from which E = -Kia @ That he oman of eer we Be ng V =H jo ov = “Bak - ot ® os ® while for a parabola (which corresponds to letting a in either (8) or (4)], vt = 2K/mr 538. An artificial (man-made) satellite revolves about the earth at height H above the surface. Determine the (a) orbital speed and (0) orbital period so that a man in the satellite will be in a state of weightlessness. (a) Assume that the earth is spherical and has radius R. Weightlessness will result when the centrifugal force [equal and opposite to the centripetal force, ie. the force due to the cen- 136 CENTRAL FORCES AND PLANETARY MOTION (CHAP. 5 tripetal acceleration) acting on the man due to rotation of the satelite just balances his attraction to the earth. ‘Then if vy is the orbital speed, mes Gm _ _gRtm R+H ~ (R+H!~ (+H If His small compared with R, this is Vo approximately. R weave distance traveled in one revelu ) ee time for one revolution, or period wus : 2(R+H) Th 7 eS ‘Then from part (a) p= mien . (2pm) [REE = 2r(238 If H is small compared with R, this is 2°VR/o approximately. 5.39. Calculate the (a) orbital speed and (b) period in Problem 5.38 assuming that the height H above the earth's surface is small compared with the earth’s radius. ‘Taking the earth's radius as 4000 miles and g = 82 ft/sect, we find (a) » = VRg 4.92 mifsec and (b) P = 2rV/Rio = 1.42 hr = 85 minutes, approximately. 540. Find the force of attraction of a solid sphere of radius a on a particle of mass m at adistance b 0; (@) P= r(r—Drll +1); (@) P= sinzrey Anis. (a) attractive; (b) repulsive; (¢) attractive if 0 1; (2) repulsive for Bn 0? Does the work depend on the path? Explain, Ans. (a) “Kir, (6) Ka b)fab ‘Work Problem 5.50 for the force field Kr. Ans. (a) —K ins, (8) —K n(a/b) A particle of mass m moves in a central force fleld defined by F = —Kry/r®, (a) Write an equation for the conservation of energy. (B) Prove that if E is the total energy supplied to the particle, then its speed is given by v = VK/m7®+2E/m. A particle moves in a central force feld defined by F =—Kréry. It starts from rest at a point on_the cirele_r = a (a) Prove that when it reaches the circle r = b ite speed will be VEK(@"—05)/5m and that (b) the speed will be independent of the path. A particle of mass m moves in a central force field F=Kry/r where K and n are constants. It starts from rest at r= a and arr with finite speed vp, (a) Prove that we must have <1 and K>0. (b) Prove that j= V2Ka-/m(n—1). (e) Discuss the physical si nificance of the results in (a). By differentiating both sides of equation (18), page 117, obtain equation (6). DETERMINATION OF ORBIT FROM CENTRAL FORCE OR CENTRAL FORCE FROM ORBIT 536. A particle of mass m moves in a central force field given in magnitude by f(r) =—Kr where K is a positive constant. If the particle starts at ra, #=0 with a speed v in a direction perpendicular to the 2 axis, determine ite orbit. What type of curve is deseribed? (a) Work Problem 5.56 (®) Discuss the cases a= the speed is v% in a direction making angle « with the positive # ax ‘and give the physical significance, 138 CENTRAL FORCES AND PLANETARY MOTION (CHAP. 5 558. A particle moving in a central force field located at_r=0 describes the spiral Prove that the magnitude of the force is inversely proportional to r° 558, Find the central force necessary to make a particle describe the lemniscate +? = a? cos 20 (see Fig. 5-16). ’ Ans. A force proportional to r-¥. acta 5.60, Obtain the orbit for the particle of Problem 5.48 and describe physically. . 561. Prove that the orbits r= e-* and r= /@ are both possible for the ease of an inverse cube field of force, Explain physically how this is possible. Fig. 516 5.82. (a) Show that if the law of force is given by Ary Bry Foose = Feoate then a particle can move in the circular orbit r= 2a.cose, (b) What can you conclude about the umiguenss of fores when the orbit is speifed? (e) Answer part (8) when the fores are 563. (a) What central force at the origin O is needed to make a particle move around O with a speed which is inversely proportional to the distance from 0. (b) What types of orbits are possible in such case? Anz. (a) Inverse cube force. 564, Discuss the motion of a particle moving in a central force field given by F = (a/r+ p/r9)r. 565. Prove that there is no central force which will enable a particle to move in a straight line, 566. Complete the integration of equation (12) of Problem 5.13, page 125 and thus arrive at equation (©) of the same problem. (Hint. Let r= 1/u] 561. Suppose that the orbit of a particle moving in, central force field GES ‘mba a! 20") respect to r- ay iven by @=o(7). Prove where primes denote differentiations with 568. (a) Use Problem 5.67 to show that if 9 = 1/r, the central force is one of attraction and varies inversely as 13. (6) Graph the orbit in (a) and explain physically. CONIC SECTIONS. ELLIPSE, PARABOLA AND HYPERBOLA aql2gg- Gravh the con, nding (0) the fk (0) the verti, (6) the length of the major axis, (d) the length of the minor axis, () the distance from the enter to the directrix. 569. The equation of a conic is r 5410, Work Problem 5.69 for the conic + 34 5 cone 541. Show that the equation of a parabola ean be written as r= p sec® (0/2) 542. Find an equation for an ellipse which has one focus at the origin, its center at the point (4,0), ‘and its major axis of length 10. Ane. r= 9/(5 + 4 cosé) CHAP. 5) CENTRAL FORCES AND PLANETARY MOTION 139 513, 5a, 515, 576, sm. In Fig, 6-17, SR or TN is called the minor axis of the hyperbola and its length is generally denoted by 2. ‘The length of the major azis VU is 2a, while the distance between the foci O and 0” is 2¢ [ie. the distance from the center C to a focus 0 or 0" is C). (a) Prove that ef = a? +b%. (®) Prove that 6 = aV@—I where « is the eocen- tricity. (©) Prove that = ae. Compare with results for the ellipse. Fig.517 Derive equation (22), page 119, for a hyperbola, In rectangular coordinates the equations for an ellipse and hyperbola in standard form are given by ey aie +Bat oa S-Ber respectively, where a and b are the lengths of the semi-major and semi-minor axes. Graph these equations, locating vertices, foci and directrices, and explain the relation of these equations to ‘equations (19), page 118, and (22), page 119. Using the alternative definitions for an ellipse and hyperbola given on pages 118-119, obtain the equations (19) and (22). Prove that the angle between the asymptotes of a hyperbola it 2 co8~! (1/e). KEPLER'S LAWS AND NEWTON'S LAW OF GRAVITATION 58. 579. 5.0, 581 583. 5a. Assuming that the planet Mars has a period about the sun equal to 687 earth days approximataly, find the mean distance of Mars from the sun. Take the distance of the earth from the sun as 98 million miles. Ane. 140 million miles Work Problem 5.78 for (a) Jupiter and (6) Venus which have periods of 4838 earth days and 225 earth days respectively. Ams, (a) 484 million miles, (6) 67 million mil Suppose that a small spherical planet has a radius of 10 km and a mean density of 5 gm/em’, (a) What would be the acceleration due to gravity at its surface? (®) What would a man weigh fon this planet if he weighed 80 ki wt on earth? If the acceleration due to gravity on the surface of a spherically shaped planet P is gp while its ‘mean density and radius are given by op and Rp respectively, prove that gp = $xGRpap where G is the universal gravitational constant. If L,M,T represent the dimensions of length, mass and time, find the dimensions of the universal gravitational constant, Ans, IAM-!T~2 Calculate the mass of the sun using the fact that the earth is approximately 150 x 10° kilometers {from it and makes one complete revolution about it in approximately 865 day: Ans. 2X 1090 ke Calculate the force between the sun and the earth if the distance between the earth and the sun is taken as 150 % 10° kilometers and the masses of the earth and sun are 6X 10% ke and 2% 10% kg respectively. Ans. 1.16 X 10% newtons ATTRACTION OF OBJECTS 585, 5.86, Find the force of attraction of a thin uniform rod of length a on @ mass m outside the rod but on ‘the same line as the rod and distance 6 from an end. Ane. GMm/b(a+ 8) In Problem 5.85 determine where the mass of the rod should be concentrated so as to give the same force of attraction. Ame. At a point in the rod a distance Vb(a+8)—b from the end 140 sar. 5.88, 538, 5.90, 501. 592. 533, 595. 5.96, 5a CENTRAL FORCES AND PLANETARY MOTION [CHAP. 5 Find the forve of attraction of an infinitely long thin uniform rod on a mass m at distance 5 from it, Ana, Magnitude is 20me/b ‘A uniform wire is in the form of an are of a circle of radius § and central angle y. Prove that the force of attraction of the wire on @ mass m placed at the center of the circle is given in magnitude by : a. 2GMm sin (6/2) 2Gem sin y/2) a or “ where M is the mass of the wire and & 1s per unit length. Di and y= nr. the m wuss the eases y In Fig, 6-18, AB is a thin rod of length 2a and m is a mass located at point C a distance b from the rod, Prove that the force of attraction of the rod fon m has magnitude GMm um sin Hat AY in a direction making an angle with the rod given by sow (Si) : SS lem 5:26. Fig. 5-18 By comparing Problem 5.89 with Problem 6.88, prove that the rod of Problem 5.89 can be replaced by a wire in the form of cirealar are DEG [shown dashed in Fig. 5-18) which has its enter at C and is tangent to the rod at H. Prove that the direction of the attraction is toward ‘the midpoint of this are. ‘A hemisphere of mass M and radius a has a particle of mass m located at its center. Find the force of attraction if (a) the hemisphere is a thin shell, (2) the hemisphere is sol Ane, (a) GMm/2a2, (6) 86 Mm/2a Work Problem 6.91 if the hemisphere is a shell having outer radius @ and inner radius 6. Deduce from Kepler's laws that if the foree of attraction between sun and planets is ‘magnitude by ym/r?, then + must be independent of the particular planet. ‘A cone has height H and radius a. Prove that the force of attraction on a particle of mass m eGam laced at ite vertex has magnitode S62 (1-H), @ ae Hi Find the force of attraction between two non-interseeting spheres. 'A particle of mass m is placed outside of « uniform solid hemisphere of radius a at a distance # on a line perpendicular to the base through its center. Prove that the force of attraction is siven in magnitude by GMm(y2— 1)/a Work (a) Problem 5:26, (8) Problem 5.27, and (¢) Problem 5.94 by frst finding the potential MISCELLANEOUS PROBLEMS 598, [A particle is projected vertically upward from the earth’s surface with initial speed vo. (a) Prove that the maximum height H reached above the earth's surface is H = vGR/(20R — #9). (©) Discuss the significance of the case where v3 = 202. (6) Prove that if His small, then it is equal to v/2g very nearly. CHAP. 5) CENTRAL FORCES AND PLANETARY MOTION 141 5.98, 5.100, 5.01, 5:02, 5:05, 5.106, 5.07, 5.08, 5.109, 5.110 san 5412, (q) Prove that the time taken to reach the maximum height of Problem 6.08 is VeVi + Baton Et} (8) Prove that if His very small compared with 2, then the time in (a) is very nearly V2ATp. (a) Prove that if an object is dropped to the earth's surface from a height H, then if air resistance is negligible it will hit the earth with a speed v= V2gRHAR+H) where R is the radius of the earth, (H) Calculate the speed in part (a) for the eases where I respectively. ‘Take the radius of the earth as 4000 miles 100 miles and HT = 10,000 miles Find the time taken for the object of Problem 5.100 to reach the eart the two eases surface in each of What most be the law of force if the speed of a particle in a central force field is to be proportional to r=" where » is @ constant? ‘What velocity must a space ship have in order to keep it in an orbit around the earth at a distance of (a) 200 miles, (6) 2000 miles above the carth’s surface? An object is thrown upward from the earth's surface with velocity vp, Assuming that it returns ‘to earth and that air resistance is negligible, find its velocity on returning. (@) What is the work done by a space ship of mass m in moving from a distance a above the earth's surface to a distance 6? () Does the work depend on the path? Explain. Ans. (a) GmM(a—B)/ab (a) Prove that it is possible for a particle to move in a citvle of radius @ field whose law of force is (7). (®) Suppose the particle of part (a) is displaced slightly from its circular orbit, Prove that it will return to the orbit, ic. the motion is stable, if af(a) + 8f(a) > 0 any central force but is unstable otherwi (©) Mustrate the result in (b) by considering f(r) = Lr and deciding for which values of x stability can occur. Ans. (c) For <3 there is stability, Jf the moon were suddenly stopped in its orbit, how long would it take to fall to the earth assuming that the earth remained at rest? Ans. About 4 days 18 hours If the earth were suddenly stopped in its orbit, how long would it take for it to fall into the sun? Ana, About 65 days Work Problem 5.84, page 13, by using energy methods Find the velocity of escape for an object on the surface of the moon. Use the fact that the acceleration due to gravity on the moon's surface is approximately 1/6 that on the earth and ‘that the radius of the moon is approximately 1/4 of the earth’s radius, Ane. 1.5 mifsee ‘An object is dropped through hole bored through the center of the earth. Assuming that the resistance to motion is negligible, show that the speed of the particle as it passes through the center of the earth is slightly less than 5 m/sec [Hint, Use Problem 5.40, page 16, In Problem 5.111 show that the time taken for the object to return is about 85 minutes. 142 5.8. 511d 518, 5.116, suit, 5.18. 5.19, 5.120, 5,21 5122. saz, 5%, 5.138. 5126, sazt. 5.128 5.29, 5:30, sast. 532. 5.138. CENTRAL FORCES AND PLANETARY MOTION [cHAP. 5 “Work Problems 6.111 and 5.112 if the hole is straight but does not pass through the center of the earth. Discuss the relationship between the results of Problems 5.111 and 6.112 and that of Problem 5.39. How would you explain the fact that the earth has an atmosphere while the moon has none? Prove Theorem 6.1, page 120 Discuss Theorem 6.1 if the spheres intersect, Explain how you could use the result of Problem 5.27 to find the force of attraction of a solid sphere on a particle. Find the force of attraction between a uniform circular ring of outer radius a and inner radius b fand a mass m located on its axis at a distance b from its center. ‘Two space ships move about the earth on the same elliptical path of eccentricity «. If they are separated by a small distance D at perigee, prove that at apogee they will be separated by the distance DL a/( +4). (a) Explain how you could calculate the velocity of eseape from a planet. (8) Use your method to ealeulate the velocity of eseape from Ma ‘Ana. (0) 5 km/sec, or about 8 mi/see Work Problem 6.121 for (a) Jupiter, (8) Venus, Ans. (a) about $8 mi/see, (2) about 6.8 mi/see ‘Three infinitely tong thin uniform rods having the same mass per unit length lie in the same plane ‘and form s triangle. Prove that force of attraction on a particle will be zero if and only if the particle is located at the intersection of the medians of the triangle. Find the force of attraction between a uniform rod of length a and a sphere of radius } if they do not intersect and the line of the rod passes through the center. ‘Work Problem 5.124 if the rod is situated so that a li line of the rod biseets the rod. drawn from the center perpendicular to the A satellite of radius a revolves in a circular orbit about planet of radius with period P. Tf the shortest distance between their surfaces is c, prove that the mass of the planct is AaYatb bOGP?, Given that the moon is approximately 240,000 miles from the earth and makes one complete revolution about the earth in 27} days approximately, find the mass of the earth Ans. 6% 10 kg. Discuss the relationship of Problem 5.126 with Kepler's third law. Prove that the only central foree field F whose divergence is zero is an inverse square force field. Work Problem 5.82, page 182, by using triple integration. A uniform solid right circular cylinder has radius @ and height H. A particle of m placed fon the extended axis of the eylinder so that it is at a distance D from one end. Prove that the force of attraction is directed along the axis and given in magnitude by BGM cay 4 at DE- VaR DTH) Suppose that the eynder of Problem 6:81 has a given volume. Prove that the force of attraction than the particle at the center of one end ofthe cylinder in maximum when a/H1 = 4(0~ VI?) Work (a) Problem 6.26 and () Problem 5.27 assuming an inverse cube law of attraction, CHAP. 5) CENTRAL FORCES AND PLANETARY MOTION 43 5m, 5.185, 5.135, 5st, 5.138, 5.40, sau, 5.182 5.183. 58, 5.18, 5.146, Do the results of Problems 5:29 and 5.80 apply if there is an inverse cube law of attraction? Explain. What would be the velocity of escape from the small planet of Problem 6.80? A spherical shell of inner radius @ and outer radius b has constant density e, Prove that the ravitational potential V(r) at distance r from the center is given by Bra(bt~ a) rea Voy = | Bao fr) — treaty arch ( 4re(6 — anya" rob If Einstein's theory of relativity of a planet becomes taken into account, the differential equation for the orbit oe Fetus Saw where y= 3K/met, © being the speed of light. (a) Prove that if axes are suitably chosen, then the position 1 of the planet can be determined approximately from mbt T¥ cco (}) Use (@) to show that a planet actually moves in an elliptical path but that this ellipse slowly rotates in space, the rate of angular rotation being 27yK/mh?.. (c) Show that in the case of Mercury this rotation amounts to 48 seconds of are per century. This was actually observed, thus offering experimental proof of the validity of the theory of relativity. , where @ = 1— yK/mkt ind the position of a planct in its orbit around the sun as a function of time t measured from where it is furthest from the sun, At apogee of 200 miles from the earth’s surface, two space ships in the same elliptical path are 500 feet apart. How far apart will they be at perigee 150 milet assuming that they drift without altering their path in any way? ‘A particle of mass is located on a perpendicular lin through the center of a rectangular plate of sides 2a and 2) at distance D from this center, Prove that the force of attraction of the plate fon the particle is given in magnitude by Oe i (Fath) ab EE DIET DH Find the force of attraction of a uniform infinite plate of negligible thicknes fon a particle at distance D from it. Ans. 220Gm ‘and density o Points where #=0 are called apsidee (singular, apsis). (a) Prove that apsides for a central foree field with potential V(r) and total energy E are roots of the equation Vir) + M/2rt = E. () Bind the apsides corresponding to an inverse square field of force, showing that there are ‘vo, one or none according as the orbit is an ellipse, hyperbola or parabola A particle moving in a central force field travels in a path which is the eycloid Find the law of force. Ans. Inverse fourth power of 1. (1 —c08 6). Set up equations for the motion of a particle in a central force field if it takes place in a medium ‘where the resistance is proportional to the instantaneous speed of the particle. A satellite has its largest and smallest orbital speeds given by Yaax and qin respectively. Prove ‘that the eccentricity of the orbit in which the satellite moves is equal to “**— "min max F Pin Prove that if the satellite of Problem 5.145 has a period equal to r, then it moves in an elliptical path having major axis whose length is Vax Yuin Chapter 6 MOVING COORDINATE SYSTEMS NON-INERTIAL COORDINATE SYSTEMS In preceding chapters the coordinate systems used to describe the motions of particles were assumed to be inertial [see page 83]. In many instances of practical importance, however, this assumption is not warranted. For example, a coordinate system fixed in the earth is not an inertial system since the earth itself is rotating in space. Consequently if we use this coordinate system to describe the motion of a particle relative to the earth we obtain results which may be in error. We are led therefore to consider the motion of particles relative to moving coordinate systems. ROTATING COORDINATE SYSTEMS In Fig. 6-1 let XYZ denote an inertial coordinate a2 ‘stem with origin O which we shall consider fixed in space, Let the coordinate system «yz having the igin O be rotating with respect to the XYZ sete, Consider a vector A which is changing with time. To an observer fixed relative to the «yz system ‘the time rate of change of A = Ayi+Aaj+ Ask is found to be aa day the a where subscript M indicates the derivative in the moving (xyz) system. dA, “ait dy +o @) However, the time rate of change of A relative to the fixed XYZ system symbolized by the subscript F is found to be [see Problem 6.1] gal _ aA dt) = dt lw where w is called the angular velocity of the xyz system with respect to the XYZ system, + oxa @ DERIVATIVE OPERATORS Let D, and D,, represent time derivative operators in the fixed and moving systems. ‘Then we can write the operator equivalence D, = Dy tex () ‘This result is useful in relating higher order time derivatives in the fixed and moving systems, See Problem 6.6. 144 CHAP. 6] MOVING COORDINATE SYSTEMS 145 VELOCITY IN A MOVING SYSTEM If, in particular, vector A is the position vector r of a particle, then (2) gives ar ar $ te tox w or Dy = Dy boxe 6) Let us write Ypyp = de/dt|,, = Dyx = velocity of particle P relative to fixed system Vpy = dr/dt|,, = Dye = velocity of particle P relative to moving system Vue = 2X2 = velocity of moving system relative to fixed system. ‘Then (4) or (5) can be written Vow = Vow +; (6) ACCELERATION IN A MOVING SYSTEM If D} = a/dt*|, and D3 = @/dt*|y are second derivative operators with respect to t in the fixed and moving systems, then application of (3) yields [see Problem 6.6] Dir = Dix + (Dye) Xr + 26x Dye + @X(wX2) « Let us write app = @r/dt|y = Dir = acceleration of particle P relative to fixed system py, = @r/dt*|y, = Dix = acceleration of particle P relative to moving system (Dyas) Xx + 2a Dyr + 0X (0X) = acceleration of moving system relative to fixed system Then (7) can be wi Bre = Bae + Mane (8) CORIOLIS AND CENTRIPETAL ACCELERATION ‘The last two terms on the right of (7) are called the Coriolis acceleration and centripetal acceleration respectively, i.e., Coriolis acceleration 2a Dye = 2a vy @ Centripetal acceleration = «x (wr) (10) ‘The second term on the right of (7) is sometimes called the linear acceleration, i.e. ir = xr = (48] \xr Linear acceleration = (Dya) xr = ($| ) x (1) and Dy is called the angular acceleration. For many cases of practical importance [e.g. in the rotation of the earth] w is constant and Dye =0. ‘The quantity ~» x (« xr) is often called the centrifugal acceleration. MOTION OF A PARTICLE RELATIVE TO THE EARTH ‘Newton's second law is strictly applicable only to inertial systems. However, by using (7) we obtain a result valid for non-inertial systems. This has the form mDjx = F — m(Dye) Xx — 2m(eX Dyz) — mo x (a Xr) (12) where F is the resultant of all forees acting on the particle as seen by the observer in the fixed or inertial system. 146 MOVING COORDINATE SYSTEMS (CHAP. 6 In praetice we are interested in expressing the equations of motion in terms of quantities as determined by an observer fixed on the earth [or other moving system]. In such ease we may omit the subscript M and write (12) as ar mae =F = mlaxr) ~ 2mloXv) — mlaX (Xx) (13) For the case of the earth rotating with constant angular « about its axis, 0 and (13) becomes ar GE =F Bmax) ~ mlex (eX) (4) CORIOLIS AND CENTRIPETAL FORCE Referring to equations (18) or (14) we often use the following terminology Coriolis force 2m(o xt) = 2mle xv) Centripetal force = mle (#*)] Centrifugal force = —m[a x (# x )] MOVING COORDINATE SYSTEMS IN GENERAL In the above results we assumed that the coordi- nate systems zyz and XYZ [see Fig. 6-1] have com- mon origin 0. In case they do not have a common origin, results are easily obtained from those already considered. ‘Suppose that R is the position vector of origin @ relative to origin O [see Fig. 6-2]. Then if R and denote the velocity and acceleration of Q relative to O, equations (5) and (7) are replaced respectively by De = B+ Dy t axe a dr = R+ Gt exe (15) Fie and Dir = it + Dix + (Dye) Xr + 20X Dye + 0X (0X) = tt Gee dart texvt oxox) (16) Similarly equation (14) is replaced by. GE =F 2mloxy) ~ mie (exe)] — mit an THE FOUCAULT PENDULUM Consider a simple pendulum consisting of a long string and heavy bob suspended vertically from a frictionless support. Suppose that the bob is displaced from its equilibrium position and is free to rotate in any vertical plane. Then due to the rotation of the earth, the plane in which the pendulum swings will gradually precess about a vertical axis. In the northern hemisphere this precession is in the clockwise direction if we look down at the earth’s surface, In the southern hemisphere the precession would be in the counterclock- wise direction. Such a pendulum used for detecting the earth's rotation was first employed by Foucault in 1851 and is called Foucault's pendulum. CHAP. 6] MOVING COORDINATE SYSTEMS a7 Solved Problems ROTATING COORDINATE SYSTEMS 6.1. An observer stationed at a point which is fixed relative to an xyz coordinate system with origin O [see Fig. 6-1, page 144] observes a vector A= Aii+Aj+Ask and At; Uy. Later, he finds that he calculates its time derivative to be i+ “at and his coordinate system are actually rotating with respect to an XYZ coordinate system taken as fixed in space and having origin also at O. He asks, “What would be the time derivative of A for an observer who is fixed relative to the XYZ coordi- nate system?” 1f GP), ana 4B] denote respectively the time derivatives of A relative to the fixed and moving systems, show that there exists a vector quantity « such that day dt Fle + ox To the fixed observer the unit vectors |, ‘observer would compute the time derivative ay May te en Oy A Jk actually change with time. Hence such an By a + Ab + as o HDs ade ln + Ange + Aage + Asae @ ince i is a unit vector, di/dt is perpendicular to i and must therefore lie in the plane of jand k. Then © w Similarly, From i+} =0, from (8). Thus a4 =a. Similarly from i+ke = 0, and ag = Tet follows that 1 Aart Ah + Age which can be written as ii ok a nm Ar Ay As ‘Then if we choose ay = oy, —og =n a; = this determinant becomes tis, aay ox Ay Ap As where w= oil teri tose 148 MOVING COORDINATE SYSTEMS [oHap. 6 From (2) and (6) we find, as required, aa\ Tl te ‘The vector quantity « is the angular velocity of the moving system relative to the fixed system. 62. Let D, and D,, be symbolic time derivative operators in the fixed and moving systems respectively. Demonstrate the operator equivalence D, = Dy + ex By detntion Dg = 8 = derivative in and eytem aa i ‘Than frm Problem 64, Dih = Dyk + aXA = (Dy +o¥0A which shows the equivalence of the operators Dp = Dy + #% 63. Prove that the angular acceleration is the same in both XYZ and xyz coordinate systems. Let A= in Problem 61. Then Since de/dt is the angular acceleration, the required statement is proved. VELOCITY AND ACCELERATION IN MOVING SYSTEMS 64, Determine the velocity of a moving particle as seen by the two observers in Problem 6.1. Replacing A by the position veetor r of the particle, we have a de a a a o If r Is expressed in terms of the unit vectors i, velocity of the particle relative to this system de _ de 1k of the moving coordinate system, then the ‘on dropping the subscript M, w= alt . and the velocity of the particle relative to the fixed aystem is from (1) 7 oo a) = Bt exe © ‘The velocity (2) is sometimes called the true velocity, while (2) is the apparent velocity 65. An zyz coordinate system is rotating with respect to an XYZ coordinate system having the same origin and assumed to be fixed in space (i.e. it is an inertial system] ‘The angular velocity of the xyz system relative to the XYZ system is given by « = 2ti— tj +(2t+4)k where ¢ is the time. The position vector of a particle at time t as observed in the xyz system is given by r = (#+1)i—6tj+4t%. Find (@) the apparent velocity and (b) the true velocity at time t CHAP. 6) MOVING COORDINATE SYSTEMS 149 (a) The apparent velocity at any time t is de/dt = 26 — 6) + 12% Attime (=1 this is i — Of + 12k, (b) The true velocity at any time ¢ is efat + eX x = (BH—6}+ 120%) + [RE — 8h + (RE Ak) x [E+ DK — 6H) + 4K Attime £=1 this is k 6 +i +]2 1 6] = si y4 mK 2-64 66. Determine the acceleration of a moving particle as seen by the two observers in Problem 6.1. , The acceleration of the particle as scen by the observer in the fixed XYZ system is Dix = Dy(D-x). Using the operator equivalence established in Problem 62, we have DylDyr) = Dy(Dyr} «%) (Dy + 0% \Dyr + ox) = DylDyr + 0%) + 0X Dye + 0X2) Dige + Dayle Xe) + aX Dye + 0X (eX) orsince DyleXe) = (Dye) Xr + «Xue, Diz = Dir + Dys) Xx + 20x (Dut) + 0X (0%2) « If r is the position vector expressed in terms of i,j,k of the moving coordinate system, then the acceleration of the particle relative to this system is, on dropping the subscript Mf, @r Pe by, be, ae ~ gal * alt aak o ‘The acceleration of the particle relative to the fixed system is given from (J) as @ Pr, de de Gel = Foe dees nex(H) + exten ® ‘The acceleration (3) is sometimes called the true acceleration, while (2) is the apparent acceleration. 6.7. Find (a) the apparent acceleration and (b) the true acceleration of the particle in Proce ei a , $= £() = fe-osmm = atom Attime (= 1 this is ai + 24k, (0) The true a ata Ft ex(exn) Attime ¢=1 this equals 2h + 2k + (41-2) + 12k) x (Rk oj + 12k) + (B12) 4.24) x (21 — 6) + 4K) + (BL= 5 +.6k) x ((21~ 5 + 6k) x (21 6) + 4k} BL + 24k + (48i—24j— 20K) + | 40i + 1845 + 36K = Aj 8k) + (14h + 2125+ 40%) 150 MOVING COORDINATE SYSTEMS [omar.6 CORIOLIS AND CENTRIPETAL ACCELERATION 68. Referring to Problem 6.5, find (a) the Coriolis acceleration, (b) the centripetal acceleration and (¢) their magnitudes at time ¢=1. (©) From Problem 6.5 we have, Coriolis acceleration = 24% de/dt = (412) +12k) x (2i— 6) + 12k) 481 — 24) — 20k (®) From Problem 6.5 we have, Centripetal acceleration = #X (wr) = (i Adi + 2195 + 40K. j + 6k) x (32+ 45 —10k) (o) From parts (a) and (#) we have Magnitude of Coriolis acceleration = V@BFF RIFF C207 = 4305 Magnitude of centripetal acceleration = VIN? > IOP > OF = 2VT1685 MOTION OF A PARTICLE RELATIVE TO THE EARTH 69. (a) Express Newton's second law for the motion of a particle relative to an XYZ coordinate system fixed in space (inertial system). _(b) Use (a) to find an equation of motion for the particle relative to an zyz system having the same origin as the XYZ system but rotating with respect to it. (a) If m is the mass of the particle (assumed constant), d2r/dt? |p its acceleration in the fixed system and F the resultant of all forces acting. on the particle as viewed in the fixed system, then Newton's second law states that (b) Using subscript M to denote quantities as viewed in the moving system, we have from Problem 6.6, Bele axes tox El + oxtexn) © Substituting this into (1), we find the required equation . F = maxx) — am (0 $F.) — mle lo 0) Cy We can drop the subscript M provided it is clear that all quantities except F are as determined by an observer in the moving system. The quantity F, it must be emphasized, is the remultant force es observed in the fixed or inertial system. If we do remove the subscript M and write dr/dt =v, then (8) can be written nk ae =F mOxx) = 2mleX) — max fox] Cy 6.10. Calculate the angular speed of the earth about its axis. Since the earth makes one revolution (2+ radians] about its axis in approximately 24 hours = £86,400 sec, the angular speed i 22 36,400 ‘The actual time for one revolution is closer to 86,164 sec and the angular speed 7.29 x 10- rad/sec, os = 127 x 10-5 rad/see CHAP. 6] MOVING COORDINATE SYSTEMS 151 MOVING COORDINATE SYSTEMS IN GENERAL 6.11. Work Problem 6.4 if the origins of the XYZ and zyz systems do not coincide. Let R be the position vector of origin @ of the sys system relative to origin 0 of the fixed (or inertial) XYZ system (see Fig. 6-8]. The velocity of the par- ticle P relative to the moving system is, as before, dr) de de, dy, de atl ~ de > alt aed + ae ka) Now the position vector of P relative to O is b= Retr and thus the velocity of P as viewed in the XYZ system is eae a a& = eel, = ale te = hte axe = R+Gte @) using equation (8) of Problem 6.4. Note that itis the velocity of Q with respect to 0. If R=0 this re- duces to the result of Problem 6.4. Fig. 6-3 6.12. Work Problem 6.6 if the origins of the XYZ and ayz systems do not coincide. Referring to Fig. 6-8, the acceleration of the particle P relative to the moving system is, as before, oe @e _ ea, , &y, , @ ihe = ae = Get + G+ ae ” Since the postion vestor of P relative to O Is p=R++4, the aceleration of P as viewed in the XY2 system is e e a aot), * ale ha Pty te a te Bet exe taxon using equation (9) of Problem 6.6. Note that it is the acceleration of Q with respect to 0. If this reduces to the result of Problem 6.6. 6.13. Work Problem 6.9 if the origins of the XYZ and xyz systems do not coincide. (@) The position vector of the particle relative to the fixed (XYZ) system isp. Then the required equation of motion Fe F © maele (8) Using the result (2) of Problem 6.12 in (1), we obtain = (a x2) = Bmax ¥) = X (xe) ® ‘where F is the force acting on m as viewed in the inertial system and where v 6.14. Find the equation of motion of a particle relative to an observer on the earth's surface. 152 MOVING COORDINATE SYSTEMS (CHAP. 6 ‘We assume the earth to be a sphere with eenter at O (Fig. 6-4) rotating about the Z axis with angular velocity # = wK. We also use the fact that the effect of the earth's rotation around the sun is negligible, so that the XYZ system can be taken as fan inertial system, ‘Then we can use equation (2) of Problem 6.12. For the ease of the earth, we have seo o B= exexm © v= - Se, cy the first equation arising from the fact that the ro- tation of the earth about its axis proceeds with eon- stant angular velocity, the second arising from the fact that the acceleration of origin Q relative to O is the centripetal acceleration, and the third arising from Newton’s law of gravitation. Using these in {ro Stel Shae caval ato ress =, — aoe) — Hare) — axtexe @ 1 that other forces acting on m [such as air resistance, ete] are neglected. ‘We can define 4 B= exten © th scr due oy 2 th) bom = B — AwXv) — eX (eXr) (0) ae Near the earth's surface the last term in (6) can be neglected, so that to @ high degree of ‘approximation, or Pr = g- 26x ” In practice we choose g as constant in magnitude although it varies slightly over the earth's surface. If other external forces act, we must add them to the right side of equations (8) or (7). 6.15. Show that if the particle of Problem 6.14 moves near the earth’s surface, then the equations of motion are given by B = 2weosry 2u cosa # + w sind 2) i @ ~9 + Qosinrd the colatitude (see Fig. 6-4] and 90°— is the latitude. From Fig. 6-4 we have where the angle 2 is KDE + OR DH + (1K (Cin a) + 05 + (cosa)k = sind i + cos ie and #0 # = eK = ~esinnd + cosa CHAP. 6] MOVING COORDINATE SYSTEMS 158 ‘Then eX = ex Gite tiy io5 k mr 0 weosd eG @ (-w cond ilk + (w cond + wind) — (w sina De ‘Thus from equation (7) of Problem 6.14 we have or gh = e- %exy : ok + 2 cosAFi — Be conrZ + wsina dj + 2osinadk Equating corresponding coefficients of i,j,k on both sides of this equation, we find, as required, B= Beery © Be cosh # + w sind 8) ® 0 + 2osinnd o 6.16. An object of mass m initially at rest is dropped to the earth’s surface from a height which is small compared with the earth’s radius. Assuming that the angular speed of the earth about its axis is a constant », prove that after time ¢ the object is deflected east of the vertical by the amount jugt* sin. ‘Method 1. ‘We assume that the object is located on the z axis at #=0, y ‘equations (1) and (2) of Problem 6.15 we have on integrating, = Wwoosrytey F = —Mwcosretusinns) +o , 2 =H [ave Fig. 6-4). From 25h wehave ¢=0, ¢=20sindh. Thus =o coed e+ wind 2) + Qo sind h wo Since at a — dot sind (cos n 2 + sind (2—A)] But since the terms on the right involving o? are very small compared with —g we can neglect them and write 7=—g. Integration yields 7=—gttey Since 2=0 at t=0, we have e=0 or bs -ot ® ‘Using equation (2) and the first equation of (1) in equation (2) of Problem 6.15 we find FH = (Pa cos rile cosd w) + (Bo sin r\(-98) tut cos? y + 2u sind ot ‘Then neglecting the first term, we have j= 20 sind gt. Integrating, b= egsinnr +e Since at t=0, wehave =O and j= og sind Integrating again, y= fog sinn 8 + os ‘Then since y=0 at t=0, ¢=0 so that, as required, y= fogsinn @ Method 2. Integrating equations (1), (2) and (8) of Problem 6.18, we have 2wcosry + ey Be cosh 2 + wsinds) + oy ot + Qo sindy + 5 154 6.7. THE 6.18. MOVING COORDINATE SYSTEMS (cHaP. 6 Using the fact that att Boh sind, cy =0. Thus z= h, we have Bo cosh y Be coshe +o mot + 2osind y Aa) + Boh sind Integrating these we find, using the above conditions, Se ® y= tanesinn — 2a cons fede = tein fs © f= ha yet + toains f yae © Since the unknowns are under the integral sign, these equations are called integral equations We shall use a method called the method of successive approzimations or method of iteration to obtain a solution to any desired accuracy. The method consists of using a first guess for 21,2 tunder the integral signs in (4), (6) and (6) to obtain a better guess. As a first guess we can try 2=0, y=0, 7=0 under the integral signs. Then we find as a second guess = 0, y= Behtsind, 2 = h— Jott Substituting these in (4), (5) and (@) and neglecting terms involving o%, we find the third guess 2 = 0, y = Bwhtsimd ~ 2osinniht— Joe) = fugHsind, = = h— doe neglecting terms involving w?, we find the fourth guess foo sing, = = A Jott Since this fourth guess is identical with the third guess, these results are accurate up to terms involving «?, and no further guesses need be taken. It is thus seen that the deflection 00 sinh, as required ‘Using these in (4), (6) and (6) and agai e=O Referring to Problem 6.16, show that an object dropped from height h above the earth's surface hits the earth at a point east of the vertical at a distance Soh sind V2A/9. From (2) of Problem 6.16 we have on integrating, 2=—Jg@+e, Since s=h at (=0, e=h and 2—h— 4g, Then at 2=0, h= 4g? or ¢= VOh/g. Substituting this value of ¢ into (3) of Problem 6.16, we find the required distance. FOUCAULT PENDULUM Derive an equation of motion for a simple pendulum, taking into account the earth's rotation about its axis. Choose the xy2 coordinate system of Fig. 6-5. Suppose that the origin O is the equilibrium position of the bob B, A is the point of suspension and the length of string AB is LI the tension in the string is T, then we have + (PDI + (Took o Since the net force acting on B is T+ mg, the equa- tion of motion of B is given by [see Problem 6.14] T+ mg — 2mla Xv) ~ max (exe) (2) Fig.65 CHAP. 6} MOVING COORDINATE SYSTEMS 155 6.19. 620. If we neglect the last term in (2), put g form as ok and use (1), then (2) ean be written in component Tell) + 2muij cos ® =T(y/l) — 2me(é cos + # sind) w B= Tal — mg + Amey sind 6 By assuming that the bob of the simple pendulum in Problem 6.18 undergoes small oscillations about the equilibrium position so that its motion can be assumed to take place in a horizontal plane, simplify the equations of motion. ‘Making the assumption that the motion of the bob takes place in a horizontal plane amounts to assuming that @ and # are zero, For small vibrations (I—2)/I is very nearly equal to one. ‘Then equation (5) of Problem 6.18 yields 0 = 7 mg + 2m sind o T= mg ~ 2mui sind w Substituting (2) into equations (#) and (4) of Problem 6.18 and simplifying, we obtain Boa 82 4 Beh SIOD 5 945 205 @ Au, Bev SI» 9,5 gag ou +. Ravi Bad 008 © ‘These differential equations are non-linear because of the presence of the terms involving ei) and yj. However, these terms are negligible compared with the others since w, 7 and y are small. Upon neglecting them we obtain the linear differential equations gel + 2uij cos w ou/t — 2aié cos © Solve the equations of motion of the pendulum obtained in Problem 6.19, assuming suitable initial conditions. ‘Suppose that initially the bob is in the yz plane and is given a displacement from the = axis ‘of magnitude A >0, after which it is released. Then the initial conditions are A,v=0 0 at e=0 © ‘To find the solution of equations (s) and () of Problem 6.19, it is convenient to place KP = gl, a = weosn ® 0 that they become Kis + ta) ® —Kty ~ 2a w It is alzo convenient to use complex numbers. Multiplying equation (4) by i and adding to (3), wwe find KX e+ iy) + Bal 18) = Ke + iy) Bialé +i) Then calling u=2+ iy, this can be written = Ku aint or H+ Bias + Ku = 0 © If w= Cer where C and y are constants, this becomes + Biny + KE = 0 0 that, y= (Rint VERY = ia ®VE 6) Now since a? =f cos?A is small compared to K?= g/l, we can write y= wie ti o 156 MOVING COORDINATE SYSTEMS [cHAP. 6 ‘Then solutions of the equation are (allowing for complex coefficients) (C4 icgeMe-M and (Cy FCoE and the general solution is = (LH ICge MOM 4 (Cy FICE IE o where C,, Co, Cy, Cy are assumed real. Using Euler's formulas eo? = cone + ising, = cose — ising fand the fact that w= 2+4y, (8) can be written et iy = (C,+ iC) (cos (a— K)t— isin (@— Kye} + (Cy iC){cos a+ Kt ~ i sin (a+ KYA) Equating real and imaginary parts, we find ® = Cyeosla—K)t + Cyain (a Kyt + Cy cos (a+ K)t + Cy sin(at Kt (10) Wy = =G; sin (a—K)t + Cy 008 («KM ~ Cy sin (a Kt + Cy cos a+ Kit « Using the initial condition 2=0 at ¢=0, we find from (10) that C,+C)=0 or C=C, Similarly, $=0 at £=0, we find from (10) that c c= = © c08 + Gai eems) Now since © cos) is small compared with Vil, we have, to a high degree of approximation, C= Cy ‘Thus equations (10) and (11) become f= C,cos(e—Kyt + Cysin (e—K)t — C, cos (a+ Kt + Crsin (a+ Kt (1a) y= ~Cysin(e—K)t + Cz cos(a—KIt + Cy sin(a+ Kit + Cyeos(a+ KE (18) Using the initial condition j= 0, (18) yields C,=0, Similarly using y=A at t=0, we find C= 44. Thus (12) and (18) become 2 = $Asin(e— Kit + fA sina Kit y= JA cosla—K)t + 4A cos(a+ Kt or = A cosKt sinat ww y= A cost cosat ey 2 = A cos Vgilt sin (wcosd t) 5) v= A cos Vaiit cose cosh t) 621. Give a physical interpretation to the solution (15) of Problem 6.20. In vector form, (15) can be written r= aity) = A cosVollin where nn = isin (o cos r)t + j cos (u cos yt unit vector ‘The period of cos Vat (namely, 2xVU79) is very small compared with the period of m [namely, aellacoan)|, Te follows that m is 4 very slowly tarning vector. ‘Thos physically the pendulum sclitatcs Wy alplane through the s axis whieh is slowly rotating (or precering) about the = axis. Now at t=0, m=4 and the bob is at yA. After a time = 2s/(4y cosa), for example, 4V2i+ Jv2j 60 that the rotation of the plane is proceeding in the clockwise direction as Tiewed trom shove the earth's surface in the northern hemisphere (where cosh > 0), In the Touthern hemisphere the rotation of the plane fs counterclockwise. ‘The rotation of the plane was observed by Foucault in 1851 and served to provide laboratory evidence of the rotation of the earth about it6 axis, CHAP. 6) MOVING COORDINATE SYSTEMS 187 MISCELLANEOUS PROBLEMS 622. The vertical rod AB of Fig. 6-6 is rotating with constant angular velocity «. A light inextensible string of length I has one end attached at point 0 of the rod while the other end P of the string: has @ ‘mass m attached. Find (a) the tension in the string and (b) the angle which string OP makes with the vertical when equilibrium conditions prevail. Choose unit vectors i and k perpendicular and parallel respectively to the rod and rotating with it. ‘The unit vector ican be chosen perpendicular to the plane of i and k. Tat, = Tsinei — loose k be the position vector of m with respect to O. Three forces act on particle m (‘The weight, mg = —mgk i) The centrifugal force, bee Hime X (eX )) = (lok) x (ok x [Ein ¢ 1 Fc08# kD} = ~m{ [ak] x (al sin 6) = mat in 0 ‘The tension, -T = —Tsinei + Teosek ‘When the partie ibrium, the resultant of all these forces is zero. ‘Then —mgk + mil singi— Tsinei + Teosek = 0 ie, (motl sine —T sino)i + (Tcoss— mk = 0 or mil sing — Tsing = 0 © Teoso—mg = 0 ® Solving (1) and (2) simultaneously, we find (a) T= mu%l, (6) @ = cos! (g/et). Since the string OP with mass m at P describe the surface of a cone the system is sometimes called a conieal pendulum. 623. A rod AOB (Fig. 6-1] rotates in a vertical plane [the yz plane] about a horizontal axis through O perpendicular to this plane (the 2 axis| with constant angular velocity ». Assuming no frictional forces, determine the motion of a particle P of mass m which is constrained to move along the rod. An equivalent problem exists when the rod AOB is replaced by a thin hollow tube inside which the particle can move. A Fig. 6-7 At time t let r be the position vector of the particle and ¢ the angle made by the rod with the y axis. Choose unit vectors j and k in the y and z directions respectively and unit vector i=}xk. Let r, be a unit vector in the direction r and 6, a unit vector in the direction of Increasing ¢, 158 6.24. MOVING COORDINATE SYSTEMS (CHAP.6 ‘There are three forees acting on P: (The weight, Gi) The centrifugal force, mgk = —mg sino r,—mg cose @ mle X (Xx) = —m[ehX (oi x rm] ~mfsilai- rrp ~ relat} —m|0— ire) = metre, (iil) The reaction force N=N@, of the vod which is perpendicular to the rod since there are no frictional or resistance forces. ‘Then by Newton's second law, OE = ok + mete, + « whiny = mma sine = mace, + mde + Ny = (mute — mg sin dey + (N= mg £08 6)8, It follows that N= mg cos# and @rid? = wr —-gsine @ Since §=., a constant, we have #=t if we assume ¢=0 at ¢=0. Then (1) becomes @r/de — er = ~o inet ® It we assume that at ¢=0, r= rq d/dt = om (Mette) aot (Bas tele + Leatnes (4B) (Geen + shames ® or in term of hyperbolic fonctions, 1 = recoshut + (2 — Ps) sinh ut + 525 ain so enthat + (52 fy) snhat + fain ot ” (a) Show that under suitable conditions the particle of Problem 6.23 can oscillate along the rod with simple harmonic motion and find these conditions. (6) What happens to the particle if the conditions of (a) are not satisfied? (a) The particle will oscillate with simple harmonic motion along the rod if and only if r= 0 fand ‘v= g/2v. In this case, r= (g/2s!) sinot. Thus the amplitude and period of the simple hharmonie motion in such eage are given by g/2st and 2r/o respectively. (6) If %=(o/2u)— erg then r= ree#t + (g/2s#) sin wt and the motion is approximately simple hharmonie after some time, Otherwise the mass will ultimately fly off the rod if it is fnite A projectile located at colatitude 2 is fired with velocity vo in a southward direction at an angle « with the horizontal. (a) Find the position of the projectile after time t. (b) Prove that after time ¢ the projectile is deflected toward the east of the original vertical plane of motion by the amount fog sind f — uve cos (e—a) # (a) We use the equations of Problem 6.15. Assuming the projectile starts at the origin, we have =0 wo * 20,250 at ‘Also, the initial velocity is vp = 2 coset + vysinak so that ® E=vpeone, F=0, F=vysina at CHAP. 6) MOVING COORDINATE SYSTEMS 159 Integrating equations (1), (2) and (8) of Problem 6.15, we obtain on using conditions (2), 2 = QWeosry + ry cose ® H = 2ecordetesinda) o 2 = ~gt + Qosinny + ep sina © Instead of attempting to solve these equations directly we shall use the method of iteration ive approzimations as in Method 2 of Problem 6.16. Thus by integrating and u (2), we find 2 = mocons Spe + osc y= teens fica ~ tes ff ede ” f= dnysinat = Jor + tosis fv o ‘As a first guess we use 2=0, y=0, 2 ‘become, neglecting terms involving 2, under the integral signs, Then (6), (7) and (8) 2 = (yeoaat o y=0 «o) = (msinalt ~ jot ry To obtain a better guess we now use (0), (10) and (11) under the integral signs in (6), (7) and @), thus arriving at 2 = (tycosaye a) ¥ = ~wup cola 2) # + foot sind «sy £ = (ipsinalt — Jott ws) where we have again neglected terms involving ot Further guesses again produce equations (42), (18) and (14), so that these equations are accurate up to terms involving af. (4) From equation (13) we see that the projectile is deflected toward the east of the #2 plane by the amount ag! sind — vy cos(a—r) @. If v= 0 this agrees with Problem 6.16, Prove that when the projectile of Problem 6.25 returns to the horizontal, it will be at ‘the distance ‘ smesinte (5 a » SFr (8 cosa cosh + sina sinX) to the west of that point where it would have landed assuming no axial rotation of the earth. ‘The projectile will return to the horizontal when z=0, ie, (wosinayt— yo@ = 0 ort = Grsinadio Using this value of t in equation (18) of Problem 6.25, we find the required result. 160 MOVING COORDINATE SYSTEMS (CHAP. 6 Supplementary Problems ROTATING COORDINATE SYSTEMS. VELOCITY AND ACCELERATION 621. 628. 620. 6.0, 632. 633. ‘An yz coordinate system moves with angular velocity = 2i~3)+5k relative to a fixed or inertial XYZ coordinate system having the same origin. If a vector relative to the zyz system ia given as a function of time t by A = sinti— cost} + e-tk, find (a) dA/dt relative to the fixed aystem, (8) dA/at relative to the moving system. Ans. (a) (6 cost ~ Se-Hi + (6 sint — 26-9} + Bint ~ 2 cost — ek () conti + sintj ~ ek Find @PA/d¢® for the vector A of Problem 6.27 relative to (a) the fixed system and (b) the moving system, Ans. (a) (6 cost — 45 sint + 160"! + (40 cost — 6 sint ~ 1e “+ (10 sin €~ 23 coat + 160k (®) “sin 4 + costs + otk 0 with angular velocity = 51—4)—10k relative to a fixed sme origin. Find the velocity of a particle fixed in the xys ‘system at the point (3, 1, 2) as seen by an observer fixed in the XYZ system, Ans. 18i—20}+17 Diseuss the physical interpretation of replacing by —« in (a) Problem 6.4, page 148, and () Problem 6.6, page 149, Explain from a physical point of view why you would expect the result of Problem 6.8, page 148, to be correct. ‘An ays coordinate system rotates with angular velocity w = costi+ sint j+k with respect to a fixed XYZ coordinate system having the same origin. If the position vector of a particle is given by r= sinti—costj-+ tk, find (a) the apparent velocity and (6) the true velocity at any time t. Ane, (a) costi + sinej +k (0) (tsing + 2costhi + sint — t cost} Determine (a) the apparent acceleration and (6) the true acceleration of the particle of Problem 6.32. Ans. (a) ~sintitcost} (0) @tcost — Bsindi + (Beost + Bsingj + (1 — Oke CORIOLIS AND CENTRIPETAL ACCELERATIONS AND FORCES 6 63. 636. 6st. 6.38. 6.29. 640. eal. ‘A ball is thrown horizontally in the northern hemisphere. (a) Would the path of the ball, if the Coriolis foree is taken into account, be to the right or to the left of the path when it is not taken into account as viewed by the person throwing the ball? (2) What would be your answer ‘to (a) if the ball were thrown in the southern hemisphere? Ans. (a) to the right, (b) to the left ‘What would be your answer to Problem 6.84 if the ball were thrown at the north or south poles? Explain why water running out of a vertical drain will swirl counterclockwise in the northern hemisphere and clockwise in the southern hemisphere. What happens at the equator? Prove that the centrifugal force acting on a particle of mass m on the earth’s surface is ‘vector (a) directed away from the earth and perpendicular to the angular velocity vector # and (8) of magnitude m.2R sin where \ is the colatitude. In Problem 6.87, where would the centrifugal force be (a) a maximum, (8) a minimum? Ans. (a) at the equator, (b) at the north and south poles. Find the centrifugal foree acting on a train of mass 100,000 kyr at (a) the equator (8) colatitude 0°. Ans. (a) 86.0 kg wt, (0) 17.5 ke wt () A river of width D flows northward with a speed vp at colatitude X. Prove that the left bank of the river will be higher than the right bank by an amount equal to (2a cos Ng? + Auto cos? n)—1 where « is the angular speed of the earth about its axis. () Prove that the result in part (a) is for all practical purpores equal to (2Duv cos \)/0- If the river of Problem 640 is 2km wide and flows at a speed of S km/hr at colatitude 45°, how much higher will the left bank be than the right bank? Ana, 2.9 em CHAP. 6) MOVING COORDINATE SYSTEMS 161 62, 6.3, ‘An automobile rounds @ curve whose radius of curvature is p. If the coeficient of friction is prove that the greatest speed with which it can travel so as not to slip on the road is Ved. Determine whether the automobile of Problem 6.42 will slip if the speed is 60 mifhr, »=.05 and (a) p = 500 ft, (0) p= 50ft, Discuss the results physically. MOTION OF A PARTICLE RELATIVE TO THE EARTH at, 6.45, 6.46, ear. 6.49, 6s. 652, ‘An object is dropped at the equator from a height of 400 meters, If air resistance is neglected, hhow far will the point where it hits the earth's surface be from the point vertically below the Initial position? Ans, 17.6 em toward the east Work Problem 6.44 if the object is dropped (a) at colatitude 60° and (b) at the north pole. Ans, (a) 15.2 em toward the east An object is thrown vertically upward at colatitude 2 with speed vy, Prove that when it returns it will be at a distance westward from its starting point equal to (4evi sin »)/30?. ‘An object at the equator is thrown vertically upward with a speed of 60 mi/hr. How far from its initial position will it land? Ans. .78 inches ‘With what speed must the object of Problem 6.47 be thrown in order that it return to a point fon the earth which is 20 ft from its original position? Ans. 406 mi/hr ‘An object ia thrown downward with initial speed 1y- Prove that after time t the object is defected east of the vertical by the amount vy sind 2 Jog sind Prove that if the object of Problem 6.49 is thrown downward from height h above the earth's surface, then it will hit the earth at a point east of the vertical at a distance Gar (Va + BOR — voR VT BOR + 209) Suppose that the mass m of @ conical pendulum of length I moves in a horizontal circle of radius a, Prove that (a) the speed is aVg/YF—a? and (b) the tension in the string is moiv= a, If an object is dropped to the earth's surface prove that its path is a semicubieal parabola. ‘THE FOUCAULT PENDULUM 653. 654, 655, Explain physically why the plane of oscillation of a Foucault pendulum should rotate clockwise ‘when viewed from above the earth's surface in the northern hemisphere but counterclockwise in the southern hemisphere. How long would it take the plane of oscillation of a Foucault pendulum to make one complete revolution if the pendulum is located at (a) the north pole, (b) colatitude 452, (c) colatitude 85°? Ans. (a) 28.94 hr, (8) 83.86 hr, (c) 92.60 hr Explain physically why a Foucault pendulum situated at the equator would not detect the rotation of the earth about its axis. Is this physical result supported mathematically? Explain. MOVING COORDINATE SYSTEMS IN GENERAL 656. 651. 658, ‘An zyz coordinate system rotates about the + axis with angular velocity « = costitsint relative to a fixed XYZ coordinate system where t is the time. The origin of the 60°, <60° or a= 60° respectively, is fred with initial velocity oi + vej + vk from the origin of a coordinate system to the earth’s surface at colatitude A, prove that its position at any later time ¢ will be given by = = Ot + ongtt cosa 1 vat = wt%(oy c08d + uy 81nd) + Jog! sind = vt — Jott + avg? sina neglecting terms involving 2, Work Problem 6.89 so as to include terms involving 4? but exclude terms involving wt, ‘An object of mass initially at rest is dropped from height h to the earth's surface at eolatitude 2. ‘Assuming that air resistance proportional tothe instantancous speed of the object i taken Ino Account as well'a the Totation of the earth about ite axis, prove that after time ¢ the object i ‘efcted east of the vertical by the amount Besimd fg —2ngn\t eH) + hte — pot + Jost neglecting terme of order ot and highe ‘Work Problem 6.91, obtaining accuracy up to and including terms of order A frictionless inclined plane of length 1 and angle a located at colatitude \ is so situated that 2 particle placed on it would slide under the influence of gravity from north to south. If the particle starts from rest at the top, prove that it will reach the bottom in a time given by Bi, 2ol sind cosa 4 2obsin coe Taina 30 and that its speed at the bottom is /2al ine ~ $al sin a cosa sind neglecting terms of order 2. (a) Prove that by the time the particle of Problem 6.98 reaches the hottom it will have undergone 48 deflection of magnitude Ble 3 Vo to the east or west respectively according as cos(s+X) is greater than or less than zero. (2) Discuss the case where cos(a+\)=0. (c) Use the result of (a) to arrive at the result of Problem 6.17. = coset) ‘Work Problems 6.93 and 6.94 if the inclined plane has coafficient of friction Chapter 7 SYSTEMS of PARTICLES DISCRETE AND CONTINUOUS SYSTEMS ‘Up to now we have dealt mainly with the motion of an object which could be considered as a particle or point mass. In many practical eases the objects with which we are concerned ‘can more realistically be considered as collections or systems of particles. Such systems are called discrete or continuous according as the particles can be considered as separated from each other or not. For many practical purposes a discrete system having a very large but finite number of particles can be considered as a continuous system. Conversely a continuous system can be considered as a discrete system consisting of a large but finite number of particles. DENSITY For continuous systems of particles occupying a region of space it is often convenient to define a mass per unit volume which is called the volume density or briefly density. ‘Mathematically, if AM is the total mass of a volume Ar of particles, then the density can be defined as o = lim wo ‘The density is a function of position and can vary from point to point. When the density is a constant, the system is said to be of uniform density or simply uniform. ‘When the continuous system of particles occupy a surface, we can similarly define a surface density or mass per unit area. Similarly when the particles occupy a line for curve] we can define a mass per unit length or linear density. RIGID AND ELASTIC BODIES In practice, forces applied to systems of particles will change the distances between individual particles, Such systems are often called deformable or elastic bodies. In some ‘cases, however, deformations may be so slight that they may for most practical purposes be considered non-existent. It is thus convenient to define a mathematical model in which the distance between any two specified particles of a system remains the same regardless of applied forces. Such a system is called a rigid body. The mechanics of rigid bodies is considered in Chapters 9 and 10. DEGREES OF FREEDOM ‘The number of coordinates required to specify the position of a system of one or more particles is called the number of degrees of freedom of the system. Example 1 ‘A particle moving freely in space requires 3 coordinates, ‘the number of degrees of freedom is 3. se (2,442), to specify its position. Thus 165 166 SYSTEMS OF PARTICLES (ouap.7 Example 2. A system consisting of N particles moving freely in space requires SN coordinates to specify its Position. ‘Thus the number of degrees of freedom is 8N. A rigid body which can move freely in space has 6 degrees of freedom, i.e. 6 coordinates are required to specify the position. See Problem 7.2. CENTER OF MASS Let rirs,...,tv be the position vectors of a system of N particles of masses ‘m1, Mz, ...ymn respectively [see Fig. 7-1]. The center of mass or centroid of the system of particles is defined as that point C having position vector + tars + sss tmery 1S e mn ae me um @) + tg FF My Fig. 741 Fig.7-2 For continuous systems of particles occupying a region R of space in which the volume density is o, the center of mass can be written Sora Soe where the integral is taken over the entire region [see Fig. 7-2]. If we write FS ++i, n= nit i+ ok then (3) can equivalently be written as FS ) a Lime, | Zim Lm w eee M M Ge and pet ©) where the total mass is given by either M= om © or Ms Sod “ CHAP. 7] SYSTEMS OF PARTICLES 167 ‘The integrals in (8), (5) or (7) can be single, double or triple integrals, depending on which may be preferable. In practice it is fairly simple to go from discrete to continuous systems by merely replacing summations by integrations. Consequently we will present all theorems for discrete systems. CENTER OF GRAVITY If a system of particles is in a uniform gravitational field, the center of mass is some- times called the center of gravity. MOMENTUM OF A SYSTEM OF PARTICLES If w= drat i, is the velocity of m,, the total momentum of the system is defined as 5 sme 7 Zmin @) We can show [see Problem 7.3] that = ME (9) where v= di/dt is the velocity of the center of mass. ‘This is expressed in the following Theorem 7.1. The total momentum of a system of particles can be found by multiplying the total mass M of the system by the velocity ¥ of the center of mass. MOTION OF THE CENTER OF MASS ‘Suppose that the internal forces between any two particles of the system obey Newton's third law. ‘Then if F is the resultant external force acting on the system, we have (see Problem 7.4) . oe ye (10) at aw at This is expressed in Theorem 7.2. ‘The center of mass of a system of particles moves as if the total mass and resultant external force were applied at this point. CONSERVATION OF MOMENTUM Putting F=0 in (10), we find that P my = constant an) ‘Thus we have Theorem 7.3. If the resultant external force acting on a system of particles is zero, then the total momentum remains constant, ie. is conserved. In such case the center of mass is either at rest or in motion with constant velocity. ‘This theorem is often called the principle of conservation of momentum. It is a generaliza- tion of Theorem 2-8, page 37. 168 SYSTEMS OF PARTICLES [onap.7 ANGULAR MOMENTUM OF A SYSTEM OF PARTICLES The quantity x a = Sminxv) a2) is called the total angular momentum [or moment of momentum] of the system of particles about origin 0. THE TOTAL EXTERNAL TORQUE ACTING ON A SYSTEM If F, is the external force acting on particle », then r,xF, is called the moment of the force F, or torque about O. The sum A Saxe, as) is called the total external torque about the origi RELATION BETWEEN ANGULAR MOMENTUM AND TOTAL EXTERNAL TORQUE If we assume that the internal forces between any two particles are always directed along the line joining the particles [ie. they are central forces), then we can show as in Problem 7.12 that an a= 2 ( Thus we have Theorem 7.4. The total external torque on a system of particles is equal to the time rate of change of the angular momentum of the system, provided the internal forces between particles are central forces. CONSERVATION OF ANGULAR MOMENTUM Putting A =0 in (14), we find that @ = Smilnxv,) = constant (as) ‘Thus we have Theorem 7.5. If the resultant external torque acting on a system of particles is zero, then the total angular momentum remains constant, i.e. is conserved. ‘This theorem is often called the principle of conservation of angular momentum. It is the generalization of Theorem 2.9, page 37. KINETIC ENERGY OF A SYSTEM OF PARTICLES ‘The total Kinetic energy of a system of particles is defined as Te 1S nt T= dmot = 2D mit (a6) WORK If Fy is the force (external and internal) acting on particle v, then the total work done in moving the system of particles from one state (symbolized by 1] to another [symbolized by 2] is xt We = 3% SFede an CHAP. 7) SYSTEMS OF PARTICLES 169 As in the case of a single particle, we can prove the following Theorem 7.6. The total work done in moving a system of particles from one state where the kinetic energy is 7; to another where the kinetic energy is T2, is We = %-T: (18) POTENTIAL ENERGY. CONSERVATION OF ENERGY ‘When all forces, external and internal, are conservative, we can define a total potential energy V of the system. In such case we can prove the following Theorem 7.7. If T and V are respectively the total kinetic energy and total potential energy of a system of particles, then T+V = constant (a9), This is the principle of conservation of energy for systems of particles, MOTION RELATIVE TO THE CENTER OF MASS It is often useful to describe the motion of a system of particles about [or relative to] the center of mass. The following theorems are of fundamental importance. In all cases primes denote quantities relative to the center of mass. Theorem 78. The total linear momentum of a system of particles about the center of mass is zero. In symbol ym = Dm = 0 (20) Theorem 7.9, The total angular momentum of a system of particles about any point 0 equals the angular momentum of the total mass assumed to be located at the center of mass plus the angular momentum about the center of mass. In symbols, 8 = exMy + 3S mieixw) en Theorem 7.10. The total kinetic energy of a system of particles about any point O equals the kinetic energy of translation of the center of mass [assuming the total mass located there] plus the kinetic energy of motion about the center of mass. In symbols, = Aye 413 no T= gue +53 mx (e) Theorem 7.11. The total external torque about the center of mass equals the time rate of change in angular momentum about the center of mass, ie. equation (14) holds not only for inertial coordinate systems but also for coordinate systems moving with the center of mass. In symbols, ao wos oF es) If motion is described relative to points other than the center of mass, the results in the above theorems become more complicated. IMPULSE If F is the total external force acting on a system of particles, then 170 SYSTEMS OF PARTICLES (CHAP. 7 Sora en is called the total linear impulse or briefly total impulse. As in the case of one particle, ‘we can prove Theorem 7.12. The total linear impulse is equal to the change in linear momentum, Similarly if is the total external torque applied to a system of particles about origin O, then . S" sae (25) is called the total angular impulse. We can then prove Theorem 7.13. The total angular impulse is equal to the change in angular momentum. CONSTRAINTS. HOLONOMIC AND NON-HOLONOMIC CONSTRAINTS Often in practice the motion of a particle or system of particles is restricted in some way. For example, in rigid bodies [considered in Chapters 9 and 10] the motion must be such that the distance between any two particular particles of the rigid body is always the same. As another example, the motion of particles may be restricted to curves or surfaces. The limitations on the motion are often called constraints. If the constraint condition can be expressed as an equation oe (26) connecting the position vectors of the particles and the time, then the constraint is called holonomic. If it cannot be so expressed it is called non-holonomic. VIRTUAL DISPLACEMENTS Consider two possible configurations of a system of particles at a particular instant which are consistent with the forces and constraints. To go from one configuration to the other, we need only give the »th particle a displacement 6r. from the old to the new position. We call 8r, a virtual displacement to distinguish it from a true displacement [denoted by dr.] which occurs in # time interval where forces and constraints could be changing. The symbol 8 has the usual properties of the differential d; for example, a(sin#) = cose 80. STATICS OF A SYSTEM OF PARTICLES. PRINCIPLE OF VIRTUAL WORK In order for a system of particles to be in equilibrium, the resultant force acting on each particle must be zero, ie. F,=0. It thus follows that F,-8r,=0 where F,- bry is called the virtual work. By adding these we then have Sees = 0 en If constraints are present, then we can write R= e+e (28) where F and F° are respectively the actual force and constraint force acting on the vth particle. By assuming that the virtual work of the constraint forces is zero [which is true for rigid bodies and for motion on curves and surfaces without friction], we arrive at CHAP. 7) SYSTEMS OF PARTICLES am Theorem 7.14. A system of particles is in equilibrium if and only if the total virtual work of the actual forces is zero, i.e. if SRP = 0 i) ‘This is often called the principle of virtual work, EQUILIBRIUM IN CONSERVATIVE FIELDS. STABILITY OF EQUILIBRIUM The results for equilibrium of a particle in a conservative force field [see page 38) can be generalized to systems of particles. The following theorems summarize the basic results. Theorem 7.15. If V is the total potential of a system of particles depending on coordinates 4,,2,,..., then the system will be in equilibrium if, (1) Since the virtual work done on the system is wv wv By = Fa, + a to (32) is equivalent to the principle of virtual work. Theorem 7.16. A system of particles will be in stable equilibrium if the potential V is a minimum, In case V depends on only one coordinate, say q,, sufficient conditions are wv av, Pa a>? Other cases of equilibrium where the potential is not a minimum are called unstable. D'ALEMBERT'S PRINCIPLE Although Theorem 7.14 as stated applies to the statics of a system of particles, it ean be restated so as to give an analogous theorem for dynamics. To do this we note that according to Newton's second law of motion, B=p or Fp =0 (30) where p, is the momentum of the vth particle, The second equation amounts to saying that a moving system of particles can be considered to be in equilibrium under a force F.—p» ie, the actual force together with the added force —p, which is often called the reversed effective force on particle v. By using the principle of virtual work we can then arrive at Theorem 7.17. A system of particles moves in such a way that the total virtual work FR —p)-8n = 0 (32) With this theorem, which is often called D’Alembert's principle, we can consider dynamics as a special case of statics. 472 SYSTEMS OF PARTICLES (CHAP. 7 Solved Problems DEGREES OF FREEDOM 71. Determine the number of degrees of freedom in each of the following cases: (a) a particle moving on a given space curve; (b) five particles moving freely in a plane; (c) five particles moving freely in space; (d) two particles connected by a rigid rod moving freely in a plane. (@) The curve ean be described by the parametric equations x= z(e), y=yle), z= (6) where ‘ls the parameter. Then the position of a particle on the curve is determined by specifying ‘one coordinate, and hence there is one degree of freedom. (®) Each particle requires two coordinates to specify its position in the plane. Thus 5+2=10 coordinates are needed to specify the positions of all 5 particles, ie. the system has 10 degrees ‘of freedom, (6) Since each particle requires three coordinates to specify its position, the system has 5+8 = 15 degrees of freedom. (@) Method 1. ‘The coordinates of the two particles can be expressed by (21,44) and (ey us), he. a total ints in a constant a (the length © that, one of the coordinates can be degrees of freedom. of 4 coordinates. However, since the distance between these of the rigid rod], we have (z,—z2)? + (yu)? = a ‘expressed in terms of the others. ‘Thus there are 4 Method 2. ‘The motion is completely specified if we give the two coordinates of the center of mass fand the angle made by the rod with some specified direction. Thus there are 2-+1=8 degrees of freedom, 72. Find the number of degrees of freedom for a rigid body which (a) can move freely in three dimensional space, (b) has one point fixed but can move in space about this point. (@) Method 1. If 3 non-collinear points of a rigid body are fixed in space, then the rigid body is also fixed in space. Let these points have coordinates (2,41) (a Wn #2) (ta ta) respectively, @ total of 8. Since the body is rigid we must have the relations (ey 20)? + (vy — a + (2129? = constant, (2225) + (va Ws)? + (aaa (2a— 2) + (vs— wi? + (4a— 219" = constant hhence 3 coordinates can be expressed in terms of the remaining 6. Thus 6 independent coordinates are needed to describe the motion, ie. there are 6 degrees of freedom. Method 2. To fix one point of the rigid body requires 3 coordinates. An axis through this point is xed if we specify 2 ratios of the direction cosines of this axis. A rotation about the axis can then be described by 1 angular coordinate. The total number of coordinates required, ie. the number of degrees of freedom, is 3+2+1=6. (©) The motion is completely specified if we know the coordinates of two points, say (Vs #2) land (ts, Ya), where the fixed point is taken at the origin of a coordinate system. But since the body is rigid we must have abt vf + af = constant, 2f-+vf-+ 2 = constant, (21 —23)*+ (v1 —¥2)?+ (e)— 4)" = constant trom which 8 coordinates can be found in terms of the remaining 8. Thus there are 3 degrees of freedo constant, CENTER OF MASS AND MOMENTUM OF A SYSTEM OF PARTICLES 73. Prove Theorem 7.1, page 167: The total momentum of a system of particles can be found by multiplying the total mass M of the system by the velocity ¥ of the center of mass. CHAP. 7) SYSTEMS OF PARTICLES 1713, Zm,r, he cnr of mas ib detnon, #= 22, ‘Then the total momentum isp = Ym,v, = Emje = Mdelde = at 74, Prove Theorem 7.2, page 16° The center of mass of a system of particles moves as if the total mass and resultant external force were applied at this point. Let F, be the resultant external force acting on particle » while fy is the internal force on Particle »'due to particle 4. We shall assome that "fy ~0, ie. particle » does not exert any force on itself By Newton's second law the total force on particle » is om _ @ B+ Bin = P= Somny “ ‘where the second term on the left represents the resultant internal force on particle » due to all other particles, Summing over» in equation (2), we nd Be BR = d{zmn} © Now according to Newton's third law of action and reaction, f,=~f, so that the double ‘summation on the left of (2) is zero. If we then write Fo dR ad t= YB ® (2) becomes r= wat “ Since F is the total external force on all particles applied at the center of mass f, the required result is proved. 75. A system of particles consists of a 8 gram mass located at (1,0,—1), a 5 gram mass at (—2, 1,8) and a2 gram mass at (3,1, 1). Find the coordinates of the center of mass. ‘The position vectors of the particles are given respectively by “M4549, my = BF +K neick on is given by ‘Thus the coordinates of the center of mass are (yy hs 76. Prove that if the total momentum of a system is constant, i.e. is conserved, then the center of mass is either at rest or in motion with constant velocity. ‘he total momentum of the aytem ie elven by : a ad [2mr,) P= me = Ems = Edm, = wg {emeh ‘Then if p is constant, 80 also is dé/dt, the velocity of the center of ms 7.2. Explain why the ejection of gases at high velocity from the rear of a rocket will move the rocket forward. Since the gas particles move backward with high velocity and since the center of mass does not move, the rocket must move forward. For applications involving rocket motion, see Chapter 8. 174 78. 79. SYSTEMS OF PARTICLES [cHaP. 7 Find the centroid of a solid region ® as in Fig. 7-3. Consider the volume clement ar, of the solid. The mass of this volume clement is AM, = ey dry = op Ae By Sty where », is the density [mass per unit volume) and ty Aypsd%) are the dimensions of the volume element. ‘Then the centroid is given approximately by Beam, _— Bred _ Stren dey ave dey aM, ~ Zed ~ Beday a3 where the summation is taken over all volume elements of the solid. oe ‘Taking the limit as the number of volume elements becomes infinite in such a way that ary of 2,0, dy, 0,2, +0, we obtain for the centroid of the solid Ae Soe | Mvwne © far foea S{S ore where the integration is to be performed over ‘R, as indicated, Writing r= ai+yjtsk, #= 41+) +2k, this can also be written in component form as SS xvsetvie Sf even Sf seseeva SiS vec’ 1 SfFeeeanae” ° "Sif esearae Find the centroid of the region bounded by the plane z+y+z=a and the planes 2=0, y=0, 2=0. ‘The region, which is a tetrahedron, is indicated in Fi the results of Problem 7.8. Tin forming the sum over all volume elements of the region, it is advisable to proceed in an ‘orderly fashion. One possibility is to add first all terms corresponding to volume elements ontained ina column such as PQ in the figure. This amounts to keeping #, and y, fixed and ‘adding over all z,. Next keep , fixed but sum over all y,. This amounts to adding ail columns such as PQ, contained in a slab RS, and consequently amounts to summing over all cubes contain‘ fn such a siab, Finally, vary 2,. This amounts to addition of all slabs such as RS. In performing the integration over ®, we use these same ideas. Thus keeping # and y constant, integrate from z= 0 [base of column PQ] to z=a—z~y {top of column PQ). Next keep 2 constant and integrate with respect to y. This amounts to addition of columns hhaving bases in the ay plane [z = 0) located anywhere from B [where y=0] to S [where 2+y =a or v a~2], and the integration is from y= 0 toy =a—z. Finally, we add all slabs parallel to the yz plane, which ‘amounts to integration from 2=0 to z=a. We thus obtain TA. To find the centroid, we u ‘o (ei + yl + 2k) dedy dx "ode dyde ase, it may be cancelled. The denominator without o is evaluated to be ai/6, and the numerator without o is (al/24)(i+j-+k). Thus the center of mass is aM\i+i+k) or 2= 0/4, 7 ald, #= al4, CHAP. 7] SYSTEMS OF PARTICLES 175 7.10. Find the centroid of a semi-circular region of radius a. Method 1. Using rectangular coordinates. Choose the region as in Fig. 7-5. The equation of the circle C is 22+ y? vere ae fem med constant, then the cortnates of the centroid are ten Sze S f eayae c . ee ° feu” (fou R e Si veea SS vivac c ® fee” ae Note that we can write #=0 immediately, since by symmetry the centroid is on the y axis. ‘The denominator for j can be evaluated without integrating by noting that it represents the semi-circular area which is Jra2. y Fig. 75 Fig. 7.6 Method 2. Using polar coordinates, ‘The equation of the circle is r=a [see Fig. 7-6]. As before, we see by symmetry that the centroid must lie on the y axis, so that 2=0. Since y=rsine and dA~rdrds in polar coordinates, we can write Sorts So Sieamorew Sees C= | ‘711. Find the center of mass of a uniform solid hemisphere of radius a. By symmetry the center of mats lies on the # axis (see Fig. 7-1]. Subdivide the hemisphere into solid cireular plates of radius r, such as ABCDEA. If the center G of such a ring is at distance 2 from the center O of the hemisphere, rete! =a, Then if dz is the thickness of the plate, the volume of each ring is ertds = s(at— 2) de and the mass is ro(a?—24)dz, ‘Thus we have Sh enter fimeae 176 SYSTEMS OF PARTICLES (CHAP. 7 ANGULAR MOMENTUM AND TORQUE 712. Prove Theorem 7.4, page 168: The total external torque on a system of particles is equal to the time rate of change of angular momentum of the system, provided that the internal forces between particles are central forces. As in equation (1) of Problem 7.4, we have a _ 4 tin = a = aim) wo ‘Multiplying both sides of (1) by x, x, we have oe BX, + Be Xty Swe Kelme) ® Since sx Zomy) = elmira) ® ve wnt Baxi = Eten) Summing over » tn (4), we find Bexw + BB = L{Emiexw} © Now the double sum in (5) is composed of terms such as WX ha + 1X he cc) which becomes on writing fy =—fy, according to Newton's third law, BX ha AXE ADK o ‘Then since we suppose that the forces are central, Le. f,, has the same direction as r,—my, it follows that (7) is zero and also that the double sum in (5) is zero, Thus equation (5) becomes sexn = Efzmnxm} oo 4-9 where A = B4,XR, @ = Emule, xn) WORK, KINETIC ENERGY AND POTENTIAL ENERGY 713. Prove Theorem 7.6, page 169: The total work done in moving a system of particles from one state to another with kinetic energies 7, and Tz respectively is T:—T'. ‘The equation of motion of the sth particle in the system is = Loni rh H+ Bin = Hlmid @ Taking the dot product of both sides with f,, we have ® Since (2) can be written 1 Fey = Beh + Black = 5 Almeh o ‘Summing over + in equation (3), we find Bre = Beek + BBMrk = $4 (met) ® CHAP. 7) SYSTEMS OF PARTICLES wT 74. Integrating both sides of (4) with respect to t from 1 to Wa = BS rena = BS eka s BES eka tay we find a1 ed = i [dene Using the fact that ,dt= dr, and the symbole 1 and 2 for the states at times t and t respectively, this can be written Bree, = Efrem + Bz faa, where Ty and Ty ate th tial Kinetic energies at and fy respectively, Sine Wa = BS'ra, © is the total work done (by external and internal forces) in moving the system from one state to ‘another, the required result follows. It should be noted that the double sum in (5) indicating work done by the internal forces, cannot be reduced to zero either by using Newton's third law or the assumption of central forces, ‘This is in contradistinetion to the double sums in Problems 7.4 and 7.12 which ean be reduced to zero. Wis + © T, - ‘Suppose that the internal forces of a system of particles are conservative and are derived from a potential Viv (rs) = Voalrn) where ny = 7a = V@i—m)'+(n— w+ GA) is the distance between par- ticles A and » of the system. (a) Prove that Y Phy-dr, = 3D Lave where fy is the internal force on particle » due to particle A. (©) Evaluate the doublesum SE ("fu-ds. of Problem 7.18. (a) The force acting on particle + is Mae ay ay, grad, Vay = —VyVay O) grad, Var WM = ta ‘The work done by these forces in producing the displacements dr, and dn, of particles » and respectively is Vay = Hy, ‘Then the total work done by the internal forces is BPPbray, = ® ‘the factor $ on the right being introduced because otherwise the terms in the summation ‘would enter twice. 118 SYSTEMS OF PARTICLES [cHAP.7 grating (8) of part (a), we have BES = FEBS me = vie - ve ® where Vi" and V{!t denote the total internal potentials lyay © 108 t, and ¢ respectively. 715. Prove that if both the external and internal forces for a system of particles are conservative, then the principle of conservation of energy is valid. If the external forces are conservative, then we have Fr, = vv, Oy frm veh B fhe = Ble, = vm . where V{"? and V{"° denote the total external potential zy. at times t; and fy respectively. f Using (2) and equation (4) of Problem 7.14(8) in equation (6) of Problem 718, we find mot, = Vie — vie + vim — sh = v= vy a where v= vee + vim and vy = ye + fhe o are the respective total potential energies [external and internal] at times ¢, and ty, We thus find from (8), Tr4Vi = Tet Ve or HV = constant 6 which is the principle of conservation of energy. MOTION RELATIVE TO THE CENTER OF MASS 746. Let r/ and v, be respectively the position vector and velocity of particle v relative to the center of mass. Prove that (a) D m.r, = 0, (b) Dmev, = 0. (a) Let x, be the position vector of particle + relative to O land # the position vector of the center of mass C relative to 0. Then from the definition of the center of mass, B= Bm © where M = Sm,. From Fig. 7-8 we have ate @ ‘Then substituting (2) into (1), we find 1 Fo= qimty = gmt from which Emr = 0 o (0) Differentiating both sides of (3) with respect to t, we have Z myv, CHAP. 7] SYSTEMS OF PARTICLES 179 717. Prove Theorem 7.9, page 169: The total angular momentum of a system of particles about any point O equals the angular momentum of the total mass assumed to be located at the center of mass plus the angular momentum about the center of mass. Let r, be the position vector of particle » relative to O, # the position vector of the center of mass C relative to O and +’ the position vector of particle » relative to C. Then nearte 0 Differentiating with respect to t, we find R+to way ® ‘where ¥ is the velocity of the center of mass relative to O, v, is the velocity of particle » relative to 0, and v5 is the velocity of particle » relative to C. ‘The total angular momentum of the system about O a Smile x wd = Sm HH xy +A) Bmteixvy + Bmx + Zmlexy) + Bmx ® Now by Problem 7.16, amicxn = femefav = 0 pean a Aran Bmx = {zmbexn = mexn ‘hen (becomes, as required, 0 = Emi_xvy + Mexn 7.18. Prove Theorem 7.10, page 169: The total kinetic energy of a system of particles about any point O equals the kinetic energy of the center of mass [assuming the total mass located there] plus the kinetic energy of motion about the center of mass. ‘The kinetic energy relative to O [see Fig. 7-8] is T= 5Emod = FBmb,-%) a Using equation (2) of Problem 7.16 we find R= PER = Ftv, ‘Thus (1) can be written T= FEmwt yyy) since By, by Problem 7.16. 180 SYSTEMS OF PARTICLES (CHAP. 7 IMPULSE 7.19. Prove Theorem 7.12: The total linear impulse is equal to the change in linear momentum, ‘The total external force by equation (4) of Problem 7.4 is Pe ed ros wot = ay — Mv where p,= MF; and py= MV, represent the total momenta at times t, and tz respectively. CONSTRAINTS. HOLONOMIC AND NON-HOLONOMIC CONSTRAINTS 720. In each of the following cases state whether the constraint is holonomic or non- holonomic and give a reason for your answer: (a) a bead moving on a circular wire; (®) a particle sliding down an inclined plane under the influence of gravity; (c) a particle sliding down a sphere from a point near the top under the influence of gravity. (@) The constraint is holonomie since the bead, which can be considered particle, is constrained to move on the circular wire. (0) The constraint is holonomic in this case a plane, ince the particle is constrained to move along a surface which is (©) The constraint is non-holonomic since the particle after reaching a certain location on the sphere will leave the sphere. ‘Another way of seeing this is to note that if r is the position vector of the particle relative to the center of the sphere as origin and a is the radius of the sphere, then the particle moves so that x? 202 Th non-holonomic constraint since it is not of the Form (20), page 170. An example of holonomic constraint would be x? = a STATICS. PRINCIPLE OF VIRTUAL WORK. STABILITY 721. Prove the principle of virtual work, Theorem 7.14, page 171. For equilibrium, the net resultant force F, on each particle must be zero, s0 that Bera, = 0 o But since F, = FO) + F° where FO and F{° are the actual and constraint forces acting on the ith particle, (2) ean be ‘written Bray + BRM = 0 ® If we assume that the virtual work of the constraint forces is zero, the second sum on the left of (2) is zero, so that we have = Which is the principle of virtual work. =o ® 722, Two particles of masses m: and m: are located on a frictionless double incline and connected by an inextensible massless string passing over a smooth pex (see Fig. 7-9 below). Use the principle of virtual work to show that for equilibrium we must have sina, _ ome sing, ~ ™m where a, and a, are the angles of the incline, OnAP. 7] SYSTEMS OF PARTICLES 181 Method 1, Let ry and rz be the respective position vectors ‘of masses m, and m, relative to 0. ‘The actual forces (due to gravity) acting on ‘my and my are respectively 2 = mg FO = mp w According to the principle of virtual work, BRO, = 0 or FO eae, + FP sa = 0 ® Fig. 7-9 where dr, and ry are the virtual displacements of m, and mz down the incline. Using (1) in (2), mga, + mygedry = 0 ” or myo Br, sina + myg Bry sinag = 0 ” ‘Then since the string is inextensible, ic. 8rj+4r,=0 oF ary=—8r,, (4) becomes (mp sina — mag sin a,)or, = 0 But since 87, is arbitrary, we must have myg sina, — mao si sin. mm a a2 = 0, ie, Method 2. When it is not clear which forces are constraint forces doing no work, we ean take into account «ll forces and then apply the principle of virtual work. Thus, for example, taking into account the reaction forees , and Ry due to the inclines on the particles and the tension forces T, and T, the principle of virtual work becomes (met +R) Bey + (mye M+R) *Br, = 0 Cy Now since the inclines are assumed smooth (xo that, the reaction forees are perpendicular to the in- lines] we have Ryser, = 0, Byrd, = 0 o Also, since there is no friction at the peg, the ten- sions T, and T, have the same magnitude, Thus 7 ‘we have, using the fact that dr, and dr, are directed down the corresponding inclines and the fact that ty, f Bry = Br, Or + Myrdry = —T yar, — Taare (Tar, = 0 6) 'y. Then using (7) and (8), (6) becomes mye mygsar, + mgs8r = 0 ‘as obtained in (8). 1 since T= 7.28. Use Theorem 7.15, page 171, to solve Problem 7.22. Let the string have length ! and suppose that the lengths of string OA and OB on the inclines, (Fig. 7-0] are z and !—z respectively. The total potential energy using a horizontal plane through O as reference level is Y= mmpesing,— ntl say "hen for eqlibium we mast have w : SE = -masingy +mersing = 0 or AM = ME It should be noted that V is not a minimum in this case so that the equilibrium is not stable, ‘as is also evident physically. 182 SYSTEMS OF PARTICLES (CHAP.7 D’ALEMBERT'S PRINCIPLE 724. Use D’Alembert’s principle to describe the motion of the masses in Problem 7.22. anal Introduce the reversed etectve foeer mF and my Im equation (#) of Problem 1.2 to obtain (mie — my %)+35 = 0 wo ‘This can be written (mg sin ay — mF)8r, + (mgg sin ay ~ ma¥s)8rg = 0 @ Now since the string is inextensible so that r; +r constant, we have artim = 0, Ath =O —¥, Thos (2) becomes, after dividing by tr; 0, mg sina — mi — mag sinay — mary = 0 slo xy — mag in 9g mF mg ‘Thus particle 1 goes down or up the incline with constant acceleration according as ‘mig sina, > mag sinay oF mg sin a, <2mgp sin.ay respectively. Particle 2 in these cases goes ‘up or down respectively with the same constant acceleration. We can also use a method analogous to the second method of Problem 7.22. MISCELLANEOUS PROBLEMS 7.25. Two particles having masses m: and mz move io so that their relative velocity is v and the veloc- m™ ity of their center of mass is ¥. If M = m:-+ma is the total mass and y. = myma/(ms + m2) is the " reduced mass of the system, prove that the total fn kinetic energy is $M0* + uv. Let ra and # bo the postin vestora with re tte 6 ot mass my mase my and the center of ase © renpectively. Fie.ta1 "From the dfnition of the center of mas, we have aaa cnteg oe myy, + many = (my + mae a If the velocity of m, relative to mg is v, then ae vos Fino) = 0 that wom ® Solving (1) and (2) simultaneously, we find we tent, ws testis ‘Then the total kinetic enerey is t= Amy + bmvt = Im (ve = Heme mpet + EMM = bao + bot CHAP. 7] SYSTEMS OF PARTICLES 183 7.26. Find the centroid of @ uniform semicircular wire of radius a, By symmetry [sce Fig. 7-12] the centroid of ‘the wire must be on the y axis, so that 2—0. If is the mass per unit length of the wire, then if de represents an element of are, we have de — ade ood Cf. at _ te 727. Suppose that n systems of particles be given having centroids at fi, ...,f and total masses M:, Ms, ...,Ms respectively. Prove that the centroid of all the systems we Maki + Mob +--+ + Mats M+ M+ + My Let system 1 be composed of mastes my, min, --- located at ryytig «+. respectively. Si Jet system 2 be composed of massea mi, may located at tyytyn'- <= Then by defnition, martin + mages + + martin + Maataa + om Fig + My ‘mantor + Mantas + + manor + Manton $+ + But the centroid for all s Gmataa + matin t *-) + (magtog + maatan + °°) + 27 + (marta + mageag +2) Chan Fag FYE ay Mae tT EE gy +g Fe) 7.28 Find the centroid of a solid of constant density consisting of a cylinder of radius a and height H surmounted by a hemisphere of radius a [see Fig. 7-13). Let Fhe the distance of the centroid of the solid from the base. ‘The centroid of the hemisphere of radius a is at dis- tance Ja+H from the base of the solid, and its mass is M, = jra°e [see Problem 7.1). ‘The centroid of the cylinder of radius a and height H fs at distance 4# from the base of the solid and its mass is B= rate. ‘Then by Problem 7.27, (arate) (Gat H) + (ra2HeN(g) ~~ Fraie + vate Ba? + Balt + or Ba + 1H 184 SYSTEMS OF PARTICLES [CHAP.7 729. A circular hole of radius a/2 is cut out of a circular region of radius a, as shown in Fig. 7-14, Find the centroid of the shaded region thus obtained. ” ¥ | rate. — Irate te je Fig. 155 By symmetry the centroid is located on the # axis, so that 7 = 0. We can replace the circular region of radius o by the mass fy = zate concentrated at its centroid j= [Fig- 7-15). Similarly, we can replace the circular hole of radius o/2 by the negative mass M,=—}rate concentrated at its centroid 2)= fa. Then the centroid is located fon the + axis at Myzy + Myty _(za%e)(a) + (~4rate)(da) “MM, ees a= 730. A uniform rod PQ [see Fig. 7-16] of mass m and length L has its end P resting ‘against a smooth vertical wall AB while its other end Q is attached by means of an inextensible string OQ of length 1 to the fixed point O on the wall. Assuming that the plane of P, Q and 0 is vertical and perpendicular to the wall, show that equilibrium oceurs if sine = > sing = ve ‘There is only one actual force, i.e. the weight mg of the A rod, Other forces acting are the force of the wall on the rod and the tension in the string, However, these are con- straint forces and can do no work. This ean be seen since if P were to slide down the wall no work would be done, 4 cud because the wall is frictionless and thus the force due 0 the wall on the rod is perpendicular to the wall. Also if @ ‘were to drop, it could only move perpendicular to the P string at Q. Pace Let ¥ be the position vector of the center of mass C [in this case also the center of gravity] relative to 0. Also me @ let i and j be unit vectors in the horizontal and vertical directions respectively so that r= zi+ vj Fig. 716 From Fig. 7-16, 0g = op+PQ o 0g = oc + cq ® ‘Then from (1), on taking the dot produet with i, OQ+i = OF-i+ Part Since OP +1 =0, this reduces to Pai or sing = Laing ® CHAP. 7) SYSTEMS OF PARTICLES 185 ‘Similarly on taking the dot product of both sides of (2) with J, 0Q+) = 06+) + C+ or loose = y+ dL eosp w Now a virtual displacement of the center of mass C is given by ar = bei t Ovi Oy ‘Since mg is the only actual force, the principle of virtual work becomes mesdr = 0 © Using (6), this becomes mgdy = 0 of ty =O ” Now from (8) and (4), we have Leosesa = L cosh 88 =Using a = by — 4bsing 3p since 1 and L are constants and since 5 has the same properties as the differential operator d. Since sy =0 from (7), these equations become Teotada = Los 4p ® Isine da = 4lsing 59 © From (6) and (@), we have on division, a. troy Now from ( sing = (it) sinew a 0 that cone = VI~ CT aint ry ‘Thus equation (10) can be ine | 1__Isina cn mainte? VEF—Painta Dividing by sing and squaring both sides, we find ae sine = ay nd from (1 Si oe and from (11 sing = ——— as) } 8 i as) ‘as required. 781. A uniform solid consists of a cylinder of radius. and height H on a hemisphere of radius a, as indi- cated in Fig. 7-17. Prove that the solid is in stable equilibrium on a horizontal plane if and only if aH > 2. By Problem 7.28 the centroid C is at a distance CB from the center B of the hemisphere given by M4 So + 6H _ GH Sort ~ Bet ie ‘Then the distance of the centroid C above the plane is cP = cD + DP SH? ~ Sat = FPA S cose +a Fig. 717 CB cose + BQ =a) 186 782, SYSTEMS OF PARTICLES (CHAP. 7 0 that the potenti energy (or potential) is st = sat v= wo (HES ome + 0) v oa 0 or ae (SEZ) sine = 0, te» Equilibrium occurs where * ‘Then the equilibrium will be stable if Bat — GEE wo (Sa) he. 8@2—6H2>0 or a/H > v3. A uniform chain has its ends suspended from two fixed points at the same hori- zontal level. Find an equation for the curve in which it hangs. Let A and B [Fig. 7-18] be the fixed points ‘An clement of the chain of length as isin equilib rium under the tensions of magnitude 7 and T+ AT due to the rest of the chain and also the weight of ds of the element of chain. Now from Fig. 7-18 if the directions of the vectors ‘corresponding to T and T+AT make angles of @ and @-+ Ao with the 2 axis respectively, ‘we have as the condition for equilibrium [neg- leeting terms of order (4)? and higher], (L447) cos (e+ Ao + (T+AT) sin (e+ 0} ~ (Teosai+ Tsing j) — egias = 0 or (+47) cos (0 + 40) T cose o (T+A7) sin(o+a0) — Tsing = egas ® Equation (1) shows that the horizontal component 7’ cos: ‘as Ty which corresponds to the tension at the lowest pr rust be a constant, which we shall take of the chain, where @=0. ‘Thus Tess = Ty « From (2) we find on dividing by 49, (P+ aN sino + oo ae eae o cof both sides of (4) as A0->0, we find a de fersing = aot © Using (8) to eliminate 7, (6) becomes Ay tan) de ©) ay Toran) = Gy or ad 0 @ where b= To/ag. Now ® ‘Thus from (7) and (8), fe 2 EE = (coss)id sete) = bacco © (sin 9(b eect) = B see tan (20) CHAP. 1 SYSTEMS OF PARTICLES 187 Integrating (9) and (10) with respect to 6, we find 2 = Bin (seco + tans) + 0 «ay y= Brcce + ey a) Let us assume that at the lowest point of the chain, ie. at ¢=0, 2=0 and y=. Then from (11) and (12) we find ey =0, ey =0. Thus 2 = bin(sece + tans) us) y= bacco ay From (18) we have seco + tang = en! (as) But secto — tante = (eece + tan )(scee—tane) = 1 ro) ‘Then di ing (16) by (15), we find seco — tang = os an ding (Ft ir ad tg (18) y= Hens 2m) = becant wo ‘This curve is called a catenary (from the Latin, meaning chain} Supplementary Problems DEGREES OF FREEDOM 18%. Determine the number of degrees of freedom in each of the following cases: (a) a particle moving cn a plane curve; (6) two particles moving on a space curve and having constant distance between them; (¢) three particles moving in space so that the distance between any two of them always constant. Ane. (a) 1, (6) 1, (@) 6 784. Find the number of degrees of freedom for a rigid body which (@) moves parallel to a fixed plane, (6) has two points fixed but can otherwise move freely. Ans. (a) 8, (2) 1 135. Find the number of degrees of freedom for a system consisting of thin rigid rod which can ‘move freely in space and a particle which is constrained to move on the rod. Ams. 4 CENTER OF MASS AND MOMENTUM OF A SYSTEM OF PARTICLES 786. A quadrilateral ABCD has masses 1, 2, 9 and 4 units located at its vertices A(-1,-2,2), BG,2,—1), C(1,—2, 4) and DG, 1,2). Find the coordinates of the center of mass. Ans. (2,0,2) 181. A system consists of two particles of masses m, and my. Prove that the center of mass of the system divides the line joining m, to ms into two segments whose lengths are in the ratio my to my. 1.38 A bomb dropped from an airplane explodes in midair. Prove that if air resistance is neglected, then the center of mass describes a parabola. 78%, Three particles of masses 2,1, 3 respectively have position vectors ry = Bt ~ 204 + (8t—2)k, ry = Qt— Hi + 02-5) + + 6t~ 30, D+ (@+2))— ek where € is the time, Find (a) the velocity of the center of mas =1 and (b) the total linear momentum of the system at f=1. Ane. (a) 81—2)—k, (0) 181—12)—6k 188 1.40, 1. 148, 1a 1.46, at, SYSTEMS OF PARTICLES [owar.7 ‘Three equal masses are located at the vertices of a triangle, Prove that the center of mass is located at the intersection of the medians of the triangle. ‘A uniform plate has the shape of the region bounded by the parabola y=2? and the line y= H in the zy plane. Find the conter of mass. LE : 9 40 Ane #=0, = 9H Find the center of mass of a uniform right circular cone of radius a = and height Hf. ‘Ans. That point on the axis at distance 3/7 from the vertex. ‘The shaded region of Fig. 7-19 is solid spherical cap of height H cut of from @ uniform solid sphere of radius a. (a) Prove that the ‘centroid of the cap is located at a distance $(2a~H)/(Ba—H) from the base AB. (b) Discuss the cases H=0, H=a and H = 2a, Fig. 7-19 Find the center of mass of @ uniform plate bounded by wosing and the 2 axis, Ana. 2= 4/2, 7 = 2/8 Find the center of mass of rod of length f whose den- sity is proportional to the distance from one end 0. Ane. jl from end 0 Find the centroid of a uniform soli planes 42+2y+2=8, 2=0,y soali-+ 25+ 4k) bounded by the = 0, Ans, # A uniform solid is bounded by the paraboloid of revalu- tion t+ H [see Fig. 7-20). ANGULAR MOMENTUM AND TORQUE 18, 1 750, 158. 154, ‘Three particles of masses 2, Sand move under the influence of a force field so that their position vectors relative to a fixed coordinate system are given respectively by ry = 2ti— 3) + Ck, fy = (+ Di+ a4) — dk and ry = Ci — t+ @t—1k whore t is the time. Find (a) the total ‘angular momentum of the system and (6) the total external torque applied to the system, taken with respect to the origin. Ana, (a) (81— 120) + (622 — 10e—12)) + (21+ 5) () 124 + (12 10)) + Jock Work Problem 7.48 if the total angular momentum and torque are taken with respect to the center of mass. Verify that in (a) Problem 7.48 and (6) Problem 7.49 the total external torque is equal to the time rate of change in. angular momentum. In Problem 7.48 find (a) the total angular momentum and (#) the total external torque taken. lahout a point whose position vector Is given by r= t—2tj-+Sk, Does the total external torque ‘equal the time rate of change in angular momentum in this ease? Explain. Verify Theorem 7.9, page 169, for the system of particles of Problem 7.48. State and prove @ theorem analogous to that of Theorem 7.9, page 169, for the total external torque applied to a aystem. Is the angular momentum conserved in Problem 7.88? Explain, CHAP. 7] SYSTEMS OF PARTICLES 189 WORK, ENERGY AND IMPULSE, 155. Find the total work done by the force field of Problem 7.48 in moving the particles from their positions at time *—1 to their positions at time ¢=2 Ans. 42 7.56, Is the work of Problem 7.55 the same as that done on the center of mass assuming all mass to be concentrated there? Explain. 751, Find the total kinetic energy of the particles in Problem 748 at times (a) #=1 and (6) t= 2 Discuss the connection between your results and the result of Problem 7.55. Ane. (a) 725, (0) 305 758 Find the total linear momentum of the system of particles in Problem 7.48 at times (a) ¢=1 and (t= 2 Ans, (a) 17144) 14k, (6) B74 Aj + 18k and discuss 759. Find the total impulse applied to the system of Problem 7.48 from ¢=1 to t ‘the connection of your result with Problem 7.68. Ans. 10i-+ 4k. 1.60, Prove Theorem 7.18, page 170, Problem 7.48. 761, Verify Theorem 7.18, page 170, for the system of particles CONSTRAINTS, STATICS, VIRTUAL WORK, STABILITY AND D’ALEMBERT'S PRINCIPLE. 1.62. In each case state whether the constraint is holonomie or non-holonomic and give a reason for ‘your answer: (a) a particle constrained to move under gravity on the inside of a vertical paraboloid of revolution whose vertex is downward; (0) a particle sliding on an ellipsoid under the influence ‘of gravity; (c) a sphere rolling and possibly sliding down an inclined plane; (d) a sphere rolling down an inclined plane parallel to a fixed vertical plane; (¢) a particle sliding under gravity on the outside of an inverted vertical cone. Ans. (a) holonomic, (b) non-holonomic, (©) non-holonomie, (d) holonomi (© holonomie 763. A lever ABC [Fig. 21] has weights W, and We fat distances a, and a from the fixed support B. _W Wa Using the principle of virtual work, prove that a necessary and sulfcient condition for equilibrium is L Wray = Weay 64, Work Problem 7.63 if one or more additional weights are placed on the lever, Fig. 7-21 785. An inextensible string of negligible mass hanging over a smooth peg at B (see Fig. 7-22] connects one mass ‘mon a frictionless inclined plane of angle « to another ‘mz. Using D'Alembert’s principle, prove that es will bo in equilibrium if m, =m sina. 1166, Work Problem 7.65 if the incline has coeficient of frie- tion Ans. m; = mysina ~ 4 c08.a) 12 161. A ladder AB of mass m has its ends on a smooth wall and for {soe Fig. 1-23)..‘The foot of the ladder is tied by an inextensible rope of negligible mass to the base C of the wall 20 that the Indder makes an angle « with the foor. Using the principle of virtual work, find the magnitude of the tension in the rope ‘Ane, 4g cote 768, Work (a) Problem 7.63 and (6) Problem 7.05 by using the po- tential energy method. Prove that the equilibrium in each case is unstable, Fig. 7.23 190 189, i, 1m. i, am. SYSTEMS OF PARTICLES [oHAP.7 ‘A thin uniform rod of length I has its two ends constrained to move on ‘the circumference of a smooth vertical cirele of radius a [see Fig. 7-24). Determine conditions for equilibrium. Is the equilibrium of the rod of Problem 7.69 stable or not? Explain. ‘A solid hemisphere of radius a is located on a perfectly rough inclined plane of angle a. (a) Prove that it is in stable equilibrium if « < sin (9/8). (#) Aze there any other values of « for which equilibrium can occur? ‘Which of these, if any, yield stable equilibrium? Fig. 7-24 Use D'Alembert’s principle to obtain the equations of motion of m: 13 m, and mz of Problem 7.65. Work Atwood's machine problem [seo Problem 7.22, page 180] by using D'Alembert’s principle. Use D’Alembert’s pri le to determine the equations of motion of @ simple pendulum, ‘MISCELLANEOUS PROBLEMS 15. 16 um, 18, 118, 180, ral. 182, 188, 184, 188, Prove that the center of mass of @ uniform circular are of radius a and central angle « is Toeated on the axis of symmetry at a distance from the center equal to (a sin )/ Discuss the eases (a) a= r/2 and (8) «=x in Problem 7.75. ‘A circle of radius a is removed from a uniform circular plate of radius b > a, as indicated in Fig. 7-25. If the distance be- tween their centers A and B is D, find the center of mass, Ans, Tae point at distance o#D/(O?~ a) below B. 6 Work Problem 7.77 if the citeles are replaced by spheres. 2 Ans. The point at distance oD/(0?— 0) below B. Prove that the center of mass does not depend on the origin of the coordinate system used. Prove that the center of mass of a uniform thin hemispherical shell of radius a is located at a distance 4a from the center. Fig. 17.25 ‘Let the angular momentum of the moon about the earth be A. Find the angular momentum of a ayatem consisting of only the earth and the moon about the center of mass. Assume the masses Of the earth and moon to be given by M, and My respectively, Ana. MyA/(M,+ My) pply in case the angular momentum is taken about an arbitrary Does Theorem 7.13, page 170, point? Explain. In Fig. 7-26, AD, BD and CD sre uniform thin rods af equal length @ and equal weight. They are Stroothly hinged at D and have ends 4, and Con 8 Soth herlzontal plane. "To prevent the motion Stienda A, Band G, we use an inoxtenable string nc ot netiibe mass which ia inthe form of en sauileteral triangle. Tf-2 weight WY Js suspended {fom D oo thatthe rods make equal angen ¢ with {he horizontal plane, prove that the magnitude of the tension inthe ering Is }VS(H7 + 3u) cote, Work Problem 7.88 if the weight W is removed from D and suspended from the center of one of ‘the rods, Derive an expression for (a) the total angular momentum and (b) the total torque of system about, fan arbitrary point. CHAP. 7) SYSTEMS OF PARTICLES 191 7.86. Prove that the torque about any point P is equal to the time rate of change in angular momentum about P if and only if (@) P ia fixed in space, (b) P coincides with the center of mass or (c) P is moving with ‘a velocity which is in the same direction as the center of mass, 781. Find the centroid of a solid of constant density consisting of a right cireular cone of radius @ and height surmounted by a hemisphere of radius a [see Fig. 7-27). Ans. At height {(a? + #2)/(2a+H) above 0. 0 the hemisphere. Ans. At height f(a? +2479)/(a +H) above 0. Fig. 1.27 1.88. Work Problem 7.87 if the density of the cone is twice the density of 7.89. A hemisphere of radius a is cut out of a uniform solid cube of side 6 > 2a [see Fig. 7-28], Find the center of mass of the remaining soi y i my, gm 9 Y Fig. 7-28 Fig. 7.29 i. 7-30 7.90. A uniform chain of 45 kg wt is suspended from two fixed supports 16 meters apart. If the saz in the middle fs 20 em, find the tension at the supports. Ane. 450 kg wt 791. A chain of length L and constant density o is suspended from two fixed points at the same horizontal level. If the sag of the chain at the middle is at a distance D below the horizontal line through the fixed points, prove that the tension at the lowest point of the chain is o(L— 408/80. 7.92, Three particles of masses mm;,ms,m, are located at the vertices of a triangle opposite sides having lengths aj, ay, 43 respectively. Prove that the center of mass Ti Disectors of the triangle if and only if m/a, = 7.93. Masses m, and my are on a frictionless circular eylinder connected by an inextensible string of negligible mass [see Fig. 7-29]. (a) Using the prineiple of virtual work, prove that the system Js in equilibrium if my sina, = my sina. (6) Is the equilibrium stable? Explain. 794, Work Problem 7.98 if friction is taken into account. 795. Derive an expression for the total kinetic energy of a system of particles relative to a point which may be moving in space. Under what conditions is the expression mathematically implifed? Discuss the physical significance of the simplification, 7.96. Find the center of mass of a uniform plate shown shaded in Fi hypocyeloid 22+ 2/9 = qi and the lines x= 0, y=, hhypocyeloid are z= a.coste, y=asinte] Ans. # 730 which is bounded by the (Hint. Parametric equations for the 560/315 192 SYSTEMS OF PARTICLES (CHAP. TAT. Let mym,my be the masses of three particles and vj;,viy,¥jg be thelr relative velocities. (@) Prove that the total kinetic energy of the system about the center of mass is mymgofy + mamgvis + mymavis mF my + () Generalize the result in (a). 7.98. A chain of variable density is suspended from two fixed points on the same horizontal level. Prove that if the density of the chain varies as the horizontal distance from a vertical line through its center, then the shape of the chain will be a parabol 799, iscuss the relationship of Problem 7.98 with the shape of a suspension bridge. 7.100. A solid consists of a uniform right circular cone of vertex angle « on a uniform hemisphere of ‘the same density, as indicated in Fig. 7-81. Prove that the solid can be in stable equilibrium on horizontal plane if and only if « > 60°. Fig. 731 Fig. 7.32 7A0L, A uniform solid [see Fig, 7-22] consists of a hemisphere of radius a surmounted by a cube of side b symmetrically placed about the center of the hemisphere. Find the condition on a and b for stable equilibrium. Ans. a/b > Y= 7102, Find the centroid of the area bounded by the eycloid 2 = ale—sing), vy = all ~ cose) and the x axis. Ans. (ea, Ba/6) 7409, Prove that if the component of the torque about point P in any direction is zero, then the component of angular momentum about Pin that direction is conserved if (a) P is a fixed point, () P coincides with the center of mass or (c) P is a point moving in the same direction as the center of mass 7404, In Problem 7.108, is the angular momentum conserved only if (a), (6) or (c) occurs? Explain. 7106, Prove that the virtual work due to a force is equal to the sum of the virtual works which cor reapond to all components of the foree 71405, Prove that it is impossible for one sphere to bo in stable equilibrium on a fixed sphere which is perfectly rough (ie. with coefficient of friction y=]. Is it possible for equili fat all? Explain, 7407. A uniform solid having the shape of the paraboloid of revolution ez = z#-+y2, ¢>0 rests on ‘the ay plane, assumed horizontal. Prove that if the height of the paraboloid is #1, then the ‘equilibrium is stable if and only if H< 4c. 7408, Work Problem 7.107 if the xy plane is inclined at an angle « with the horizontal. CHAP. 7) ra, za. mana, 193 In Fig. 7-88, AC and BC are frietionless wires in a vertical plane making angles of 60° and 20° respectively with the horizontal, Two bends of mancen dm and 6 gm are located. on tho wires. Connected by a thin Tod ef ngliible mass. Prove thet the ayatem wil be in equibriam tren the red makes an angle with the horizontal given by tan! (RV). 4 B 6 gm Prove each of the following theorems due to Pappus. (a) If a closed eurve C in a plane is revolved about an axis in the plane which does not interseet it, then the volume generated is equal to the area bounded by C multiplied by the distance traveled by the centroid of the area. () If an are of a plane curve (closed or not) is revolved about an axis in the plane which does not intersect it, then the area of the surface generated is equal to the length of the are ‘multiplied by the distance traveled by the centroid of the arc, Use Pappus’ theorems to find (a) the centroid of a semicircular plate, (b) the centroid of a semi- cireular wire, (¢) the centroid of a plate in the form of u right triangle, (q) the volume of a cylinder. Find the (a) surface area and (2) volume of the doughnut shaped region obtained by revolving fa circle of radius @ about a its plane at a distance b>a from ite center. Ans. (a) de%ab, (0) 2o%ed Chapter 8 APPLICATIONS to VIBRATING SYSTEMS, ROCKETS and COLLISIONS VIBRATING SYSTEMS OF PARTICLES If two or more particles are connected by springs [or interact with each other in some equivalent manner), then the particles will vibrate or oscillate with respect to each other. As seen in Chapter 4, a vibrating or oscillating particle such as the simple harmonic oscillator or bob of a simple pendulum, has a single frequency of vibration. In the case of systems of particles, there is generally more than one frequency of vibration. Such frequencies are called normal frequencies. The motions of the particles in these cases are often called multiply-periodic vibrations, A mode of vibration [i.e. a particular way in which vibration occurs, due to particular initial conditions for example] in which only one of the normal frequencies is present is called a normal mode of vibration or simply a normal mode. See Problems 8.1-8.3, PROBLEMS INVOLVING CHANGING MASS. ROCKETS Thus far we have restricted ourselves to motions of particles having constant mass. An important class of problems involves changing mass. An example is that of a rocket which moves forward by expelling particles of a fuel mixture backward, See Problems 8.4 and 8.5, COLLISIONS OF PARTICLES During the course of their motions two or more particles may collide with each other. Problems which consider the motions of such particles are called collision or impact problems. In practice we think of colliding objects, such as spheres, as having elasticity. The time during which such objects are in contact is composed of a compression time during which slight deformation may take place, and restitution time during which the shape is restored. We assume that the spheres are smooth so that forces exerted are along the common normal to the spheres through the point of contact [and passing through their centers}. A collision can be direct or oblique. In a direct collision the direction of motion of both spheres is along the common normal al the point of contact both before and after collision. A collision which is not direct is called oblique. Fundamental in collision problems is the following principle called Newton's collision rule which is based on experimental evidence. We shall take it as a postulate. Newton's collision rule. Let v,, and vj, be the relative velocities of the spheres along the common normal before and after impact. Then v, fa = Mn 194 CHAP. 8] APPLICATIONS TO VIBRATING SYSTEMS, ROCKETS AND COLLISIONS 195 ‘The quantity «, called the coefficient of restitution, depends on the materials of which the objects are made and is generally taken as a constant between 0 and 1. If «=0 the collision is called perfectly inelastic or briefly inelastic. If ¢=1 the collision is called perfectly elastic or briefly elastic. In the case of perfectly elastic collisions the total kinetic energy before and after impact is the same. CONTINUOUS SYSTEMS OF PARTICLES For some problems the number of particles per unit length, area or volume is so large that for all practical purposes the system can be considered as continuous. Examples are a vibrating violin string, a vibrating drumhead or membrane, or a sphere rolling down an inclined plane. ‘The basic laws of Chapter 7 hold for such continuous systems of particles. In applying them, however, it is necessary to use integration in place of summation over the number of particles and the concept of density. THE VIBRATING STRING Let us consider an elastic string such as a violin or piano string which is tightly stretched between the fixed points =0 and x=l of the + axis [see Fig. 8-1]. If the string is given some initial displacement [such as, for example, by plucking it] and is then rium position, released, it will vibrate or oscillate about the equil Fig. 8 Fig. 82 If we let ¥(2, t) denote the displacement of any point x of the string from the equilibrium position at time t [see Fig. 8-2], then the equation governing the vibrations is given by the partial differential equation ey _ oY = OR (1) where if T is the (constant) tension throughout the string and o is the (constant) density [mass per unit length of string], c= Tho (2) ‘The equation (1) holds in case the vibrations are assumed so small that the slope é¥/ax at any point of the string is much less than one. BOUNDARY-VALUE PROBLEMS The problem of solving an equation such as (1) subject to various conditions, called boundary conditions, is often called a boundary-value problem. An important method for solving such problems makes use of Fourier series. 196 APPLICATIONS TO VIBRATING SYSTEMS, ROCKETS AND COLLISIONS [CHAP. 8 FOURIER SERIES Under certain conditions {usually satisfied in practice and outlined below] a funetion f(z), defined in the interval y<20 such that =B < f(z) e8NSG) ee hea By expanding the determinant ay of Problem 8.30 in terms of the elements in the first row, | dy = eda ~ Syne o Also, a a= end ” Putting N =2 in (1), we see that equations (2) are formally satisfied if we take ay =1, Thus conditions consistent with (1) and (2) are d= ase 6 ‘To solve the difference equation (1), assume that y= p¥ where p is a constant to be determined. Substituting this into (1), we find on dividing by p¥-2, pimep+t orp If we call ¢=2cose, then p = cose + ising = ox ‘Thus solutions of the difference equation are ("= el = cope + isinNe and (e-#) = eNO = cosNo ~ isin No Since constants multiplying these solutions and sums of solutions are also solutions [as in ‘the case of linear differential equations], we see that the general solution is ay = GeosNe + H sinNe ® Now from equations (8) we have S)=1, 4,=2cos# so that G vote. Thus dy = conne + Sateense © ine exl(N+Y), « yssoyNe Thus using (3) This is equal to zero when sin(N+t)e=0 or of Problem 8.30, we find = (1 - cosy) o CHAP. 8] APPLICATIONS TO VIBRATING SYSTEMS, ROCKETS AND COLLISIONS 217 832, Solve for A. in Problem 8.30 and thus find the transverse displacement Y, of particle v. From equations (1) and (2) of Problem 8,30 we have [on using the normal frequencies (@) of Problem 8.81 and the superscript « to indicate that the A’s depend on al, 7 ° o AWD, + 2A? cos GE ~ ‘together with the end conditions Ah =o, Af, =e « ‘The equation (1) subject to conditions (2) can be solved in a manner exactly like that of Problem 8.31, and we find wei where C, are arbitrary constants. In a similar manner if we had assumed ¥, = B, sin ot [see remarks ‘at the end of Problem 8.30] we would have obtained AO = Cysin BoP = Dy sings ‘Thus solutions are given by oF cost ans sin sin wt i nae and since sums of solutions are also solutions, we have NST ‘The constants C, and Dg are determined from initial conditions, (Cq c08 ut + Da sin ot) ‘The analogy with the continuous vibrating string is Supplementary Problems VIBRATING SYSTEMS OF PARTICLES 833, 834 835, 836 837, Find the normal frequencies of the vibr and masees are all different, jons in Problem 8.1, page 197, if the spring constants ‘Two equal masses m on a horizontal frictionless table as shown in Fig. 8-26 are connected by equal springs. ‘The end of one spring is fixed at A and the masses are set into mation. (a) Set up the ‘equations of motion of the system. (0) Find the normal frequencies of vibration. (e) Describe the normal modes of vibration, Fie aa on = ot = no MLE ig. 8-26 @ Work Problem 8.24 if the spring constants and masses are different, ‘Two equal masses m are attached to the ends of a spring of constant « which is on a horizontal frictionless table. If the masses are pulled apart and then released, prove that they will vibrate with respect to each other with period 2rvV/m/2x, ‘Work Problem 886 if the masses are different and equal to M, and M, respectively. Ans. 2eV qin where x = MyMy/(M; + M,) 218 APPLICATIONS TO VIBRATING SYSTEMS, ROCKETS AND COLLISIONS [CHAP.8 838. In Fig, 827 equal master m lying on a horizontal ‘tictiontem table are comected fo each ather and to fixed pointe A and B by means of elaric svings of Constant tension and length 1, f the daplacements ‘om the siulforiom portion Af! af the maoner ara ¥,"and Yy respectively, show thatthe equations of maton are given by ¥, = ,-2¥y, where «= 87/nt. y= W(¥—2¥ 4) 4 8.89. Prove that the natural frequencies of the vibration in Problem 8.38 are given = respectively by a et en a Vm Br Vm ian and describe the modes of vibration. " 840. Find the normal frequencies and normal modes of vibration for the system of io) particles of masses my and mg connected by springs as indicated in Fig. 828, Fig. 8.28 CHANGING MASS. ROCKETS 841. (a) Prove that the total distance traveled by the rocket of Problem 8.5 in time t is given by. w= wo fee (ME2)n(ME)} - doe (6) What is the maximum height which the rocket can reach and how long will it take to achieve this maximum height? 842, Suppose that a rocket which starts from rest falls in a constant gravitational field, At the instant it starts to fall it ejects gas at the constant rate « in the direction of the gravitational field and at speed v with respect to the rocket. Find its speed after any time f. aan at ~ inate) 848, How far does the rocket of Problem 8.42 travel in time ¢? fae my = at) mo — at Ane. 196 vofe+( ) mn (Maas } 8.44, Deseribe how Problem 842 ean be useful in making @ “eoft landing” on a planet or allite? 845. Discuss the motion of @ two-stage rocket, ie. one in which one part falls off and the other rocket takes over. COLLISIONS OF PARTICLES 846. A gun fires a bullet of mass m with horizontal velocity v into a block of wood of mass M which rests on a horizontal frictionless plane. If the bullet becomes embedded in the wood, (a) determing the subsequent velocity of the system and (b) find the loss in kinetic energy. Ans, (a) mv/(M +m) (b) mAtv2/2(M +m) 8.47. Work Problem 8.46 If the block is moving away from the gun with velocity V. 848. A ball which is dropped from a height H onto a floor rebounds to a height ho ¥s0,9=0, YEN =, YO = 0, Yile,0) SI. 862, 863, 864, (6) Give a physical interpretation of the problem in (a). (ys Bai (n= Pe sin @n—1)t am Yen = BS Solve the boundary-value problem Ye= Varo 00 ¥(0,1) = 0, Ye ‘and interpret physically. ans. Yee) = teeta % ¥(e,0) = pele—2), ¥e(e,0) = 0 gelatin tn Bec ttn—e ~ jon (o) Pind wiaton of the equation 2Y = 42% which satiaten the cantons ¥(0,2) = 0, ¥ (x,t) = 0, ¥(z,0) = 0.1 sin +0.01 sin dz, Y¢(2,0)=0 for 0<#<-,t>0. (b) Interpret physi- ithe Butary eons In) and the Seton Ans. (a) ¥(e,t) = 0.1 sin x cos 2¢+ 0.01 sin 4x cos to) Save the toundarywaive protien 24 = 98% subject to the emus ¥%0,9 =0, Y(2,0) = 0, ¥(x,0) = 0.052(2— 2), ¥; (x, 0) 9, where 0<#-< 10. (Interpret pigs 1s g 1, Qnmter,, 8— Set Ane (8) Yl = EES Bg in BEE yg BBE Dt Solve Problem 863 with the boundary conditions for Y(e,0) and Y;(z,0) interchanged, Le. ¥(z,0) = 0, ¥;(2,0) = 0.052(2—2), and give a physical interpretation, 1 gi Qn—Dee 4,5 SEn— Det we 2 z Ans. Ya.) MISCELLANEOUS PROBLEMS 865 866, 867. ‘A spherical raindrop falling in a constant gravitational field grows by absorption of moisture from its surroundings at a rate which is proportional to the instantaneous surface area. Assumi that it starts with radius zero, determine its acceleration. Ams. 4g ‘A cannon of mass M rests on @ horizontal plane having coeficient of friction g. It fires a projectile of mass m with muzzle velocity v¥) ina direction making angle « with the horizontal. Determine how far back the cannon will move due to the recoil. ‘A ball is thrown with speed vp onto a smooth horizontal plane in a direction making angle « with the plane. If « is the coefficient of restitution, prove that the velocity of the ball after the impact is given by mV1—G—e)sinte in a direction making angle tan~1(c tan) with the horizontal. CHAP. 8] APPLICATIONS TO VIBRATING SYSTEMS, ROCKETS AND COLLISIONS 221 8.69, 870. an. 872, 873, am, 875, 876, am, 818, sa. Prove that the total theoretical time taken for the particle of Problem 8.67 to come to rest is BHT (+ 9/9) Prove that while the particle of Problem 821, page 213, moves from the top to the bottom of the cline, the incline moves a distance (ml cos a)/(M +m). Prove that the loss of kinetic energy of the spheres of Problem 8.9 is u(vy cos 4, ~vy cos #,)*(1—e) where y is the reduced mass myma/(m, +,). Prove that the acceleration of a double incline of mass M (Fig. 8-29] which is on a smooth 4,608.05 — my sin ag O82) OE my sina + my ana table is given by If A is the acceleration of the incline in Problem 8.71, prove that the accelerations of the masses ‘my(A cota, + 9 sin ay) + my(A cos an ~ 9 sin an) relative to the incline are given numerically by oe ~ if Ss Ly "is a — ‘A mass m slides down an inclined plane of the same mass which is on a horizontal plane with coefficient of friction y. Prove that the inclined plane moves to the right with acceleration equal to (18/81). See Fig. 8-30. A tun of mass Af Is located on an incline of angle @ which in turn is on a smooth horizontal plane. ‘The gun fires a bullet of mass m horizontally away from the incline with speed vy. Find the recoil speed of the gun, Ans. (mv cosa)/M up the ineline How far up the plane will the gun of Problem 8.74 move before it comes to reat if the incline is (@) frictionless, (b) has cooficient of friction 4? A weight W is dropped from a height H above the plate AB of Fig. 831 which is supported by a spring of constant x. Find the 4 B speed with which the weight rebounds, A ball is thrown with speed vp at angle @ with a horizontal plane. If it rebounds successively from the horizontal plane, determine its location after n bounces. Assume that the coefficient of restitution ie c and that air resistance is negligible. Fig. 8.31 Work Problem 8.77 if the horizontal plane is replaced by an inclined pla ball is (@) thrown downward, (b) thrown upward. of angle # and the Obtain the equation (1), page 195, for the vibrating string by considering the equations of motion for the N particles of Problem 8.29, page 215, and letting N— «. Prove that as N= the normal frequenci for the continuous vibrating string. jven in Problem 8.31, page 216, approach those Prove that for 05 () atena) = SB — (SPE 4 BH, Sons .) ( aena) = (SR singe 222 382 sat, 885, 886, 8st. 358, 8.90, sat, 892, 893, 895. APPLICATIONS TO VIBRATING SYSTEMS, ROCKETS AND COLLISIONS (CHAP. & Use Problem 8.81 to show that os x = @ 32-3 © 30 8 Prove that Y = fle+et) + o(e—ct) is a solution of the equation ey _ 4aY eee ane tnd alco the connection ofthis slutin with the problem of the vibrating sting, (0) Prove that the ttl potential eneray ofa vibrating stings v= Ff (SE) ae BT § as ret neet)* (8) Thos show that V = HT & wt (a, cos + 5, sin) (a) Prove that the total kinetic energy of the vibrating string is KE, ott 2, sot neet\* 0) thw ew tat KB. = HE Se (seat sant) (©) Can the kinetic energy be infinite? Explain. Prove that the total energy of a vibrating st 2 + 08 Find the potential energy, kinetic energy and total energy for the string of (a) Problem 8.20, page 210, (b) Problem 8.28, page 214. If damping proportional to the instantaneous transverse velocity is taken into account in the : ae ere aY. ay problem of the vibrating string, prove that its equation of motion is E+ pSk = OE. Prove that the frequencies of vibration for the damped string of Problem 8.88 are given by Vare2TE = BH, n= 1,2,8, 0. Solve the problem of the damped vibrating string if the string is fixed at the endpoints 2 = 0 and 2=1 and the string ia (a) given an initial shape (2) and then released, (6) in the equilibrium position and given an initial velocity distribution g(z), (0) given an initial shape f(z) and velocity Aistribution 9(2). Work the problem of the damped vibrating string if gravitation is taken into account. Work (a) Problem 8.84(a), (6) Problem 8.86(a), (e) Problem 8.86, (2) Problem 8.88 for the case where the string is replaced by NV particles as in Problem 828, pege 215. In Fig. 8.82 the double pendulum system is free to vibrate in a vertical plane. Find the normal frequencies and normal modes assuming small ‘Work Problem 893 if there is an additional mass m, suspended from m, by a string of length |. Generalize the motion of (a) Problem 8.1, (0) Problem 8.4 to N equal particles and springs. In Problem 895 investigate the limiting case as N=. Discuss the physical significance of the results CHAP. 8] APPLICATIONS TO VIBRATING SYSTEMS, ROCKETS AND COLLISIONS 223 a1, Seve the boundary-aie problem ee oe aga t a sinet ¥(e,0) = fl), ¥,(2,0) = YO.) = 0, YO = and give a physical interpretation, 898, Work Problem 8.97 if the eon ion Y; (2,0) =0 is replaced by Y¢(,0) = ote). 899, Work Problem 8.97 if the partial differential equation is replaced by ay , gm ay e+ a = ahh + asinat and interpret physically. 8.100, Set up the differential equations and initial conditions for the motion of a rocket in an inverse ‘square gravitational field. Do you believe these equations can be solved? Explain, 8.101, Two bodies {such as the sun and earth or earth and moon) of masses m, and mz move relative to each other under their mutual inverse square attraction according to the universal law of gravitation. If r, and rz are their position vectors relative to a fixed coordinate system, and F=1,—¥,, prove that thelr equations of motion are given by =") Gme.—n) ‘This ts called the problem of two bodies. 8.102, In Problem 8.101 choose a new origin at the center of mass of the two bodies mr; Pars =0 ‘Thos show that if we let x be the position vector of m, relative to ma, then Glog + mes Gioms+ mee oa ae aa Gone + mae on aubtractng, ¢ = —Seutoee ‘Thus show that the motion of m, relative to mg tly the same as if the body of mass m, were fixed and its mass increased to my + ma. $108. Using Problem 8.102, obtain the orbit of mass m, relative to m, and comp Chapter 5. Are Kepler's first and second laws modified in any way? Explain. with the results of 8.104, If P is the period of revolution of m, about m, and a is the semi-major a ‘m, about mz, prove that it my, prove that = Fe Gm, Fm) of the elliptical path of Compare this result with Kepler's third law: In the case of the earth (or other planet] and sun, does this modified Kepler law have much effect?) Explain 8.105. Set up equations for describing the motion of & bodies under a mutual inverse square law of attraction, 8.108. Transform the equations obtained in Problem 8.105 so that the positions of the bodies are described relative to their center of mass. Do you believe these equations can be solved exactly? 8407, Work Problems 8.105 and 8.106 for N bodies. Chapter 9 PLANE MOTION of RIGID BODIES RIGID BODIES A system of particles in which the distance between any two particles does not change regardless of the forces acting is called a rigid body. Since a rigid body is a special case of a system of particles, all theorems developed in Chapter 7 are also valid for rigid bodies. ‘TRANSLATIONS AND ROTATIONS A displacement of a rigid body is a change from one position to another. If during ‘a displacement all points of the body on some line remain fixed, the displacement is called a rotation about the line. If during a displacement all points of the rigid body move in lines parallel to each other the displacement is called a translation. EULER'S THEOREM. INSTANTANEOUS AXIS OF ROTATION The following theorem, called Euler's theorem, is fundamental in the motion of rigid bodies. Theorem 9.1. A rotation of a rigid body about a fixed point of the body is equivalent to a rotation about a line which passes through the point. ‘The line referred to is called the instantaneous axis of rotation. Rotations can be considered as finite or infinitesimal. Finite rotations cannot be represented by vectors since the commutative law fails, However, infinitesimal rotations can be represented by vectors. GENERAL MOTION OF A RIGID BODY. CHASLE’S THEOREM In the general motion of a rigid body, no point of the body may be fixed. In such case the following theorem, called Chasle's theorem, is fundamental. Theorem 9.2. The general motion of a rigid body can be considered as a translation plus a rotation about a suitable point which is often taken to be the center of mass. PLANE MOTION OF A RIGID BODY ‘The motion of a rigid body is simplified considerably when all points move parallel to a given fixed plane. In such case two types of motion, called plane motion, are possible. 1. Rotation about a fixed axis. In this case the rigid body rotates about a fixed axis perpendicular to the fixed plane. ‘The system has only one degree of freedom (see Chap- ter 7, page 165] and thus only one coordinate is required for describing the motion. 224 CHAP. 9} PLANE MOTION OF RIGID BODIES 225 2 General plane motion. In this case the motion can be considered as a translation parallel to the given fixed plane plus a rotation about a suitable axis perpendicular to the plane. This axis is often chosen so as to pass through the center of mass. The num- ber of degrees of freedom for such motion is 8: two coordinates being used to describe the translation and one to describe the rotation. ‘The axis referred to is the instantaneous azis and the point where the instantaneous axis intersects the fixed plane is called the instantaneous center of rotation (see page 229). ‘We shall consider these two types of plane motion in this chapter. The motion of a rigid body in three dimensional space is more complicated and will be considered in Chapter 10, MOMENT OF INERTIA A geometric quantity which is of great importance in discussing the motion of rigid bodies is called the moment of inertia. ‘The moment of inertia of a particle of mass m about a line or azis AB is defined as T= mt r where ris the distance from the mass to the line. a ‘The moment of inertia of a system of particles, with masses ms, ma, ...,my about the line or axis AB is defined as T= Smri = mete mgp to + mary where ri,7s,...,7y are their respective distances from AB. The moment of inertia of a continuous distribution of ‘mass, such as the solid rigid body of Fig. 9-1, is given by a @ where r is the distance of the element of mass dm from AB. Fig. 9-1 RADIUS OF GYRATION Let 1 = 3 mar? be the moment of inertia of a system of particles about AB, and ‘M = & my be the total mass of the system. ‘Then the quantity K such that 1 Sm read ® M =m is called the radius of gyration of the system about AB. For continuous mass distributions (4) is replaced by Sram ms ps © am 226 PLANE MOTION OF RIGID BODIES (cuar. 9 ‘THEOREMS ON MOMENTS OF INERTIA 1. Theorem 9.3: Parallel Axis Theorem. Let I be the moment of inertia of a system about axis AB and let Ic be the moment of inertia of the system about an axis parallel to AB and passing through the center of mass of the system. Then if b is the distance between the axes and M is the total mass of the system, we have T= 1,4 Mot (6) 2. Theorem 94: Perpendicular Axes Theorem. Consider a mass distribution in the ay plane of an zyz coordinate system. Let J., 1, and I, denote the moments of inertia about the x, y and z axes respectively. Then hokt+h 1a) SPECIAL MOMENTS OF INERTIA ‘The following table shows the moments of inertia of various rigid bodies which arise in Practice. In all cases it is assumed that the body has uniform [i.e. constant] density. Rigid Body Moment of Inertia 1. Solid Girewlar Cylinder of radius @ and mass M jaar about axis of eylinder. 2, Hollow Cireular Cylinder of radius @ and mass M about axis of cylinder. Wall thickness is negligible. Ma® 8, Solid Sphere of radius @ and mage M gue bout a diameter, 4 Hollow Sphere of radius @ and mass M about a Sphere Mar negligible. 5. Rectangular Plave of sides @ and 6 and mass Mf about an ax perpendicular to the plate through the center of mass. 6. Thin Rod of length a and mass Af about an axis perpendicular syilor to the rod through the center of mass. Mo +08) COUPLES A set of two equal and parallel forces which act in opposite directions but do not have the same line of action {see Fig. 9-2) is called a couple. Such a couple has a turning effect, and the moment or torque of the couple is given by Fx F, ‘The following theorem is important. CHAP. 9) PLANE MOTION OF RIGID BODIES 227 Theorem 9.5. Any system of forces which acts on a rigid body can be equivalently replaced by a single force which acts at some specified point together with a suitable couple. KINETIC ENERGY AND ANGULAR MOMENTUM ABOUT A FIXED AXIS Suppose a rigid body is rotating about a fixed axis with angular velocity » which has the direction of the axis AB {see Fig. 9-8]. Then the kinetic energy of rotation is given by T= ue ® where J is the moment of inertia of the rigid body about the axis, Similarly the angular momentum is given by a=t ©) MOTION OF A RIGID BODY ABOUT A FIXED AXIS ‘Two important methods for treating the motion of a rigid body about a fixed axis are given by the following theorems. Theorem 96: Principle of Angular Momentum. If A is the torque or the moment of all external forces about the axis and = Je is the angular momentum, then a 7 A = Fille) = Ie = Ie (10) where a is the angular acceleration. Theorem 9.7: Principle of Conservation of Energy. If the forces acting on the rigid body are conservative so that the rigid body has a potential energy V, then T+V = He +V = EB = constant (a) WORK AND POWER Consider a rigid body ® capable of rotating in a plane about an axis O perpendicular to the plane, as indicated in Fig. 9-4. If 4 is the magnitude of the torque applied to the body under the influence of force F at point A, the work done in rotating the body through angle d# is aw = sd (12) and the instantaneous power developed is aw ea Ma ae (as) where « is the angular speed. Fig. 94 ‘We have the following Theorem 98. The total work done in rotating a rigid body from an angle 6, where the angular speed is o, to angle 6, where the angular speed is », is the difference in the kinetic energy of rotation at », and »,. In symbols, . Sido = ang - ng (uy 228 PLANE MOTION OF RIGID BODIES [cHAP. 9 IMPULSE. CONSERVATION OF ANGULAR MOMENTUM ‘The time integral of the torque ee Adt (15) is called the angular impulse from time ¢; to ts. We have the following theorems, Theorem 9.9. The angular impulse is equal to the change in angular momentum. In symbols a Adt = %-a% (16) Theorem 9.10: Conservation of Angular Momentum. If the net torque applied to a rigid body is zero, then the angular momentum is constant, i.e. is conserved. THE COMPOUND PENDULUM Let & (Fig. 9-5] be a rigid body which is free to oscillate in a vertical plane about a fixed horizontal axis through O under the influence of gravity. We call such a rigid body a compound pendulum. Let C be the center of mass and suppose that the angle between OC and the vertical OA is #. Then if [> is the moment of inertia of ® about the horizontal axis through O, M is the mass of the rigid body and a is the distance OC, we have for the equation of motion, + Me sino = 0 (an) For small oscillations the period of vibration is P = 2n/T/Mga (18) ‘The length of the equivalent simple pendulum is t= Wie a) ‘The following theorem is of interest. Theorem 9.11. The period of vibration of a compound pendulum is a minimum when the distanee OC =a is equal to the radius of gyration of the body about the horizontal axis through the center of mass. GENERAL PLANE MOTION OF A RIGID BODY ‘The general plane motion of a rigid body can be considered as a translation parallel to the plane plus a rotation about a suitable axis perpendicular to the plane. Two important methods for treating general plane motion of a rigid body are given by the following theorems. Theorem 9.12: Principle of Linear Momentum. If r is the position vector of the center of mass of a rigid body relative to an origin O, then dati) = F (20) where M is the total mass, assumed constant, and F is the net external force acting on the body. CHAP. 9] PLANE MOTION OF RIGID BODIES 229 Theorem 9.13: Principle of Angular Momentum. If I, is the moment of inertia of the rigid body about the center of mass, « is the angular velocity and A, is the torque or total moment of the external forces about the center of mass, then a . Ao = Bee) = Mee (en) Theorem 9.14: Principle of Conservation of Energy. If the external forces are conser- vative so that the potential energy of the rigid body is V, then T+V = dm 4H +V = E = constant (22) Note that jmi?= mvt is the kinetic energy of translation and HI," is the kinetic energy of rotation of the rigid body about the center of mass. INSTANTANEOUS CENTER. SPACE AND BODY CENTRODES ‘Suppose a rigid body ® moves parallel to a given fixed plane, say the zy plane of Fig. 9-6. Consider an 2’y’ plane parallel to the zy plane and rigidly attached to the body. As the body moves there will be at any time t a point of the moving 2’y’ plane which is instan- taneously at rest relative to the fixed ry plane. ‘This point, which may or may not be in the body, is called the instantaneous center. The line perpen- dicular to the plane and passing through the instan- taneous center is called the instantaneous azis. Figo ‘As the body moves, the instantaneous center also moves. The locus or path of the instantaneous center relative to the fixed plane is called the space locus or space centrode. ‘The locus relative to the moving plane is called the body locus or body centrode. The motion of the rigid body can be described as a rolling of the body centrode on the space centrode. ‘The instantaneous center can be thought of as that point about which there is rotation without translation. In a pure translation of a rigid body the instantaneous center is at infinity. STATICS OF A RIGID BODY ‘The statics or equilibrium of a rigid body is the special case where there is no motion. ‘The following theorem is fundamental. Theorem 9.15. A necessary and sufficient condition for a rigid body to be in equilibrium is that, F=0, A=0 (23) where F is the net external force acting on the body and A is the net external torque. PRINCIPLE OF VIRTUAL WORK AND D’ALEMBERT'S PRINCIPLE Since a rigid body is but a special case of a system of particles, the principle of virtual work and D’Alembert’s principle [see page 171] apply to rigid bodies as well. 230 PLANE MOTION OF RIGID BODIES [cHar. 9 PRINCIPLE OF MINIMUM POTENTIAL ENERGY. STABILITY At a position of equilibrium the net external force is zero, so that if the forces are conservative and V is the potential energy, F=-vV¥=0 (24) a . — ay Oz In such case V is either a minimum or it is not a minimum. If it is a minimum the equilibrium is said to be stable and a slight change of the configuration will restore the body to its original position. If it is not a minimum the body is said to be in unstable equilibrium and a slight change of the configuration will move the body away from its original position. We have the following theorem. Theorem 9.16. A necessary and sufficient condition for a rigid body to be in stable equilibrium is that its potential energy be a minimum. or in components, (25) Solved Problems RIGID BODIES 9.1. A rigid body in the form of a triangle ABC (Fig. 9-7] is moved in a plane to position DEF, ie. the vertices A, B and C are car- ried to D, FE and F respectively. Show that the motion can be considered as a transla- tion plus a rotation about a suitable point. Choose a point @ on triangle ABC which cor- responds tothe point H on triangle DEP. Perform the translation in the direction GH ao that triangle ABC ia carried to A°B'C”. Using H as center of rotation perform the rotation of triangle A’B'C through the angle indicated so that A'B’C’ is carried to DEF. Thus the motion has been accom plished by @ translation plus a rotation. Fig. 97 9.2. Give an example to show that finite rotations cannot be represented by vectors. Let Az represent a rotation of a body [such as the rectangular parallelepiped of Fig. 9-8(@)) ‘about the # axis while A, represents a rotation about the y axis. We assume that such rotations ‘ake place in a positive or counterclockwise sense according to the right hand rule, @ CHAP. 9] PLANE MOTION OF RIGID BODIES 231, s ¥ Tr v v ¥ z @ o : © Fig.9-9 In Fig, 9-8(a) we start with the parallelepiped in the indicated position and perform the rotation ‘Az about the 2 axis as indicated in Fig. 9-8(6) and then the rotation about the y axis as indi- caled in Fig, 9-S(e). Thus Fig. 9-8(c) is the result of the rotation A,+ Ay on Fig. 9-8(0). In Pig. 9-9(a) we start with the parallelepiped in the same position as in Fig. 9-8(a), but this time. we first perform the rotation A, about the y axis as indicated in Fig. 9-9(0) and then the rotation A. about the x axis as indicated in Fig. 9-9(c). Thus Fig. 9.0(e) is the result of the rotation A, +A, on Fig. 9-8(0). ‘Since the position of the parallelepiped of Fig. 9-8(c) is not the same as that of Fig. 9-0(¢), ‘we conclude that the operation A, +A, ig not the same as A, +A,. Thus the commutative Taw is not satisfied, so that A, and A, cannot possibly be represented by vector MOMENTS OF INERTIA 93. Two particles of masses m: and m2 respectively are connected by a rigid massless rod of length a and move freely in a plane. Show that the moment of inertia of the system about an axis perpendicular to the plane and passing through the center of mass is ja? where the reduced mass - = myma/(m, + m2). = Let r; be the distance of mass m, from the oon center of mass C. Then a— 7 is the distance ™, Sfimase| mr fm C2 Since ‘Ca the conar wo of mass, oO" rig.s10 = mfa—n) — tromwhich on = and or, = SE mir = mfa—r) fromwhich ry = pee and 8 at me ‘Thus the moment of inertia about an axis through C is 9.4, Find the moment of inertia of a solid circular cylinder of radius a, height k and mass M about the axis of the cylinder. y Method 1, using single integration. Subdivide the eylinder, a cross section of which appears in Fig. 3-11, into concentric rings one of which is the ele- and the element of mass is dm = 2verh dr. Fe ce Poti, 1 =f" aeorhar = jeatat Oy Fig. 941 232 PLANE MOTION OF RIGID BODIES [cuar.9 ‘Then since the m M = Sf" arerhar = orath : ® we find 1 = 4a Method 2, sing double integration. Using polar coordinates (r, 6), we see from Fig. 9-12 that the moment of inertia of the element of mass dm distant from the axis is vdm = Mehrdrds = ohrSdrde since ir drdo is the volume element and ¢ is the mass per unit volume (density). Then the total moment of inertia is 1 = SG" chrtarde = yooh tt) ‘The mass of the eylindor i given by Fig. so12 -) which can also be found directly by noting that the volume of the cylinder is rah. Dividing equation (1) by (2), we find 1/M = fat or I= 4Mat Go charade = ora © 9.5. Find the radius of gyration, K, of the cylinder of Problem 9.4, Since K2 = 1/M = 4o?, K = aiVE= pov. 96. Find the (a) moment of inertia and (b) radius of gyration of a rectangular plate with sides a and b about a side. Method 1, using single integration, (a) ‘The element of mass shaded in Fig. ind its moment of inertia about the y axis is (ob ds)z! = ebztdz. ‘Thus the total moment of inertia is SL ,rtstte = jeter ‘Since the total mass of the plate is M-= abs, we have I/M = fo or 1 (0) K2= 11M = ot of K = alVB= jay. ' ’ do I 7 _. a — | 2 | : Figs Pie ‘Method 2, using double integration. ‘Assume the plate has unit thickness. If dm = ody dz is an clement of mass [see Fig. 9-14), the moment of inertia of cm about the side which is chogen to be on the y axis is 2?dm = ea! dy da, ‘Then the total moment of inertia is Ce be. Then, as in Method 1, we find ‘The total mass of the plate is Mf Mat and K CHAP. 9 PLANE MOTION OF RIGID BODIES 238 9.7. Find the moment of inertia of a right circular cone of height 4 and radius @ about its axis. Method 1, using single integration. ‘The moment of inertia of the circular eylindrical dise one quarter of which is represented by PQR in Fig. 9-15 is, by Problem 9.4, Mertedey(r2) = Jrortdz since this dise has volume «de and radius r. ae From Fig. 9-15, 2= ‘Then the total moment of inertia about the = axis is tom def fete} = eee Also, Fig. 9.18 = AL bl Method 2, using triple integration. Subdivide the cone, one quarter of which is shown in Fig. 9-16, into elements of mass dm as indi- cated In the figure, In eylindrical coord mass dm of the cylinder, «is the density. ‘The moment of inertia of dm about the 2 axis is dm = or drdede A As in Method 1, A=# == = ‘Then the total moment of inertia about the's axis is Fig. 9418 ene ‘The total mass of the cone is GL Se wettest which can bo obtained directly by noting that the volume of the cone is fra. ‘Thos P= Mat, 98, Find the radius of gyration K of the cone of Problem 9. HS UM = Aya and K = aV = sav. THEOREMS ON MOMENTS OF INERTIA 99. Prove the parallel axis theorem [Theorem 9.8, page 226). Let 0@ be any axis and ACP a parallel axis through the centroid C and distant b from OQ. In Fig, 9-17 below, OQ has been chosen as the z axis so that AP is perpendicular to the ey plane at P. 234 9.10. 9.1. PLANE MOTION OF RIGID BODIES (CHAP. 9 If by is @ unit vector in the direction OP, then the vector OP is given by b = 0b o) ‘where 6 is constant and is the distance between Let ry and xf be the position vectors of ‘mass m, relative to O and C respectively. If t is the position vector of C relative to O then wwe have me ® ‘The total moment of inertia of all masses 1m, about axis OQ 1 = 3S mirwye Cy Fig. 917 ‘The total moment of inertia of all masses m, about axis ACP is to = 3 mich-bye w ‘Then using (2) we find = 3 micheby + ee bye oe ee = SZ mteebit + 2S miceebyie-dy) + 3 maieebye = eew(Zan)utede = eeu zm [Problem 7.16, page 178]. since +b; ‘The result is easily extended to continuous mass systems by using integration in place of summation. Use the parallel axis theorem to find the moment of inertia of a solid circular cylinder about a line on the surface of the cylinder and parallel to the axis of the cylinder. ‘Suppose the cross section of the cylinder is represented as in Fig. 9-18. Then the axis is represented by C, while the line on the surface of the cylinder is represented by A. If a is the radius of the cylinder, then by Problem 9.4 and the parallel axis theorem we ha In = Ie-+ Ma? = Mat + Mat = jMat Fig. 9-18 Prove the perpendicular axes theorem [Theorem 9.4, page 226). Let the position vector of the particle with mass m, in the ay plane be na atta ‘The moment of inertia of m, about the (see Fi 2 axis is mr, ‘Then the total moment of inertia of all particles about the # axis CHAP. 9} PLANE MOTION OF RIGID BODIES 235 Zmat Smet = non where J, and I, are the total moments of inertia about the 2 axis and y axis respectively. ‘The result is easily extended to continuous systems. 9.12. Find the moment of inertia of a rectangular plate with sides a and b about an axis perpendicular to the plate and passing through a vertex. Choose the rectangular plate (see Fig. 9-20] in the ay plane with sides on the and y axes. Choose the ¥ axis perpendicular to the plate at a vertex. From Problem 9.6 we have for the moments of inertia about the # and y axes, 1, = 4M, 1,=4Mat ‘Then by the perpendicular axes theorem the moment of inertia about the z axis is Ty = Intl, = 4M +02) EM(at 438) COUPLES 9418. Prove that a force acting at a point of a rigid body can be equivalently replaced by a single force acting at some specified point together with a suitable couple. Tet the force be F, acting at point P, as in », "ig, 9-21. If Q is any specified point, it is scen that the effect of F, alone is the same if we apply two forces fy and ~f, at Q. % In particular if we choose f,=—F,, Le. if f, 4 hhas the same magnitude as F, but is opposite in direction, we see that the effect of Fy alone is the e ‘same as the effect of the couple formed by Fy and 1)=—F, [which has moment r, x Fy] together with the force Fy. 944. Prove Theorem 9.5, page 227: Any system of forces which acts on a rigid body can be equivalently replaced by a single force which acts at some specified point together with a suitable couple. By Problem 9.18 we can replace the force F, at Py by the force F, at @ plus a couple of moment te XF,. Then the system of forces Fy,Fy,.. "Fy at points’ Py,Ps,...,Py ean be combined into’ forces Fy Fy. .Fy at Q having resultant FR tree try together with couples having moments FOB Beas oy ty X Ey which may be added to yield a single couple, Thus the system of forces can be equivalently replaced by the single force F acting at Q together with a couple, Fig. 9.22 236 PLANE MOTION OF RIGID BODIES (CHAP. 9 KINETIC ENERGY AND ANGULAR MOMENTUM B 9.15. If a rigid body rotates about a fixed axis with angular velocity #, prove that the kinetic energy of rotation is T= 4Ju* where J is the moment of inertia about the axis. Choose the axis af AB in Fig. 9:29. A particle P of ruse my mill rotate about the axa with angela speedo: Then Wt wil deserbe a circle PORSP with near speed trots where vy ie its dntace from arin AB. Thor te Kinetic neray of rotation about AB to fof = myo? tnd the total Kinet energy of all partis is r= Sanen where 1 = 3 impr? is the moment of inertia about AB. ‘The result could also be proved by using integration in place of summation 9.16. Prove that the angular momentum of the rigid body of Problem 9.15 is = Ie. ‘The angular momentum of particle P about axis AB is m,r}«. Then the total angular momentum ofall paviles about as A is (Bme)e = a Bm where I= & yr} is the moment of inertia about AB. ‘The result could also be proved by using integration in place of summation. MOTION OF A RIGID BODY ABOUT A FIXED AXIS 9.17. Prove the principle of angular momentum for a rigid body rotating about a fixed axis [Theorem 9.6, page 227] By Problem 7.12, page 176, since a rigid body is a special case of a system of particles, A= dO/dt_ where A is the torque or moment of all external forces about the axis and @ is the total angular momentum about the axis. da a Since = Ta by Probem 236, 8 = Sito) 9.18. Prove the principle of conservation of energy for a rigid body rotating about a fixed axis [Theorem 9.7, page 227] provided the forces acting are conservative. ‘The principle of conservation of energy applies to any aystem of particles in which the forces acting are conservative. Hence in particular it applies to the special case of a rigid body rotating bout « fxed axis, If T and V are the total kinetic energy and the potential energy, we thus have T4+V = constant = E Using the result of Problem 9.15, this can be written jfut + V = E. CHAP. 9) PLANE MOTION OF RIGID BODIES 237 WORK, POWER AND IMPULSE 9.19. Prove equation (12), page 227, for the work done in rotating a rigid body about a fixed axis, Refer to Fig. 9-4, page 227. Let the angular velocity of the body be «=k where k is a lunit vector in the direction of the axis of rotation. The work done by F Is aw = Poe = eelae = Bede = Peexnat = xPed = A dt = Avdt = ade where in the last two steps we use A= Ak, «=k and w= dé/dt 9.20. Prove equation (13), page 227, for the power developed. From Problem 9.19 and the fact that dé/dt = w, P= aWiat = Ardejat = dw 921. Prove Theorem 9.8, page 227. We have A= Ida/dt so that A=Tde/dt. Then from Problem 9.19 and the fact that eae have : So eat = [tote = yuk te Work done =f nan 922, Prove Theorem 9.9, page 228: The angular impulse is equal to the change in angular momentum, “ae = f' Ba = aa, 923. Prove Theorem 9.10, page 229, on the conservation of angular momentum if the net torque is zero. From Problem 9.22, if A then @, = Oy, THE COMPOUND PENDULUM ¥ 924. Obtain the equation of motion (17), page 228, for — CN pendulum is chosen au the xy plane (Fig. 8-2N] where ON the = axis through origin O is the horitontal aie of sonpension. Let point C have the position vector a relative to 0. Since the body is rigid, [al =a is constant and is the distance from 0 to C. ‘The only external force acting on the body is its Weight Mfg =—Mgj acting vertically downward. ‘Thus we have A = total external torque about = axis, aXMg = -aXMgj = aMgsinek (a) Fig. 9.24 ‘where kis @ unit vector in the positive = direction [out of the plane of the paper toward the reader}. Also, the instantaneous angular velocity is = ~ik ® 80 that if Zp is the moment of inertia about the z axis 238 PLANE MOTION OF RIGID BODIES (CHAP. 9 © = angular momentum about 2 axis = lye Substituting from (1) and (2) into A = dO/at, = 2a 4 Mine = amg sinek = F(-tbw) or + MB sing = 0 @) Method 2. ‘The force Mg ~—Mgi is conservative, so that the potential energy V is such that aoe ae ~ us ce WV = i - Me = -M9i x , Deo from which V = Moy+e = —Moancose +e CO) since y=—a.cos¢. This could be seen directly since y= —a cose is the height of C below the axis taken as the reference level. By Problem 9.16, the kinetic energy of rotation is jlq?= gi Then the principle of conservation of energy given T+V = fit — Maemo = constant = 5 © ing equation (5) with respect to t, dé + Moasines = 0 cor, since # is not identically zero, Io'+ Mga sins = 0 as required, 9.25. Show that for small vibrations the pendulum of Problem 9.24 has period P= 2rV/Mgall.. For small vibrations we can make the approximation i Ddecomes 0 = 6 0 that the equation of motion o mn ‘Then, as in Problem 4.28, page 102, we find that the period is P = 2eVTq/Moa. 9.26. Show that the length 1 of a simple pendulum equivalent to the compound pendulum of Problem 9.24 is I= Io/Ma, ‘The equation of motion corresponding to a simple pendulum of length 1 suspended vertically from 0 is [see Problem 4.23, equation (2), page 102] G4 Seine = 0 © Comparing this equation with (1) of Problem 9.25, we seo that 1 = Ip/ifa. GENERAL PLANE MOTION OF A RIGID BODY 9.27. Prove the principle of linear momentum, Theorem 9.12, page 228, for the general plane motion of a rigid body. ‘This follows at once from the corresponding theorem for systems of particl page 167), since rigid bodies are special ca [Theorem 7-1, 9.28. Prove the principle of angular momentum, Theorem 9.18, page 229, for general plane motion of a rigid body. ‘This follows at once from the corresponding theorem for systems of particles [Theorem 7-4, page 168], since rigid bodies are special cases. CHAP. 9) PLANE MOTION OF RIGID BODIES 289 929. A solid cylinder of radius @ and mass M rolls without slipping down an inclined plane of angle «. Show that the acceleration is constant and equal to 3g sina. Suppose that initially the eylinder has point O in contact with the plane and that after time t the cylinder has rotated through angle ¢ [see Fig. 9-25) ‘The forces acting on the cylinder at time ¢ are: () the weight Mg acting verti- cally downward at the center of mass C; (ii the reaction R of the inclined plane act- ing perpendicular to the plane; (iii) the frictional force f acting: upward along the Incline. Fig. 9.25 Choose the plane in which motion takes place as the zy plane, where the # axis is taken as positive down the ineline and the origin is at 0. If ris the position of the center of mass at time ¢, then by the prinetple of linear momentum, ME = Met Re But g=gsinai-geosaj, R= Rj, {=—fi. Hence (1) can be written ME = (Mo sina fi + (R~ Mg cosa) ® ‘The total external torque about the horizontal axis through the center of mass is A= 0KMg + OxR + CBXE = CBXE = (-a)Xxf) = —afk ch ‘The total angular momentum about the horizontal axis through the center of mass @ = Ice = Iel-ik) = =Ieke w where I is the moment of inertia of the eylinder about this axis, Substituting (8) and (4) into A= dO/dt, we find ~afk =k or Io¥ = af. Using = 21++ yh in (2), we obtai ME = Mosine~f, MY = R- Mgcosa © Now jf there is no slipping, #=as or ¢ = 2/a. Similarly, since the eylinder remains on the Incline, = 0; hence from (5), R= Aly cose. Using @=2/a in fei'=af, we have = Ic¥/a2. From Problem 9.4, [e= $Me’, Then substituting /= {Mf ¥ into the frst equation of (6), we obtain = 3g sina as required. 930. Prove that in Problem 9.29 the coefficient of friction must be at least 4 tana. ‘The covfficient of friction is» = /IR. From Problem 9.29 we have f= 4M¥= Mp sina and R=Mg cosa, ‘Thus in onder that slipping will not oceur, » must be at least f/ = } tan ws 931. (a) Work Problem 9.29 if the coefficient of friction between the cylinder and inclined plane is « and (b) discuss the motion for different values of u. (@) In equation (5) of Problem 9.29, substitute f= wR = My cosa and obtain oisina — u cos.) Note that in this case the center of mass of the eylinder moves in the same manner as 4 particle sliding down an inclined plane. However, the cylinder may slip as well as coll 240 PLANE MOTION OF RIGID BODIES (oHAP. 9 OF _ alg cosa Te yaar (sina ~ 9p cosa). ‘The acceleration due to rolling ai 2g cosa. ‘The acceleration due to slipping is Z— a’ () If (sina 34 co¥a) > 0, ie. 4 < f tana, then slipping will occur. If (sina — 3» cosa) = 0, ie. n= ftana, then rolling but no slipping will occur. These results are consistent with those of Problem 9.30, 9.82. Prove the principle of conservation of energy [Theorem 9.14, page 229]. ‘This follows from the corresponding theorem for systems of particles, Theorem 7-7, page 169. ‘The total kinetic energy 7 is the sum of the kinetic energy of translation of the center of mass plus the kinetic energy of rotation about the center of mass, ie, T= Amit + Heat If V is the potential energy, then the principle of conservation of energy states that if F is a constant, T+V = mitt Hatt V =F 933. Work Problem 9.29 by using the principle of conservation of energy. ‘The potential energy is composed of the potential energy due to the external forces [in this cease gravity] and the potential energy due to internal forces [which is a constant and can be ‘omitted]. Taking the reference level az the base of the plane and assuming that the height of the center of mass above this plane initially and at any time ¢ to be H and h respectively, we have MEE You + Moh = MoH or, using H—h sing and i= 3+j2?=% since j BME + Host = Moz sine Substituting «= 4 = /a and fe = 4Mo?, we find i= tox sina. Differentiating with respect to 1, we obtain 222 = fobsing or F = Gosing INSTANTANEOUS CENTER. SPACE AND BODY CENTRODES 934. Find the position vector of the instantaneous center for a rigid body moving parallel to a given fixed plane. Choose the XY plane of Fig. 9-26 as the fixed plane and the 2y plane as the plane attached to fand moving with the rigid body . Let point P of the zy plane [which may or may not be in the Figid body] have position vectors Rand r relative to the XY and zy planes respectively. If v and vq are the respective velocities of P and A relative to the XY system, va take vy + XR) where Ry is the position vector of A relative to 0. 11 P is to be the instantaneous center, then v= 0 so that #X@-R) = -v4 « Multiplying both sides of (2) by « X and using (7), page 5, ele R—Ry)) ~ (R—RYlo-a) ‘Then since « is perpendicular to R—Rq, this becomes (R-Ryst = wx vq oF @ CHAP. 9} PLANE MOTION OF RIGID BODIES 241 935. A cylinder moves along a horizontal plane. Find the (a) space centrode, (b) bo centrode. Discuss the case where slipping may occur. (a) The general motion is one where both rolling. and slipping may occur. Suppose the cylinder is moving to the right with velocity vq [the velocity of its center of mass] and i rotating about A with angular velocity «. Since w = “uk and vq = val, we ha 2X vq = “wval 80 that (8) of Problem 9:36 becomes (ooh R= n,-% In component form, MAY) = Xt aj—(uqloli or X= Xy Y= a ylo ‘Thus the instantaneous center is located vertically above the point of contact of the eylinder with the ground and at height a —v,/s above it ‘Then the space centrode is a line parallel to the horizontal and at distance a—v4/o above it. If there is no slipping, then v, = ao and the space centrode is the X axis while ‘the instantaneous center is the point of contact of the cylinder with the X a (©) The body centrode is given by |r| = vy/u, or a circle of radius v/s. In case of no slipping, % =u and the body centrode is the circumference of the cylinder. 936. Solve Problem 9.29 by using the instantaneous center. By Problem 9.85, if there is no slipping then the point of contact’ P of the eylinder with the plane is the instantaneous center. ‘The motion of P ig parallel to the motion of the center of mass, so that we can use the result of Problem 7.86(¢), page 191, ‘The moment of inertia of the cylinder about P's, by the parallel axis theorem, 4a? + Ma? = {Ma2. The torque about the horizontal axis through P is Mgasine, Thus GliMet) = Mya sine 3 = Wane Since 2 = a8, the acceleration is ¥ = 39 sin, Fig. 9.28 STATICS OF A RIGID BODY 937. A ladder of length 1 and weight W: has one end ‘against a vertical wall which is frictionless and the other end on the ground assumed horizontal. The ladder makes an angle « with the ground. Prove that a man of weight Wm will be able to climb the ladder without having it slip if the coefficient of friction between the ladder and the ground is at least r= 3" cot, Let the ladder be represented by AB in Fig. 9-29 and ‘choose an zy coordinate system as indicated, 242 PLANE MOTION OF RIGID BODIES (CHAP. 9 "The most. dangerous situation in which the Iadder would slip occurs when the man is at the top of the ladder. Hence we would require that the ladder be in equilibrium in such ease. ‘The forces acting on the ladder are: (j) the reaction y= Rit of the wall; (ji) the weight Wa = —Wrj of the man; (if) the weight W; =—Wij of the ladder concentrated at C, the center of gravity; (¥) the reaction Ry = Raj of the ground; (v) the friction force {= —fi. For equilibrium we require that F=0, A=0 « where F is the total external force on the ladder and A the total external torque taken about a which we shall take as the horizontal axis through A perpendicular to the xy plane. FS RF Wy tt R +E = (RP + (Wy Wit RY = 0 it Ry-f= 0 and -Wy- Wi +R, = 0 ® Also, A= (XR, + (0) Wy + (AC) XW; + (AB) >< Ry ++ (AB) XE = OX (RI) + 0) (-Wal) + (Wleos at ~ Hsin a x (—Wid) + eos Usina fx (yi) + (Loose it ~ Esina j) x (Ai) = -HW,cosak + UR; cosok — Ifsinak = 0 it YW, cose + Ry cosa ~ fsine = 0 « Solving simultaneously equations (2) and (8), we find P= R= (Wet 4W) cote and By = We tM "Then the minimum coefficient of friction necessary to prevent slipping of the ladder is MISCELLANEOUS PROBLEMS 938. Two masses m; and m: are connected by an inextensible string of negligible mass which passes over a frictionless pulley of mass M, radius a and radius of gyration K which can rotate about a horizontal axis through C perpendicular to the pulley. Discuss the motion. Choose unit vectors i as shown in Fig. 9-80. If we represent the acceleration of mass m, by Aj, then the acceleration of mass my is Aj. and j in the plane of rotation CChoode the tensions and Ty in the string a8 shown inthe figure. By Newton's second law, mai = Tt me = TS mH my Aj = Ty + mye = Te + mag) (2) Thus mA = mo—T mA = Tmo — ) of amen, T= meta mg ‘The net external torque about the axis through C is Fig.9-30 A= CanxCry + XCM) = aT—THk © ‘The total angular momentum about O 0 = Ice = Teck = Tete © Since A = d0/at, we find from (6) and (6), a(r,-7) o CHAP, 9] PLANE MOTION OF RIGID BODIES 248 If there is no slipping about the pulley, we also have 4 ® Using (4) in 7, -n © Using (6) in), A feu woe (o) ih mg + METAR ‘Thus the masses move with constant acceleration given in magnitude by (10). Note that if M = 0, ‘the result (10) reduces to that of Problem 3.22, page 76. 939, Find the moment of inertia of a solid sphere about a diameter. Let 0 be the center of the sphere and AOB be the ameter about which the moment of inertia is taken [Fig 9-31. Divide the sphere into discs auch as QRSTQ perpendicular to AOB and having center on AOB at P.- Take the radius of the sphere equal to «, OP = s, SP = and the thickness of the dise equal to'de, Then by Problem 8-4 the moment of inertia of the disc about AOB is Here deye2 From triangle OSP, 72 = a?—8. Substituting into (1), the total moment of inertia is Jeortae ® Fig. 9.51 1 S. frola?—amtde = Svea (2) ‘The mass of the sphere is Mo= Sf rolet-ayde = rate « which could also be seen by noting that the volume of the sphere is From (2) and (8) we have I/M = gat or I= gMat. 9.40. A cube of edge s and mass M is suspended vertically from one of its edges. (a) Show that the period for small vibrations is P = 27 \/2\/s/8g. (b) What is the length of the equivalent simple pendulum? (a) Since the diagonal of « square of side « has length Vera: the distance OC from axis O to the center of mass is jeVZ. ‘The moment of inertia I of a cube about an edge is the same as that of a square plate about a side, Thus by Problem 9.6, T= 4M(8+ 8) = get, ‘Then the period for small vibrations is, by Problem 9.25, P = oe VferMalge/Ey) = 2 VEVETG wi, °, (©) The Tength of the equivalent simple pendulum is, by Problem 926, 1 = gMotiM(gev3)) = avE0 Fig.0-82 941. Prove Theorem 9.11, page 228: The period of small vibrations of a compound pendulum is a minimum when the distance OC =a is equal to the radius of gyration of the body about a horizontal axis through the center of mass. If Ic is the moment of inertia about the center of mass axis and Ip is the moment of inertia, bout the axis of suspension, then by the parallel axis theorem we have 244 PLANE MOTION OF RIGID BODIES (CHAP. 9 Ip = To Mat ‘Then the square of the period for small vibrations is given by - fhe _ at (Te 402/Ké ) = Gat = FGete) = Ft where K2 = Ic/M is the square of the radius of gyration about the center of mass axis. Setting the derivative of Pt with respect to a equal to zero, we find dapy = tt (_KE Sen = S(-Fr1) = 0 from which a= Ke. This can be shown to give the minimum value since @4(P/da? <0. ‘Thus the theorem is proved. ‘The theorem 10 true even if the vibrations are not assumed small, See Problem 9.147 9.42. A sphere of radius a and mass m rests on top of a fixed rough sphere of radius b. ‘The first sphere is slightly displaced so that it rolls without slipping down the second sphere. Where will the first sphere leave the second sphere? Let the zy plane be chosen so as to pass through the centers of the two spheres, with the center of the fixed sphere as origin O [see Fig. 9-83). Let the position of the center of mass C of ‘the first sphere be measured by angle , and sup- pose that the pasition vector of this center of mass C with respect to O is x. Let r, and unit vectors as indieated in Fig. 9-88. Resolving the weight W=—mgj into com- ponents in directions 7, and 0, we have [compare Problem 1.48, page 24] Werpey + OWE, (moierpey + (moi mg sine ry — mg cos. 8, force N and frictional force f #0... Using Theorem 9.12, age 228, together with the result of Problem 1.49, Fig. 9.88 page 26, we have F = ma = m(F— rite + (rd + Be = WHNGt (Ng sin ode, + F— mg 05.0), from which m( rit) = N--mgsing, —mrd+2%8) = f — mgcos @ Since + +b [the distance of C from Q], these equations become = f— mo cose We now apply Theorem 9.13, page 229, The total external torque of all forces about the center of mass C is [since W and N pass through C], A= (an)xt = (ar) xf) = ~afk ‘Also, the angular acceleration of the first sphere about C is —mat 2 = N= mg sine, — ma+2) «= —Betoe = -G+i« Since there is only rolling and no Then g=s/2—o and y 1g it follows that are AP equals are BP, or b9 = ay. so that ip (bfa)al2— es -G+Hm cs CHAP. 9] PLANE MOTION OF RIGID BODIES 245 Since the moment of inertia of the first sphere about the horizontal axis of rotation through C is 1= gma’, we have by Theorem 9.13, ‘ ino(2E2)ie ee amen Using this value of f in the socond equation of (1), we find Te afk § = -qiije = ~ aba cose ® ‘Multiplying both sides by # and integrating, we find after using the fact that Oo +f = a sino) ® Taso Using (8) in the first of equations (1), we find N= Jmg(t7 sin leaves the second sphere where N= 0, ie. where ¢ = sin=! 10/17. 10), ‘Then the first sphere Supplementary Problems RIGID BODIES 943. Show that the motion of region R of Fig, 9-94 can be carried into region ’ by means of a translation plus a rotation about a suitable point. 944, Work Problem 9.1, page 280, by first applying a translation of the point A of triangle ABC. 945. If A,,Ay A, represent rotations of a rigid body about the 2, y'and axes respectively, is it true that the associative law applies, ie. is A, +(A,+A) (A,+A,) +4,2 Justify your answer. Fig. 9.34 MOMENTS OF INERTIA 846, Three particles of masses 9,5 and 2 are located at the points (—1,0,1), (2 —1,8) and (-2,2,1) respectively. Find (a) the moment of inertia and (b) the radius of gyration about the 2 axis, Ane 71 947, Find the moment of inertia of the system of particles in Problem 9.46 about (a) the y axis, (@) the z axis. Ane. (a) 81, (0) 44 248, Find the moment of inertia of uniform rod of length 1 about an axis perpendicular to it and Passing through (a) the center of mass, (b) an end, (c) a point at distance W/4 from an end, Ans, (0) AME, (0) AMP, (0) MP 849, Find the (a) moment of inertia and (8) radius of gyration of a square of side @ about a diagond Ava. (0) Met, (0) fay 950, Find the moment of inertia of a cube of edge a about an edge. Ans. Ma? 951. Find the moment of inertia of a rectangular plate of sides @ and 6 about a diagonal. Ans. 4Matb2/(a2 +62) 952. Find the moment of inertia of a uniform parallelogram of sides a and b and included angle @ bout an axis perpendicular to it and passing through its center. Ana. M(a!+ 88) sin? a 246 PLANE MOTION OF RIGID BODIES [cHAP. 9 958. Find the moment of inertia of a cube of side a about a diagonal. 954. Find the moment of inertia of a cylinder of radius a and height h about an axis parallel to the axis of the eylinder and distant b from its center. Ane. 4M(a? +268) 955. A solid of constant density is formed from a cylinder of radius a and height hand a hemisphere of radius a as shown in Fig. 9-85. Find its moment of inertia about @ vertical axis through their centers. Ane, M(2a¥+ 15a%h)/(10a + 15h) 956. Work Problem 9.55 if the eylinder is replaced by a cone of radius a and height A. 957. Find the moment of inertia of the uniform solid region bounded by the paraboloid cz=a2t+y? and the plane <= about the zaxi, Ane, Moh Fig. 9.5 958 How might you define the moment of inertia of a solid sbout (a) a point, (8) a plane? Is there any physical significance to these results? Explain. 259. Use your definitions in Problem 9.58 to find the moment of inertia of @ cube of side a about (o) avvertex and (b) a face, Ans. (a) Mat, (0) 4M? KINETIC ENERGY AND ANGULAR MOMENTUM 960. A uniform rod of length 2 ft and mass 6Tb rotates with angular epeed 10 radians per second ‘about an axis perpendicular to it and passing through its center. Find the kinetic energy of rotation, Ana, 100 Tb ft/sec? 961. Work Problem 9.60 if the axis of rotation is perpendicular to the rod and passes through an end. Ana. 400 Th ft/sec? 962, A hollow cylindrical disk of radius @ and mass M rolls slong @ horizontal plane with speed 2. Find the total kinetic energy. Ans. Mut 963. Work Problem 9.62 for solid cylindrical disk of radius a, Ans. §Mut 964. A flywheel having radius of gyration 2 meters and mass 10 kilograms rotates at angular speed of 6 radians/sec about an axis perpendicular to it through its center. Find the kinetic energy of rotation. Ans, 1000 joules 9.65. the angular momentum of (a) the rod of Problem 9.60 (8) the fywheel of Problem 9.64, Ana. (a) 6 Th £82/sec, (B) 200 kg m?/see 966. Prove the result of (a) Problem 9.15, page 298, (b) Problem 9.16, page 286, by using integration in place of summation. 961. Derive a “parallel axis theorem” for (a) kinetic energy and (®) angular momentum and explain the physical significance. MOTION OF A RIGID BODY. THE COMPOUND PENDULUM. WORK, POWER AND IMPULSE 868. A constant force of magnitude Fy is applied tangentially to a flywheel which can rotate about a fixed axis perpendicular to it and passing through its center. If the flywheel has radius a, radi of gyration K and mass M, prove that the angular acceleration is given by Foa/MK2, 9.69. How long will it be before the flywheel of Problem 9.68 reaches an angular speed uy if it starts from rest?) Ans, MKay/Fyt 270, Astuming that the fywheel of Problem 9.68 starts from rest, find (a) the total work done, (@) the total power developed and’ (@) the total impulse applied in getting the angular speed SD to vy. Ane. (2) AMRP3, (0) Fytay, (0) MR%oy CHAP. 9} PLANE MOTION OF RIGID BODIES 247 10 newtons, Work (a) Problem 9.68, (b) Problem 9.69 and_(e) Problem 9.70 if Fy K=05 meter, M=20 kilograms and wy = 20 radians/sec. Ans. (a) 2 rad/see®; (b) 10 see: (2) 250 joules, 200 joules/sec, 100 newton see 1 meter, 942 Find the period of small vibrations for a simple pendulum assuming that the string supporting the bob is replaced by a uniform rod of length 1 and mass M while the bob has mass im. “Ana, ty 4{2QEE Smt BOT Bm 97%. Discuss the eases (a) M=0 and (8) m=0 in Problem 972, 974, A rectangular plate having edyes of lengths «and b respectively hangs vertically from the edge of length a. (a) Find the period for small oscillations and (b) the length of the equivalent simple pendulum, Ans. (a) 2=V36/39, (B) 3b 945. A uniform solid sphere of radius a and mass M is suspended vertically downward from a point on its surface. (a) Find the period for small oscillations in a plane and (b) the length of the equivalent simple pendulum. Ans. (a) 2eVTa/bg, (8) Ta/5 9.16. A yo-yo consists of a cylinder of mass 80 gm around which a string of length 60.em is wound. If the end of the string is kept fixed and the yo-yo is allowed to fall vertically starting from rest, find its speed when it reaches the end of the string. Ans, 280 em/sec 9.47. Find the tension in the string of Problem 9.76, Ans. 19,600 dynes 918, A hollow cylindrical disk of mass M moving with constant speed vp ‘comes to an incline of angle a, Prove that if there is no slipping it will rise @ distance vi/(g sina) up the incline, 919, If the hollow disk of Problem 9.78 is replaced by a solid disk, how high will it rise up the incline? Ans, 30i/(4g sin e) 980. In Fig. 9-26 the pulley, assumed frictionless, has radius 0.2 meter and its radius of gyration is 0.1 meter. What is the acceleration of the 5 kg mass? Ane. 2.45 m/sec INSTANTANEOUS CENTER. SPACE AND BODY CENTRODES: S81. A ladder of length 1 moves so that one end is on a vertical wall and the other on a horizontal floor. Find (a) the space centrode and (b) the body centrode. Ans. (a) A circle having radius and center at point O where the floor and wall meet. (b) A circle with the ladder as diameter 982. A long rod AB moves so that it remains in contact with 4 the top of a post of height h while its foot # moves on a horizontal line CD [Pig. 9-37]. Assuming the motion to be in one plane, find the locus of instantaneous centers, 982. What is the (a) body centrode and (6) space eentrode in Problem 9.82? al 981, Work Problems 9.82 and 9.85 if the post is replaced by a fixed cylinder of radius a STATICS OF A RIGID BODY 9.85. A uniform ladder of weight W and length 1 has its top against a smooth wall and its foot on a oor having coeficient of friction s. (a) Find the smallest angle « which the ladder can make with the horizontal and still be in equilibrium, (3) Can equilibrium occur if x= 0? Explain. 248 PLANE MOTION OF RIGID BODIES (cHaP.9 986. Work Problem 9.85 if the wall has coefficient of friction ‘987. In Fig. 9-88, AZ ia a uniform bar of length I and weight W supported at C. It carries weights at A and W; at D so that AC=a and CD=b. Where must a weight W, be placed on AC so that the system will be in equilibrium? Ww, We Ws . c D DUD A eee ee Fig. 9.38 Fig 9.89 988 A uniform triangular thin plate hangs from a fixed point O by strings OA, OB and OC of lengths a, b and e respectively. Prove that the tensions 7, 7 and 7, in the strings are such that Tyla = Tyo = Tye. 989. A uniform plank AB of length I and weight W is supported at points C and D distant a from A and b from B respec- tively [Fig 9-89]. Determine the reaction forces at C and D respectively. In Fig. 9:40, OA and OB are uniform rods having the same density and connected at O a0 that AOB is a right angle. ‘The system is gupported at O so that AOB is in a vertical plane. Find the angles a and A for which equilibrium occurs, Ane. «= tant (a/d), p= 2/2 —tan-t (a/b) MISCELLANEOUS PROBLEMS 891. A cirular cylinder has radius @ and best h, Prove that the moment of Inertia about an axis perpendicular to the axis ofthe cpinder and passing theough the centro i jg Af(#-+ 3a) 992 Prove that the effect of a force on a rigid body is not changed by shifting the force along its line of action. .out slipping down an inclined plane of Prove that when it reaches the 992. A cylinder of radius @ and radius of gyration K rolls ‘angle « and length 1, starting from rest at the top of the inel bottom of the incline is speed will be V/{Bgla? sin «(a+ KA) 994. A cylinder resting on top of a fixed cylinder is given a slight displacement so that it rolls without slipping. Determine where it leaves the fixed cylinder. Ans. @= sin-4/7 where ¢ is measured as in Fig. 9-98, page 244. 995. Work Problem 9.42 if the sphere is given an initial speed vp. 996, Work Problem 9.94 if the cylinder is given an init speed 997. A sphere of radius a and radius of gyration K about a diameter rolls without slipping down fan incline of angle a. Prove that it descends with constant acceleration given by (ga sin o)/(a? + K®). $9, Work Problem 027 if the aphere is () ali, (6) hollow and of neti thiknen ‘Anz. (0) fg sina, () 29 sina 298A stow ahora has ter radian o and outer radius 6, Prove that if 0 Inf soan hen the out tn ta theo ir cena tt oth It at + Wt) ee CHAP. 9] PLANE MOTION OF RIGID BODIES 249 9.100, Wooden plates, all having the same rectangular shape are stacked one above the other as indicated in Fig. 9-41. (@) If the length of each plate is 2a, prove that equilibrium conditions will preva if the (n+ I)th plate extends a maximum distance of a/n beyond the nth plate where x =1,2,3,... (®) What is the maximum horizontal’ distance which can be reached if more and more plates are added? 9.401. Work Problem 9.100 if the plates are stacked on f sphere of radius R instead of on a flat surface ‘as assumed in that problem, Fig. 941 9.4102 A cylinder of radius a rolls on the inner surface of a smooth cylinder of radius 2a. Prove that the period of small oscillations is 2V8a/29. 9.108. A ladder of length 1 and negligible weight rests with one end against a wall having cooficient of friction 4, and the other end against a floor having coefficient of friction ya. It makes an angle « ‘with the floor. (a) How far up the ladder can @ man climb before the ladder slips? (b) What is the condition that the ladder not slip at all regardless of where the man is located? Ana. (2) pallny + tam a) (ayaa + Vy (0) tana > Vag 9104, Work Problem 9.103 if the weight of the Indder is not negligible. 9105. A ladder AB of length 1 (Fig. 9-42] has one end A on an incline of angle « and the other end B on a vertical wall. ‘The ladder is at rest and makes an angle f with the incline, If the wall is smooth and the incline has coefficient of friction 4, find the smallest value of x s0 that a man of weight W'q will be able to climb the ladder without having it \ slip. Check your answer by obtaining the result of Problem 9.37, page 241, a8 a special case, ~ 3 9.106. Work Problem 9.105 if the wall has coefficient of friction 4. Fig. 9-42 9407. A uniform rod AB with point A fixed rotates about vertical axis so that it makes a constant angle q with the vertical [Fig. 9-43]. If the length of the rod ie J, prove that the angular speed needed to do this is w= V(p seea)/3l. \ Fi, 9.108 suspended from the ceiling by a wire as shown in ‘The cylinder is given an angular twist @ and is then released. If the torque is assumed proportional to the angle through which the eylinder is turned and the constant of proportionality is A, prove that the cylinder will undergo simple harmonic motion with period 2ravm/2h. 9.109. Find the period i Ans. 2raVInT 2110, Work (a) Problem 9.108 and (6) Problem 9.109 if damping proportional to the instantaneous angular velocity is present. Discuss physically, 9.411, A uniform beam AB of length I and weight W [Pig. 9-45] is supported by ropes AC and BD of lengths a and b respectively making angles « and with the ceiling CD to which the ropes are fixed. If equilibrium conditions prevail, find the tensions in the ropes. Problem 9.108 if the cylinder ia replaced by a sphere of radius a. 250 912, 941s, ous, 916, sanr, oats. ais, 942. saz, 9424 9.35. PLANE MOTION OF RIGID BODIES (CHAP. 9 In Fig, 9-46 the mass m is attached to a rope which is wound around fa fixed pulley of mass M and radius of gyration K which can rotate freely about O. If the mass is released from rest, find (a) the angular speed of the pulley after time ¢ and (#) the tension in the rope. Prove that the acceleration of the ms patho? + K2), mm in Problem 9.112 Describe how Problem 9.112 can be used to determine the radius of gyration of a pulley. ‘A uniform rod AB [Fig. 9-47] of length 1 and weight W having its ‘ends on a frictionless wall OA and floor OB respectively, slides Starting from rest when its foot B is st a distance d from 0. Prove that the other end A will leave the wall when the foot B is at a | distance from O given by 4VEF>4a. A cslinder of mass 10 Ib rotates about a fixed horizontal axis through its center and perpendicular to it. A rope wound around it carries a mass of 201b, Assuming thet the mass starts from rest, find its speed after 5 seconds, Ans, 128 ft/sce Fig. 047 ‘What must be the length of a rod suspended from one end so that it will be a seconds pendulum fon making small vibrations in a plane? Ana, 149 cm [A solid sphere and a hollow sphere of the same radius both start from rest at the top of an inclined plane of angle « and toll without slipping down the incline, Which one gets to the bottom first? Explain, Ans, The solid sphere 1 compound pends of mata AC ad rave of gyration K shat feiotal axis is deleed A gman peda ot att the wotcl ands thn rene. Prove that i the cna of cee th tie thes the rastn fren onthe axle even BF eg UF cat Te FT TNTE A rctnasae paral of sido and is suspended vertielly from the sie of lnath Fab he prod at smal elaon Find the least coefficient of friction needed to prevent the sliding of a eireular hoop down an incline of angle «. Ane. 3 tana Find the period of small vibrations of a rod of Iength 1 suspended vertically about a point 4! from one end. A. pulley system consists of two solid disks of radius ry and ry respectively rigidly attached to each other and capable of rotating freely about a fixed horizontal axis through the center O. A weight W is suspended from a string wound around the smaller disk as shown in Fig. 948. If the radius of gyration of the pulley system is K land ite weight is w, find. (a) the angular acceleration with which the ‘weight descends and (b) the tension in the string. Ans, (@) Wor/(Wel + 0K), (0) WekP/(Wr5 + 0K ‘A solid sphere of radius & rolls on the inside of a smooth hollow aphere of radius @. Prove that the period for small oscillations is, given by 2ev a= Bb0~ ‘A thin circular solid plate of radius a is suspended vertically from a horizontal axis passing through a chord AB (see Fig. 9-49]. If it makes small oscillations about this axis, prove that the frequency fof such oscillations is greatest when AB is at distance a/2 from the center, Fig. 9-49 CHAP. 9] PLANE MOTION OF RIGID BODIES 251 9126. A unorm rd of length 5 essen vertically fom a string of length 2! which has it other cod feet Prove at the normal tegen for smal omatins in plane are 24f% and : and describe the normal mode a T 9.427. A uniform rod of mass m and length 1 is suspended from one of its ends, What is the minimum speed with which the other end should be hit so that it will describe a complete vertical circle? 9.128, (a) If the bob of a simple pendulum is a uniform solid sphere of radius a rather than a point ‘mass, prove that the period for small oscillations is 2rVU/g> 207501. (®) For what value of 1 is the period in (a) 2 minimum? 9.129, A sphere of radius a and mass M{ rolls along a horizontal plane with constant speed vy. It comes to an incline of angle a. Assuming that it rol's without slipping, how far up the incline will it travel? Ane, 100%/(7g sin a) 9.130. Prove that the doughnut shaped solid or torus of Fig. 9-50 has ‘@ moment of inertia about its axia given by 4M(da?+ 482) Q S181, A cylinder of mass m and radius a rolls without slipping down ‘8 45° inclined plane of mass M which is on a horizontal frietion- less table. Prove that while the rolling takes place the incline ‘will move with an acceleration given by mg/(3M + 2m), 9.4182 Work Problem 9.181 if the incline is of angle « Anns. (ng sit. 2a)/(8M + 2m —m cos 2a) 9.4138. Find the (a) tension in the rope and (b) accelers tion of the system shown in Fig. 9-61 if the radius of gyration of the pulley is 0.5m and its mass is 20 ke. 9134. Compare the result of Problem 9.193 with that obtained assuming the pulley to have negligible mass. 9135. Prove that if the net external torque about an axis fs zero, then it is also zero about any other axis. Fig. 9.81 9436. A solid cylindrical disk of radius @ has a circular hole of vadius b whose center is at distance from the center of the disk. If the disk rolls down an inclined plane of angle , find its acceleration. [See Fig. 9-52.) v Fig. 9.52 Fig. 0.53 Find the moment of inertia of the region bounded by the lemniscate 12 = a? cos2s [see Fig. 9-55] about the 2 axis. Ans. MaX(Sr~8)/48 Find the largest angle of an inclined plane down which a solid cylinder will roll without slipping if the coeficient of friction is». 9.489, Work Problem 9.188 for a solid sphere, 252, PLANE MOTION OF RIGID BODIES [cHAP. 9 9.40, Discuss the motion of a hollow cylinder of inner radius a and outer radius bas it rolls down an inclined plane of angle a. D141, A table top of negligible weight has the form of an equilateral triangle ABC of side s. The legs fof the table are perpendicular to the table top at the vertices. A heavy weight Wis placed on the table top at point which is distant a from side BC and 6 from side AC. Find that part of the weight supported by the legs at A, B and C respectively. 2Wa 20 ( sat) me a ae 9.42, Discuss the motion of the disk of Problem 9.186 down the inclined plane if the coeficient of friction is 9.143. A hill has a eross section in the form of a eycloid z= alo sing), y = all ~ cose) as indicated in Fig. 9-54. A solid sphere of radius b starting from rest at the top of the hill slight displacement so that it rolls ping down the hill. Find the speed — of its center when it reaches the bottom of the hill Ans. Vi0g@a= 677 Fig. 9.54 9.444, Work Problem 3.108, page 85, if the masses and moments of inertia of the pulleys are taken into account, 9.145, Work Problem 9.38, page 242, if frietion is taken into account. 9146, A uniform rod of length 1 is placed upright on a table and then allowed to fall, Assuming that its point of contact with the table does not move, prove that its angular velocity at the instant when it makes an angle # with the vertical is given in magnitude by Vg{i ~ cos e)/2I. 8147, Prove Theorem 9.11, page 228, for the case where the vibrations are not necessarily small, Compare Problem 9.41, page 248. 9.148, A rigid body moves parallel to @ given fixed plane. Prove that there is one and only one point fof the rigid body where the instantaneous acceleration is zero 9.149, A solid hemisphere of radius a rests with its convex surface on a horizontal table. If it is displaced slightly, prove that it will undergo oscillations with period equal to that of a simple pendulum of equivalent length 4a/8. 9450, A solid cylinder of radius @ and height A is suspended from axis AB indicated in Fig. 9-55, Find the period of small oscillations about, 9451, Prove that a solid sphere will roll without slipping down an inclined plane of angle « if the coefficient of friction is at least tan a. 9.4152, Find the least coefficient of frletion for an inclined plane of angle @ in order that solid cylinder will roll down it without slipping. Ans. ftane Chapter 10 SPACE MOTION of RIGID BODIES GENERAL MOTION OF RIGID BODIES IN SPACE In Chapter 9 we specialized the motion of rigid bodies to one of translation of the center of mass plus rotation about an axis through the center of mass and perpendicular to a fixed plane. In this chapter we treat the general motion of a rigid body in space. Such general motion is composed of a translation of a fixed point of the body [usually the center of mass] plus rotation about an axis through the fixed point which is not necessarily restricted in direction. DEGREES OF FREEDOM ‘The number of degrees of freedom (see page 165] for the general motion of a rigid body in space is 6, i.e. 6 coordinates are needed to specify the motion. We usually choose 3 of these to be the coordinates of a point in the body [usually the center of mass] and the remaining 8 to be angles [for example, the Euler angles, page 257] which describe the rotation of the rigid body about the point. If a rigid body is constrained in any way, as for example by keeping one point fixed, ‘the number of degrees of freedom is of course reduced accordingly. PURE ROTATION OF RIGID BODIES Since the general motion of a rigid body can also be expressed in terms of translation of a fixed point of the rigid body plus rotation of the rigid body about an axis through the point, it is natural for us to consider first the case of pure rotation and later to add the effects of translation. To do this we shall first assume that one point of the rigid body is fixed in space. The effects of translation are relatively easy to handle and can be obtained by using the result (10), page 167. VELOCITY AND ANGULAR VELOCITY OF A RIGID BODY WITH ONE POINT FIXED ‘Suppose that point 0 of the rigid body of Fig. 10-1 is fixed. Then at a given instant of time the body will be rotating with angular velocity » about the in- stantaneous axis through 0. A particle P of the body having position vector r, with respect to O will have an instantaneous velocity v, given by Wh = oxry a See Problem 10.2. Fig. 104 258 254 SPACE MOTION OF RIGID BODIES [OHAP. 10 ANGULAR MOMENTUM ‘The angular momentum of a rigid body with one point: fixed about the instantaneous axis through the fixed point is given by a = Lmivxw) D melt x (w X)} ® where m» is the mass of the sth particle and where the summation is taken over all particles of R. MOMENTS OF INERTIA. PRODUCTS OF INERTIA Let us choose a fixed xyz coordinate system having origin O and let us write Q= 4+) +0k, «= ites tok (8) tei + yj + ee aw ‘Then equation (2) can be written in component form as [see Problem 10.3}. OQ, = Ta, + Tyo, + Lye,| = Tate + Tyty + Te O) Tagg + Tagg + Teste where Te = Smuts), Iw = Dmelet tat), le = Loma +e) 6 Ty = —Dmew = Ie Te = — Lime = by (6) Tn = —Dmyt2 = Nee The quantities /.., I), Je: are called the moments of inertia about the x, y and z axes respec- tively. The quantities I:y, Is, are called products of inertia, For continuous mass distributions these can be computed by using integration. Note that the products of inertia in (6) have been defined with an associated minus sign. As a consequence minus signs are avoided in (4). MOMENT OF INERTIA MATRIX OR TENSOR, ‘The nine quantities Ire, Izy, ...,fe can be written in an array often called a matrix or tensor given by _ fer Toy Te Tez Tv Tye ” Tee Tey Ta and each quantity is called an element of the matrix or tensor. The diagonal consisting of the elements Ise,[y,Jee is called the principal or main diagonal. Since Tuy = Tuy Tee = Tay Tye = Toy @) it is seen that the elements have symmetry about the main diagonal. For this reason (7) is often referred to as a symmetric matrix or tensor. KINETIC ENERGY OF ROTATION ‘The kinetic energy of rotation is given by T = ML,o2 + 1,03 + Lot + 20,0, + 21,00, + 2I,.0,0,) de (9) CHAP. 10] SPACE MOTION OF RIGID BODIES 255 PRINCIPAL AXES OF INERTIA A set of 3 mutually perpendicular axes having origin O which are fized in the body and rotating with it and which are such that the products of inertia about them are zero, are called principal azes of inertia or briefly principal azes of the body. ‘An important property of a principal axis [which can also be taken as a definition] is that if a rigid body rotates about it the direction of the angular momentum is the same as that of the angular velocity. Thus Oo = le (10) where J is a scalar, From this we find [see Problem 10.6) that Na, + Taye + Tas, = 0 Bye + Og Dy + yt = Taste + Lgey + lag Dy q ° In order that (11) have solutions other than the trivial one 0, we require that | aid Tey Tex jee tie | 0 (12) Tex Ty ‘This leads to a cubic equation in J having 8 real roots I1,/2,Js. These are called the principal moments of inertia. The directions of the principal axes can be found from (11), as shown in Problem 10.6 by finding the ratio », :0,:», An axis of symmetry of a rigid body will always be a principal axis. ANGULAR MOMENTUM AND KINETIC ENERGY ABOUT THE PRINCIPAL AXES If we call 00, and 0,0, the magnitudes of the angular velocities and angular momenta about’ the principal axes respectively, then oe = Tyg 5 (as) ‘The kinetic energy of rotation about the principal axes is given by T= alot + Teoh + Lye8) (14) which can be written in vector form as [compare equation (9)] T = joo (18) ‘THE ELLIPSOID OF INERTIA Let n be a unit vector in the direction of #. Then «= on = o(cosai + cos fj + cosy k) (16) where cos a, cos 8, cosy are the direction cosines of » or nm with respect to the 2, y and z axes. Then the kinetic energy of rotation is given by T = ie «an 256 SPACE MOTION OF RIGID BODIES (HAP. 10 where 1 = 1,,costa + 1,, cos + I,,costy + 2,, cosacosf + 21,, cos cosy + 2F,, c08« C08 y (18) By defining a veetor p= aT (19) where p=p,itp,i+,k, (18) becomes 1,03 + yp} + Lot + 2Lpp, + 2lypyp, + 2epp, = 1 (20) In the coordinates p,.p,,p, equation (20) represents an ellipsoid which is called the ellipsoid of inertia or the momental ellipeoi If the coordinate axes are rotated to coincide with the principal axes of the ellipsoid, the equation becomes Toh + Let + Lys = 1 (er) where p,-p.sp, Tepresent the coordinates of the new axes. EULER'S EQUATIONS OF MOTION It is convenient to describe the motion of a rigid body relative to a set of coordinate axes coinciding with the principal axes which are fixed in the body and thus rotate as the body rotates. If A,,A,,\, and «,,0 0, tepresent the respective components of the external torque and angular velocity along the principal axes, the equations of motion are given by Hi, + (hoy, = Ay 14,4 (,—Boon, = Ay (e2) Nis + Boon ‘These are often called Huler’s equations. FORCE FREE MOTION. THE INVARIABLE LINE AND PLANE Suppose that a rigid body is rotating about a ° fixed point O and that there are no forces acting on the body [except of course the reaction at the fixed point]. Then the total external torque is zero. Thus the angular momentum vector @ is 5 Tovar constant and so has a fixed direction in space as mane indicated in Fig. 10-2. The line indicating this direction is called the invariable tine. Since the kinetic energy is constant [see Problem 10.84], we have from (15) avait tine fy #°0 = constant (28) Fig. 10-2 This means that the projection of » on @ is constant, so that the terminal point of describes a plane. This plane is called the invariable plane. As the rigid body rotates, an observer fixed relative to the body coordinate axes would see a rotation or precession of the angular velocity vector # about the angular momentum vector a. CHAP. 10] SPACE MOTION OF RIGID BODIES 257 POINSOT'S CONSTRUCTION. POLHODE. HERPOLHODE. SPACE AND BODY CONES As noted by Poinsot, the above ideas can be geometrically interpreted as a rolling without slipping of the ellipsoid of inet corresponding to the rigid body on the i le plane. ‘The curve described on the le plane by the point of contact with the ellipsoid is called the herpolhode [see Fig. 10-3). ‘The corresponding eurve on the ellipsoid is called the polhode. ‘To an observer fixed in space it would appear that the vector w traces out cone which is called the space cone. To an observer fixed on the rigid body it would appear that # also traces out a cone which is called the body cone. The motion can then be equivalently described as a rolling without slipping of one cone on the other. See Problem 10.19. Fig. SYMMETRIC RIGID BODIES. ROTATION OF THE EARTH Simplifications occur in the case of a symmetric rigid body. In such case at least two principal moments of inertia, say I, and Is, are equal and the ellipsoid of inertia is an ellipsoid of revolution. We can then show (see Problem 10.17] that the angular velocity vector « precesses about the angular momentum vector with frequency given by p= plBptla e where the constant A is the component of the angular velocity in the direction of the axis of symmetry. In the case of the earth, which can be assumed to be an ellipsoid of revolution flattened slightly at the poles, this leads to a predicted precession period of about 300 days. In practice, however, the period is found to be about 430 days. The difference is explained as due to the fact that the earth is not perfectly rigid, THE EULER ANGLES In order to describe the rotation of a rigid body about a point we use 8 angular coordinates called Euler angles. These coordinates denoted by 4,0, are indicated in Fig. 10-4. In this figure the xyz coordinate system can be rotated into the x'y’z’ system by successive rotations through the angles ¢ and then @ and then ¢ [see Problem 10.20]. ‘The line OA is sometimes called the line of nodes. In practice the 2',y/,2’ axes are chosen as the principal axes or body azes of the rigid body while the x, yand 2 axes or space azes are fixed in space. Fig. 10-4 258 SPACE MOTION OF RIGID BODIES [CHAP. 10 ANGULAR VELOCITY AND KINETIC ENERGY IN TERMS OF EULER ANGLES In terms of the Euler angles the components w,,»,.», of the angular velocity along the #',y' and 2’ axes are given by dsingsing + dsing j sino cosy — ésiny (25) good +h The kinetic energy of rotation is then given by T= Mot + Td + Ty) 26) where /,,/:,/s are the principal moments of inertia. MOTION OF A SPINNING TOP y An interesting example of rigid body motion occurs when a symmetrical rigid body having one point on the symmetry axis fixed in space is set spinning in a gravitational field. One such example ma : is that of a child’s top as shown in Fig. 10-5, where point O is assumed as the fixed point. For a discussion of the various kinds of motion which can occur, see Problems 10.25-10.32 and 10.36. Fig 1055 GYROSCOPES Suppose a circular disk having its axis mounted in gimbals [see Fig. 10-6] is given a spin of angular velocity w. If the outer gimbal is turned through an angle, the spin a of the disk will tend to point in the same direction as previously [see Fig. 10-7]. This assumes of course that friction at the gimbal bearings is negligible. In general the direction of the spin axis remains fixed even when the outer gimbal, which is attached to some object, moves freely in space. Because of this property the mechanism, which is called a gyroscope, finds many applications in cases where maintaining direction [or following some specified course] is important, as for example in navigation and guidance or control of ships, airplanes, submarines, missiles, satellites or other moving vehicles. A gyroscope is another example of a symmetric spinning rigid body with one point on the symmetry axis [usually the center of mass] taken as fixed. CHAP. 10) SPACE MOTION OF RIGID BODIES 259 Solved Problems GENERAL MOTION OF RIGID BODIES IN SPACE 10.1. Find the number of degrees of freedom for a rigid body which (a) can move freely in space, (b) has one point fixed, (c) has two points fixed. (@) 6 [see Problem 7.2(a), page 172] (0) 8 [see Problem 7.206), page 172 (©) If two points are fixed, then the rigid body rotates about the axis joining the two fixed res of freedom is 1, such as for example the angle of rotation 102. A rigid body undergoes a rotation of angular velocity # about a fixed point 0. Prove that the velocity v of any particle of the body having position vector r relative to O is given by v= «Xr. ‘This follows at once from Problem 6.1, page 147, on noting that the velocity relative to the moving system is dr/dtlyy = dr/dt|y 0. ANGULAR MOMENTUM. KINETIC ENERGY. MOMENTS AND PRODUCTS OF INERTIA 10.8. Derive the equations (4), page 264, for the components of angular momentum in terms of the moments and products of inertia given by equations (5) and (6), page 254. ‘The total angular momentum is given by miley X (eH) where we have used Problem 10.2 applied to the sth particle. Now by equation (7), page 5, we have eX (eXE) = alte (oxi + ogi + obled +B + 3) (eh td + alent, + oy + 0a) ~ ner) = fol +2) — oytaty — ont yayhh + (oyG8+ 2) — erry — eee + fosed + vB) ~ oxtty ~ onzsdk ‘Then multiplying by m,, summing over » and equating the coofficients of i, j and k to M,% and O, respectively, we find as required = [Smatsohass {Ramada + Smenhe Tests + Heyy + Teste a a Taga + Testy + Iai Zmerafe, + {3 mabe = Irate + Tysty + Tee For continuous mass distributions of density 6, we can obtain the same results by starting with a= Scone = Sstextoxmar 260 SPACE MOTION OF RIGID BODIES (CHAP. 10 104. If a rigid body with one point fixed rotates with angular velocity » and has angular momentum , prove that the kinetic energy is given by T= Je+Q. T= g3msb = 4 EmG,-4) PE me Xe (e XH) = 4 Zmter lel x mI} = Ra Xlexn) = er “where we have used the abbreviation ¥ in place of 3 . 105. Prove that the energy in Problem 10.4 can be written T= Ml,ot + Tol + Lyot + l,yoa, + 2l,,0,0, + 2 oo.) From Problem 10.4, we have T = forO = Huitaitow-tt+os+u9 = Hoste to0y +000) Neoaate + Igy + Fost) + ely + Lys + Tee) + eldeste + Iayty + Tete) Hleso! + lye + Lal + Beyogey + Brey + Baap) using the fact that Tey Teas Tye, Tyo Tee PRINCIPAL MOMENTS OF INERTIA AND PRINCIPAL AXES 106. Derive equations (11), page 255, for the principal moments of inertia and the directions of the principal axes. Using o-n w together with equations (9) and (, page 254, we have Taag + Tnyty + Teste = Tee Taste + Typ + Iyete = Ig ete + Dyy + Hetty = Hay o (ae Die + Fapty + Tests = Type + (yy ~ Dy + Bye Tete + Tyaty + Une Doe = ° ° ® o ‘The principal moments of inertia are found by setting the determinant of the coefficients of epee in (2) equal to zero, i Tego t Tey Te In | = 0 Ia Tee~ ‘This is @ cubic equation in I leading to three values J,,J3,13 which are the principal moments of inertia. By putting T=, in (2) we obtain ratios foro, uy, which yields the direction of » or the direction of the principal axis corresponding to 1,. Similarly, by substituting 1, and 1, ‘we find the directions of the corresponding principal axes. CHAP. 10] SPACE MOTION OF RIGID BODIES 261 10.7. Find the (a) moments of inertia and (b) products of inertia of a uniform square plate of length a about the z,y and z axes chosen as shown in Fig. 10-8. (©) The moment of inertia of an element dz dy of the plate about the 2 axis if the density is o is eydz dy. Then the moment of inertia of the entire plate about the x axis is ta = SS etdedy = foot = tae (0 Or the plata =o slmtry he moment of inert of the pint sou since the ms = 4Mat (2) is also evident by symmetry. Fig. 108 ‘The moment of inertia of dz dy about the z a inertia of the entire plate about the axis is 66s eo fis o(2?-+ 92) de dy, and so the moment of ‘This Iso follows from the perpendicular axes theorem [see page 226). (6) ‘The product of inertia of the element de dy of the plate about the 2 and y axes is exydedy, ‘and go the product of inertia of the entire plate about these axes is in = = fi Sena = it = ae ‘The product of inertia of the element dz dy of the plate about the 2 and z axes is the product of ode dy by the distances to the yz and zy planes, which are 2 and 0 respectively. ‘Thus we most have ae land similarly Tye = Tey = 0 © 108. Find the (a) principal moments of inertia and (b) the directions of the principal axes for the plate of Problem 10.7. («) By Problem 10.6 and the results (1)-(5) of Problem 10.7, we obtain Mat —1 — Mat ° <4Ma 4 Mat — ° a0 o ° 0 gMat 1 or (gate? — ngMter — 1) — (—yMer(—pMad || — 1) = 0 which can be written [Pe gMor + ZpMeat|[gaa?— 1) = 0 Setting the first factor equal to zero and using the quadratic formula to solve for I, we find for the three roots of (1), T= qgMa®, y= yyMat, y= goto ® which are the principal moments of inertia. (8) To find the direction of the prineipal axis corresponding to I, we let T ‘the equations (Mato, — Yar, + 00, —4Mata, + (JMa?—Day + Ou, Oa, + Ouy + (gMa—De, = 3 262 10.9. 10.10. Find the pri SPACE MOTION OF RIGID BODIES (CHAP. 10 ‘The first two equations yield =, while the third gives o=0. ‘Thus the direction of the principal axis is the same as the direction of the angular velocity veetor ai tos tek = od tod = ot) ing to 1, is in the direction 1+ 5 ‘Then the principal axis correspot Similarly, by letting = Ty = q_Ma® in (8) we find = wes de 0-89 that the direction of the correspond- ing principal axis is @ = od— ud = ay(t—}) oF If we let Ma? in (8) we find ey =0 while o, is arbitrary. This gives = shows that the third principal axis is in the direction i. ‘The directions of the principal axes are indicated by i+j, Ij and k in Fig. 10-9. Note that these are mutually perpendicular and that 1+) and 1—J have the directions of the diagonals of the squace plate which are lines of symmetry. ‘The prineipal moments of inertia can also be deter- mined by recognizing the lines of symmetry. Fig. 109 ipal moments of inertia at the center of a uniform rectangular plate of sides a and b. y axis and 2 axis [the last of which is perpendicular to the xy plane] h ‘The principal axes lie along the directions of symmetry and thus must be along the 2 axis, in Fig. 10-10, By Problems 9.6, 9.9 and 9.11 the principal moments of inertia are found to be [y= jMa% TMB, 1, = yfyMla2 + 24), v : — a Fig. 10-10 Fig. 10.11 Find the principal moments of inertia at the center of the ellipsoid @ ve a@tyte One cighth of the ellipsoid is indicated in Fig. 10-11. "The moment of inertia of the volume =1 clement dr of mass dr about the 2 axis or “" axis is (2+ y2}edr, and the total moment of Inertia about the = axis is Integration with respect to = gi n= af" eee (tt vededyae ef fe ee my TET ty te ‘To perform this integration let x= aX, y= 6Y where X and Y are new variables. Then the integral can be written oe tte fo cozt ern ATT ar ax CHAP. 10] SPACE MOTION OF RIGID BODIES 263, Introducing: polar coordinates R,0 in this XY plane, this becomes soobe f° fe coe + Pmt site) VT=TE a = acaba 49 f° VTS Ram = freabat+¥9 where we use the substitution 1—R? =U? in evaluating the last integral M= Since the volume of the ellipsoid is frabo, the mass By symmetry we fd f= JMOE+e, fy = EMO +e!) 10.11. Suppose that the ellipsoid of Problem 10.10 is an oblate spheroid such that a while ¢ differs slightly from @ or b. Prove that to a high degree of approximation, (s— hy = 1 = ela. wae _ @-olare, From Problem 10.10, if a= 6 $$ - S35 only slightly from @ then abe 2a and a?+e?~ 2a. ‘Thus, approximately, (y-1)lt = (a~eay/2at = 1 ee But if ¢ differs 10.12. Work Problem 10.11 for the case of the earth assumed to be an oblate spheroid. ‘Since the polar diameter or distance between north and south poles is very nearly 7900 miles while the equatorial diameter is very nearly 7026 miles, then taking the polar axis as the “3” axis we have 2¢= 7000, 2a 7926 or ¢= 3050, a= 2068. ‘Thus by Problem 10.11, (I~ 1j)/fy = 1~8860/3068 = .00328. ELLIPSOID OF INERTIA 10.18. Suppose that the moments and products of inertia of a rigid body ® with respect to an zyz coordinate system intersecting at origin O are Tez Iv Tsey Tey, Tea Tye PeSpeC~ tively. Prove that the moment of inertia of about an axis making angles 4,8, y with the x, y and z axes respectively is given by 1 = Ieecosta + Im costB + Tes €08*y + Bay C08 @ C08 f) + Bae COS COS y + Bye C08 f C08 y A unit veetor in the direction of the axis is given by n= copai + cos i + cosy ‘Then if m, has position veetor r, its moment of inertia about the axis OA is m,D? where D, = |r, %n|, But iio k nxn = | te fe cosa cos 8 cosy = (yp cony — = €08 Ah + ey cosa~ x, c08 i + (zy €08 8 — ay cos alk and [ep X nl? = (yy cosy — 2, e089)? + (2, cosa ~ 2, cos 7}? + (ey €08 B — vy cos a® = G+ costa + («E+ 2B) cost a + (abt uB) cost Bay cos a cos f — 22,5, co8a cosy — 2s £08 f £08 Y 264 SPACE MOTION OF RIGID BODIES [onar.10 Thun the total moment of inertia of all masses my is 1 = 3 mp? = [zmateahoore+ {Bmerrehh cote + {3 mat t-sD} cory + 2{ mau} conacoes + 2{—S may} ccna cany +2{-3maay} corp cory Tug C03 a + Iyy 6082 + Tug C08? + Way cosa cos B + By, cos. cosy + Bly cos 8 608 ¥ 10.14. Find an equation for the ellipsoid of inertia corresponding to the square plate of Problem 10.7. We have from Problem 10.7, Tog = AMO, Iyy = AMO, Tey = GMa, Tyg = —1Me?, Tyg ‘Then the equation of the ellipsoid of inertia is by equation (20), page 256, {Motel + |Mateh + gMo%E — [Malye = 1 or OEE ME Hay = Mat EULER'S EQUATIONS OF MOTION 10.15, Find a relationship between the time rate of change of angular momentum of a rigid body relative to axes fixed in space and in the body respectively. If the vittid body axes are chosen as principal axes having directions of the unit vectors ind e, respectively, then the angular momentum becomes @ = Iyyey + Taeste + Tyasey Problem 6.1, page 147, if » and b refer to space (fixed) and body (moving) axes respectively, then el, + axe 1 + Tabyes + Tabves F (ores + ose oe) X Useres + Tevet + Lue) (id, + p= Hodones}ey + ada + (ly — Kaowenden + shy + (a= Towss}es 10.16. Derive Euler’s equations of motion (22), page 256. By the principle of angular momentum, we have at 4-2 o) where A is the total external torque. Writing A= Aer + Aves + Aves ® where Ay,Ay, As are the components of the external torque along the principal axes and making use of (1) and Problem 10.15, we find 11d, + (ly—Iaesee = Ae Ing + (y— Hagan = Aa ® Ing + Ua Teva = Aa CHAP. 10) SPACE MOTION OF RIGID BODIES 265 FORCE FREE MOTION OF A RIGID BODY. ROTATION OF THE EARTH 10.17. A rigid body which is symmetric about an axis has one point fixed on this axis. Discuss the rotational motion of the body, assuming that there are no forces acting other than the reaction force at the fixed point, Choose the axis of symmetry coincident with one of the principal axes, say the one having direction e;- Then T,= Ty and Euler's equations become Ndy + Us— dows = 0 w Iida + Uy Inhegs = 0 @) ian = 8 ® Frm (0) 5 = catant = As ht) and) Ye ater die by Is—hy 5+(478am = 0 ® () o : ® where : ® os eam ” “hen rom (0) we ae = Comat an "Thus the angular velocity is = aye, + ores + ae Coosxtey + Csinatey + Ae (11) From this it follows that the angular velocity is constant in magnitude equal tow = |x| = VCP A? and precesses around the “S" axis with frequency f a, fs indicated in Fig, 10-13, Note that the vector w describes a cone about the “3” axis, However, this motion is relative to the body principal axes which are in turn rotating in space with angular velocity «. Fig. 10413, 10.18. Calculate the precession frequency of Problem 10.17 in the case of the earth rotating about its axis. Since the earth rotates about its axis once in a day, we have vs =A = 2r radians/day. ‘Then the processional frequency is from Problems 10.12 and 10.17, ey Na 1 -2) 2 = radians r= E(2pB)a = L(v-s)a = Laman) = ona ndinter "The period of precession is thus P = 1/f = 805 days. The actual observed period is about 430 days ‘and is explained as due to the non-rigidity of the earth. 266 SPACE MOTION OF RIGID BODIES [CHAP. 10 ‘THE INVARIABLE LINE AND PLANE. POLHODE, HERPOLHODE, SPACE AND BODY CONES 10.19. Describe the rotation of the earth about its axis in terms of the space and body cones. From Problem 10.17 the angular velocity # and angular momentum @ are given respectively by = oye + yt, + sey = Cleoe nt ey + sinxt es) + Aes B= ye, + Lyesty + Toesty = IC (coset e, + sin at ey) + Tye Let a be the angle betwoen we; = Ae; and Q. Then [es IO) cose = AVECTTAF cose = IyAt A and — a Vane + ae Similarly, let @ be the angle between ws and e, ‘Then este = legllel cone = AVC AT cosp = AP A and cop = 4 O° lara o From (1) and (2) we see that sina Co ee Ver Bat Vere ae 1c Thus tana = BE, w oe or Pay 4 @ Now for the earth or any oblate spheroid [flattened at the poles) we have I, Jy and ‘the space cone is outside the body cone [see Problem 10.121]. 1044 CHAP. 10) SPACE MOTION OF RIGID BODIES 267 THE EULER ANGLES 10.20. Show by using three separate figures how the xyz coordinate system of Fig. 10-4, page 257, is rotated into the 2’y’z’ coordinate system by successive rotations through the Euler angles 4, 0 and g. Refer to Figures 10-15, 1016 and 1017. Fi 10-16 indicates a rotation through angle ¢ of the + a ord if ” axes into an X and Y axis respectively while keeping the same # or Z axis In Fig. 10.16 a rotation about the X axis through angle is indicated so that the Y and Z axes of y Fig, 10-15 are carried into the Y" and Z' axes of Lg Fig. 10-16 respectively. In Fig. 10-17 a rotation about the 2" or 2" axis through angle y ia indicated eo that the X’ and Y" axes are carried into the 2 and y' axes respectively. In the figures we have indicated unit vectors on the sine axes; X,¥,Z axes, X'Y',2" axes and 2',y',2! axed by 1,i,k; L4,K; V,3',K and j,k! respectively. Fig. 10-15 z Fig. 1046 Fig. 10617 1021. Find the relationships between the unit vectors (a) i,j,k and 1,J,K of Fig. 10-15, (0) LJ, and VK’ of Fig. 10-16, (0) V,J’,K’ and i’, j,k’ of Fig. 10-17. (a) From Fig, 1015, i DE + GDS + GK = cong — sing d J = WDE + U-dS + UWK = sing T+ coses Ko= UM + Geos + eK = K (0) From Fig 10-16, T= er +a +b aeKK = FS GW + GSI + FRI = cose y= sine KE KS ORT + OFS FORK = sing + cose K (0) From Fig. 10-17, vo= qenr say + arene sing Fo Wii + iY + ae + cosy i! He (ADH + ROY + (YE = 268 SPACE MOTION OF RIGID BODIES (CHAP. 10 10.22. Express the unit vectors i,j,k in terms of i,j’, From Problem 10.21, i = cosgt— sings, J = singh +cosgJ, k= K T= 1, 4 = cosos sine RK’, K = sins J’ + cose K Vo = cosyi'-sinyy, Jt = sing + cosy s, W’ = ‘Then cospl — sing = cospl — sing cose’ + sing sine cos ¢ cosy i’ — cose sing ~ sing eos sing — sing cose cosy ff + sing sing ki (cos # cos y — sin # cos ¢ sin yi + (cos siny ~ sing cos cosy)j’ + sing sing k” = sing 14 cose = sing I" + cosy cone J! ~ cong sing R’ sin gcoay ¥ — sing siny + corp core sing I’ + cosy cose cosy i! ~ cose sine k’ = (sing cory + cong cose sin y)i + (ing sin y + cos cone cos yi’ ~ cos sing k’ k= sine J’ + cose’ = sing sing + sine cosy J + cose k 10.28. Derive equations (25), page 258. = ee ee ee = wk Gsinosing 7 +g sina cosy i’ + $eosek’ + boop it~ Gsing i’ + Gk (sing siny + § cosy)” sin yi’ + (Scone + Hk’ ‘Then since = api? + ost ok’, 10.24. (a) Write the kinetic energy of rotation of a rigid body with respect to the principal axes in terms of the Euler angles. (b) What does the result in (a) become if 11 = I? (a) Using Problem 1028, the required kinetic eneray is seen to be T= Mld + lad + Ty) {14(3 sin o sing + & cosy)? + afb sine cony ~ b sing)! + [6 cone + $F (0) TE y= ty, the result can be written T= MyGtsinte 4) + Bld cose +H MOTION OF SPINNING TOPS AND GYROSCOPES 10.25. Set up equations for the motion of a spinning top having fixed point O [see Fig. 10-18]. HAP. 10) SPACE MOTION OF RIGID BODIES 269 Let zyz represent an inertial or fixed set of axes having origin O. Let 2'y's' represent principal axes of the top having the same origin. Choose the orientation of the 2'y’ plane so that Oz, Oz’ and Oy’ are coplanar. ‘Then the 2' axis is in the xy plane. The line ON in the 2'y’ plane making an angle y with the 2 axis is assumed to be attached to the top. ‘The angular velocity corresponding to the rotation of the 2'y's/ axes with respect to v the 2ye axes is = oye, + ext + axes 0 In obtaining the angular momentum we must use the fact that in addition to the com- ponent ws; due to rotation of the 2’y/2" system ‘there is also the component # = ee) = jes since the top is spinning about the 2’ axis, ‘Then the angular momentum is Fig. 1048 B= Toye, + ses + Iles + es ® Now if we let subscripts / and denote the fixed system and body system respectively, we have by Problem 6.1, page 147, aa a ao = Bl rexa « ‘Using (1) and (2) in (8), we find = Unb + Uy Ideas + Tyeet)ey Isher ~ Laebes w + Usa +8) + Uy T)eyendes + Wake + ‘The total torque about O is A= (les) x (me) es + (keeles + (ke3)e, = 08 (w/2 = sino e; + cose leg) (mo) © Since k= ee the ors is A= matin ah) = mot ine o Then oning A= $2), wth 1,1, we fink from (and y+ y= Donn + Tne = maine 13s-+(h~2ownn Ione = 0 ” iuistd = 0 10.26. Express equations (7) of Problem 10.25 in terms of the Euler angles 6 and ¢ of Fig. 10-18. ‘The components We find ‘en; ean be obtained from Problem 10.23 by formally letting y oy = 4 = Gsing, oy = gone o ‘Then equations (7) of Problem 10.25 become 270 SPACE MOTION OF RIGID BODIES [cHap. 10 13 + Uy M)# sin cone + Ist mol sin 6 1G sino + 35 cos6) + (ly —1)83 cose — Ibe = 0 ® 1Gcose — gasino +’) = 0 ‘The quantities g, 6 and e are often known as the magnitudes of the angular velocity of precession, ‘of mutation and of spin respectively. 10.27, Prove that the equations (2) of Problem 10.26 can be written as (a) hi — Ng*sin9 cosd + LASsing = mgtsing (0) L($sin6 + 244 cosa) — LAI = 0 where A is a constant. From the third equation in (7) of Problem 10.25, ete A or 8 = Amey wo ‘Then substitution into the first and second of equations (7) yields Thy ~ Tyayey + Tag = mol sine ® Ip + haya Tord = 0 ® Using the results (1) of Problem 10.26, we find that equations (2) and (8) reduce to the required ‘equations 10.28. (a) Find the condition for steady precession of a top. (0) Show that two precessional frequencies are possible. Since ¢ is constant so that ¥=0, we have from Problem 10.27(a), (34 cose — 43+ mad sine = 0 o 112 coss ~ IhAS + mat = 0 + VEAP tol wo from which WAS Vea enallicase) ® 2 wae ‘Thus there are two frequencies provided that TEAS > Amgll cose ® If HA®=d4mglf, cose only one frequency is possible If A is very large, eg. if the spin of the top is very great, then there are two frequencies, one large and one small, given by BAM, 6086), mollgA ® 10.29. Prove that (a) (i? +4? sint@) + HA? + mglcos# = constant = E (b) hg sin?@ + [nA cose constant K and give a physical interpretation of each result. (a) Multiply equations (7) of Problem 10.25 by wy, 42 and os +s respectively, and add to obtain Iy(aydy + opin) + Tnog +9034 8) = mot sin which can be written as 4 anit +e) + phlestar) = Zc-mat cose) EQneh +o) + alert ae) = Gy-mateo A as well as the results j= and op=¢sing, we find into) + pled? + molcose = E ” CHAP. 10] SPACE MOTION OF RIGID BODIES 271 where # is constant. The re since the Kinetic energy is, t is equivalent to the principle of conservation of energy, 1 = G2 + point) + Aya? ® while the potential energy is V = mgl cose @ and T+V=E is the total energy. (8) Multiplying the result of Problem 10.27(6) by sin, sin? @ + 21,68 sin ¢ cose — IyAé sir o which can be weitten 4 Haag soto + 1A cove) = 0 Intgrating, Lp sinte + 1yA cone = constant = K w ‘To interpret this result physically, we note that the vertical component of the angular ‘momentum is [,d sin? #+I,A cos¢ (see Problem 10.123], and this must be constant since the torque due to the weight of the top has zero component in the vertical direction. 10.30. Let «= cos @. Prove that: @) 12 = (e—puy(L—w) — (yu = flu) where a= 2(E-4sA%\/h, B mals, y= Ki, 8 = 1sA/h; au 0) t= Sf + constant V E(u) (a) Prom Problem 1029, HG! + Gainey + ALA + maton = wo Ipainte + ly cone = K ” 5 - Kotaheme From (2), 3 od @ Substituting this into (1), (&~ bh en 4 Eafe O80" 5 gigA® + mglcose = E and sinte K~ Iau ay eg (Rcd? maint Thus e+ (Sy, ae which can be written as #2 = Amd— wh) — ya = fy ® where <= QB—AMy B= BmOly Y= Klly 8 o Note that with this notation (2) can be written x 272 SPACE MOTION OF RIGID BODIES [CHAP. 10 (8) From the result of (a) we have, since i> 0, a ee Cicleste” “Peaglh a cas at Vie Integrating, v= Settee io ‘The integral can be evaluated in terms of elliptic functione which are periodic. 10.31. (a) Prove that @=0 at those values of « for which (a) From Problem 10.80(a), i = fw) = (a—Aw)(t~w (uw) = (a Buy(1 — ut) ~ (y— Su)? = 0 (0) Prove that the equation in (a) has three real roots 1, us, ws but that in general not all the angles corresponding to these are real. ~~ o 0 where é=0 oF flu) ‘Thus §=0 at the = Buy(d = a8) ~ (y= au)? = 0 @ Since i= sing 3, it follows that roots of the equation as To) () Equation (1) can be written as flu) = Bud ~ (88+ aut + (2y8— puta ye ? Since > 0, it follows that fey = 9, foe) = 2 WA) = 7-8 fet) = 04 ‘Thus there is a change of siz from = to + as u goes from 1 to =, and | consequently there must be a root, say tig, between 1 and = as indieated in Fig. 10-19. Now we know that in order for the motion of the top to take place we must have Also, since 0505/2, e0Susl, It thus follows that there must be two roots 1 and u, between 0 and 1, as indicated in the figure. It follows that in general there are ‘ovo corresponding angles ¢, and ¢, such that cose; = mj, cos42 = uz. In special cases it could happen that 1, = oF on a Fig. 1049 10.22. Give a physical interpretation of the results found in Problem 10.81. ‘The fact that there are two roots m; and vw, corresponding to # and ¢, respectively, shows that the motion of the top is such that its axis always makes an angle # with the vertical which lies between 4, and #;, ‘This motion, which is a bobbing ap and down of the axis between the limits 0; and #, is called mutation and takes place at the same time as the precessional motion of the axis of the top about the vertical and the spinning of the top about its axis. Because the motion can be expressed in terme of elliptic functions [see Problem 10.104], we can show that it ts periodic. In general the tip of the axis of the top will describe one of various types of curves such as indicated in Figs. 10-20, 10-21 and 10-22. The type of curve will depend on the root of the equation [see equation (6) of Problem 10.30] CHAP. 10) SPACE MOTION OF RIGID BODIES 273, If this root given by y/6 is greater than 1s, the curve of Fig. 10-20 occurs, If it is the same as tu the curve of Fig. 10-21 obtains, If it is between 1, and uy the curve of Fig. 10.22 occurs. Other ceases can arise if the root is the same as 1 oF is less than 1; [see Problems 10.124), ty < 95 Se Fig. 10-22 Aside from the general motion which is made up of nutation and precession, there are various special cares which can arise. One of these is the case of steady precession with no hnutation [sce Problem 10.28]. In this eate 1 =v, s0 that @;—¢, or ¢=eonstant, Another cease is the “sleeping top” which oceurs where 1; ~ y= 1 and the axis of the spinning top always remains vertical [see Problem 10.36). MISCELLANEOUS PROBLEMS 10.38. If T is the total Kinetic energy of rotation of a rigid body with one point fixed, prove that d7/dt=«-A where all quantities refer to the body principal axes. Multiplying both sides of the Euler equations (8) of Problem 10.16 by wypwzvus respectively and adding, we obtain ibs + Heenda + Iyeady = rds bande teats w ie _ at But Hy, + Iaoada + Taoady = ES ut tad t hd) = 2 cy and rds teat cay = (overt gta + ones) (Ayes + Maes + Ase) sea ® ‘Ths (1) becomes avidt = wb 6 10.34. (a) Prove that if there are no forces acting on a rigid body with one point fixed, then the total kinetic energy of rotation is constant. (b) Thus prove that #0 = 27 = constant. (a) Since there are no forces, A=0, Then by Problem 10.33, @T/at=0 or constant. (0) Since @= Ire, + Iaats + oyts and «= oy taste testy = 1d + Mead + Ih = 27 = constant 10.35. Find the precession frequency of Problem 10.17 in terms of the kinetic energy and angular momentum of the rigid body. ‘The kinetic energy T= Myo + Iyed + tye) = hak + Tod + wD = AC? + LA 0 that 1G + 1,A* = 2 24 SPACE MOTION OF RIGID BODIES (CHAP. 10 ‘The angular momentum is © = Bayes + Treats + Foeaty = Treier + Loree + Losses TsCcost ey + sin xt es) + IyAes so that @ = |0| = VROFRA? or Ho + Rat = a ® Solving (2) and (2) simultaneously, we find 2h ~ Ma=T - cy) 10.36. Find the condition for a “sleeping top”. For a “sleeping top” we must have #=0 and vertical ‘and no nutation can take place. ‘Then from Problem 10.29, IyA = K, IgA? = 218mg, ‘Alto, from Problem 10.20 we has and y= 8, and #0 Flu) = (= Puy 8) — (y= St = aft WA = 08) = A= we = A wAfad +m — 77) It follows that f(u) = 0h a= Lally, = 2mgllly, y= WAM, 6 = LAM. Thus «= 8 4 double root atu =1, while the third root is given by 2 « nal Then the top will “sleep” if this root is greater than oF at most equal to 1, x0 that A = Amott 8 Of course, even though this condition may apply at the beginning, energy will in practice be diminished due to friction at the support so that after some time we will have A < 4mglfy/I, Tn such case precession combined with mutation will be introduced. Further loss of energy will ‘ultimately cause the top to fall down. 10.37. Find the torque needed to rotate a rectangular plate of sides a and b [see Fig. 10-23) about a diagonal with constant angular velocity «. . By Problem 10.9 the principal moments of inertia of the plate at the centar O are given by 1 Mat, Ty= Mit, Iy= MCR + 0) On! . — + (obs : ! San a Vate vabtet Fig. 10-28 =v Thus “ a. o Varo Substituting (1) and (8) into Euler's equations Thy + Us—Taenan = Ay Taba + (hy Tones Iyhis + a= Woven A As CHAP. 10] SPACE MOTION OF RIGID BODIES 275 _ Mi? aPabut ne wefind A, =0, A:=0, y= MUFSO Thus the required torque about O i M(W# = a®adst . Toa + 6) - Note that if the rectangular plate is a square, ic. if 2, then A. Supplementary Problems GENERAL MOTION OF RIGID BODIES IN SPACE. 10.38. Find the number of degrees of freedom of (a) @ sphere free to roll on a plane, (b) an ellipsoid free to rotate about a fixed point, (c) an airplane moving in space. Anz. (a) 8, (0) 8, (0) 6 1039, In Fig. 10-24 a displacement of a tetrahedron space is indicated. Show directly that the di placement can be accomplished by a translation plus a rotation about e suitable axis, thus Mlustrating Chasle’s theorem [page 224] for space. 10.0, Give an illustration similar to that of Problem 10.39 involving a rigid body whose surfaces are not plane surfaces. A , 1041, Derive the result of Problem 102, page 259, Co) ® without using Problem 6.1, page 147. Fig. 10-24 ANGULAR MOMENTUM. KINETIC ENERGY. MOMENTS AND PRODUCTS OF INERTIA 1042. A rigid body consists of 8 particles of masses 2,1,4 located at (1)—1,1), (20,2), (—1,1,0) respee- tively. Find the angular momentum of the body’ if it is rotated about the origin with angular velocity © =i 2j-+4k, Ane, ~6)+42k 10.43. Determine the (a) moments of inertia about the x, y and z axes and (6) the products of inertia for the rigid body of Problem 1 Ame. (a) Loe = 32, Tyy = 36, Fee = 16; (©) Tey Tye = 2, Tee = 8 10.44. What is the kinetic energy of rotation for the system of Problem 1042? Ans. 180 1045. Find the (a) moments of inertia and (6) products of inertia of a uniform rectangular plate ABCD of sides AB=a and AD=b taken about axes AB,AD and the line perpendicular to the plate at B. calling axes through AB and AD the 2 and y axes respectively. 10.45, Find the (a) moments of inertia and (6) products of inertia of a cube of side a taken about 2,y, axes coinciding with three intersecting edges of the cube. Ana (0) a BM 02,8) Tay = ye = Lee = Mat 1047, Find the (a) angular momentum and (#) kinetic energy of rotation of the cube of Problem 10.46 shout the point of intersection O of the three edges if the cube has an angular velocity = 2+5)— 8k about O. Ans. (a) ;fyMa2(l0i + 43} — 45k), (0) 1854a%/12 216 SPACE MOTION OF RIGID BODIES [CHAP. 10 1048. Find the (a) moments of inertia and (6) products of inertia of the uniform solid sphere at+y'+ zt =a? in the first octant, Le. in the region 2 =0, y =O, 2 =0. Ame. (0) Teg = Ty = Tan = BMO2, (0) Tay = Ig = Teg = ~2MO*/B PRINCIPAL MOMENTS OF INERTIA. PRINCIPAL AXES. ELLIPSOID OF INERTIA 1048. Prove that the principal moments of inertia for a system consisting of two particles of masses ‘my and my connected by a massless rigid rod of length {ate [, =, = mimgt/im, +m), Iy'= 0. 1050, Find the (@) principal moments of inertia and (2) directions of the principal axes for the system of Problem 10.42. Ane, (@) y= 18, y= 18-V, y= 184 VB 0) Sk, AU+VBV-j+k YA-VB-I+k, 1051, Determine the (a) principal moments of inertia and (@) directions of the principal axes for right triangle ABC of Fig, 10-25 about point C. 1052. Find the principal moments of inertia at the center of @ parallelogram of sides a and b and acute angle a. 1053, Find the (a) principal moments of inertia and (®) diree- tions of the principal axes for the cube of Problem 10.46. Fig 10-25 Ih= Hie, I, associated with I; is in the direction of the diagonal from the origin. Axes associated with [, and [y have any mutually perpendicular directions in a plane perpen- dicular to this diagonal 1054, of a uniform eylinder of radius @ and height 4, Ans. 1, = Iy= pyM(Sa2+ 12), Ty = 4Mat 1055, Obtain the principal moments of inertia and directions of principal axes for a rectangle of les a and b by using Problem 10.45 and equations (11), page 255. Compare with Problem 10.9, page 262. 1056, Find the lengths of the axes of the cllipsoid of inertia corresponding to the rectangle of Problem 10.55, Ane. 4V/3/Ma?, 4/3/62, 4V/3/M(a? +6) 1057, Find the lengths of the axes of the ellipsoid of inertia corresponding to the cube of Problem 10.46. Ans. 4VSME?, 457M, 2678 1058. Prove that the ellipsoid of inertia for a regular tetrahedron is a sphere and determine its radius. 1059. If J.J, are the principal moments of inertia, prove that hehtl, hihtlh hehth 1060, Under what conditions do any or all of the equality signs hold in Problem 10.50? 1061, Prove that if a rigid body is a solid of revolution about a line L, then L is a principal axis corresponding to any part of L. 1052. Suppose that a rigid body is symmetrical about a plane P. Prove that if L is a line perpendicular to P at point O, then L is a principal axis corresponding to point O. EULER'S EQUATIONS OF MOTION 1063, A rigid body having one point O fixed and no external torque about O, has two equal principal faxes of inertia, Prove that it must rotate with constant angular velocity. CHAP. 10] SPACE MOTION OF RIGID BODIES 217 1064. Write Eul significanes of plane motion of a rigid body and 's equations for the ca jscuss thelr physical 1065. Solve the problem of a compound pendulum by using Euler’s equations. 1066. Describe how Euler's equations can be used to discuss the motion of a solid eylinder rolling down an inclined plane, 61. Write Euler's equations in case the axes are not principal axes, FORCE FREE MOTION. INVARIABLE LINE AND PLANE. POLHODE, HERPOLHODE, SPACE AND BODY CONES 168, If two principal moments of inertia corresponding to the fixed point about which a rigid body rotates are equal, prove that (2) Poinsot’s ellipsoid is an ellipsoid of revolution, (6) the polhode is a circle and (2) the herpolhode is a circle. 1069, Discuss the (a) invariable line and plane, (b) polhode and herpolhode and (c) space and body ‘cones for the case of a rigid body which moves parallel to a given plane, ie. plane motion of a rigid body. 1070, (a) How would you define the instantaneous axis of rotation for space motion of a rigid body? () What is the relationship between the instantaneous axis of rotation and the space and body 1071, Prove that relative to ite center of mass the axis about which the earth spins in a day will rotate about an axis inclined at 28.5° with respect to it in 25,780 years. THE EULER ANGLES 10:72, Using the notation of Problem 10.20, page 267, find: (a) 1,J,K in terms of 1,4,k; (0) Va K’ in terms of 1,4,K; (0) 144K’ in terms of 1',J',K’. Ans. (a) 1 = cosgt+singj, J = -sing i+ cose), K = k @) =, J = copes taineK, K’ = sine + cone © ¥ = cosy tl +sinys, J = —siny + cosy, K 1018, Prove the results up = dcosg + Gsins sing oy = Gsing ~ Fsine cose w= bt vee wn it prove that the kinetic energy of rotation of & rigid body referted to principal snes in P= fi(Get 492+ 25 cone). MOTION OF SPINNING TOPS AND GYROSCOPES, 10.75. A top having radius of gyration about its axis equal to 6em is spun about its axis. ‘The spinning point is fixed and the center of gravity is on the axis at a distance Sem from this fixed point, If it ie observed that the top precesses about the vertical at 20 revolutions per minute, find the angular speed of the top about its axi Ang, 8.10 rev/see or 19.5 rad/see 1016, A uniform solid right circular cone of radius a and height A is spun so that its vertex is fixed fand ite axis is inelined at a constant angle « with the vertical. If the axis precesses about the vertical with period P, determine the angular speed of the cone about its axis. 10.77, Work Problem 10.76 if the cone is surmounted by a uniform solid hemisphere of radius a and the same density. 10178, Explain physically why the spin axis of the gyroscope of Figures 10-6 and 10-7, page 258, should maintain its direction. 278 SPACE MOTION OF RIGID BODIES (CHAP. 10 1079, Explain how & gyroscope can be used to enable a ship, airplane, submarine or missile to follow some specified course of motion. MISCELLANEOUS PROBLEMS 1080, A uniform solid cube of side a and mass M has its edges lying on the positive 2, y and z axes of a coordinate system with vertex at the origin O. If it rotates about the = axis with constant angular velocity «, find the angular momentum, Ane. ~lyMa%o(3i + 3] ~ 8k) ost, 1d the moment of inertia of a uniform solid cone of radius a, height A and mass M about () the base, (b) the vertex. Ans. (a) jy Ma?, (0) yM(Qh? + a?) 1082, Find the principal moments of inertia at the center of a uniform elliptical plate having semi-major faxis a-and semi-minor axis b. Ane. Ty = M2, 1, = [Mat, [y= }M(a2 +09) 1083, A top has the form of a aolid circular disk of radius @ and mass M ‘with a thin rod of mass m and length I attached to its center [see Fig. 10-26]. Find the angular velocity with which the top should be spun to as to “sleep”. Assume that the base point O is fixed, 1084, Work Problem 10.88 for a cone of radius a, height h and mass M. 1085, Work Problem 10.88 for a cone of radius a, height hand mass Mf surmounted by a hemisphere of radius a and mass m. 1086. A coin of radius a is set spinning about a vertical axis with angular velocity « [see Fig. 10-27], Prove that the motion is stable if «2 > Ag/a. 4081. Suppore that the coin of Problem 10386 is spun with angular speed ‘about a diameter which is inclined at an angle « with the verti r— find the angular speed with which the coin precesses about the vertical. 1088, Discuss how gyroscopes can be used to control the motions of a ship on a stormy sea. Fig. 10.27 1089, ‘The vertex of a uniform solid cone of radius a, height h and mass M is fixed at point O of @ horizontal plane. Prove that if the cone rolls on the plane with angular velocity w about an axis, : fon ig SMAMG? + BHAT perpendicular to the plane through O, then the kinetic energy of rotation is SMITE TES 10.90. Explain how the principal axes of a rigid body can be found if the direction of one of the principal axes is known. 1091. A uniform solid cone has the radius of its base equal to twice its altitude, Prove that the ellipsoid of inertia corresponding to its vertex is a sphere. 1092, Explain how a gyroscope can be used as a compass, often called a gyrocompass. 1093, A dumbbell consists of two equal masses Mf attached to = rod ABC of length ! and negligible mass [see Fig. 10-28) ‘The aystem rotates about a vertical axis DCE with constant angular velocity « such that the rod makes a constant langle ¢ with the vertical. Prove that the angular momentum Q of the aystem describes a cone of angle =/2—0 about « and has magnitude $MPo sin CHAP, 10) SPACE MOTION OF RIGID BODIES 279 1094, 1095. 10.96. 10398, 10,100. 10101 r0.402, 10104, 108, 10.06. (a) Prove that the magnitude of the torque needed to keep the system of Problem 10.98 in motion JMS sin2e, (6) What is the direction of the torque? ‘Work (a) Problem 10.98 and (6) Problem 10.94 if the rod ACB has mass m. ‘A thin solid uniform circular plate of radius a has its center BR attached to the top of a thin fixed vertical rod OA (see \ Fig. 10-29), It is spun with constant angular speed wy about fan axis which is inclined at angle « with the normal OB to the plate, (a) Prove that the angular velocity vector « recesses about the normal OB with period 2x/(ay cosa). (8) Prove that the axis OB describes a space cone with period 2el(oyV1 ¥8 costa). Im Problem 10.96 find the angle through which the plate ‘turne during the time it takes OB to describe the space cone. Find the principal moments of i of radius, helt h and ma (0) center of mass Ane. (0) I, =I gyMlat +48), Ty = Mot 0) I= y= RMF + 4, Ly = yo? jertia of a uniform solid cone ‘M taken about the (a) vertex, ‘A compound pendulum of mass Mf oscillates about a horizontal axis which makes angles a, 2,7 ‘with reapect to the principal axes of inertia, If the principal moments of inertia are [y.ls:Ts respectively and the distance from the center of mass to the axis of rotation is 1, prove that for small oscillations the period is eV MII where I= MI®-+ I, cosa +I, coat + I, cos*y. Find the period of small oscillations of a uniform solid cone which rotates about horizontal axis attached to, the vertex of the cone. ‘An elliptical plate [see Fig. 10-80] having semi-major 5 ‘and semi-minor axes of lengths a and 8 respectively is rotated with constant angular speed xp about an axis 7 ‘making a constant angle « with the major axis. Find the torque required to produce this motion. ‘Work Problem 10.101 if the elliptical plate is replaced by an ellipsoid. Fig. 10.30 Given Euler’s equations of motion for a rigid body having zero external torque about a fixed point 0, i.e, Ids + Uy Taba + Uy— Tsay, = 0, Taba + a Tienes = 0 prove that Io + yeh + Iga = constant = 27° and Hel + thd + tof = constant = HP Prove from Problem 10.108 that oy, 0; and uy satisfy a differential equation of the form dy/éz = Y=) 2), and thus show that the angular velocity can be expressed in terms of elliptic funetions. Find the moment of inertia of « uniform solid cone of radius @, height hand mass Mf about a Tine which Ties in its surface, Ane. MaMa! + GN8)/(0! +14) ‘The moments and products of inertia of a rigid body about the 2,y and axes are Iz. = 3, Igy = 1073, Tee = 8/3, Iny = 4/3, Teg = —4/3, Tye = 0. Find (a) the principal moments of inc (B) the directions of the principal axes. Ane. (@) 1h =3, y=, y= 4 @) e, =1-21- 2k, e 2i+j— 2k, ~ 2+ 280 SPACE MOTION OF RIGID BODIES (CHAP. 10 107, A cone having semi-vertical angle « rolls with constant angular speed » on a horizontal plane with its vertex fixed at a point O. Prove that the axis of the cone rotates about the vertical axis through O with angular speed « tan « ‘A horizontal plane rotates about a vertical axis with constant angular velocity «, A uniform solid sphere of radius @ is placed on this plane. Prove that the center describes a circle with angular velocity given in magnitude by fs, 10109, Work Problem 10.108 if the sphere is not necessarily of constant density Ans. oK3/(K2 +02) where K is the radius of gyration about a diameter 10,10, Show how to find the relative maximum and minimum distances trom the origin to the ellipsoid @ = Ast + By? + Cat + Day + Ky2 + Fes = 1. [Hint. Maximize or minimize the function v= ae2+y2+2? subject to the condition © =1. ‘To do this use the method of Lagrange multipliers, ie. consider the function G=¥-+6 where Ais the (constant) Lagrange multiplier and set 9G/2z, aG/ay, 3G/2z equal to zero.) 10,11, Explain the relationship of Problem 10.110 to the method of page 255 for obtaining principal moments of inertia and directions of prineipal axes. (a) Find the relative maximum and Dat + 1Oy? + 822+ day — dz = 3. (®) Discuss the connection of the results of (a) with those of Problem 10.106, imum distances from the origin to the ellipsoid 10.113, Find the moment of inertia of the system of particles of Problem 10.42 about « li point (2,—1,3) in the direction ai—2)-+4k. through the 1114, Prove that the motion of the “sleeping top” of Problem 10.86 is stable if A? = dmglly/. 10.15, Find the moment of inertia of the lemniscate +2 cos2¢ about the z axis, Ans. §Mat 10.116. A plane rigid body (lamina) has an xy and 2'y' coordinate system with common origin O such that the angle between the «and 2’ axes is « (se Fig. 10-31]. Prove that (0) Tey = Iqy C088 ~ Bey Sin a C08 @ + yy Si? () Typ = Tessin + 2yy sina cosa + yy Sint Fig. 10.31 10417, Use Problem 10.116 to prove that Teg + Iyy = Test ley fand give a physical interpretation. 1418. Referring to Problem 10.116, find an expression for Izy in terms of Fem layslyy 8nd 10118. Use the results of Problems 10.116 and 10.118 to prove that for plane region having moments fand products of inertia defined by Izy Izy» lyy corresponding to a particular xy coordinate system, the principal axes are obtained by a rotation of these axes through an angle « given by tan 2a = Toy/(lpy~Lay)- . Prove that the lengths of the principal axes in Problem 10.116 are given by Mast y) * VE lee Tn CHAP. 10] SPACE MOTION OF RIGID BODIES 281 10, |. Discuss Problem 10.19, page 266, if I, > Is. 10122, Find the moment of inertia of @ uniform semicircular wire of mass M and radius a about its center. Ans. 2M(x~2)0%/y 10128, Prove that the exprestion on the left side of equation (4) in Problem 10.29 is the vertical component of the angular momentum. 10124, Discuss Problem 10.82 if the root of equation (J) is (a) equal to 1, (b) less than uy. 10:25, A rigid body consists of 3 particles of masses my, my and my. The distance between m, and ma; ‘my and mg; mg and m; are ly, la and lay Tespectively. Prove. that the moment of inertia of the system about an axis perpendicular to the plane of the particles through thelr center of mass is ae + mamta + mamath ‘my ma FM mymatts 10.12%, Derive a “parallel axis theorem” for products of inertia and illustrate by means of an example. 10427, Prove that the principal moments of inertia of a triangle of sides a,b,c and mass Mf about the center of mass are given by hah = Malte cena OEM, y= Kereta 1028. A coin of radius 1.5 em rolls without slipping on horizontal table such that the plane of the foin makes an angle of 60° with the table. If the center of the coin moves at a speed of 8 m/sec, prove that the coin moves in a cireular path and find its radius. Ans. 2.5 m Chapter 11 LAGRANGE'S EQUATIONS GENERAL METHODS OF MECHANICS Up to now we have dealt primarily with the formulation of problems in mechanics by Newton’s laws of motion, It is possible to give treatments of mechanics from rather general viewpoints, in particular those due to Lagrange and Hamilton. Although such treatments reduce to Newton's laws, they are characterized not only by the relative ease with which many problems can be formulated and solved but by their relationship in both theory and application to such advanced fields as quantum mechanics, statistical mechanics, celestial mechanies and electrodynamics. GENERALIZED COORDINATES ‘Suppose that a particle or a system of N particles moves subject to possible constraints, as for example a particle moving along a circular wire or a rigid body moving along an inclined plane. Then there will be a minimum number of independent coordinates needed to specify the motion, ‘These coordinates denoted by dy eon « are called generalized coordinates and can be distances, angles or quantities relating to them. The number » of generalized coordinates is the number of degrees of freedom [see page 165). Many sets of generalized coordinates may be possible in a given problem, but a strategie choice ean simplify the analysis considerably. NOTATION In the following the subscript « will range from 1 to n, the number of degrees of freedom, while the subscript » will range from 1 to N, the number of particles in the system. ‘TRANSFORMATION EQUATIONS Let r= sityd-+zk be the position vector of the vth particle with respect to an ye coordinate system. The relationships of the generalized coordinates (1) to thé position coordinates are given by the transformation equations Be = Adi Tes «+» Int) Yr = lds Gay +4 Inv t) @ Se = ld Ges oy ust) where t denotes the time. In vector form, (2) can be written By = iy Dy «24 Qu t) @) ‘The functions in (2) or (2) are supposed to be continuous and to have continuous derivatives 282, CHAP. 11] LAGRANGE'S EQUATIONS 283 CLASSIFICATION OF MECHANICAL SYSTEMS ‘Mechanical systems can be classified according as they are scleronomic or rheonomic, holonomic or non-holonomic, and conservative or non-conservative as defined below. SCLERONOMIC AND RHEONOMIC SYSTEMS In many mechanical systems of importance the time t does not enter explicitly in the equations (2) or (8). Such systems are sometimes called scleronomic. In others, as for example those involving moving constraints, the time ¢ does enter explicitly. Such systems are called rheonomic. HOLONOMIC AND NON-HOLONOMIC SYSTEMS Let 41, 42, ---, qu denote the generalized coordinates describing a system and let t denote the time. If all the constraints of the system can be expressed as equations having the form 6(q1, 42, ---,d» #) = 0 or their equivalent, then the system is said to be holonomic; otherwise the system is eaid to be non-holonomic. Compare page 170. CONSERVATIVE AND NON-CONSERVATIVE SYSTEMS If all forces acting on a system of particles are derivable from a potential function {or potential energy] V, then the system is called conservative, otherwise it is non-con- servative. KINETIC ENERGY. GENERALIZED VELOCITIES The total kinetic energy of the system is = 1Smit tg zm (4) ‘The kinetic energy can be written as a quadratic form in the generalized velocities Ge If the system is scleronomic [i.e. independent of time ¢ explicitly], then the quadratic form has only terms of the form dapfads. If it is rheonomic, linear terms in Ge are also present. GENERALIZED FORCES If W is the total work done on a system of particles by forces F, acting on the »th particle, then aw = Zodn CY where = pre © is called the generalized force associated with the generalized coordinate qu. See Problem 11.6. LAGRANGE'’S EQUATIONS The generalized force can be related to the kinetic energy by the equations [see Problem 11.10] (ar a) ~ a = * ” 284 LAGRANGE'S EQUATIONS (CHAP. 11 If the system is conservative so that the forces are derivable from a potential or potential energy V, we can write (7) as dab) _ ab a (sie) ~ ade (6) where L=T-v ) is called the Lagrangian function of the system, or simply the Lagrangian. ‘The equations (7) or (8) are called Lagrange’s equations and are valid for holonomic systems which may be scleronomic or rheonomic. If some of the forces in a system are conservative so as to be derivable from a potential V’ while other forces such as friction, ete., are non-conservative, we can write Lagrange's equations as % (10) where L=T—V’ and ®, are the generalized forces associated with the non-conservative forces in the system. GENERALIZED MOMENTA We define ar a eS to be the generalized momentum associated with the generalized coordinate qa. We often call pa the momentum conjugate to qa, or the conjugate momentum. If the system is conservative with potential energy depending only on the generalized (11) coordinates, then (17) can be written in terms of the Lagrangian L = T—V as aL. a B 12) Ps ode (a2) LAGRANGE'S EQUATIONS FOR NON-HOLONOMIC SYSTEMS ‘Suppose that there are m equations of constraint having the form DAadg + Adt = 0, LBadqa + Bat = 0, ... (13) or equivalently = SAede +A = 0, LB +B =O, (4) We must of course have m ” ‘The quantity m= © is called the generalized momentum or conjugate momentum associated with the generalized coordi- nate da: CHAP. 11) LAGRANGE'S EQUATIONS 289 11.11. Suppose that the forces acting on a system of particles are derivable from a potential function V, i.e. suppose that the system is conservative. Prove that if L=T—-V is the Lagrangian function, then ab) ab ait) ~ ie = 9 If the forces are derivable from a potential V, then (see Problem 11.7], = ww bl qa Since the potential, or potential energy is a function of only the ¢'s [and possibly the time ¢), 2 2p = ‘Then from Problem 1.10, 4 (ab) _ at | at aia) ~ 40 11.42, (a) Set up the Lagrangian for a simple pendulum and (b) obtain an equation describing its motion. (@) Choose as generalized coordinate the angle # made by string OB of the pendulum and the vertical OA [sce Fig. 11-4). If Lis the length of OB, then the kinetic energy is T= gomst = jolt = gmt o) | ee ‘The potential energy of mass m [taking as reference level # horizontal plane through the lowest point Ais given y V = mo(0A 00) = molt ~ t cos) = mgl(1 — cos 6) cc} ‘Thus the Lagrangian is L = TV = ymbit — mpllt — cone) 0) 0) tagrange' equation A) -# = From (8), 3 = mgt sine, 2 = mb 0 Substituting these in (), we find mbG + motsine = 0 or 74 Psine = 0 o ‘which is the required equation of motion (compare Problem 4.28, page 102]. 11.13, A mass M; hangs at one end of a string which passes over a fixed frictionless non- rotating pulley [see Fig. 11-5 below]. At the other end of this string there is a non-rotating pulley of mass M, over which there is a string carrying masses m: and ma. (@) Set up the Lagrangian of the system. (b) Find the acceleration of mass Mz. Let X, and X, be the distances of masses M, and Mz respectively below the center of the fixed pulley. Let 2; and ay be the distances of masses m, and mz respectively below the center of the ‘movable pulley My. Since the strings are fixed in length, Xy+ Xz = constant vy +2 = constant = b 290 LAGRANGE’S EQUATIONS (CHAP. 11 ‘Then by differentiating with respect to time ¢, kek oe oon and hth=0 0 &=-% ‘Thus we have Velocity of My Vecity of fy = ¥) = —f, —,.,r~—C ‘ = Sate, Velocity of my = $06,422) tay ‘Then the total Kinetic energy of the ayatem is Tom gk + ame + gmcky +08 + gmat, Fae co) ‘The total potential energy of the system measured from ah of the fixed pulley as reference ‘zontal plane through the center Vo = -MyoX, ~ MyoX, ~ molXy +2) ~ miki +9) = -MyoX, ~ Myala =X) ~ ma, +2) — ml, +8~24) ® Then the Lagrangian is L=r-v AMAT + gon + gy, +28 + gmat, 28 + MoX, + Myg(a~X) + mol, +a) + molX,+b—a) rc) Lagrange's equations corresponding to X, and 2; are @ (aL) a ae) a = ® From (8) we have Mag ~ Mag + mg + mg = (My— Maem tmio aks + May + my +a) + my — 2) = My + My tam t+ made + omy — may mg — mg = (mmo mys + dy) — maty— #8) = (my — may + (om tomy ‘Thus equations (4) become (y+ My + my t+ mds + Om — m4) B= (My — Mat rms ma my — madi, + (my +ma) y= (my mao Solving simultaneously, we find (Bt, = Melon +13) + Amma Be = FM Nom, Fy) + Arm ‘ 2Ma(my— ma) B= GE, Fm) + Sram? ‘Then the downward acceleration of mass My is constant and equal to Se = MET MaNGom + a — Arg SX GF Rain Fm) F omy 7 CHAP. 11) LAGRANGE’S EQUATIONS 201 11.14, Use Lagrange’s equations to set up the differential equation of the vibrating masses of Problem 8.1, page 197. Refer to Fa: 87 and 8 of page 197. The kinetic energy ofthe system is 7 = gil + Amid o Since the stretches of aprings AP, PQ and QB of Fig. 8-8 are numerically equal to ey, %,—%1 and zp respectively, the potential energy of the system is V = dealt daleg— ai? + ded @) ‘Thus the Lagrangian is L = PV = mit + 4mih — gaat — deley— 21)" — dae @) d (ab) _ ab aE) =o w Lagrange’s equations are a (ab\_ ab £(2)-% nose mantle © ert Be = ey noe ‘equations (4) become mE, = wlag—2xy), my = xle,— 2x9) 6) agreeing with those obtained in Problem 8.1, page 197. 1115. Use Lagrange’s equations to find the differential equation for which oscillates in a vertical plane about a fixed horizontal axi Let the plane of oscillation be represented by the zy plane of Fig. 11-8, where O is its intersection with the axis of rots ¥ tion and C is the center of mass. ‘compound pendulum Suppose that the mass of the pendulum is M, its moment of inertia about the axis of rotation In fy = MR [K'= radius of gyration), and distance OC = h. 1 # is the instantaneous angle which OC makes with the vertical axis through O, then the kinetic energy is T= lui? [S028 The potential energy relative to a horizontal plane through 0 is V = ~Mgh cons, Then the Lagrangian is L Since dL/#¢ =—Mgh sing and aL/sb = MK, Lagrange ‘equation is Aye MEO + Mohsine = 0 oF Bheing = 0 Compare Problem 9.24, page 287. Fig, 1 T= = MEOH + Mgh cose 11.16. A particle of mass m moves in a conservative force field. Find (a) the Lagrangian function, (b) the equations of motion in cylindrical coordinates (p, $,2) [see Problem 1.147, page 82). (a) The total kinetic energy T = Jmm[j?-+ 0438+ 44]. The potential energy V = Vio,#2)- ‘Then the Lagrangian function is b= POV = dnl t +H] —Voves2) 292 LAGRANGE’S EQUATIONS (OMAP. 11 (0) Lagrange’s equations are E(B) E = 0 te Loni - (mois 2) ° (ak) _ a (8) * 48)" oe ie. Gplmot +E = 0 or omy 4 ¥. fe od te doy+% = 0 o wee — 11.17. Work Problem 11.16 if the particle moves in the zy plane and if the potential depends only on the distance from the origin. Tn this cate V depends only on p and += 0. Then Lagrange’s equations in part (2) of Problem 11.6 become mG-o) = -%, Suri) = 0 ‘These are the equations for motion in a central force fleld obtained in Problem 6.8, page 122. LAGRANGE'S EQUATIONS FOR NON-HOLONOMIC SYSTEMS 11.48. Derive Lagrange’s equations (15), page 284, for non-holonomic constraints. ‘Assume that there are m constraint conditions of the form BAg dag + Adt , PBaday + Bat w where m<, the number of coordinates. gar ‘As in Problem 11.10, page 288, we have = 4(at) _ mt tos (GG) ~ ae = BSE % If ar, are virtual displacements which satisfy the instantaneous constraints [obtained by eons ing that time ¢ is a constant}, then te, = BR oe cy Now the virtual work done is aw = Yoko, = PEmi Toe = B¥ebte % Now since the vitoal work can be written in terms af the generalize forces y as BW = Seater 6) ‘we have by subtraction of (4) and (5), . 20e- Pa) 8d oO o Since the 84, are not all independent, we cannot conclude that ¥. Lagrange’s equations as obtained in Problem 11.10, ‘From (1), since is constant for instantaneous contraints, we have the m equations BAatia = 0% ZBadde = 0 wo re which would lead to ‘Multiplying these by the m Lagrange multipliers 4, ds, ... and adding, we have Pde + Bet) ae = 0 @ CHAP. 11] LAGRANGE’S EQUATIONS 293 Subtraction of (#) and (8) yields Beer = ida Maa 9) Bay = 0 o Now because of equations (7) we can solve for m of the quantities 8qq_ [sty 841, ---» Sau] in terms. of the remaining 8a, (S8¥ 8¢myin++or8¢q)- ‘Thus in (9) we ean consider 89, ..»)8dm 88 dependent fand 8¢q41-+++8dq 88 independent. Let us arbitrarily set the coefficients of the dependent variables equal to 2070, i.e, 6 1,2,..49m ao) Yq ~ Gq — Mg ~ Dae — ‘Then there will be left in the sum (0) only the independent quantities Sq, and since these are ar- bitrary it follows thst their coefficients will be zero. ‘Thus Yq q— Midg = PaBg 2 = = MH a a Equations (2) (10) and (11) thus lead to aan) _ at S(2)- bg + hidg + Be + (RE) ~ HE = eet fas required, These equations together with (1) lead to n-+m equations in n-+-m unknowns. aa L 2am ay 11.19. Derive equations (16), page 284, for conservative non-holonomic systems. From Problem 11.18, dar) _ ar aan) _ an 4 hide + Ba te a( x) ~ agg = tet Nhe + Be + o ‘Then if the forces are derivable from a potential, y= "Thus (2) ean be written aV/aq where V docs not depend on 4 (ab) _ ab S028) — HE = ate tbe + ® where L=T—V. 11.20. A particle of mass m moves under the influence of gravity on the inner surface of the paraboloid of revolution 2'+y?=az which is assumed frictionless [see Fig. 11-7]. Obtain the equations of motion, By Problem 11.16, the Lagrangian in eylindrieal eo- ‘ordinates is given by 1 = jmp ott i9 — moe wo ee eee oan Aare a o ee ee oes peta ine oo news Ay=%, Ap=0, As=—0 ® ee ae (SH) — Re = Me apey_m (4) -% Using (1), these become 204 LAGRANGE'S EQUATIONS (OHAP. 11 mGi— 688) = Dre ® mot) = 0 © mg — 0 © We also have the constraint condition : ay ab = 0 o ‘The four equations (4), (6), (@) and (7) enable us to find the four unknowns p,9,#,As. 1121. (a) () @) @ © Prove that the particle of Problem 11.20 will describe a horizontal circle in the plane z= provided that it is given an angular velocity whose magnitude is o= y2gla. Prove that if the particle is displaced slightly from this circular path it will undergo oscillations about the path with frequency given by (1/x)/2g/a. Discuss the stability of the particle in the circular path, ‘The radius of the circle obtained as the intersection of the plane =A with the paraboloid pear is i aoa a Letting « = A in equation (6) of Problem 11.20, we find dy = —mola ® ‘Then using (1) and (2) in equation (4) of Problem 11.20 and calling $=., we find (po 2 moles ot a? = 29/0" trom Which ‘melee - o = Viera © circular path are given respectively by =i? ® ‘The period and frequency of the part P= te and From equation (8) of Problem 1.20, we find e'§ = constant = A 6) Assuming that the particle starts with angular epecd o, we find A = ahe oo that 3 = ahalpt Since the vibration takes place very neatly in the plane ==, we find by letting in equation (0) of Problem 11:20 that A = —mo/a Using (6) and (2) in equation (4) of Problem 11.20, we find Pm athtat/p® = —2gp/a 8) Now ifthe path departs slightly from the circle, then » will depart slightly from pp. ‘Thus we fare Ted to make the transformation p= mtu () fn (f), where is amall compared with py. ‘Then (8) becomes = wt Wey ty a Oe Gotu on Ce But to @ high degree of approximation, 1 top = Alt wie ~ Altar a by the binomial theorem, where we have neglected terms involving wu%,.... Using the ‘values of py and ws given by (1) and (8) respectively, (£0) becomes CHAP. 11] LAGRANGE’S EQUATIONS 295 % + Gola = 0 © whose solution isu = «coe VEg/at + «sin VEg/at, Thus p= pp bu = Vak +n con Vig7at + ep sin VEg7a ¢ It follows that if the particle is displaced slightly from the circular path of radius 9 = Vah, it will undergo oscillations about the path with frequency or period Py Vs o It is interesting that the period of oscillation in the eireular path given by (4) is twice the period of oscillation about the eireular path given by (7). (e) Since the particle tends to return to the cireular path when it is displaced slightly from it, the motion is one of stability | 11.22, Discuss the physical significance of the Lagrange multipliers 4s, A, ... in Problem 11.18. In case there are no constraints the equations of motion are by Problem 11.10, = ot te + det MBa Ho It follows that the terms 44g + %sBa + +++ correspond to the generalized forces associated with constraints, Physically, the Lagrange multipliers are associated with the constraint forees acting on the system. Thus when we determine the Lagrange multipliers we are essentially taking into account the effect of the constraint forces without actually finding these forces explicitly. LAGRANGE’S EQUATIONS WITH IMPULSIVE FORCES 11.28, Derive the equations (20), page 285. For the ease where forces are finite we have by Problem 11.10, ajar) _ at at(gi,) ~ ae 7 where = BREE o Integrating thes 1) wth rept to from = 0 to aries : SA(B)a- fa = fh nae o a a ‘Taking the limit ax +0, we have »{(@)-(@S - w 296 LAGRANGE'S EQUATIONS [OHAP. 11 : (7 GE), = Bk = wang tf SE 0 ace Ett, and i fe 11.24. A square ABCD formed by four rods of length 21 and mass m hinged at their ends, rests on a horizontal frictionless table. An impulse of magnitude .J is applied to the vertex A in the direction AD. Find the equations of motion. After the square is struck, its shape will in gen- eral be a rhombus [Fig. 11-8] Suppose that at any time ¢ the angles made by sides AD (or BC) and AB (or CD) with the 2 axis fare 0; and 0p respectively, while the coordinates of the center M are (e,y). ‘Thus 2,14 #4, are the gen- ‘eralized coordinates, From Fig. 11-8 we see that the position vectors of the centers E,F,G,H of the rods are given re- spectively by (= Leoaepi + (y — Lain opi (e+ Leos ali + (y — Usin €s)5 (e + Leos mii + (y+ Ein gyi ty = (Leos aah + (y+ Lin ooh Fig 11-8 ‘The velocities of H, F,G and H at any time are given by ve = be = tT sing, Api + Leones 5 we = E= Using bgt + (Leos oy ve = Tsing, ADL + (I+ Leos: vu = iy = E+ Lain oy bgt (H+ L008 60 bi ‘The kinetic energy of a rod such as AB is the same as the kinetic energy of a particle of mass m located at its center of mass H plus the kinetic energy of rotation about an axis through E perpendicular to the zy plane. Since the angular velocity has magnitude 4, and the moment of inertia of a rod of length 2 about its center of mass is Iyy = }ml2, the total energy of rod AB is Tan = Ymrd + Mani Similarly, the total kinetic energies of rods BC, CD and AD are Tee = Amib + Machi, Teo = amid + Mevit, Tao = Ami + Aandi = yma Thus the total kinetic energy is [using the fact that J fe T = Ts + Tac + Ten + Tan fmGh +P + eb +t + 163+ ip = yale + apt + BP +208) + ymPGt + Bist + 2) + $m +) Let us assume that initially the rhombus fs a aquare at rest with ite sides parallel to the coordinate axes and its center located at the origin. ‘Then we have 2=0,y=0, 0 = 4/2, Oy B=0,9=0, 4, If we use the notation (); and ()q to denote quantities before and after the impulse is applied, we have hao @). = (ami), = 0 @, = Umi, = 0 a Gmrs), = 0 i, = mri.) = 0 CHAP. 11] LAGRANGE’S EQUATIONS ar a ), = Mae = ami Then - ory (st (R)-Gh), om were for simplicity we have now removed the subserDt a Tofind Fy Fy Foy Fo, we note that fo = where .J, are the impulsive forces. We thus have ary ap are itp Fe = IGE t Ia Fe t+ Ie Fe + In Fe ot arg ate arp y= IGA Ie GP + So Gy + So Fe ‘Now from Fig. 11-8 we find the position vectors of A,B,C,D given by ta = (e— Leone, ~ Losey) + (y ~ Using, + Usin 93) Fp = (@—Leose, + cos opi + (y —Isin 6, — I sin o3)} (2 + Leos, + Leoni + (y + Usin ay — I sin 693 ty = (@ +1 e086, = Leoni + (y + I sino, + Usin op Since the impulsive force at A is initially in the direction of the positive y axis, we have K= Thus equations (6){9) yield f= aS Then equations (1)(4) become Ami = 0, diy = J, $mPti, = — Jl cone, Fmlti = Jl e080 Geos, Fa, = Sh eosoy 297 o ® ® w © © o @ CO (10) ay us) 11.25. Prove that the kinetic energy developed immediately after application of the impulsive forces in Problem 11.24 is T= ,*/2m. From equations (12) of Problem 11.24, we have 35. 35 eee ace 298 LAG ‘ANGE'S EQUATIONS (CHAP. 11 Substituting these values in the kinetic energy obtained in Problem 11.24, we find (cos? + cos? a.) o But immediately after application of the impulsive forces, 6 =s/2 and ¢=0 appro Thus (1) becomes T= J2/2m, tely. MISCELLANEOUS PROBLEMS 11.26. In Fig. 11-9, AB is a straight frictionless wire fixed at point A on a vertical axis OA such that AB rotates about OA with constant angular velocity #. A bead of mass m is constrained to move on the wire. (a) Set up the Lagrangian. () Write Lagrange’s equations. (c) Determine the motion at any time. (a) Let r be the distance of the head from point A of the wire at time f The rectangular coordinates cof the bead are then given by y = reine sinet where it the #z plane and that the distance from 0 to A is h. Fig. 11-9 ‘The kinetic energy of the head is P= ymtseee 4mm(GF sin « con ot — or sina sin at)? + (Fsima sinat +r sin. cost)? + ¢ jm? + 08 into) ‘the potential energy, taking the xy plane as reference lve, V = mgs = mg(h ~ rc ‘Then the Lagrangian Le = T—V = $G2 ate sate) ~ moth — 1 cose) # eosa)t} () We have “ mate sin? + mg cosa, a i mo 4 (ak) _ ate al®)-F a land Lagrange’s equation is ~ (mate sinta + mg cosa) = 0 ey FH (aintayy = geore o (o) The general solution of equation (1) with the right hand side replaced by zero is eleanor 4 eye osinot fon ig 72282 Since the right hand side of (1) is a constant, a particular solution is |. Thus the general solution of (1) is = qqeteninae 4 ege-onine — gos. w ‘This result can also be written in terms of hyperbolic functions as + = ogcouh(u sina) + sinh winedt — $222 © CHAP. 11) LAGRANGE'S EQUATIONS 299 11.27. Suppose that in Problem 11.26 the bead starts from rest at A. How long will it take to reach the end B of the wire assuming that the length of the wire is 1? Since the bead starts from rest at ¢=0, we have r=0,#=0 at ¢=0, ‘Then from equa- tion (2) of Problem 11.28, ror rte, = Ze and aa =o Thos 6, = 6: = gGSBBE and (of Problem 11.25 neces fm BEBE (ctesmee s geno, = i gee / o p= GEE (conh(o sino 1) ® which can also be obtained from equation (8) of Problem 11.28. When r=T, (2) yields ‘cosh (w sina)t 0 that the required time is ae © = Samzeomet(1 + = skim 9) + 11.28 A double pendulum [see Problem 11.1(c) and Fig. 11-8, page 285] vibrates in a vertical plane. (a) Write the Lagrangian of the system. (b) Obtain equations for the motion. (@) The transformation equations given in Problem 11.2, page 286, 2 = boone, we = si yield The kinetic energy of the system is T= amity + aml + iD = gOAEA + dolla + BIE + 2h ds cos (6, — 09] ‘The potential energy of the system [taking as reference level a plane at distance {5+ ly below the point of suspension of Fig. 11-2) is Vi = moll +t2— 1 e004) + maplly + le — (hy 2086s + by co8 44) ‘Then the Lagrangian b=r-v = amalldt + dmalltdt + BH + 2ttaisés cos (6, ~o4)] aw malls + la — be e084] — mmaplla + ly ~ (h 086, +f 608 9)} (®) The Lagrange equations associated with 6, and #, are 4 (ab) _ ab 4(%)-2 300 LAGRANGE’S EQUATIONS. (CHAP. 11 From (1) we find = mama labyds sin (6,42) — mygl, sino, — magh sin 6 4 mal iy + mya’ c08 (0) = A imaliyiids sin (6, — 64) — mygly sin 4 — myghy sin and amalf By + mala, c08 (6, ~ 42) — mablaiG,~ 4) sin 049) = masa sin (0,— 62) — mapl sin og which reduce respectively to + mglyle By cos (8) — 69) + mal;ly63 sin (0, — 62) —(m, + mg)aly ) and amghh By + mails Bi, €08 (0,— 03) — malls sin(e,— ep) = —magly sing w 11.29, Write the equations of Problem 11.28 for the case m:=m=m and lt Letting m= ma t= ly in equations (@) and (4) of Problem 11.28 and simplifying, they can be written o ® BH, + Va, con (ayn) + WE sim 0) + Vig — Wi sing Vi, cos ( 11.30. Obtain the equations of Problem 11.29 for the case where the oscillations are assumed to be small. Using the approximations sing = 0, cos# tions (1) and (2) of Problem 11.29 become and neglecting terms invalving #, the equa- i, + 1, = 20m Ti+ thy = 0, 1131. Find the (a) normal frequencies and (b) normal modes corresponding to the small oscillations of the double pendulum. (a) Let 0 =A, cost, 6 = Ag coset [or Aye, Azelt] in the equations of Problem 11.0. ‘Then they can be written 2g Lada = Wy = 0) let, + (gle), = 0 o In order for A, and A, to be different from zero, we must have the determinant of the coeficients, ‘equal to 26r9, ie, 2g— ba) let a or Put digu?+2g? = 0. Solving, we find lg + V16E9* = BP g* (2+ v2W0 ee ee TT CHAP. 11] LAGRANGE’S EQUATIONS 801, ‘Thus the normal frequencies are given by 1, [@+Vv2i0 Dl () Substituting «=f =(@+VE)git im equations (1) of Part (a) yields A, = -VEA, 0 ‘This corresponds to the normal mode in which the bobs are moving in opposite directions, Substituting = f= (2—V)o/l in equations (1) of Part (a) yields Ay = VBA: o ‘This corresponds to the normal mode in which the bobs are moving in the same directions. and fa = 5° = 3, « 11.82, (a) Set up the Lagrangian for the motion of a symmetrical top [see Problem 10.25, page 268) and (0) obtain the equations of motion. (e) The kinetic energy in terms of the Euler angles (se Problem 10.24, page 268) ts T= ht Tach I) = MUG! sito + 3 + Al con + BF “ The pots fas seen from Fig. 10-18, page 269, since distance OC =1 and the height of the center of ‘mass C above the xy plane is therefore cos #. Thus T= TV = BiGtsite +9) + 4lAG cone + 9 — motcone ® energy is V = mot cose Oy o aL/ae = Iyit sine cone + 159 cone + 9-3 sine) + mol sine aLas = 1 aLlig = 0 AL/9§ = Tb ainte + 144 cone + $) cane | ably = 0 able = TAG cose +H) ‘Then Lagrange’s equations are W/L) wy 4/9) aby a agi) a= ae 8 a o H~ LP sins cone + 1G cosa + $15 sing — mol sing = 0 ® Aen tenn 2 ith sinte + 159 cone +H) cose] = 0 © AUG cone +9) = 0 o 11.33. Use the results of Problem 11.32 to obtain agreement with the equation of (a) Problem 10.29(b), page 270, and (b) Problem 10.27(a), page 270. (a) From equations (6) and (6) of Problem 11.82 we obtai on integrating, Ty sin? + 1,6 cose + }) cose = constant = K w feuree = A rn Using (2) in (1), we find ot IA cose = K (®) Using (2) in equation (4) of Problem 11.82, we find 1,8 — 13% sino cove + IyA3 sine = mglsine 302 LAGRANGE’S EQUATIONS [OHAP. 11 11.34, Derive Euler's equations of motion for a rigid body by use of Lagrange’s equations. ‘The kinetic energy in terms of the Euler angles is [see Problem 10.24 page 268} T= {sts tool + tad) = Gs ‘Then aT/ag = Iy(G sine siny +b cosyN(g sin cosy — 3 siny) sing +H cony)? + MyG@ sine cosy ~ & sing)? + AMS cone + H* + 146 sine cory ~ & sin y)i-$ sing siny — 3 eos) Ayeyeg + Ty(og)(—oy) (hho aio = Wide th) = tes ‘Then by Problem 11.10, page 288, Lagrange’s equation corresponding to y is (at) _ or a(S) a = fe or Igy + (h-Mayer = % O) ‘This is Buler’s third equation of (22), page 256. The quantity ty represents the general ized foree corresponding to a rotation y about an axis and physically represents the component 1g of the torque about this axis [see Problem 11.102]. ‘The remaining: equations Iidy + Ua Tewes = At cy Indy + ()—Idose, = a Cy can be obtained from symmetry considerations by permutation of the indices. They are not directly obtained by using the Lagrange equations corresponding to # and ¢ but can indirectly be deduced {rom them [see Problem 11.79] 11.85. A bead slides without frietion on a frictionless, wire in the shape of a cycloid (Fig. 11-10] with equations x = a(9—sind), y = a(1 + c0s6) where 0302+. Find (a) the Lagrangian function, (b) the equation of motion. Fig. 11-10 (o) Kinetic energy = 7 = mit +i) rna(C1 ~ con = maX(1 — cos 06? + [sing ) Potential energy = V = mgy = myga(i + cos) ‘Then Lagrangian = L = T—V = ma%1—coss)é*— mga(1 + cos) © (8) 8L = 0, ie. Sfamat(t — cosh] — [mat sine # + mga sin] = 0 a) « 5 Bad o Lacon — gsine dt — Laine = 0 which can be writen (1 cons) + sine ising = 0 11.86. (a) Show that the equation of motion obtained in part (b) of Problem 11.35 can be written 7 Ge + Gu = 0 where w= cos (0/2) omar. 11] LAGRANGE'S EQUATIONS 308 and thus (b) show that the bead oscillates with period 2-y/4a/g. (a) If w= cos(o/2), then f sin 6/2) 3 — 4 cos 22 at © in the sno os ~J sin 0/23 — 4 cos erie + Zcos(ei2) = 0 ‘whieh ean be written as + J eot (0/2) cot (0/2) = 0 o 2a 0s(o/2) _ 2 sin (o/2) sin (6/2) 2 aint (6/2) it follows that equation (1) is the same as that obtained in Problem 11.86(6). Since cot (6/2) i cose (®) The solution of the equation is u = cos(e/2) = c, cos V4algt + en sin Vialgt from which we see that cos(¢/2) returns to ite original value after a time 2eVa/g which is the required period. Note that this period is the same as that of a simple pendulum with An application of this is the cyeloidal pendulum, See Problem 4.86, page 112. 11.37. Obtain equations for the rolling sphere of Problem 9.42, page 244 by use of Lagrange’s equations. Refer to Fig. 9-88 in which and y represent generalized coordinates, Since the sphere of radius CP =a rolls without slipping on the sphere of radius OP =, we have bdgidt = adpldt or b6 = ab Which shows that if @=0 when y=0, then be = ap oy Thus 6 andy (and therefore dy and dy or 89 and 44] are not independent. The Kinetle energy of the rolling sphere ts T= 4mlat bri + gre dma OR + Jaman sje using the fact that T= mat is the moment of inertia of the aphere sbout a horizontal axis Ehrough its enter of mass ‘The potential energy of the rolling sphere [taking the horizontal plane through O as reference level] is V = mg(a+b) coss ‘Thus the Lagrangian is Lo = TH V = jolas OM + yratetbe ~ mola+b) cosy ” Meo Lees tin) ps fe ohne Fe hv bbg—asy = 0 (3) see meat .o4 mt ey erm ein af Pen 38 clam AL=b Ay wo ‘Thus equations (16), page 284, become AQ) : @ 304 LAGRANGE'S EQUATIONS (CHAP. 11 Substitution of (2) inté (6) and (6) yields nat O89 + gmarF+$) — moles) sine = x6 o gmaxS+9) = re © Substituting y= (bap [from (1)} into (7) and (8), we find inla+ B25 + fmarA+b/a)F — molatb) sing = Xb © ma%(1 + b/a) 3 (20) Now from (10) we have dy = —9mlet oy ‘and using this in (9) it becomes after simplifying and solving for ¢ 3 ‘This is the same equation as that of (2) in Problem 9.42, page 244, with g=/2—¢. To find the required angle At which the sphere falls off, see Problem 11.104, 11.88, (a) Solve the equations of motion obtained in Problem 11.24, page 296, and (b) inter- pret physically. (@) From the first of equations (12) in Problem 11.24 we have @ = constant = 0 o since #=0 at Similarly, from the second of equations (12) we have ® since y=0 at or on integrating, Thus since oy =/2 at t have 9 = 2/2. ‘From the fourth of equations (12) in Problem 11.24 we have similarly, 8g vende = Pa age or on integrating, Sint 7% - oe Now when wan(: tg = Ea aaerteetoiny (b) Equations (1) and (2) show that the center moves along the y axis with constant speed .J/4m. ‘The rods AD and BC are always parallel to the y axis while rods AB and CD slowly rotate until finally {t > «| the rhombus collapses, so that all four rods will be on the y a CHAP. 11) LAGRANGE'S EQUATIONS 305 Supplementary Problems GENERALIZED COORDINATES AND TRANSFORMATION EQUATIONS 1138. Give a set of generalized coordinates needed to completely specify the motion of each of the follow- ing: (a) a bead constrained to move on a circular wire; (6) a particle constrained to move on a phere; (c) a compound pendulum [see page 228|; (d) an Atwood’s machine [see Problem 3.22, page 76); (¢) a circular disk rolling on a horizontal plane; () a cone rolling on a horizontal plane. 11.40, Write transformation equations for the motion of a triple pendulum in terms of a suitable set of ‘generalized coordinates. HAI, A particle moves on the upper surface of a frictionless paraboloid of revolution whose equation is 28+ y? = cz, Write transformation equations for the motion of the particle in terms of a suit- able set of generalized coordinates. 11.42, Write transformation equations for the motion of a particle constrained to move on a sphere, CLASSIFICATION OF MECHANICAL SYSTEMS 1143, Classify each of the following according as they are (i) scleronomic or rheonomic, (ii) holonomic oF non-holonomic, and (if) conservative or non-conservative: (@) a horizontal cylinder of radius a rolling inside a perfectly rough hollow horizontal cylinder of radius b> a; (8) a cylinder rolling [and possibly sliding] down an inclined plane of angle a; (©) a sphere rolling down another sphere which is rolling with uniform speed along a horizontal plane; (@) & particle constrained to move slong a line under the influence of @ force which is inversely proportional to the square of its distance from a fixed point and a damping force proportional ‘to the square of the instantaneous speed. Ans. (a) seleronomie, holonomic, conservative () scleronomie, non-holonomic, conservative (6) rheonomic, non-holonomic, conservative (@) seleronomic, holonomic, non-conservative WORK, KINETIC ENERGY AND GENERALIZED FORCES: AA, Prove that if the transformation equations are given by ry the time ¢ explicitly, thon the kinetic eneray ean be written as r Bu(diede «+4 Gye Le do not involve where agg are functions of the da 1145, Discuss Problem 11.44 in ease the transformation equations depend explicitly on the time ¢. 1146, If FOz,.v,d«) = d* Fla, y,2) where 2 is a parameter, then F is anid to be a homogeneous function of order n.’ Determine which (if any) of the following functions are homogeneous, giving the order in each ease (a) Pb yet aytyetae (6) 29 tanMiy/z) (8) Se = 2y + 42 Asinzy (©) aye + 2xy +202 + Dye ) e+ut settee +e @ (etytae Ans. (a) homogeneous of order 2, (6) homogeneous of order 1, (@) non-homogencous, (d) homo- sneous of order sete, (e) homogeneous or erder 8, (f) nonhomogeneous, ) homogeneous of 1147, If Fle, y,2) is homogeneous of order oF 4 4 ax + Voy oe Problem 11.46), prove that, 1F ‘This i called Euler's theorem on homogencous functions. (Hint, Differentiate both sides of the identity F(\x,Ay,d=) = d*F(2,y,2) with respect to A and then place = 1.) 1148, Generalize the result of Problem 11.47, 306 1149, LAGRANGE’S EQUATIONS [OHAP. 11 Prove that if the transformation equations do not depend explicitly on time ¢, and T is the kinetic energy, then wa sai sgt. fag, + Gay, “a, = * Spee prove thin anecywithont the ute of Esler theorem on homagennas funtion [Problem aq]? LAGRANGE'S EQUATIONS 1150. 31 11532, 1158, 1154 1155, 1156, ust. 1158, 1138, (a) Set up the Lagrangian for a one dimensional harmonic oscillator and (6) write Lagrange’s equations. Ans. (a) L= $mi*—Jex?, (6) m¥ + (a) Set up the Lagrangian for a particle of mass m falling freely in a uniform gravitational field and (b) write Lagrange's equations. ‘Work Problem 11.61 in case the gravitational force field varies inversely as the square of the dis- tance from a fixed point O assuming that the particle moves in a straight line through 0. Use Lagrange's equations to describe the motion of a particle of mass m down a frictionless in- clined plane of angle «. Use Lagrange's equations to describe the motion of a projectile launched with speed vp at angle ‘with the horizontal Use Lagrange’s equations to solve the problem of the (a) two-dimensional and (b) three- dimensional harmonic oscillator. A particle of mass m is connected to a fixed point P on a horizontal plane by a string of length 1. The plane rotates with constant angular speed w about a vertical axis through a point O of the plane, where OP =a. (a) Set up the Lagrangian of the system. (0) Write the equations of motion of the particle. ‘The rectangular coordinates (x, v,2) defining the position of a particle of mass m moving in a force field having potential V are given in terms of spherical coordinates (r,¢,¢) by the transformation uations = 2 = rsingcose, y = rsingsing, 2 = rcose ‘Use Lagrange’s equations to set up the equations of motion. Ana, {it~ rit ri? costa] = —' or Ae uv [eed + vita genee] = EE FA mdisiany = aha ‘Work Problem 11.66 if the particle does not neceasarily move in a straight line through 0. Work Problem 4.28, page 102, by use of Lagrange’s equations. LAGRANGE'S EQUATIONS FOR NON-HOLONOMIC SYSTEMS 1180. 181. 1162. (a) Work Problem 120, page 293, if the paraboloid is replaced by the cone at+ y= ot! (@) What modification must be made to Problem 11.21, page 294, in this ease? Use the method of Lagrange’s equations for non-holonomic systems to solve the problem of a particle of mase m sliding down a frictionless inclined plane of angle «. Work Problem 3.74, page 82 by using the method of Lagrange's equations for non-holonomic systems, LAGRANGE’S EQUATIONS WITH IMPULSIVE FORCES 11.68. ‘A uniform rod of length and mass M is at rest on a horizontal frictionless table, An impulse of ‘magnitude .) is applied to one end A of the rod and perpendicular to it. Prove that (a) the Telocity given to end A is 4,J/M, (b) the velocity of the center of mass is J/M and (e) the rod Totates about the center of mass with angular velocity of magnitude 6,9/3!0 CHAP. 11] LAGRANGE’S EQUATIONS 307 1164, In Fig. 11-11, AB and BC represent two uniform rods having the same length {and mass M smoothly hinged at B and at rest on a horizontal frictionless plane. ‘An impulse is applied at C normal to BC in the di- A. rection indicated in Fig. 11-11 so that the initial velocity of point C is vy, Find (a) the initial vel- cities of points A and B and (8) the magnitudes of the initial angular velocities of AB and BC about their centers of mass Fig. 1-11 Ans. (a) vo/7,— 2%; (b) B0y/Tl, ~ Sve! TL 1145. Prove that the total kinetic energy developed by the system of Problem 11.64 after the impulse fn gato 11.66, A square of side @ and mase M, formed from 4 uniform rods which are smoothly hinged at their edges, rests on a horizontal frictionless plane. An impulse is applied at a vertex in a direction ff the diagonal through the vertex so that the vertex is given a velocity of magnitude 1. Prove that the rods move about their centers of mass with angular speed Sv9/4a. 1161. (a) If Jf is the magnitude of the impulse applied to the vertex in Problem 11.66, prove that the Kinetic eneray developed by the rods is given by 6,42/4M. (b) What is this kinetic energy in terms of ‘vo? (e) Does the direction of the impulse make any difference? Explain. 11.68. In Problem 11.24, page 296, suppose that the impulse is applied at the center of one of the rods in ‘a direction which is perpendicular to the rod. Prove that the kinetic energy developed is .2/8m. MISCELLANEOUS PROBLEMS 11,69. A particle of mass m moves on the inside of a smooth hollow hemisphere of radius a having its vertex on a horizontal plane. With what horizontal speed must it be projected so that it will remain in a horizontal circle at height h above the vertex? ‘A particle of mass m is constrained to move inside a y thin hollow frictionless tube [see Fig. 11-12] which is rotating with constant angular velocity » In a hor- iontal zy plane about a fixed vertical axis through ~\ 0. Using Lagrange’s equations, deseribe the motion. 11.21, Work Problem 11.10 if the zy plane is vertical. ‘m 11.72, A particle of mass m moves in a central force fleld having potential Vir) where + is the distance from the force center. Using spherical coordinates, (0) set up the Lagrangian and (b) determine the equa- tions of motion. Can you deduce from these equa- tions that the motion takes place in a plane (compare Problem 6.1, page 121]? Fig 11-12 11.73, A particle moves on a frictionless horizontal wire of radius a, proportional to the instantaneous speed. If the particle position of the particle at any time t. Ans. ¢ = (mv/s)(1 — e-stima) where @ is the angle which a radius drawn tom makes with a fixed radius such that @=0 at t=0, and « is the constant of proportionality. ted upon by a resisting force which tziven an initial speed vy, find the 1.74, Work Problem 11.78 if the resisting foree is proportional to the square of the instantaneous speed. we (BEE) 11.45. A spherical pendulum is fixed at point O but is otherwise free to move in any direction. Write ‘equations for its motion. 11.46. Work Problem 9.28, page 289, by use of Lagrange's equations 308 mu. 1178, a7, 11.80, m3. 1182, 1183, 115, 1186, nat, 1188, 1189, LAGRANGE’S EQUATIONS [CHAP..11 Work Problem 11.20 if the paraboloid of revolution is replaced by the elliptic paraboloid az = bat + cy where a,b,c are positive constants. Prove that the generalized force corresponding to the angle of rotation about an axis physically represents the component of the torque about this axis, (a) Obtain Lagrange’s equations corresponding to @ and ¢ in Problem 11.84, page 302, and show that these are not the same as equations (2) and (3) of that problem. (b) Show how to obtain ‘equations (2) and (8) of Problem 11.84 from the Lagrange equations of (a). ‘Two circular disks, of radius of gyrations Kj,Ky and masses my my WU respectively, are suspended vertically on a wire of negligible mas Fig. 1113). They are set into motion by twisting one or both of the disks in their planes and then releasing. Let a and #, be the angles made with some specified direction. (6) Prove that the potential energy is V = Jryet + ralea~ 004) ‘where 1 and ry are torsion constants, ie. the torques required to rotate the disks through one radian, (c) Set up Lagrange’s equations for the motion. Fig. 1143 Solve the vibrating system of Problem 11.80, finding (a) the normal frequencies and (b) the normal ‘modes of vibration. Generalize the results of Problem 11.80 and 11.81 to 3 or more disks (a) Prove that if m; mz and 1, # ly in the double pendulum of Problem 11.28, then the normal frequencies for small oscillations are given by w/e where (mat mitt l) = Vem F mim = BFE ma aE Bilas ’ (®) Discuss the normal modes corresponding to the frequencies in (@) e Examine the special case 1, = la, my % my_ in Problem 11.88, ‘Use Lagrange’s equations to describe the motion of a sphere of radius a rolling on the inner surface cof a smooth hollow hemisphere of radius b >. |A particle on the inside surface of a frictionless paraboloid of revolution az = *+y? at a height Hy above the vertex is given a horizontal velocity ty. Find the value of vy in order that the particle ‘oscillate between the planes #=/, and 2=Hz Ans. vy = V2gHs Find the period of the oscillation in Problem 11.86. [A sphere of radius a is given an initial velocity v up a frictionless inclined plane of angle « im ‘irection which ia not along the line of greatest slope, Prove that its center describes a parabola. ‘A bead of mass m is constrained to move on @ frictionless horizontal circular wire of radius @ ‘which is rotating at constant angular speed © about a fixed vertical axis passing through # point ‘on the wire. Prove that relative to the wire the bead oseillates like # simple pendulum. CHAP. 11) * LAGRANGE’S EQUATIONS 309 11,90, If a particle of mass m and charge ¢ moves with velocity v in an electric feld B and magnetic fleld B, the force acting on it is given by F = E+ ¥xB) In terms of a scalar potential 4 and a vector potential A the flelds ean be expressed by the relations: E= -Ve—aasot, B= VxXA Prove that the Lagrangian defining the motion of such a particle is L = 4mvtt e(ary)— ob Work Problem 10.86, page 278, by use of Lagrange’s equations. 1192, A uniform rod of length 1 and mast M has its ends constrained to move on the circumference of ‘smooth vertical circular wire of radius a> U/2 which rotates about a vertical diameter with con- stant angular speed s. Obtain equations for the motion of the rod. a Prove that the quantity r+ev- 34% 11.93, Suppose that the potential V depends on G, as well ai in & constant, ILM, Use Lagrange’s equations to set up and solve the two body problem as discussed in Chapter 5 [see for example page 121.) 11.96, Find the acceleration of the § gm mass in the pulley system of Fig. W-ld. Ane, T1g/622, 11.96, A cireular eylinder of radius @ having radius of gyration K with respect to its center, moves down an inclined plane of angle c. If the coefficient of friction is », use Lagrange’s ‘equations to prove that the cylinder will roll without slipping x it» < qipgitana. Discuss the cases where » does not satiaty this inequality 1191, Use Lagrange's equations to solve Problem 8.27, page 213. 11,98. Describe the motion of the rods of Problem 11.64 at any time t after the impulse has been applied. 1199, In Fig. 11-18, AB represents a frictionless horizontal plane having’ a small opening at 0.” A string of length 1 which A. B passes through O has at its ends a particle P of mass m and o P f particle Q of equal mass which hangs freely. The particle P is given an initial velocity of magnitude uo at right angles ‘to string OP when the length OP =a. Uct + be the ine stantaneous distance OP while # is the angle between OP and @ some fixed line through 0. Fig. 1-15 (©) Set up the Lagrangian of the system. (6) Write a diferential equation for the motion of P in terms of r (0) Find the speed of P at any position. Ane, (a) L = ymlai2 +2882] + mg(l—n) () F = ert o (oF = Vea F Bpla—n) = Baer 11,100, Work Problem 11,99 if the masses of particles P and Q are m, and m, respectively. 310 LAGRANGE’S EQUATIONS (OHAP.11 |. Prove that if vp = Va the particle P of Problem 11.99 remains in stable equilibrium in the circle r=a and that if ft is slightly displaced from this equilibrium position it oscillates about this position with simple harmonie motion of period 2-V2a/3g. 11,102, Prove that the quantity #, in Problem 11.84, page 902, physically represents the component As of the torque. 11.103. Describe the motion of the system of (a) Problem 11.68 and (b) Problem 11.66 at any time ¢ after ‘the impulee has been applied. 11.104, Show how to find the angle at which the sphere of Problem 11.37, page 308, falls off. 11.105, (@) Set up the Lagrangian for the triple pendulum of Fig. 11-16. (@) Find the equations of motion. 11.106, Obtain the normal frequencies and normal modes for the triple pendulum of Problem 11.105 assuming small oscillations. 11,07, Work Problems 11.205 and 11.106 for the ease where the masses and lengths are unequal. Fig. 1116 11,108, A vertical spring (Fig. 11-17] has constant « and mass M. If a mass m is placed on the spring and ‘et into motion, use Lagrange’s equations to prove that the system will move with simple harmonic motion of period 2eV(MH + 8m)/3e. Chapter 12 HAMILTONIAN THEORY HAMILTONIAN METHODS In Chapter 11 we investigated a formulation of mechanics due to Lagrange. In this chapter we investigate a formulation due to Hamilton known collectively as Hamiltonian ‘methods or Hamiltonian theory. Although such theory can be used to solve specific prob- Jems in mechanics, it develops that it is more useful in supplying fundamental postulates in such fields as quantum mechanics, statistical mechanies and celestial mechanics. THE HAMILTONIAN Just as the Lagrangian function, or briefly the Lagrangian, is fundamental to Chapter 11, s0 the Hamiltonian function, or briefly the Hamiltonian, is fundamental to this chapter. ‘The Hamiltonian, symbolized by H, is defined in terms of the Lagrangian L as Ho= Spie-b w and must be expressed as a function of the generalized coordinates qs and generalized momenta ps. To accomplish this the generalized velocities qa must be eliminated from (1) by using Lagrange’s equations [see Problem 12.3, for example]. In such case the function H can be written 1d t) @ or briefly H(Pa, qe, t), and is also called the Hamiltonian of the system. HDs, -..5 Pm HAMILTON'S EQUATIONS In terms of the Hamiltonian, the equations of motion of the system can be written in the symmetrical form ot = aie) ue @) Sa These are called Hamilton's canonical equations, or briefly Hamilton's equations. ‘The equations serve to indicate that the p, and q» play similar roles in a general formulation of mechanical principles. THE HAMILTONIAN FOR CONSERVATIVE SYSTEMS If a system is conservative, the Hamiltonian H can be interpreted as the total energy (kinetic and potential) of the system, ie., H=T+V ) Often this provides an easy way for setting up the Hamiltonian of a system. BL 312 HAMILTONIAN THEORY (CHAP. 12 IGNORABLE OR CYCLIC COORDINATES A coordinate q. which does not appear expli or cyclic coordinate. In such case tly in the Lagrangian is called an ignorable Da ) 50 that pa is a constant, often called a constant of the motion. In such case we also have aH/@q2 PHASE SPACE ‘The Hamiltonian formulation provides an obvious symmetry between the px and da which we call momentum and position coordinates respectively. It is often useful to imagine a space of 2n dimensions in which a representative point is indicated by the 2n coordinates (Diy 6p Pay Gin so o4 Mn) @ Such a space is called a 2n dimensional phase space or a pq phase space. Whenever we know the state of a mechanical system at time t, i.e. we know all position and momentum coordinates, then this corresponds to a particular point in phase space. Conversely, a point in phase space specifies the state of the mechanical system. While the mechanical system moves in the physical 8 dimensional space, the representative point describes some path in the phase space in accordance with equations (8). LIOUVILLE’S THEOREM Let us consider a very large collection of conservative mechanical systems having the same Hamiltonian. In such case the Hamiltonian is the total energy and is constant, i.e., HO which can be represented by a surface in phase space. Let us suppose that the total energies of all these systems lie between Hy and Es. Then the paths of all these systems in phase space will lie between the two surfaces H=B, and H=Bs as indicated schematically in Fig. 12-1. —_ Since the systems have different initial condi- tions, they will move along different paths in the phase space. Let us imagine that the initial points are contained in region R: of Fig. 12-1 and that after time t these points occupy region Re. For example, the representative point corresponding to one particular system moves from point A to point B. From the choice of R; and Rs it is clear that the number of representative points in them are the same. What is not so obvious is the follow- Fig. 124 ing theorem called Liowville’s theorem. 1Pm Uy «+04 e) = constant = E ” Bs Theorem 12.1: Liouville’s Theorem. The 2n dimensional volumes of R: and R: are the same, or if we define the number of points per unit volume as the density then the density is constant. CHAP. 12] HAMILTONIAN THEORY 313 ‘We can think of the points of , as particles of an incompressible fluid which move from Ri to Rein time t. THE CALCULUS OF VARIATIONS A problem which often arises in mathematics is that of finding a curve y = Y(z) joining the points where x =a and x= such that the integral Sl Peniar ® where y’ = dy/dz, is a maximum or minimum, also called an extremum or extreme value. ‘The curve itself is often called an extremal. Tt can be shown [see Problem 12.6] that a necessary condition for (8) to have an extremum is is(o) ~ which is often called Buler’s equation. This and similar problems are considered in a branch of mathematics called the caleulus of variations. 0 @) HAMILTON'S PRINCIPLE ‘The obvious similarity of (9) to Lagrange’s equations leads one to consider the problem of determining the extremals of So eas can ds rt) or briefly, “Lt where L=T—V is the Lagrangian of a system. We can show that a necessary condition for an extremal is d (aL aL a) - 5 = ° @) which are precisely Lagrange’s equations. ‘The result led Hamilton to formulate a general variational principle known as Hamilton's Principle. A conserv: tz in such a way that mechanical system moves from time f; to time Sona (22) sometimes called the action integral, has an extreme value. Because the extreme value of (12) is often a minimum, the principle is sometimes referred to as Hamilton's principle of least action. ‘The fact that the integral (12) is an extremum is often symbolized by stating that a af Lat = 0 (13) where 8 is the variation symbol. a4 HAMILTONIAN THEORY (CHAP. 12 CANONICAL OR CONTACT TRANSFORMATIONS ‘The ease in solution of many problems in mechanics often hinges on the particular generalized coordinates used. Consequently it is desirable to examine transformations from one set of position and momentum coordinates to another. For example if we call qa and pg the old position and momentum coordinates while Qa and P, are the new position and momentum coordinates, the transformation is Pa = Pa(ps, - denoted briefly by nr Gy var Oar Ay Qa = QalPry Gedy) (LD Pu = Pa(Pes dest), Qe = Qulay des t) (15) We restrict ourselves to transformations called canonical or contact transformations for which there exists a function .4{ called the Hamiltonian in the new coordinates such that (16) In such case we often refer to Q, and Px as canonical coordinates. ‘The Lagrangians in the old and new coordinates are L(ps, qt) and £(Pa, Qu t) re- spectively. They are related to the Hamiltonians H(po,qe,t) and .1(Ps,Qu,t) by the equations : Fi Ho= Dode-L, H = TPeQe-L (7) where the summations extend from a = 1 tom. CONDITION THAT A TRANSFORMATION BE CANONICAL The following theorem is of interest. Theorem 12.2. The transformation Pa = Pa(Pasdart), Qu = QalPar dant) (48) is canonical if LTredda — Y PudQe (19) is an exact differential. GENERATING FUNCTIONS By Hamilton's principle the canonical transformation (14) or (15) must satisfy the con- ditions that [" Lat and J" cat are both extrema, ie, we must simultaneously have 8 Ldt = 0 and = 0 (20) ‘These will be satisfied if there is a function G such that Gone ) dt See Problem 12.11. We call G a generating function. By assuming that G is a function, which we shall denote by cJ, of the old posi ordinates qa and the new momentum coordinates P, as well as the time ¢, i.e. CHAP. 12] HAMILTONIAN THEORY 315 G = Sta Post) (e) ‘we can prove that [see Problem 12.13] = 8S go 8d, ye m= 2 = 38, ow (28) where 2H G, = ah (2) Qa” ~ @Pa Similar results hold if the generating function is a function of other coordinates [see Problem 12.12]. THE HAMILTON-JACOBI EQUATION If we can find a canonical transformation leading to .#/ = 0, then we see from (24) that Pz and Qz will be constants [i.e., Ps and Qz will be ignorable coordinates). ‘Thus by means of the transformation we are able to find p. and qu and thereby determine the motion of the system. The procedure hinges on finding the right generating function. From the third equation of (23) we see by putting .s = 0 that this generating function must satisfy the partial differential equation SF + Hpadut) = 0 (5) or 8S 4 (3S qt) = 0 (26) This is called the Hamilton-Jacobi equation. SOLUTION OF THE HAMILTON-JACOBI EQUATION ‘To accomplish our aims we need to find a suitable solution of the Hamilton-Jacobi equation. Now since this equation contains a total of +1 independent variables, i.e. %) % ..., Qn and t, one such solution called the complete solution, will involve n+1 con- stants. Omitting an arbitrary additive constant and denoting the remaining n constants by Buy Ba, ...,8x (none of which is additive} this solution can be written SF = Sq de ooo Dny Bry By «+++ Boyt) (e7) ‘When this solution is obtained we can then determine the old momentum coordinates by =, Pe = 5 (28) Also, if we identify the new momentum coordinates P. with the constants (Ba, then a, = 38 (29) ae where y,,a=1,...,n are constants, : Using these we can then find g, as functions of 8, y, and t, which gives the motion of the system. CASE WHERE HAMILTONIAN IS INDEPENDENT OF TIME In obtaining the complete solution of the Hamilton-Jacobi equation, it is often useful to assume a solution of the form 316 HAMILTONIAN THEORY (CHAP. 12 oS = Ss(as) + Sa(q2) + +> + alan) + FUE) (80) where each function on the right depends on only one variable {see Problems 12.15 and 12.16]. This method, often called the method of separation of variables, is especially useful when the Hamiltonian does not depend explicitly on time. We then find that F(t) =—Et, and if the time independent part of of is denoted by S = Sila) + Solas) + +++ + Solan) (81) the Hamilton-Jacobi equation (26) reduces to as = H(B a) = E (32) where # is a constant representing the total energy of the system. ‘The equation ($2) can also be obtained directly by assuming a generating function S which is independent of time. In such ease equations (28) and (24) are replaced by as aS, a-n=8 (88) atl where qe Oe (4) PHASE INTEGRALS. ACTION AND ANGLE VARIABLES Hamiltonian methods are useful in the investigation of mechanical systems which are periodic. In such case the projections of the motion of the representative point in phase space on any Pade Plane will be closed curves C,. The line integral Je = § radan (28) is called a phase integral or action variable. We can show [see Problems 12.17 and 12.18] that S = Slay... du Jy (36) as. where Pe =o, Qu ~ ada” en It is customary to denote the new coordinates Qu by wa so that equations (37) are re- placed by _ #8 @ 3st (39) in this ease depends only on the constants J.. Then from the second equa- where 4 tion in (39), wa = fat +00 where fo and ca are constants. We call we angle variables. The frequencies f'« are given by fa= yy a ‘See Problems 12.19 and 12.20. CHAP. 12) HAMILTONIAN THEORY 817 Solved Problems THE HAMILTONIAN AND HAMILTON'S EQUATIONS 12.1. If the Hamiltonian H = pads — L, where the summation extends from a@=1 to n, is expressed as a function of the coordinates qa and momenta Pa, prove Hamilton's equations, ne og Ope regardless of whether H' (a) does not or (b) does contain the variable time ¢ explicitly. (a) H doce not contain t explicitly. ‘Taking the differential of H = Spqdq—L, we have GH = Spedig + Shedre — S3L day — BHF din “ ‘Then using the fact that Pq )L/aig and }_ = dL/2qq, this reduces to dH = Siéadea — Bieta @) But since ZT is expressed as a fonction of pa tnd das We have aH @ Comparing (2) and (3) we have, at (0) H doen contain t czpliity. In this ease equations (1), (2) and (9) of part (o) are replaced by the equations aH = Spadlig + Biedra — Bjgedte — ~ See 0) aH = Siedre — Biatlaa ~ Bat @ = 32 ay 4 HH qH = Bip et Bag Me + ae @) ‘Then comparing (5) and (6), we have i = aH Lat aoa a ae 122. If the Hamiltonian H is independent of t explicitly, prove that it is (a) a constant and is (0) equal to the total energy of the system. (a) From equation (2) of Problem 12.1 we have an = Siads - Bieta = 0 ‘Thus H is a constant, s (8) By Euler's theorem on homogeneous functions [see Problem 11.47, page 305), 318 HAMILTONIAN THEORY (CHAP. 12 where T is the kinetic energy. Then since py = 0L/@4q = 0T/@qq {assuming the potential V does not depend on jg], we have Zpady = 27. Thus as required, a Spade L ee ieee ee eee coe 123. A particle moves in the zy plane under the influence of a central force depending only on its distance from the origin. (a) Set up the Hamiltonian for the system. (b) Write Hamilton’s equations of motion. (@) Assume that the particle is located by its polar coordinates (r,4) and that the potential due to the central force is Vir. Since the kinetic energy of the particle is T— 4m(2-+r%), the Lagrangian is L = TV = gmitteiy — vey o We hate Py = ab /ok = mb, py = ablab = mot © 20 that = pum 5 = pelt cy pit + pad — (mbi2+ 298) — Vie) )-fin(hemgh)-ve} (&) Hamilton's equations are dy aii ‘Thus # = aiep, = pilm, 3 = aHlapy = pole 0 be = ~aH/ar = pi/mr Vir), by = —dHi/o0 = 0 @ Note that the equations (6) are equivalent to the corresponding equations (3). PHASE SPACE AND LIOUVILLE’S THEOREM 12.4, Prove Liouville’s theorem for the case of one degree of freedom. We can think of the mechanical system as eing described in terms of the motion of rep- resentative pointe through an element of vol- lume in phase apace. In the case of « mechanical system with one degree of freedom, we have & two dimensional (p,q) phase space and the vol- tume element reduces to an area element dpdg [Fig. 122). Let p=lprast) be the density of rep- resentative points, ie, the number of repre- sentative points per unit area as obtained by fan appropriate limiting procedure. Since the speed ‘with. which representative points enter through AB is 4, the number of representative points which enter through AB per unit time is eidp © Fig. 122 ‘The number of representative points which leave through CD is foi + Zenae} a» ® CHAP. 12] HAMILTONIAN THEORY 319 ‘Thus the mumber which remain in the element is (1) minus (2), oF — Fei dda Similarly the number of representative points which enter through AD and leave through BC are respectively iu mt fine Zenon} . Linn ” rae mewn pie ot og on Alem) , aod) -{ es sph ada Since this is equal to 28dpdq, we must have te , [Hod , ob) _ a+ hee} =o . dy My dog, Oy ey - tht at ose + eb = 0 © Now by Hamilton's equations > =—0H/ag, = aH/ap so that a FH aH ip ~~ pag" aq ~ Baap ‘Thus since we suppose that the Hamiltonian has continuous second order derivatives, it follows that 9p/ap = —ag/dq. Using this in (5), it becomes fe ge doy 8 4542p = 0 © But this can be written plat = 0 o which shows that the density in phase space is constant and thus proves Liouville's theorem. 12.5. Prove Liouville’s theorem in the general case. In the general case the clement of volume in phase space is aV = day--+ day dpy--+ dg In exactly the same manner as in Problem 12.4 the increase of representative points in dV is found equal to 22a, wo must have ‘This can be written as ° 0 820 HAMILTONIAN THEORY (CHAP. 12 Now by Hamilton's equations, =—dH/0qq, da = ati/@p_ so that tig _ 4H in _ OH a a Aiql@qq and (1) becomes es 3 (Bit B) = 0 ® ie, delat = 0 @ ‘or p = constant, Note that we have used the fact that if = elQy,.--1@uPiy-+-+Pq ) then oe 3 (2% bo te! a> 2 laa tea CALCULUS OF VARIATIONS AND HAMILTON'S PRINCIPLE 126. Prove that a necessary condition for 1 = f° Fl,1my’)de to be an extremum [maxt- (a) aE 0!" mum or minimum] is oy Suppose that the curve which makes J an extremum is given by y= Ye), «sesh « ‘Then y = Y@) tet) = Yto ® where « is independent of #, is a neighboring curve through z=a and 2=0 if we choose nla) = 9) = 0 ® ‘The value of I for this neighboring curve is 1) = Sf Fart Vite) de w ‘This is an extremum for «= 0, A necessary condition that this be so is that {Z|,_,=0. But by differentiation under the integral sign, assuming this is valid, we find which can be written on integrating by parts as Fea By ¥ - fee = where we have used (). Since y is arbitrary, we must have Ht) g oe A (M)- a0 dy dz \ay’, dz a which is called Euler's or Lagrange’s equation. ‘The result is easily extended to the integral Sf. Fle mths tertb rte de and leads to the Euler's or Lagrange’s equations CHAP. 12] HAMILTONIAN THEORY 321 ae\ng) ~ Oe By sing a Taylor serie expansion we fd rom 4) that to ~ 10) = « f° (Ea + Sy’) de + heb ondertarmmin #y ete, eco ei 54 aad th srt of tn eal an onto f tesnie re tut tat f° Fes) de ts on extrema i ton indeed By Lf Menards = 0 127. Discuss the relationship of Hamilton's principle with Problem 12.6. By identifying the function F(z, y,y’) with the Lagrangian L(t, q,@) where 2,y and y’ are re- placed by ¢,4,4 respectively, we see that a necessary condition for the action integral five ) to be an extremum {maximom or minimum) is given by @ (ab) _ ab a(@)- 3 = ° ® Since we have already seen that (2) describes the motion of a particle, it follows that such motion fan also be achieved by requiring that (1) be an extremum, which is Hamilton's princ FFor systems involving degrees of freedom we consider the integral (1) where = Utah dade turbo Which lead to the Lagrange equations 10)-& 128. A particle slides from rest at one point on a frictionless wire in a vertical plane to another 9 point under the influence of gravity. Find the total time taken. Let the shape of the wire be indicated by curve C in Fig. 12-8 and suppose that the starting and fin- hing pointe are taken to be the origin and the point Alay, vo) Feapectively. Let P(z,y) denote any position of the particle which we assume has mass m. From the principle 0 @=42 00.50 ‘Alum of conservation of energy, if we choose the horizontal ¥ line through A as reference level, we have Fig. 12-3 Potential energy at 0 + Kinetic energy at 0 potential energy at P + kinetic energy at P or mays + 0 = molyo—w) + dm(da/aty® where de/d is the instantancous speed of the particle at time t. Then delat = =v w If we measure the arc length # from the origin, then # increases as the particle moves. Thus do/dt is positive, so that de/dt = VBgy or at = da/V2gy. ‘The total time taken to go from y=0 to y= vp is 322 HAMILTONIAN THEORY (CHAP. 12 ® 129. If the particle of Problem 12.8 is to travel from point O to point A in the least pos- sible time, show that the differential equation of the curve C defining the shape of the wire is 1+y*+2yy” = 0. ‘A necessary condition for the time r given by equation (2) of Problem 12.8 to be a minimum aa a(e)_ oF £(5)-# = 0 wo where Paty yaya . Now aPlay = (ty yy, Akay = —4t yyw Substituting these in (1), performing the indicated differentiation with respect to x and simplifying, ‘we obtain the required differential equation. ‘The problem of finding the shape of the wire is often called the brachiatochrone problem. 12.10. (a) Solve the differential equation in Problem 12.9 and thus (b) show that the required curve is a cycloid. (@) Since 2 ia missing in the diferential equation, let y/ = «so that du _ du dy day _ du = de ~ dy dz dy" dy a+ od 2udu , dy L+wt amg = 0 or Pee = 0 Integration yields n(+u + iny = Ind or = Fwy = where b is @ constant, Thus -y-%- Cee ee New since the slope must be positive, Separating the variables and integrating, we find 2 = SVPGere Letting y= 6 sin, this ean be written -! wf owed re =f a-cumde se = yoermainan +6 Bsint# . 2p gine cose de + 6 ‘Thus the parametric equations of the required curve are 2 = UQe—sin29) +0, y = bainte = 4b(1 ~ cose) Since the eurve must pass through the point 2=0,y=0, we have ¢= 0 ‘Then letting p aw oem the required parametric equations are 2 = alg—sing), ¥ (1 ~ cos ¢) ® CHAP. 12] HAMILTONIAN THEORY 323 (8) The equations (2) are parametric equations of a eycloid (see Fig. 12-4}. The constant @ must bbe determined so that the curve passes through point A. ‘The eyeloid is the path taken by & fixed point on a circle as it rolls along a given line (see Problem 12.89), Fig. 24 CANONICAL TRANSFORMATIONS AND GENERATING FUNCTIONS 12.11, Prove that a transformation is canonical if there exists a function g such that dgidt = and 3 ff" cat =0 ‘Thus by subtraction, G-xat = 0 ‘This can be accomplished if there exists a function g such that b=“ = dglét ince in such ease s(Glt)— 9} = 0 ‘The function ¢ is called generating function. 12,12, Suppose that the generating function is a function T of the old and new position co- ordinates qu and Qu respectively as well as the time t, ie. T = T(qe, Qz,t). Prove that a ete 3 ad m= G2, Pe where Pe = So, Qe = Soe By Problem 12:1, ar aos f. Ge = Uk = Breda H- {BP aea~ = Brade- BPads +. -H or at = Zpedig— BPatQy + (s- Hat 0 But if T= T(gus art, then = 30 2 ag, + 9 ar = Btu + BQ, + Tae ® Comparing (1) and (2), we have as required 8 pag ee a ae ‘The equations b= 3h a oe follow from the fact that .4{ is the Hamiltonian in the coordinates Pq, Qq so that Hamilton's equa- tions hold se in Problem 121. 24 HAMILTONIAN THEORY [cHap. 12 12.13. Let 0, it is a hyperbola. This agrees with the results of Chapter 5. ‘The equation (11) when integrated yields the position as a function of time, 328 HAMILTONIAN THEORY (CHAP. 12 PHASE INTEGRALS AND ANGLE VARIABLES 12.17. Let J be a complete solution of the Hamilton-Jacobi equation containing the n constants BurssssBu Let Ju = § padga. Prove that the Jz are functions of the fa only. We have S = Silay Bis -+-y Bad + 20+ + SalGar Bay +--+ By) — Bat a where the constant p, =, the total energy. Now aS aSy fo te ~ Hie . ae Thos Ju = Grete = $ Bag, But in this integration g, in integrated out, so that the only quantities remalning are the con- stanta fy, Bye Ths we have the » equations Je = SelB cseyBe) 2 tye ® Using (4) we cam solve for (iy... fy in terms of Jy,...yJq and express (1) in terms of the Jy. 12.18. (a) Suppose that the new position and momentum coordinates are taken to be wa and Ja respectively. Prove that if is the new Hamiltonian, 21/004, tra = d.H/Ada (8) Deduce trom (2) that Ja = constant and wu = fat tea where fe and cy are constants and fs = 3.1/0. (a) By Hamilton's equations for the canonical coordinates Qu, Pay By _HIQa Qe = 2,3/0Py ‘Then since the new position and momentum coordinates are taken as Q. these equations hecome a = ald the = a.$105q ® () Since ./ = E, the new Hamiltonian depends only on the J, and not on the w,. ‘Thus from (2) we have constant = fy ® where fy = 0,9[/idq. From (8) we find, as required, Jaq = constant, Wa = fat + eq wo ‘The quantities Jj, are called action variables while the corresponding integrals § ratte = 4. © where the integration is performed over a complete eyele of the coordinate qq, are called phase integrate. The quantities w, are called angle variables 12.19. (a) Let awe denote the change in w. corresponding to a complete cycle in the partieu- lar coordinate qr. Prove that [ 1 ifasr we = lo tame CHAP. 12] HAMILTONIAN THEORY 329 (0) Give a physical interpretation to the result in (a). ° ane = Site = Sa (B)on = Fare (S) a Epa B= (here aS in, We = Yo ene where we have used the fact that w, = aS/8Jq sumed that the order of differentiation and integration wwe Problems 12.17 and 12.18] and have as- immaterial. () From (a) it follows that 1, changes by one when gq goes through a complete eyele but that: there is no change when any other q goes through a complete cycle. It follows that gq is a periodic function of w, of period one. Physically this means that the f, in equation {4) of Problem 12.18 are frequencies, 12.20. Determine the frequency of the harmonic oseillator of Problem 12.15. A complete cyele of the coordinate q (see equation (10), Problem 12.18) consists in the motion from q=—VBfie to q=+V2Ble and back to q=~V2p/s. ‘Then the action variable is Be WRT Fo= Gee = 26 vem pe an = 4 6 vane ae = tpVare ri a zl Thus eae=Zle= x ana = Ho ae 12.21. Determine the frequency of the Kepler problem [see Problem 12.16). A complete cycle of the coordinate r consists in the motion from r= rn tO Tax and back to = Meine Where Tin And Tygx are the minimum and maximum values of given by the zeros of the quadratie equation (see equation (£0), Problem 12.16] 2mBy + 2mKir — pire = 0 wo We ten have from equations () and of Pablo 1238 a3, a = fe = $80 = $e = ne een on t= Girt = $a = Ge af v= Ae semK VIR ~ 2 o From (and () we bare on Smnatin of Jo J, = tomb Sines y=, (snl ent nt ae Get Te oe see FTP Then th revere ae 1H _ det? aH 4s2mK? f= a, c Since these two frequencies are the same, system is degenerate Ii? CTF there is only one frequency, we say that the 330 HAMILTONIAN THEORY (CHAP. 12 MISCELLANEOUS PROBLEMS 12.22. A particle of mass m moves in a force field of potential V. Write (a) the Hamiltonian and (b) Hamilton's equations in spherical coordinates (r, 6, 4). (@) ‘The Kinetic energy in spherical coordinates is T= YG + 18+ re ainte 3) o "Then the Lagrangian is L = PAV = ot + 1280 + 98 sinto 3) — Verwe.9) © We have ’ A ‘ mi, py = ab/03 = mr8é, py = ab/06 = met sinte 3 ® and #=%, joan ny ‘The Hamiltonian is given by H = Spadg—b = De t+ web + pod — dmbi? + v5? + v2 sin? e 32) + Virvo,9) we Pe PB = Be et tag t+ Vine) ° ‘where we have used the results of equations (5). We can also obtain (5) directly by using the fact that for conservative systems the Hamiltonian is the total energy, ie. H= T+. ie ae 8) Hamiton's equations are da = 24, jg =—2H. Then from part (e), (©) Hamilton’ equat Ga = ME, a= - BE then trom part ( poMoe, Ga =e age = ang ~ Pain? #,_ 8 as * ar sinte ~ Gr san _ sheose ov ante! eee ae Ae re Pe ae ae 12.23. A particle of mass m moves in a force field whose potential in spherical coordinates is V =—(K cos#)/r*, Write the Hamilton-Jacobi equation describing its motion. By Problem 12.22 the Hamiltonian is a Writing p,= o ® Dh 12,24. (a) Find a complete solution of the Hamilton-Jacobi equation of Problem 12.23 and (b) indicate how the motion of the particle can be determined. (@) Letting of = (2) of Problem 12.23, can be written 1) + S,(6) + Sle) — Bt in equ CHAP. 12] HAMILTONIAN THEORY 381 | E(B) +s) +artn(S) Est = so (BE) mae = ~~ + mare Since the left side depends only on r while the right side depends on @ and ¢, it follows that ‘each side must be a constant which we shall call @y. Thus (Smee = 5 : snd ' : ‘Multiplying equation (8) by sin? and rearranging terms, | (25) = anaes ~ ts ae (2) . Since the left side depends only on while the right side depends only on ¢ each side must be & constant which we ean eall 8, However, since ts _ a n= a Se ‘we can write ,= pj. This is a consequence of the fact that ¢ is a cyclic or igmorable co- Salt Pa fy eon a2)? os an ants cone ~ ayante ~ site ($8)P = og By solving equations (2), (6) and (6), we obtain 5, = f VinB Fa dr, 8 = f VinK eee pate Ti de, 84 = Dye where we have chosen the positive square roots and omitted arbitrary additive constants, ‘The ‘complete solution = SVR A e+ f RETA Hee + re ~ (8) The required equations of motion are found by writing as, as, a eT BT ‘and then solving these to obtain the coordinates r, 9 as functions of time using initial condi- tions to evaluate the arbitrary constants, 12.25. If the functions F and G depend on the position coordinates qe, momenta ps and time +, the Poisson bracket of F and G is defined as wel = EGR ae ~ Ream) Prove that (a) [F,G] = ~[G, Fl, (b) [F:+F:,G) = [FG] +(FnG], (0) (Fea. aF apr, (d) (F,, pr] = —aF faa. © ha = ) = -3(22 3 (2 S228) 6,7 ‘This shows that the Poisson bracket does not obey the commutative lav of algebra, B82 HAMILTONIAN THEORY (CHAP. 12 a+ Pe) ag _ HER, . intrya) = 3 {Fe 36 MTD oe ee (OF s 3G 5 ) + 8G ie ae (FG) + [FG] ‘This shows that the Poisson bracket obeys the distributive law of algebra, ar OF 4G oF F, ar ite _ ar Mr) oF ® Wed = 3 (irae aacam) ~ ter since ag/qq=1 for «=r and 0 for a%r, while a4,/2Pq=0 for all «, Since r is ar Ditrary, the required result follows. @ since dp,/ bitrary, the required result follow 12.26, If H is the Hamiltonian, prove that if f is any function depending on position, momenta and time, then df _ af R= te o ‘But by Hamilton's equations, ro) ‘Then (2) can be written a a Supplementary Problems ‘THE HAMILTONIAN AND HAMILTON'S EQUATIONS 1227, A particle of mass m moves in force field of potential V. (a) Write the Hamiltonian and (6) Ham- itoa’s equations in rectangular coordinates (#18). ‘Ana. (a) H = (p+ p+ pD/2m + Viz.w3) (©) B= palm, i= pm, = pdm, Be = ~OV U0, 3, —aV oy, B, = ~8V/08 12.28. Use Hamilton's equations to obtain the motion of a particle of mass m down a frictionless inclined plane of angle «. 1229, Work the problem of small oscillations of simple pendulum by using Hamilton's 1230, Use Hamilton's equations to obtain the motion of @ projectile launched with speed v at angle « ‘with the horizontal, CHAP. 12] HAMILTONIAN THEORY 333 1231, Using Hamilton's equations, work the problem of the harmonic oscillator in (a) one dimension, (6) two dimensions, (e) three dimensions. 1282 Work Problem 3.27, page 78 by using Hamilton's equation. PHASE SPACE AND LIOUVILLE'S THEOREM 1283. Explain why the path of a phase point in phase space which represents the motion of a system of particles ean never eross itself 1284, Carry out the details in the proof of Liouville’s theorem for the case of two degrees of freedom. CALCULUS OF VARIATIONS AND HAMILTON'S PRINCIPLE 1235. Use the methods of the calculus of variations to find that curve connecting two fixed points in a plane which has the shortest length. 1286, Prove that if the function F in the integral Is an extremum if P—y'Fy =e where ¢ Plzwyv)de is independent of 2, then the integral constant, 12.87. Use the result of Problem 12.86 to solve (a) Problem 12.9, page 322, (b) Problem 12.86. 1288, It is desired to revolve the curve of Fig. 12-5 hav- ing endpoints fixed at P(e,,y;) and Q(2, ys) about ‘the 2 axis 40 that the area I of the surface of revolution is a minimum, (0) Show that 1= 27S” witytas. (®) Obtain the differential equation of the curve, (©) Prove that the required curve is a catenary, Ane. (b) wy!" = 14 (y? Qeov9 1239. Two identical circular wires in contact are placed in a soap solution and then separated so as to form a soap film. Explain why the shape of the soap film surface is related to the result of Prob- Tem 12.38, Fig. 125, 1240, Use Hamilton's principle to ind the motion of a simple pendulum. 1241. Work the problem of a projectile by using Hamilton's principle. 1242, Use Hamilton's principle to find the motion of a solid cy angle a. ider rolling down an inclined plane of CANONICAL TRANSFORMATIONS AND GENERATING FUNCTIONS 1243, Prove that the transformation Q = p, P -q_ is canonical. 1244, Prove that the transformation Q = q tan p, P = In sin is canonical, 1245, (@) Prove that the Hamiltonian for a harmonic oscillator ean be written in the form H = $pi/m-+ deat. () Prove that the transformation q = -[P/Vesin@, p = JmPVrcos@ is canonical. (©) Express the Hamiltonian of part (a) in terms of P and Q and show that @ is eyelic. (@ Obtain the solution of the harmonic oscillator by using the above results

You might also like